Sunteți pe pagina 1din 363

P.F.

Combes

2e cycle universitaire / coles d'ingnieurs

Micro-ondes
1.

Lignes, guides et cavits


Cours et exercices

Micro-ondes
1. Lignes, guides et cavits
Cours et exercices

Micro-ondes
1. Lignes, guides et cavits
Cours et exercices
Paul Franois
Combes
9
Professeur d'universit
Docteur s sciences

Avant-propos
Les micro-ondes se sont beaucoup dveloppes depuis les annes 1940 et elles
ont actuellement des applications nombreuses et trs importantes pour les tl
communications tant terrestres que spatiales, pour le radar et les systmes de
radionavigation, pour lobservation et la tldtection de la Terre ainsi que pour
le chauffage industriel et domestique. Elles prsentent aussi une grande utilit en
mdecine o lhyperthermie micro-onde est tudie pour le traitement des
tumeurs cancreuses, en radioastronomie o les grands radiotlescopes micro
ondes ont permis des progrs dcisifs dans la connaissance de lUnivers, en
recherche physique aussi bien en spectroscopie que pour le traitement et la
caractrisation des matriaux. Par ailleurs, les trs grandes puissances fournies
par des tubes micro-ondes comme les klystrons sont indispensables dans les
acclrateurs de particules, les dispositifs tudis pour la fusion thermonuclaire
contrle ou mme le projet trs ambitieux de satellite de puissance solaire.
Lenseignement des micro-ondes a suscit beaucoup douvrages en langue
anglaise mais relativement peu en langue franaise. Pourtant les chercheurs fran
ais se sont intresss trs srieusement aux micro-ondes ds les annes 1930 et,
aprs la coupure de la Seconde Guerre mondiale, un nouvel lan, surtout indus
triel, a t pris dans les annes 1950.
Mais il aura fallu attendre les annes 1970 pour que la recherche universitaire
micro-ondes devienne trs active en France, comme en tmoigne, depuis lors,
lorganisation rgulire de Journes nationales de Micro-ondes prsentant les
travaux de nombreux centres de recherches dynamiques. Cependant, cette acti
vit a donn lieu des publications trs spcialises et des ouvrages de syn
thse dun niveau trop lev pour nos tudiants dIUT, de licence et de matrise,
dIUP et de formation continue ainsi que pour la plupart des lves des coles
dingnieurs.
Le prsent ouvrage a pour but de proposer une prsentation pdagogique
de lenseignement des micro-ondes, adapte aux niveaux du premier cycle
(2e anne) et du second cycle de lenseignement suprieur, cest--dire un
stade de formation o les jeunes gens ont dcouvrir les micro-ondes et
acqurir une solide formation de base. Il est le fruit de ma dj longue exp
rience de lenseignement des micro-ondes non seulement luniversit Paul
Sabatier tant lIUT quen matrise EEA, en licence de tlcommunications et
lIUP mais aussi en troisime anne dcoles dingnieurs, notamment Sup-

Aro, Sup-Tlcom, lENSEEIHT et lENAC et enfin en formation continue. La


rdaction de ces chapitres a donc t teste et renouvele travers un enseigne
ment complet des promotions dtudiants, dlves-ingnieurs et de stagiaires
de divers niveaux et cursus.
Cet ouvrage est form de deux volumes que les ditions Dunod publieront en
1996 et en 1997. Le premier volume traite des notions de base, prsentes en
deux parties :
Lune concerne les lignes utilises pour la transmission et les circuits. Leur
tude est effectue laide des concepts de tension et courant qui permettent
un expos didactique et facilitent la comprhension des principales proprits
caractristiques du phnomne de propagation. Les sujets traits approfondis
sent galement la rflexion sur une ligne, le diagramme de Smith et le pro
blme de ladaptation. Les cas des lignes avec pertes et des lignes en rgime
transitoire font lobjet de deux chapitres. Enfin sont exposes les applications
de ces bases thoriques pour les lignes bifilaires, coaxiales, microbandes et
fentes ainsi que pour la ralisation des impdances et des circuits accords,
essentielle pour les circuits de llectronique micro-onde.
Lautre partie de ce premier volume concerne les guides d'ondes et les cavits
qui sont videmment tudis laide des concepts de champs lectrique et
magntique. La progression pdagogique adopte, permet dtudier dabord la
rflexion et la rfraction des ondes lectromagntiques une interface entre
deux milieux. Puis le guide d ondes rectangulaire est prsent comme une
structure dans laquelle les ondes se propagent par rflexions successives sur
les surfaces mtalliques planes qui le dlimitent ; cest seulement aprs avoir
donn cette prsentation physique des phnomnes quest effectue ltude
mathmatique de la propagation dans les guides dondes mtalliques partir
des quations de Maxwell. On retrouvera ces deux aspects complmentaires
dans ltude des guides dondes dilectriques. Enfin, le chapitre sur les cavits
lectromagntiques ne traite pas seulement des conditions de rsonance dune
cavit et de sa modlisation, il sintresse galement au couplage dune cavit
par un ou deux accs.
Afin que ce livre soit un outil dtude et de travail complet, des noncs dexer
cices sont proposs la fin de la plupart des chapitres, avec indication du para
graphe auquel se rapporte chaque exercice. En gnral, il sagit dexercices qui
doivent pouvoir tre traits en quinze trente minutes maximum pour un tu
diant qui a bien appris son cours. La solution complte de chacun des 82 exer
cices est donne la fin du livre. En faisant lui-mme ces exercices, en tudiant
ensuite la solution propose, le lecteur de ce livre dispose dune relle possibilit
dapprofondissement et dassimilation du sujet tudi.
Je tiens remercier trs vivement les collgues qui mont fait lamiti de vrifier
lexactitude des solutions fournies, et simultanment, deffectuer la relecture des
chapitres correspondants : Messieurs Michel Aubs et Gabriel Soum tout parti
culirement ; Mesdames Isabelle Chnerie et Christine Galy ; Messieurs JeanLouis Amalric, Jacques David et Roger Kt.

Le second volume de cet ouvrage sur les micro-ondes devrait tre publi en 1997 ;
il traitera, en trois parties, des sujets importants que sont :
les circuits passifs rciproques et non rciproques,
la propagation des ondes lectromagntiques en espace libre,
les antennes,
et il inclura, de mme, des exercices avec solutions compltes.
Comme il faut beaucoup de persvrance pour mener bien une telle uvre et
que les encouragements de ma famille ont t essentiels, je veux dire, en termi
nant, que je ddie cet ouvrage la mmoire de mes chers parents, mes filles
Marie-Christine et Florence et Claude, ma femme.

Toulouse, le 20 juillet 1995

Table des matires


Chapitre
1.1
1.2
1.3
1.4
1.5
1.6

1. Les micro-ondes et leurs applications


Dfinition et classification
Historique et applications
Les lignes pour la transmission et les circuits
Les systmes terrestres de transmission en espace libre
Les systmes spatiaux de transmission en espace libre
Attribution des bandes de frquences micro-ondes

1re partie
Lignes pour la transmission et les circuits

1
1
3
6
13
17
22

25

Chapitre 2. Propagation sur une ligne en haute frquence


27
2.1
Modlisation de la ligne
27
2.2
quation de propagation
29
2.3
tude des solutions de lquation de propagation
32
2.4
Expression de la tension du courant et de limpdance
34
2.5
Cas particulier : ligne termine par
Ondes progressives 35
2.6
Cas particulier : ligne en court-circuit ou en circuit ouvert
37
2 7 Cas particulier : ligne quart donde - ligne demi-onde
38
2.8 La vitesse de groupe
39
E x e r c ic e s

Chapitre 3. tude de la rflexion lextrmit d une ligne


3.1
quations correspondant aux nouvelles hypothses
3.2
Coefficient de rflexion
3.3
Cas particulier o TR 1 Rgime dondes stationnaires
3 4 Cas gnral des lignes coefficient de rflexion quelconque
3 5 Mesures de tensions sur une ligne
3.6
Mesure de limpdance de charge dune ligne
E x e r c ic e s

Chapitre
4.1
4.2
4.3
4.4

4. Diagramme de Smith
Intrt
Principe et construction du diagramme
Proprits du diagramme
Dtermination de limpdance de charge dune ligne

E x e r c ic e s

44

47
47
49
49
54
58
60
62
65
65
66
69
73
75

Chapitre 5. Les dispositifs dadaptation


5.1
Le problme de ladaptation
5.2
Conditions dadaptation
5.3
Adaptation par ligne quan donde
5.4
Adaptation laide dun stub
5.5
Adaptation laide de deux stubs
5.6
Utilisation du diagramme de Smith
5.7
Adaptation par rseau dimpdances et tronon de ligne
E xercices

Chapitre 6. Lignes avec pertes


6.1 tude du paramtre de propagation et de limpdance caractristique
6.2
Importance de la condition dHeaviside - Moyens de la raliser
6.3
Expressions de la tension, du courant et de limpdance
6.4
tude des variations de la tension et du courant
6.5
Variations de limpdance et du coefficient derflexion
6.6
Puissance transporte par une ligne
E xercices

77
77
78
79
81
83
84
88
90

93
93
95

97
98
101
103
106

Chapitre 7. Lignes en rgime impulsionnel


7.1
Introduction
7.2
tude en rgime dimpulsion de tension
7.3
tude en rgime dchelon de tension

107
107
108
113

Chapitre 8. Lignes bifilaires et coaxiales


8.1
Paramtres primaires
8.2
Paramtres secondaires de la ligne coaxiale
8.3
Dimension optimale dune ligne coaxiale
8.4
Puissance transportable par une ligne coaxiale
8.5
Paramtres secondaires de la ligne bifilaire
8.6
Paramtres secondaires des lignes utilises en basse frquence
8.7
Exemples de lignes utilises en tlcommunications
8.8
Abaque dimpdances caractristiques

121
121
123
125
126
127
129
130
133
135

E xercices

Chapitre 9. Les lignes bandes et fentes


9.1
Les principaux types de lignes
9.2
Permittivit effective des lignes microbande - Longueur donde
et vitesse de propagation
9.3
Impdance caractristique des lignes microbande
9.4
Affaiblissement dune ligne microbande
9.5
La ligne triplaque
9.6
La ligne fente
E xercices

137
137
139
141
145
147
150
153

Ta b l e des m atires
Chapitre 10. Ralisation des impdances et des circuits rsonnants
10.1 quivalence entre un tronon de ligne et une inductance ou un
condensateur
10.2 Ralisation dinductances et condensateurs
10.3 Ralisation de circuits rsonnants
10.4 quivalence entre une ligne X/4 ou "k/2 et un circuit rsonnant
10.5 Ralisation des impdances par des lments constantes
localises
E xercices

2e partie
Guides d'ondes et cavits
Chapitre 11. Rflexion et rfraction des ondes lectromagntiques
11.1 Introduction
11.2 Rflexion sur un plan conducteur sous incidence normale
11.3 Rflexion sur un plan conducteur sous incidence oblique
11.4 Rflexion et transmission linterface de deux dilectriques
E xercices

Chapitre 12. Les guides dondesrectangulaires


12.1 Les divers types de guide dtmdes tudis
12.2 Propagation entre deux plans parallles
12.3 Propagation dans un guide dondes rectangulaire
12.4 Longueurs donde de propagation guide et de coupure
12.5 tude du mode fondamental
12.6 tude des modes TEmo ou TEon
12.7 Dimensions et bande passante dun guide dondes rectangulaire
12.8 Attnuation dans les guides dondes rectangulaires
12.9 Guides rectangulaires surdimensionns
12.10 Les guides dondes nervure
12.11 La ligne ailettes
E xercices

Chapitre 13. tude gnrale de la propagation


en guides dondes mtalliques
13.1 Introduction
13.2 quations de propagation des ondes guides
13.3 Les diffrents types dondes de propagation guide
13.4 Les guides dondes rectangulaires
13.5 Les guides dondes circulaires

155
156
158
161
164
166
173

175
177
177
179
183
188
200

203
203
204
207
210
213
216
218
219
222
223
225
228

231
231
232
237
243
248

Xi

13:6
13.7

tude gnrale de la rflexion des ondes guides


Le concept dimpdance, dimpdance donde et
dimpdance rduite

E xercices

Chapitre 14. Les guides dondes dilectriques


14.1 Introduction
(
14.2 Propagation en guides dilectriques structure plane
14.3 Propagation en guides dilectriques structure cylindrique
14.4 Les fibres optiques
14.5 Caractristiques des fibres optiques
14.6 Thorie lectromagntique des guides dilectriques
14.7 Caractristiques de la propagation
E xercices

Chapitre
15.1
15.2
15.3
15.4
15.5
15.6
15.7
15.8

15. Les cavits lectromagntiques


Introduction
tude des conditions de rsonance dune cavit
Principaux types de cavits rsonnantes
Coefficient de surtension dune cavit
Modlisation dune cavit - Impdance dentre
Cavits couples par un accs
Cavits couples par deux accs
Applications des cavits

E xercices

255
260
263
265
265
266
269
271
274
278
282
285

287
287
288
291
297
301
306
310
314
319

Solutions des exercices


Exercices du chapitre 2
Exercices du chapitre 3
Exercices du chapitre 4
Exercices du chapitre 5
Exercices du chapitre 6
Exercices du chapitre 8
Exercices du chapitre 9
Exercices du chapitre 10
Exercices du chapitre 11
Exercices du chapitre 12
Exercices du chapitre 13
Exercices du chapitre 14
Exercices du chapitre 15

321
321
324
327
333
338
340
343
345
347
349
353
357
359

Bibliographie

363

Conventions
de notations
Nous voulons reprsenter par la premire lettre de lalphabet une quantit sca
laire (par exemple une tension ou un courant) ou vectorielle (par exemple un
champ lectrique ou magntique) qui est une fonction de lespace (reprsent
par la variable r) et du temps (reprsent par la variable t).
Nous noterons a(r, t) la valeur instantane complexe de cette quantit. En
rgime sinusodal :
a(r, t) = A(r) e
A(r) est lamplitude complexe dont le module est A et la phase <p :
A(r) = A (r)eiv(r)

En notation relle, la valeur instantane relle est :


a(r, t) = 91 [a(/\ 0] = A(r) cos [m + cp(r)]
Nous utiliserons, en gnral, la notation complexe qui a le grand avantage de
permettre la mise en facteur du terme e imt reprsentant la variation temporelle,
ce qui nest pas possible en notation relle. Ainsi, il est possible de conduire tous
les calculs intermdiaires en amplitudes complexes, ce qui permet de bien sim
plifier les calculs.
Pour la notation des logarithmes, les conventions sont les suivantes :
- logarithme nprien ln,
- logarithme dcimal log.

Chapitre 1
Les m icro-ondes
et leurs applications

1.1

D fin itio n et c la s sific a tio n

1.1.1 Dfinition des micro-ondes


Le mot micro-ondes est la traduction littrale de langlais microwaves (wave =
onde). Le terme spcifiquement franais est hyperfrquences, mais lutilisation
de micro-ondes est plus largement rpandue.
Les micro-ondes sont des ondes lectromagntiques dont la frquence est com
prise entre 300 MHz et 300 GHz (MHz = Mgahertz = 106 Hz et GHz = Gigahertz = 109 Hz).
La frquence/dune onde est relie sa longueur donde X0 dans lair ou le vide
par X0 = c /fo c = 3 108 m/s. Cest la vitesse (ou clrit) des ondes lectro
magntiques dans lair ou le vide.
300 MHz, la longueur donde est X0 = 1 m,
300 GHz, la longueur donde est X0 = 1 mm.
Les micro-ondes sont donc des ondes :
dcimtriques entre 300 MHz (X0 = 1 0 dm) et 3 GHz (X0 = 1 dm),
centimtriques entre 3 GHz (X0 = 10 cm) et 30 GHz (X0 = 1 cm),
millimtriques entre 30 GHz (k0 = 10 mm) et 300 GHz (.0 = 1 mm).

videmment, les frontires infrieure et suprieure ne sont pas brutales et lon


pourra tre amen sintresser des dispositifs, des lignes ou des types de pro
pagation qui concernent :
les ondes sub-millimtriques jusque vers 1 000 GHz (X0 = 0,3 mm),
les ondes mtriques jusque vers 100 MHz (X0 = 3 m).

1.1.2 Situation des micro-ondes


dans le spectre lectromagntique
Dans le spectre des ondes lectromagntiques, on trouve successivement, en par
tant des ondes les plus longues :
Les ondes radio-lectriques de Aq = 10 km pour/ = 30 kHz (kHz = kilohertz =
103 Hz) X0 = 1 m pour/ = 300 MHz.
Les micro-ondes de X0 = 1 m pour/ = 300 MHz X0 = 1 mm pour/ = 300 GHz.
Les ondes infra-rouges de Xq = 1 mm pour / = 300 GHz Xq = 1 pm pour
/ = 300 THz (THz = Terahertz = 1012 Hz).
Les ondes visibles de Xq = 0,9 pm X0 = 0,5 pm ( 1 pm = 1 micron = 10 6 m).
Les ondes ultraviolettes de X0 = 0,5 pm p o u r/= 600 THz X0 = 10 nm (1 nm =
1 nanomtre = 10 9 m) pour/ = 30 1015 Hz.
Les rayons X et les rayons gamma pour des frquences suprieures 3 1016 Hz
soit pour des longueurs donde infrieures 10 8 m.
Les micro-ondes sont donc situes plutt dans la moiti infrieure, si lon rai
sonne en frquences, du spectre des ondes lectromagntiques tandis que les
ondes visibles de loptique sont situes plutt dans la moiti suprieure.

1.1.3 Classification des micro-ondes


en bandes de frquences
ou en gammes de longueurs d'ondes
On distingue successivement :
Les ultra hautes frquences (en anglais : ultra high frequencies) ou UHF, de
300'MHz 3 000 MHz. Elles correspondent aux ondes dcimtriques (de
X0 = 10 dm Xq = 1 dm).
Les supra hautes frquences (en anglais : supra high Jrequencies) ou SHF, de
3 GHz 30 GHz. Elles correspondent aux ondes centimtriques (de X0 = 10 cm
X0 = 1 cm).
Les extra hautes frquences (en anglais : extra high frequencies) ou EHF,
de 30 GHz 300 GHz. Elles correspondent aux ondes millimtriques (de
Xq = 10 mm Xq = 1 mm).

Pour la partie des micro-ondes situe entre 1 GHz et 100 GHz, les utilisateurs
ont classifi un certain nombre de sous-bandes qui sont indiques dans le tableau
ci-dessous, avec les frquences et les longueurs donde correspondantes.
Bande

Bande d e /

Gamme de X#

L
S

1 2 GHz
2 4 GHz
4 8 GHz
8 12 GHz
12 18 GHz
18 27 GHz
27 40 GHz
40 60 GHz
60 80 GHz
80 100 GHz

30 15 cm
15 7,5 cm
7,5 3,75 cm
3,75 2,5 cm
2,5 1,67 cm
1,67 1,11 cm
1,11 0,75 cm
7,5 5 mm

c
X
Ku
K
Ka
U
V
w

1.2

5 3,75 mm
3,75 3 mm

H is t o r iq u e et a p p lic a tio n s

Le thoricien fondateur de llectromagntisme moderne et, par consquent, des


bases thoriques des micro-ondes est James Clerck Maxwell qui formula, dans
les annes 1860, les clbres quations (vol. 2, chapitre 5) qui portent son nom
et quil publia en 1873 dans son Trait sur l lectricit et le magntisme.
Une vingtaine dannes plus tard, en 1888, Heinrich Hertz fut le premier pro
duire exprimentalement et dtecter des ondes lectromagntiques une fr
quence de lordre de 1 GHz. Cest pourquoi, on appelle souvent ondes hert
ziennes, les ondes dcimtriques. Lord Rayleigh, pour sa part, dmontra thori
quement en 1897, la possibilit de faire propager des ondes dans des tuyaux
mtalliques creux section rectangulaire ou circulaire, que lon appelle guides
dondes.
la suite des travaux de Hertz, la radiolectricit connut un dveloppement trs
important. Les expriences de Marconi, dans les annes 1890, montrrent quil
tait possible dtablir une liaison entre deux points de la Terre par propagation
dondes radiolectriques en espace libre. Kennelly et Heaviside, au dbut du
xxe sicle, dcouvrirent les proprits rflchissantes, vis--vis dondes dcamtriques, de certaines couches de lionosphre vers 100 km de hauteur. Les radio
ou tlcommunications modernes (la TSF, tlgraphie sans fil , comme lon
disait alors) taient nes [1 ].

Les ondes radiolectriques ncessaires pour ces liaisons de tlcommunications


taient produites par des tubes lectroniques invents en 1907 par Lee de Forest.
Pendant 50 ans, jusqu lavnement des transistors et des dispositifs ltat
solide, ces tubes - triodes et ttrodes - furent universellement utiliss.
Des techniques radiolectriques nouvelles virent ensuite le jour. En 1920, les
premires missions de radiodiffusion eurent lieu, notamment en France, depuis
un metteur situ la Tour Eiffel, sous limpulsion du gnral Ferri qui trans
posa ainsi dans le domaine civil, les progrs effectus par llectronique dans le
domaine militaire pendant la Premire Guerre mondiale.
Dans les annes 1930, la mise au point du premier tube micro-onde, le magntron, et de la premire antenne micro-onde, le rflecteur parabolodal, permirent
le dveloppement dun systme spcifiquement micro-onde : le radar (pour
Radio Dtection and Ranging). Des recherches importantes furent alors effec
tues dans les grands pays industrialiss. Les quipes franaises obtinrent des
rsultats importants, concrtiss par limplantation de radars bord des navires,
notamment en 1935, bord du paquebot Normandie. Les radars anglais, gale
ment trs performants, permettaient, ds 1939, une surveillance efficace de les
pace arien britannique. Lapport des tats-Unis pendant la Seconde Guerre
mcrndiale fut considrable et les recherches du Massachusetts Institute of
Technology (le clbre MIT) furent publies entre 1945 et 1950, en une collec
tion [2] de 25 volumes qui servirent de bible aux tudiants et chercheurs en
micro-ondes du monde entier. Certains de ces volumes ont une telle valeur de
rfrence quils ont t nouveau publis ces dernires annes !
Ds lors, le dveloppement des micro-ondes fut considrable et les applications
nombreuses et importantes :
Chauffage industriel et domestique (fours micro-ondes) [3]
Grce lutilisation de magntrons comme tubes de puissance et la pro
prit des micro-ondes, de pntrer au cur de matriaux de type dilec
triques pertes, il est possible dy dissiper de lnergie. Ceci permet un
chauffage plus rapide et plus homogne que par les mthodes tradition
nelles. La frquence utilise est de 2 450 MIL .
Mdecine
Lhyperthermie micro-ondes est tudie pour le traitement de tumeurs can
creuses. Le problme tant de n'irradier une temprature prcise, que la
zone occupe par la tumeur. Inversement, une exposition prolonge (plu
sieurs heures) aux micro-ondes peut cire dangereuse, de mme dailleurs qu
nimporte quel type de rayonnement lectromagntique (celui du soleil, par
exemple). Le seuil ne pas dpasser est de 1 m W /cnr soit 10 W /nr.
Radioastronomie [4]
On a dcouvert que le rayonnement des toiles et des galaxies est trs riche
dans le domaine des micro-ondes et que lon peut en retirer des informa-

tions complmentaires de celles recueillies dans ie domaine du visible.


C est ainsi quon t mis au point des radiotlescopes munis dune grande
antenne parabolique, de 100 m de diamtre ou plus (vol. 2, chap. 13 Les
antennes rflecteurs ), et d amplificateurs trs faible bruit (Masers),
refroidis si ncessaire, la temprature de lhlium liquide (4 K). La radio
astronomie a permis de faire de grands progrs dans la connaissance de
lunivers, notamment par lobservation plusieurs centaines de millions,
voire plusieurs milliards dannes-lumire, de galaxies ou de sources nou
velles telles que les pulsars et les quasars.
lectronique
On sait maintenant raliser des circuits et des dispositifs trs performants
qui accomplissent en micro-ondes les grandes fonctions de llectronique
classique : oscillation, am plification, mlange et m ultiplication de fr
quence [5], Cela a t possible grce la conception et llaboration de
composants actifs tels que diodes et transistors spcifiques, et de compo
sants passifs appropris (vol. 1, chap. 10 et vol. 2, chap. 1). La miniaturisa
tion de ces circuits, grce lutilisation dune technologie d intgration
hybride ou monolithique, les rend particulirement aptes tre utiliss dans
les techniques spatiales.
- Radiomtrie micro-onde [6]
Cela consiste mesurer laide dun rcepteur trs sensible (radiomtre
puissance totale ou radiomtre de Dicke), la puissance mise en micro
ondes par une zone couverte par le diagramme d une antenne relie au
radiomtre. Ces radiomtres peuvent tre aroports ou sur satellite et sont
utiliss en tldtection pour lvaluation de caractristiques physiques
(humidit, par exemple) ou naturelles (ressources agricoles, par exemple)
de la zone observe. Lavantage de la radiomtrie micro-onde est quelle
reste oprationnelle, aussi bien la nuit que le jour et mme en prsence
dune couverture nuageuse et (ou) de prcipitations.

O- Radionavigation
Outre le radar [7], plusieurs systmes [8] permettent dassurer le reprage
et le guidage des avions : le V.O.R. ( VHF Omnidirectionnal Range) pour
indiquer la direction, le D.M.E. (Distance Measurement Ecjuipment) et le
T.A.C.A.N. (militaire) pour indiquer la distance ainsi que lI.L.S. (Instru
ment Landing System) et bientt le M.L.S. (Microwave Landing System)
pour latterrissage automatique.
Acclrateurs de particules
Ce sont des klystfons (tubes spcifiquement micro-ondes) de trs fortes
puissances (100 kW en rgime permanent) qui fournissent aux particules
lnergie ncessaire
pour les acclrer des vitesses relativistes (vitesses
o
proches de 3 - 10 m/s). De mme, dans les dispositifs tudis pour la fusion
thermonuclaire contrle, on compte sur des superklystrons pour produire,

dans un espace confin, la temprature de plusieurs dizaines de millions de


degrs, qui est ncessaire pour produire la fusion des atomes dhydrogne.
Recherche physique
Les micro-ondes sont trs utilises aussi bien en speclroscopie que pour le
traitement (par polymrisation) et la caractrisation (mesure de la permitti
vit dilectrique) des matriaux, notamment les matriaux composites. Des
mesures laide de guides dondes (chap. 12 et 13) ou cavits rsonnantes
(chap. 15) sont particulirement performantes.
Citons, enfin, le projet trs ambitieux de Satellite de puissance solaire
(S.P.S., Solar Power Satellite) [9] qui a pour but de placer en orbite une
gigantesque station spatiale (20 000 tonnes) transformant lnergie solaire
en micro-ondes et la rayonnant vers la Terre grce un faisceau produit par
une antenne de plus dun kilomtre de diamtre.
Ce bref survol des principales applications des micro-ondes a laiss volontaire
ment de ct des sujets que nous avons choisi de dvelopper maintenant en
raison de leur importance et de leur troite relation avec des thmes traits dans
plusieurs chapitres de cet ouvrage ; il sagit des lignes utilises pour les tlcom
munications et les circuits ( 1.3) et des transmissions en espace libre ( 1.4).

1.3

L e s l ig n e s p o u r l a t r a n s m is s io n
ET LES CIRCUITS

Certaines de ces lignes servent pour les tlcommunications terrestres : cest le


cas des lignes bifilaires, coaxiales et des fibres optiques ; dautres, telles que les
lignes bandes sont trs utilises pour raliser les composants passifs et le sub
strat des circuits lectroniques micro-ondes ; enfin, les guides dondes mtal
liques trouvent toujours une place essentielle dans les techniques de mesure en
laboratoire et dans la ralisation des circuits passifs. Nous tudierons ces
diverses lignes et les guides dans le premier volume de cet ouvrage.

1.3.1 Les principaux types de lignes et de guides


a) La ligne bifilaire

Figure 1.
Ligne bifilaire.

La ligne bifilaire est historiquement le premier type de ligne


qui a t utilis pour les liaisons tlgraphiques et tlpho
niques. Ses deux conducteurs (fig. I ) taient maintenus dis
tance constante au moyen de supports isolants rgulirement
espacs. Avec deux conducteurs espacs de 20 cm environ et

lair comme dilectrique, laffaiblissement de ces lignes tait trs faible et per
mettait des liaisons de plusieurs dizaines de kilomtres sans amplification. Mais
on ne pouvait transmettre avec une ligne quune seule communication tlpho
nique do une infrastructure trs lourde ds quil fallait transmettre plusieurs
dizaines de communications.
b) La ligne coaxiale
Cette ligne (fig. 2) est plus labore que la prcdente car
lenveloppe extrieure qui sert de blindage pour le conduc
teur central joue aussi le rle de conducteur de retour, en
gnral mis la terre. Ce conducteur est maintenu exacte
ment dans laxe du conducteur extrieur par un manchon
cylindrique de dilectrique.

Figure 2.
Ligne coaxiale.

Les lignes coaxiales prsentent des bandes passantes importantes, ce qui permet
dacheminer simultanment plusieurs centaines de communications tlpho
niques pralablement chelonnes en frquences, grce au procd de multi
plexage.
Laffaiblissement prsent par cette ligne dpend de la qualit du dilectrique
employ. Lamlioration des cbles coaxiaux a t rendue possible par llabora
tion dexcellents dilectriques dont le tableau ci-aprs donne les principales
caractristiques.
Nature
du dilectrique

Constante dilectrique
20 C

Facteur de pertes
20 C

Polythylne

2,26
de 1 3 000 MHz

0,0002
1 MHz
0,0005 3 000 MHz

Chlorure
de polyvinyle 100 %

3,2 2,8
de 60 3 000 MHz

0,008
0,006

Polystyrne

2,56
de 60 3 000 MHz

0,0001 100 MHz


0,003 3 000 MHz

Polyttrafluorthylne
(Tflon)

2,1
de 60 3 000 MHz

0,002 100 MHz


0,00015 3 000 MHz

100 MHz
3 000 MHz

c) Les lignes bandes et fentes (strip and slot Unes)


Les principaux types de lignes sont :
La microbande (en anglais, microstrip) qui comporte un substrat en dilec
trique compltement mtallis sur lune de ses faces, et recouvert dune bande
mtallique sur lautre face (fig. 3a).

La ligne fente (en anglais, slot line) o les deux conducteurs formant la
ligne, sont dposs sur la mme face du substrat dilectrique (fig. 3b).
La ligne coplanaire (en anglais, coplanar waveguide) qui prsente trois bandes
mtalliques spares par deux fentes dun mme ct du substrat (fig. 3c).

Figure 3.
a. Ligne microbande,
b. Ligne fente,
c. Ligne coplanaire.

Ces lignes qui peuvent tre fabriques en mettant en uvre la technique trs pr
cise et bon march des circuits imprims, se prtent particulirement bien la
ralisation et la miniaturisation des circuits actifs micro-onde* pour les faibles
puissances.
d) Les guides d'ondes mtalliques
Ce sont des tuyaux mtalliques, de section rectangulaire ou circulaire remplis,
en gnral, par de lair la pression normale (fig. 4).

Figure 4.
Guides dondes
mtalliques section
rectangulaire et
circulaire.

Comme leurs dimensions transversales sont de lordre de la longueur donde, ils


ne sont utiliss quen micro-ondes entre 3 GHz et 90 GHz.
Leurs pertes sont trs faibles car :
ils utilisent comme dilectrique lair, dont les proprits isolantes sont remar
quables,
les conducteurs dont la section est relativement grande, causent de trs faibles
pertes par effet Joule, dautant quil est possible de les dorer ou de les argenter
intrieurement.
Leur construction est relativement facile et ils offrent aux ondes quils transmet
tent un blindage total.
e) Les guides d'ondes dilectriques
La partie centrale de ces guides, appele cur, est un dilectrique compltement
entour par un autre dilectrique, appel gaine, dont la permittivit dilectrique

est lgrement plus petite. La structure transversale est le plus souvent sym
trie de rvolution (fig. 5).
L a p ro p a g a tio n des o n d es s e ffectu e par
rflexions successives linterface des deux
dilectriques, de la mme faon que dans les
guides mtalliques o il y a rflexion linter
face dilectrique-mtal.
C est aux frquences optiques que lon trouve
des d i le c triq u e s tels que la silice et ses
drivs, prsentant des pertes trs faibles, inf
rieures au dcibel par kilomtre. D o le nom
de fibres optiques donn aux guides utiliss
ces frquences.

Figure 5.
Guide donde
dilectrique.

Comme le diamtre du cur va de quelques dizaines de microns dans les fibres


dites multimodes, quelques microns dans les fibres monomodes, on conoit la
difficult de leur ralisation et de leurs raccordements. En contrepartie, les
dilectriques quelles utilisent sont trs abondants et bon march.

1.3.2 Les cbles utiliss


en tlcommunications
a) Les cbles tlphoniques lignes bifilaires
La u g m e n ta tio n
trs
rapide du nombre de liai
sons t l p h o n iq u e s a
dterm in la ralisatio n
de cbles regroupant des
centaines de lignes bifi
Figure 6.
laires, appeles paires
Cble 2 x 91 paires
par les te ch n ic ie n s des
coaxiales,
t l c o m m u n ic a tio n s .
daprs [10, page 12].
C e s t a in si q u o n t t
fa b riq u s des c b le s
urbains et inter-urbains
182 p aires (fig. 6) et
1 792 paires. Ces paires
A enveloppe de plomb
C feuillards de fer
sont groupes par deux,
B jute goudronn
c o n s titu a n t ain si une
quarte . La ligne bifilaire lmentaire est constitue de conducteurs en fil de
cuivre, de diamtre compris entre 0,5 et 2 mm, isols par du papier sec ou du
polythylne. Ces cbles peuvent tre enterrs ou supports par les poteaux tl
phoniques existants lorsquils ne sont pas trop gros.

b) Les cbles tlphoniques lignes coaxiales


Pour transmettre les bandes de frquences de plusieurs MHz labores par le
procd de multiplexage, il faut une ligne dont laffaiblissement par km suit
relativement faible et ne varie pas trop avec la frquence dans la bande consi
dre. Seule la ligne (ou paire ) coaxiale rpond ces exigences.
Trois types de cbles circuits coaxiaux sont utiliss en Fiance :
le cble 2,6/9,5 (fig. 7) qui comprend 4 paires coaxiales ayant chacune un
conducteur intrieur de 2,6 mm et un conducteur extrieur de 9,5 mm. Sa
bande passante maximale est de 12 MHz (soit 2 700 voies) ;
le cble 1,2/4,4 (fig. 8) qui comprend de 4 28 paires coaxiales et qui a une
bande passante maximale de 12 MHz (soit 2 700 voies) ;
le cble 3,7/13,5 qui comprend de 4 10 paires coaxiales et qui a une bande
passante maximale de 60 MHz (soit 10 800 voies).
Le pas damplification est de 4,5 km pour le premier cble et de 2 km pour les
deux autres.
Figure 7.
Cble interurbain
4 paires coaxiales,
12 quartes en toile
et une paire centrale,
daprs [10, p. 12].

Figure 8,
Cble 28 paires
coaxiales rparties
en 7 torons
de 4 paires chacun,
daprs [11, p. 139].

c) Les cbles tlphoniques fibres optiques


Les fibres optiques sont de plus en plus utilises pour les tlcommunications.
Parmi les diverses techniques utilises pour llaboration de cbles runissant
plusieurs dizaines de fibres, nous dcrirons la technique dite jonc cylindrique
rainur .
Prenons lexemple dun cble 70 fibres (fig. 9). Ce cble rassemble 7 joncs en
plastique (fig. 9b) qui servent de support. Chaque jonc (fig. 9a) supporte
10 fibres qui sont dposes dans des rainures graves la surface du jonc.
Lensemble des joncs est protg par une enveloppe en aluminium et une
gaine plastique de 22 mm de diamtre. Le raccordement de deux tronons de
cble pose de dlicats problmes de connectique qui sont aujourdhui bien
rsolus.

Figure 9.
Cble fibres
optiques,
a. Cble 10 fibres,
b. Cble 70 fibres,
daprs /I2, p. I 024].

d) Les cbles sous-marins


Depuis les annes 1960, les liaisons intercontinentales de tlcommunications
par cbles sous-marins ont concurrenc celles qui seffectuent par satellite artifi
ciel. Les deux systmes se sont dvelopps en prsentant des performances tech
niques et des cots d exploitation peu prs comparables. Jusquen 1986, les
cbles sous-marins utilisaient la technologie coaxiale, en modulation analogique.
Le cble pos le plus performant ft le TAT 6 (TransAtlantique n 6) qui permet
tait de transmettre 4 000 voies tlphoniques avec des rpteurs bidirectionnels
espacs de 9,5 km.
Depuis 1986, les cbles sous-marins poss utilisent la technologie fibre optique
monomode, en modulation numrique, qui offre une qualit de transmission
et un nombre de voies bien suprieurs ceux des cbles coaxiaux. En 1988 a
t pos le cble TAT 8 constitu de deux paires de fibres par liaison, avec des
rpteurs espacs de 60 km (affaiblissement de 0,35 dB/km). Chaque paire de
fibres permet un dbit numrique de 280 Mbit/s, correspondant la transmission
de 23 000 com m unications tlphoniques (ou lquivalent, en transm ission
de donnes et en canaux de tlvision). Le TAT 9, pos en 1991, offre un dbit
de 560 Mbit/s par paire de fibres avec des rpteurs espacs de 120 km (affai
blissement de 0,22 dB/km). La dure de vie prvue de ces systmes est de
25 ans.
Les nouveaux systmes TAT 12 (1995) et TAT 13 (1996) utiliseront deux cbles
comportant chacun deux paires de fibres optiques dopes l erbium et des
amplificateurs optiques distants de 45 km. Chaque paire de fibres pourra trans
mettre un dbit de 2,5 Gbit/s. Les performances auront donc t multiplies par
un facteur de 10 en 10 ans, ce qui est remarquable.
La mise en place de ces cbles pose de dlicats problmes. D une part, ils sont
soumis des tractions trs importantes lors de la pose, do la mise au point de
structures spcifiques (cble minitube en acier avec armature double - fig. 10).
D autre part, afin de les protger des chaluts trans par les bateaux de pche
jusqu de trs grandes profondeurs (plus de 1 000 m), ils sont enfouis au fond
dune tranche par une charrue sous-marine tlcommande depuis le navire
cblier.

structure fibres avec conducteur


composite (diamtre extrieur : 8 mm)
gaine isolante
(diamtre extrieur : 14 mm)
fils dacier galvanis

couches de filins de polypropytne

couches de bitume agglomr


diamtre extrieur du cble 36,8 mm

Figure 10.
a. Coupe transversale
dun cble minitube
en acier avec armure
double,
b. Partie centrale
(sans armure double)
du cble,
daprs [13, p. 8).

gel thixotropique
fibres optiques

tube dacier
(paisseur : 0,2 mm
diamtre extrieur : 2,3 mm)

cbles toronns

conducteur composite
(diamtre extrieur : 8 mm)

U
gaine isolante
(diamtre extrieur : 14 mm)

b)

1 -4

L es s y s t m e s t e r r e s t r e s
DE TRANSMISSION EN ESPACE LIBRE

1.4.1 Les tlcommunications


par faisceau hertzien [ 14]
a) Structure d'une liaison
Puisque le faisceau hertzien utilise la propagation des ondes, une liaison doit
comporter dans chaque sens de transmission (fig. 11):
un metteur E et un rcepteur R,
un modulateur M et un dmodulateur D,
des antennes.
S

Figure II.

Schma structurel
dune liaison
hertzienne.

Le modulateur modifie les caractristiques dune onde lectromagntique pour


lui faire porter linformation transmettre. Le dmodulateur effectue lopration
inverse : il doit fournir un signal S aussi semblable que possible celui qui a t
appliqu au modulateur.
Lmetteur produit une onde de puissance et de frquence convenables pour
quelle puisse transporter linformation travers latmosphre. Le rcepteur
labore, partir de londe quil reoit, un signal utilisable par le dmodulateur.
Les antennes sont des dispositifs de couplage entre des lignes de transmission
relies lmetteur ou au rcepteur et lespace libre o se propage londe lec
tromagntique.
Si les deux points relier sont suffisamment rapprochs (courbure de la Terre
ngligeable) et dgags pour que les antennes soient en visibilit directe, la
liaison peut tre tablie en un seul bond.
En revanche, si la distance entre les deux points est trop grande, la liaison doit
tre tablie en plusieurs bonds en utilisant des stations relais (fig. 12) qui ampli
fient les ondes reues avant de les rmettre.
Mais dans le cas de rgions montagneuses o un obstacle sinterpose entre deux
stations rapproches, il est possible dutiliser un relais passif constitu par un
rflecteur plan, en visibilit directe de chacune des stations (fig. 13).

Figure 12.
Liaison hertzienne
en plusieurs bonds
avec relais.

Figure 13.
Liaison hertzienne
avec un relais passif.

b) Modulations utilises
Pour le faisceau hertzien, P affaiblissement de londe porteuse en cours de propa
gation est sujet des fluctuations. D autre part, certains lments de la chane
micro-onde ont une rponse non linaire en amplitude. Toutes ces causes de dis
torsions d amplitude de londe porteuse font que la modulation d amplitude
nest pas souhaitable pour les liaisons hertziennes.
La modulation angulaire, en revanche, est trs peu sensible aux conditions de
propagation et aux rponses non linaires en amplitude des quipements. Elle
offre en outre, une bonne protection contre le bruit et la plupart des brouillages
qui naffectent que lamplitude de la porteuse.
C est la raison pour laquelle la trs grande majorit des faisceaux hertziens ana
logiques utilise la modulation angulaire. Ils sont employs pour transmettre :
des multiplex analogiques de tlphonie dont la capacit peut aller jusqu
2 700 voies tlphoniques,
du tlex ou des transmissions de donnes moyenne et grande vitesse,
les signaux vido de la tlvision.
En modulation numrique, les faisceaux hertziens prsentent, dans des conditions
normales de propagation, une qualit presque parfaite puisque lon sait qu rap
port signal/bruit identique les modulations numriques permettent une meilleure
rgnration du signal que les modulations analogiques. Mais si les conditions de
propagation viennent se dgrader, la rgnration du signal nest plus possible,
en modulation numrique, ds que le rapport signal/bruit tombe en dessous dun
certain seuil alors que la rception est encore possible en modulation analogique.

Les faisceaux hertziens numriques servent transmettre :


des m ultiplex num riques de tlphonie dont le dbit peut aller jusqu
140 Mbit/s,
des donnes numriques grandes vitesses,
du visiophone et de la tlvision code.

1.4.2 Les radiocommunications


avec les mobiles
a) Description du systme
Les systmes de radiocommunication avec les mobiles sont en plein dveloppe
ment [15]. Ils permettent un abonn dappeler ou dtre appel au tlphone.
C et appel s effectue par l interm diaire d une station de base qui est un
metteur-rcepteur muni d une antenne assurant la couverture d une zone ou
cellule. Une rgion gographique relativement tendue peut ainsi tre couverte
par un rseau multicellulaire de plusieurs stations, o une frquence spcifique
est utilise pour chaque cellule du rseau.
Il existe, par exemple, des rseaux 7 frquences (fig. 14). Cet ensemble de
frquences peut tre rutilis en dehors de la rgion couverte par le rseau multi
cellulaire. videmment, la puissance de chaque metteur doit rester moyenne
(< 10 watts) pour que sa porte ne dborde pas des limites du rseau, tout en res
tant suffisamment efficace lintrieur de la cellule desservie.

Figure 14.
Exemple
de rutilisation
des frquences
dun rseau
multicellulaire
7 frquences.

Lorsque lutilisateur se dplace, sa communication tlphonique est traite par la


station de base du rseau grce laquelle le bilan de la liaison est le plus favo
rable. La taille dune cellule lmentaire desservie par une station de base, est
dun diamtre de 10 km environ. Dans des cas bien prcis - en zone urbaine o
la propagation des ondes est plus difficile et o les mobiles se dplacent plus
lentement il est envisag de raliser des rseaux microcellulaires o le dia
mtre d une cellule lmentaire est de 1 km environ.

Il apparat donc quau prix de limplantation dun nombre suffisant de stations


de base, il est possible dassurer la couverture de rgions tendues. videmment,
ce sont les zones forte densit de population et les axes autoroutiers que les
socits de radiotlphonie quipent en priorit. Les antennes utilises sur les
pylnes des stations de base sont du type diples (vol. 2, chap. 11 Les diples
rayonnants ) et la connaissance de la propagation des ondes (vol. 2, chap. 7
Influence de la Terre et de latmosphre sur la propagation des ondes ) est
essentielle pour faire des prvisions de couverture radiolectrique.
Les stations mobiles sont de trois types :
les stations montes dans les vhicules, avec antenne lextrieur du vhicule,
les stations portables avec antenne amovible, qui peux eut tre portes la
main ou montes dans un vhicule,
les stations portatives avec antenne lie au bloc metteur-rcepteur (masse
500 g, volume 500 cm3, autonomie de 24 heures en veille et de 2 3 heures
en communication).
b) Les divers systmes existants
Au cours des annes 1980, le dveloppement des systmes de radiocommuni
cation analogique sest effectu dune manire assez diverse d un pays lautre.
Les tats-Unis et le Canada ont dvelopp d une manire unifie le systme
AMPS (10 millions d abonns) dans la bande des 800 MHz.
Le Royaume-Uni, lItalie, lEspagne et lAutriche ont adopt le systme
TACS, driv de lAMPS, qui opre dans la bande des 900 MHz.
Le systme NMT 900, galement dans la bande des 900 MHz, sest dvelopp
dabord en Scandinavie et ensuite en Suisse, aux Pays-Bas, en Espagne et en
France.
LAllemagne a adopt le systme C-NETZ et lItalie le systme RTMS, tous
deux dans la bande des 450 MHz.
En France, Matra a dvelopp le rseau Radiocom 2000 dans les bandes
174-223 MHz et 400-430 MHz.
Comme on le voit, lEurope a adopt, sans aucune concertation, des systmes
diffrents. Heureusement, depuis une dizaine dannes (1987), une norme euro
penne a t dfinie : le GSM (Global System fo r Mobile Communications) qui
est un systme numrique oprant entre 890 et 915 MHz pour lmission des
mobiles (voie montante) et entre 935 et 960 MHz pour lmission des stations
fixes (voie descendante). Cette norme se dveloppe en France sous limpulsion
de deux oprateurs : la SFR (Socit Franaise de Radiotlphonie) et France
Tlcom (Rseau Itinris).
Un systme complmentaire, le DCS 1800 est en cours de dfinition en vue
dune utilisation en milieu urbain des frquences voisines de 1 800 MHz et
pour des portes rduites de lordre de quelques kilomtres au maximum.

1-5

L e s s y s t m e s s p a t ia u x
DE TRANSMISSION EN ESPACE LIBRE
[ 16, 17, 18]

1.5.1 Les divers types de liaisons


par satellites artificiels
Ces liaisons qui utilisent les micro-ondes, servent satisfaire des besoins de plus
en plus importants, tels que :
les tlcommunications avec des stations fixes ou mobiles,
la radiodiffusion,
la radiolocalisation et la radionavigation,
les liaisons avec les sondes spatiales,
lobservation de la Terre.
a) Les tlcommunications
Le service fix e de tlcommunications assure des liaisons entre stations fixes
pour la transmission :
de communications tlphoniques,
de messages sous forme de tlcopie, de textes,
de donnes numriques haut dbit pour la tlmatique,
de signaux audio et vido pour les tlconfrences,
de canaux de tlvision pour les changes internationaux dmissions.
Les satellites qui assurent ce genre de liaisons sont principalement les satellites
gostationnaires Intelsat au niveau international, Eutelsat au niveau europen
ainsi que des satellites nationaux comme, par exemple, Telecom en France. Le
tableau de la page suivante donne les principales caractristiques de ces satel
lites.
Le service mobile de tlcommunications assure des liaisons entre deux stations
mobiles ou entre une station fixe et une mobile, par lintermdiaire dun satellite
assurant le relais. Ce service sest dabord dvelopp pour les liaisons maritimes
grce aux satellites gostationnaires Inmarsat ; il sest progressivement tendu
aux liaisons ariennes.
b) La radiodiffusion
La radiodiffusion par satellite permet, depuis un Satellite gostationnaire met
tant une forte puissance (200 250 W), de diffuser des missions de TV ou de
radio FM pouvant tre directement captes par un usager quip dun systme

Caractristiques de quelques satellites


Type

Anne

Masse Voies tel. + TV Puissance


(W)2
(kg)1 ou dbit num.

Dure
de vie

Bandes de F.
(GHz)3

Intelsat 6

1989 4 200 40 000 + 2 TV

5 000

10 ans

6/4 et 14/11

Intelsat 7

1995 3 600 60 000 + 3 TV

3 900

12 ans

6/4 et
14,2/11,1 ; 11,6;
11,8; 12,6

Eutelsat 1 1983

600

2 000 + 8 TV

300

7 ans

14/11 et 14/12

Eutelsat 2 1990

900

8 0 0 0 + 16 TV

800

7-10 ans

14 14,5/11
12,75

r
Telecom 1 1984

700

2 500 + 2 TV

200

7 ans

6/4 et 14/12

Telecom 2 1991 1 500

5 200 + 4 TV

600

10 ans

6/4 et 14/12

TDF 1/2

1988 1 300

5 TV

1 000

8-9 ans 17,3 18,1/11,7


12,5

1. Il sagit de la masse du satellite en orbite.


2. Il sagit de la puissance radiofrquence.
3. Le premier chiffre (le plus lev) correspond la frquence dans le sens montant / le second
chiffre (le plus faible) la frquence dans le sens descendant.

de rception constitu dune antenne parabolodale de 40 50 cm de diamtre et


d un rcepteur hyperfrquence faible bruit. En Europe, plusieurs satellites de
radiodiffusion sont en fonctionnement, notamment TV-S A pour lAllemagne et
TDF pour la France (voir le tableau). Leurs programmes sont diffuss selon la
norme haute dfinition D2 Mac Paquet qui donne une image excellente et un son
haute fidlit. Des satellites de tlcom m unications comme Intelsat VI ou
Eutelsat I diffusent aussi des missions de TV, mais avec une puissance plus
rduite (20 40 W), ce qui ncessite une antenne de rception de 1,50 m de dia
mtre.
c) La radiolocalisation et la radionavigation
La radiolocalisation des mobiles la surface de la Terre est possible grce aux
satellites dfilement placs en orbites basses (850 ou I 000 km) quasipolaires
du systme international de dtresse et de scurit Sarsat (tats-Unis), Cospars
(URSS). Ces satellites captent les signaux mis 121,5 MHz ou 406 MHz
(nouveau systme) par des balises actives manuellement ou automatiquement
en cas de dtresse. Il existe divers types de balises : maritimes (par exemple
ARGOS), aronautiques ou terrestres. La densit actuelle des stations sol et le
nombre de satellites de ce systme permettent dalerter les services de sauvetage
en une heure, en moyenne, et avec une prcision de quelques kilomtres.

Il existe aussi des services commerciaux de radioreprage par satellites autori


sant lchange de brefs messages tels que les systmes Locstar en Europe et
Geostar aux tats-Unis. La prcision de ces systmes (de lordre de 100 m), la
lgret des terminaux bord des mobiles (antenne de 10 cm) sont particulire
ment intressants pour les transports terrestres de marchandises ou de passagers
par la route ou par le rail.
La radionavigation est assure par les satellites GPS (Global Positionning
System). Il sagit dune constellation de 24 satellites lancs entre 1983 et 1993,
qui effectuent un tour de la Terre en 12 heures une altitude de 20 185 km. Ils
sont regroups par 4,. rgulirement rpartis dans 6 plans orbitaux. Chaque satel
lite met deux frquences, lune 1 575 MHz et lautre 1 227 MHz, qui sont
porteuses de signaux cods occupant des bandes de frquence de 20 MHz. En
service payant, les prcisions obtenues sont de 21 m en position horizontale et
de 34 m en position verticale ; elles sont multiplies par cinq en service gratuit.
Les antennes de rception sont miniaturises (Micro-ondes, vol. 2, chap. 15
Les antennes lments imprims ).
d) Les liaisons avec les sondes spatiales
Grce ces liaisons, qui se situent en bande X, il est possible de communiquer
avec des sondes explorant notre systme solaire et qui se trouvent des distances
de centaines de millions voire de milliards de kilomtres. Citons notamment :
les sondes sovitiques Vnra et les sondes amricaines Mariner et Pioneer,
pour lexploration de Vnus ;
les sondes sovitiques Mars et Phobos et les sondes amricaines Mariner et
Viking, pour lexploration de Mars ;
les sondes amricaines Pioneer 10 et 11 et Voyager 1 et 2, pour lexploration
de Jupiter et Saturne ;
la sonde Voyager 2 qui, aprs avoir survol Jupiter le 9 juillet 1979 et Saturne
le 26 aot 1981 survola Uranus le 24 janvier 1986 et Neptune le 24 aot 1989,
transmettant des images dune qualit remarquable et dun intrt inestimable
pour la connaissance du systme solaire ;
la sonde Giotto qui a transmis le 13 mars 1986 les images de sa rencontre avec
la comte Halley.
Actuellement, la sonde spatiale Galile fait route vers Jupiter quelle devrait
atteindre en dcembre 1995. En 1997 sera lance la sonde spatiale Cassini qui
devrait atteindre Saturne et son satellite Titan en 2004.
e) L'observation de la Terre
Ces applications se sont notamment dveloppes depuis le dbut des annes
1980 :
Satellites mtorologiques comme Mtosat (1977-1981 et 1989) suivis par
les MOP lancs en 1989, 1990 et 1993 ;

Satellites dobservation des ocans comme Topex-Posidon (1992; qui permet


de mieux connatre les courants ocaniques, la position des glaces polaires,
ltat de la mer et de mesurer le niveau des surfaces marines avec une prci
sion de 2 4 cm !
Satellites photographiant la surface de la Terre, comme SPOT qui fournit des
images de surfaces de 60 x 60 km2 avec une rsolution de 10 m ou comme le
satellite militaire Hlios (1995) dont la rsolution serait de 1 m ;
Satellites de tldtection passive ( laide de radiointres) ou active ( laide
de radars) comme ERS-1 (1991). Celui-ci permet dtudier lvolution des
zones agricoles, des forts, des calottes polaires. 11 peut mesurer la tempra
ture et le niveau de la surface des ocans, la hauteur des vagues, la vapeur
deau atmosphrique. Comme il est dot dinstruments qui fonctionnent en
micro-ondes (radiomtre, radar) il observe mme la nuit et travers une cou
verture nuageuse, voire des prcipitations.

1.5.2 Organisation des liaisons par satellites


Les satellites artificiels sont les points centraux dun rseau de liaisons qui sef
fectuent en micro-ondes. Les lments essentiels en sont les antennes et les cir
cuits passifs (rpteurs) que lon trouve tant au niveau des stations terriennes
que du satellite lui-mme. Ces antennes et ces circuits passifs seront tudis dans
le second volume de cet ouvrage, ainsi que le bilan de la liaison satellite-Terre.
a) Liaisons de tlcommunications
Le satellite doit permettre la liaison simultane entre plusieurs couples de sta
tions la surface de la Terre (fig. 15). Le rpteur du satellite assure lamplifica
tion et la rgnration des signaux.

Figura 15.
Organisation
dune liaison de
tlcommunications
par satellite.

La capacit dun rpteur (1 000 voies tlphoniques pour une largeur de bande
de 36 MHz sur Intelsat ou Eutelsat) permet dcouler le trafic entre plusieurs

couples de stations, condition dutiliser une technique daccs multiple pour


rpartir :
en frquence, des liaisons se faisant sim ultanm ent (A ccs M ultiple par
Rpartition en Frquence, AMRF ; en anglais : Frequency Division Multiple
Access, FDMA) ;
dans le temps, des liaisons se faisant dans la mme bande de frquences (Accs
Multiple par Rpartition dans le Temps, AMRT ; en anglais : Time Division
Multiple Access, TDMA).
Notons aussi quune combinaison judicieuse de lutilisation des deux types
d accs (A ccs M ultiple par Rpartition Code en temps et en frquence,
AMRC ; en anglais : Code Division Multiple Access, CDMA) permet dutiliser,
au mieux, la capacit des rpteurs du satellite.
Les antennes utilises sont des rflecteurs (vol. 2, chap. 13 Les antennes
rflecteur ) aliments par un rseau de sources primaires. Leur rayonnement
qui est faisceaux multiples ou faisceau conform, permet de sadapter au
mieux au type de couverture qui doit tre assur.
b) Liaisons de radiodiffusion (fig. 16)
Les missions transmettre sont envoyes vers le satellite depuis la station
d m ission T t . D autres signaux de tlm esure et de tlcom m ande sont
envoys par une station de contrle oprationnel.
rpteur

Figure 16.
Organisation
dune liaison
de radiodiffusion
par satellite.

Aprs traitement (amplification et changement de frquence) par le ou les rp


teurs, les missions destines au grand public sont rayonnes vers la Terre par
une antenne qui assure la couverture de la zone desservir. Cette couverture
peut tre obtenue soit par un faisceau dcoupe circulaire ou elliptique,
engendr par une antenne rflecteur aliment par une source primaire unique,

soit par un faisceau conform, en6endr par une antenne rflecteur aliment
par un rseau de plusieurs sources primaires.
Des bandes de frquences ont t attribues aux divers systmes de radiodiffu
sion. Chaque bande de frquence est partage en un certain nombre de canaux,
chaque canal tant rserv un programme de TV ou de radio FM. Par exemple,
la bande 11,7-12,5 GHz a t partage en 20 canaux mis en polarisation circu
laire gauche et 20 autres en polarisation circulaire droite. Les frquences cen
trales de deux canaux mis dans la mme polarisation sont spares de 40 MHz
environ et celles de deux canaux mis en polarisations orthogonales sont spa
res de 20 MHz environ. En Europe, chacun des 8 pays utilisateurs de cette
bande sest vu allouer 5 canaux.
Le satellite dun pays utilisateur (tel que TDF1 pour la France ou TVSAT pour
lAllemagne) va donc mettre ces 5 canaux sur sa zone de couverture. Ce satel
lite a 5 rpteurs qui amplifient chacun 1 canal. Un multiplexeur d mission
permet de rassembler ces canaux, sans les mlanger, pour les envoyer vers lan
tenne dmission.

1.6

A t t r ib u t io n d e s b a n d e s
DE FRQUENCES MICRO-ONDES

partir de 100 MHz (A0 = 3 m)


Les principales attributions lgalises en France sont les suivantes :
la radiodiffusion FM (Bande II) de 87,5 108 MHz ;
la radionavigation pour laviation civile : ILS de 108 112 MHz et VOR de
112 118 M H z;
les radioamateurs de 144 148 MHz ;
la radiodiffusion TV (Bande III) et les radiocommunications avec les mobiles
terrestres de 174 223 MHz ;
la radionavigation pour laviation civile : ILS de 328 335 MHz ;
la radiodiffusion TV (Bandes IV et V) de 470 830 MHz ;
les radiocommunications avec les mobiles terrestres (systme GSM) de 890
960 MHz ;
la radionavigation pour laviation civile et les armes (DME, Tacan, Radar) de
960 1 260 MHz et de 1 260 1 350 MHz ;
la radioastronomie de 1 400 1 427 MHz ;
les liaisons m obiles pour les reportages TV : 1 429-1 452 MHz, 1 4601 484 MHz et 1 492-1 515 MHz ;

la radionavigation (systm e GPS) de 1 217 1237 MHz et de 1 565


1 585 MHz ;
les tlcommunications par satellite avec les mobiles (bateaux ou avions) via
le systme Inmarsat de 1 530 1 544 MHz (liaison descendante) et 1 626
1 645 MHz (liaison montante) ;
les radiocommunications avec les mobiles autour de 1 800 MHz ;
les faisceaux hertziens de 2 100 2 300 MHz ;
le chauffage industriel et domestique (fours micro-ondes) 2 450 MHz ;
la radionavigation pour laronautique de 2 700 2 900 MHz.
Au-del de 3 GHz (k0 = 10 cm)
Les micro-ondes sont surtout utilises pour les tlcommunications par fais
ceaux hertziens et par satellites artificiels ainsi que pour laviation civile.
Les p rin cip ales bandes u tilises po u r les faisceau x h ertzien s sont les
suivantes : 3,8 4,2 GHz ; 4,4 4,99 GHz (armes) ; 5,4 7,1 GHz ; 7,4
7,9 GHz (armes) ; 8,2 8,5 GHz ; 10,7 11,7 GHz ; 12,7 13,2 GHz ; 14,4
15,35 GHz ; 17,7 19,7 GHz.
Les principales bandes utilises pour laviation civile sont : 4,2 4,4 GHz
(systmes daltimtrie); 5 5,1 GHz (MLS, microwave landing System) ; 5,35
5,47 GHz (radars de bord).
Les principales bandes utilises pour les liaisons par satellite (Rgion I) sont
les suivantes :
- de 3,4 4,2 et de 5,725 7,075 GHz pour les tlcommunications civiles,
- de 7,25 7,75 et de 7,9 8,4 GHz pour les tlcommunications militaires,
- de 10,7 11,7 GHz pour les tlcommunications civiles et la radiodiffusion,
- de 11,7 12,7 GHz et de 14 14,8 GHz pour la radiodiffusion,
- de 17,3 19,7 GHz pour les tlcommunications civiles.
Comme on le remarque, certaines de ces bandes sont utilises en partage avec
les faisceaux hertziens, selon des conventions dexploitation trs prcises.
Au-del de 20 GHz (>.0 = 1,5 cm)
Les bandes de frquences ne sont pas encore dfinitivement attribues. Des
recherches sont en cours pour lutilisation de ces frquences, en dehors des raies
dabsorption de la vapeur deau (vers les 22 GHz).

1re partie
LIGNES POUR
LA TRANSMISSION
ET LES CIRCUITS

Chapitre 2

Propagation
sur une lig n e
en h au te fr q u e n c e

2 .H

o d l is a t io n

d e l a l ig n e

2.1.1 Le modle

Figure 1.
Ligne de longueur .
G, gnrateur ;
Z R, impdance de
charge.

Soit une ligne de transmission, de longueur f, alimente une extrmit par un


gnrateur de tensions H.F. et ferme lautre extrmit sur une impdance
(fig. 1). En haute frquence, du fait que la longueur de la ligne est, en gnral,
grande devant la longueur donde X, il en rsulte que la tension et le courant (ou
les champs lectrique et magntique, si les prcdents concepts ne peuvent pas
tre utiliss) varient le long de la ligne.

Figure 2.

H ?

(
L

---- i_*.

J . Flf

L,

x+ u
1_*_

Modlisation
par un quadriple
a. de la ligne
b. dun tronon de
ligne de longueur
unit.

Pour faire ltude de ces phnomnes, il convient dadopter une modlisation de


la ligne. Le modle utilis est un rseau (fig. 2) qui comporte :
en srie, une rsistance R et une inductance L, pour reprsenter respectivement
les pertes dnergie active et ractive dans les conducteurs de la ligne,
en parallle, une conductance G et une capacit C pour reprsenter les pertes
dnergie active et ractive dans le dilectrique de la ligne.
En haute frquence, la difficult du problme vient de ce quil est impossible du moins ds que la ligne nest pas de longueur trs faible devant X - de la
modliser par un seul quadriple (fig. 2a) que lon pourrait considrer comme
ferm une extrmit sur le gnrateur et l autre sur la charge. Il faut
alors faire lapproximation qui revient dcomposer la ligne en une suite dl
ments identiques, llment unit tant pris trs petit devant X pour quon
puisse le reprsenter avec une erreur ngligeable par le quadriple constantes
localises de la figure 2b. ,, L ,, G {, C j reprsentent respectivem ent la
rsistance, linductance, la conductance et la capacit rparties de cet lment
unit.

2.1.2 Domaine de validit


/?], L Gj, C, sont appeles les constantes ou paramtres primaires de la ligne.
Les seules lignes pour lesquelles il sera possible de les calculer sans difficult,
sont les lignes dites T.E.M. o les notions de tension et courant gardent un sens.
Sur ces lignes, en effet, les champs E et H se trouvent dans des plans perpendi
culaires aux conducteurs (do lappellation T.E.M. = Transversal Electric and
Magnetic fields) et il est donc possible den dduire les tension et courant. Les
lignes coaxiales, bifilaires et triplaques en sont de bons exemples (voir chapitres
1,8 et 9).
Par contre, ds que les champs ont une composante longitudinale (cest--dire
dans la direction des conducteurs), la notion de tension na plus de sens et les
constantes primaires ne peuvent plus tre calcules directement. C est le cas
pour les guides dondes mtalliques ou dilectriques ainsi que pour les lignes
microbandes, encore que, dans ce dernier cas, il soit possible de faire des
approximations du type T.E.M.

Par ailleurs, il est trs important de noter, ds maintenant, que ltude qui sera
faite dans ce chapitre et les suivants est spcifique dans ses dtails, mais nan
moins gnrale dans ses conclusions. En effet, les quations que nous crirons,
les relations que nous tablirons en appliquant les concepts tension et courant au
modle dcrit au paragraphe 2.1.1, ne sont valables dans leur forme et dans le
dtail des calculs que pour les lignes de type TEM. Cependant, tout ce qui sera
expos dans ces chapitres concernant la nature et les caractristiques des
rgimes de propagation, le diagramme de Smith, le problme de ladaptation, les
rgimes impulsionnels, est galement valable pour dautres types de lignes
comme les guides dondes.

2.1.3 Intrt de ce modle


Il provient de ce que, propos dune tude particulire aux lignes bifilaires et
coaxiales laide des concepts tension-courant et du modle {Rj, L x, G j, C x),
nous tablirons des rsultats et expliquerons des phnomnes que lon pourrait
trouver pour les guides dondes, par exemple, en utilisant les concepts de champ
lectrique et magntique.
Il faut dailleurs noter que les lignes T.E.M. pourraient aussi tre considres et
tudies comme des guides dondes lectromagntiques, les champs se propa
geant dans le dilectrique o les conducteurs sont plongs. Ce fut le point de vue
des premiers thoriciens de la radio-lectricit (Hertz et Poincar notamment).
Malheureusement, lexpos de cette mthode est assez ardu et peu explicite
tandis que lutilisation des concepts tension et courant permet, tout en restant
correct, de donner un expos simple et significatif.

2 .2

q u a t io n

d e p r o p a g a t io n

Nous tudions une ligne de transmission, de longueur f, alimente, une extr


mit, par un gnrateur de tensions H.F. et ferme, lautre extrmit, sur une
impdance ZR (fg. 1). Pour les applications, lorigine des abscisses sera prise
soit au gnrateur, soit la charge, soit en un point de la ligne o se trouve une
discontinuit tudier. Si lorigine est au gnrateur ( la charge), laxe des abs
cisses est orient vers la charge (le gnrateur) afin quun point de la ligne ait
toujours une abscisse positive. Pour ce chapitre, nous supposerons que laxe est
' orient du gnrateur vers la charge avec origine au gnrateur.
Nous allons traiter le cas gnral dune ligne avec pertes. Plaons-nous en un
point dabscisse x par rapport lorigine et raisonnons sur llment compris
entre x et x + d r (fig. 3). Soient v(jc, t) et i(x, t) les valeurs complexes instanta
nes de la tension et du courant au point dabscisse x.

Figure 3.
Reprsentation
quadripolaire
dun tronon de ligne
de longueur dx.

x+ dx

Grce lutilisation de la nota


tion com plexe, il est possible
dcrire que les valeurs instanta
nes complexes de la tension v(x,
t) et du courant /(x, t) sont le pro
duit des amplitudes complexes
V(x) et /(x), qui ne dpendent
que de la variable spatiale x, par
la fonction F(t) reprsentant les
v ariatio n s dans le tem ps. En
rgime sinusodal : EU) = e Jt/ ; nous avons donc :
Ft,dx

L.dx

v(x, t) = V(x) e jw/

avec

/(x, t) /(x) e j a v e c

V(x) = V(x) e

/(x) = /(jc) e Jlp' ^

V(x) et /(jc) sont les amplitudes relles ou modules.


tpw(jc) et <p(-(jc) sont les phases.
Les valeurs instantanes relles sont :
v(jc,

t) = Ui [v(x, /)] = V(x) cos Lcot + <pv(jc)3

/(jc, t) = 3 i [/(jc, /)] = /(jc) cos [c/ + /x )]


En notation relle, les variables t et x sont lies dans la fonction cosinus et ne
peuvent tre spares. Cest pourquoi il est plus intressant dutiliser la notation
complexe dans laquelle la dpendance temporelle F(t) e ^ peut tre mise en
facteur.
La variation de tension dv quand on se dplace de de sur la ligne est gale la
somme :
de la variation de tension dbc / due la rsistance ;
de la variation de tension - L, de (di/dt) due linductance.
3v
Soit :

di
Ri i L i
dx
1'
1 dt

( 1)

La variation dintensit de courant di quand on se dplace de dx sur la ligne est


gale la somme :
du courant scoulant par dfaut d isolement - Gj dx v ;
du courant scoulant par la capacit - Cj dx (dv/dt),
nb

Le signe - provient
de ce qil s agit de
chutes de tension et
de pertes de courant.

Soit:

di
dv
= - G j V - C 1dx
dt

(2)

partir de (1) et (2), on montre facilement que / et v satisfont les quations,


dites de tlgraphistes.

d2v

d2v
3v
-L, C, ^ (/?i C, + L j G,)
dt2
dt
dx2 ~

d2i
dx2 ~

G, v = 0

d2i
di
L x C, - ^ - ( R x C x + L, G,) - / ? , G, / = 0
dt2
dt

(3)

(4)

Dans le cas o le rgime est sinusodal, (1), (2), (3) et (4) deviennent :
av
-(/?! + j L x co)Z
ar

(5)

a/
T- = (G, + j Gi co) V
dx

(6)

a 2v
dx

Posons :

= (/?, + jL , co) (Gj + j Cj C) V

(7)

a 2/
~ = {Rx + jL , co)(G| + j Cj co)/
dx

(8)

y = ^ /(R , + j L | co)(G, + j C, co)

(9)

Cest une quantit complexe que lon appelle la constante de propagation et dont
nous ferons ltude plus loin. (7) et (8) scrivent donc :
d2V
2
~ ^-= y 2 V
a*2

d2I 2
y = y2 1
r2

(10a)

(10b)

Ce sont les quations de propagation de la tension et du courant le long de la


ligne.
(10a) et (10b) admettent des solutions de la forme :
V(x) = V i c 1x + V r eyx

(H a)

I_(x) = l i e - y x + l r eyx

(11b)

Vj, [j7 Vr, Lr sont des constantes dintgration. Il est vident daprs (5) ou (6)
quelles sont lies deux deux puisque lon a en effet :
Y.i

-/

+ '}L\

\ J

Gj + j Cj CO

quantit qui est homogne une impdance.

MB

Posons :

y et Zc. sont aussi


appels les para
mtres secondaires
de la ligne-

(13)

quantit que lon appelle limpdance caractristique de la ligne et dont nous


verrons plus tard la signification et limportance.

2 -3

tude

d e s s o l u t i o n s d e l ' q u a t io n

DE PROPAGATION
V et / ayant la mme forme, il suffit de faire cette tude uniquement pour V, par
exemple.

2.3.1. Mise en vidence de !a suoerposition


de deux ondes
Nous voyons, daprs (9), que y est de la forme a + JP- Donc :
v(jc,

/) = V(*)eJt0/ = V ,e - CLte-)(w - &*>+

e * 10'-P*>

(14)

Cette expression est la somme de deux termes :


Lun dont lamplitude diminue lorsque x augmente, cest--dire pour un dpla
cement du gnrateur vers le rcepteur ; il caractrise une onde incidente
(indices /).
Lautre dont lamplitude diminue lorsque x diminue, c est--dire pour un
dplacement du rcepteur vers le gnrateur ; il caractrise une onde rflchie
(indice r). Par consquent, la tension sur la ligne rsulte de la superposition de
deux ondes se propageant en sens contraire. Mme chose pour le courant.

2.3.2. Caractristiques de ces ondes


tudions le terme

v,(jc,

t) = Vl e ~ <XKe

~^

(15)

Passons aux valeurs instantanes relles : v,-(x, t) = fi [vf(x, /)J


_V tant complexe, nous crirons : V( = V( e ^
Donc :

v((jc,

/) =

Vj e ~ ax

cos (COt + <f>- Px)

(16)

Il sagit dune onde dont lamplitude diminue exponentiellement mesure que


lon se rapproche du rcepteur.

En un point donn de la ligne, la tension est une fonction sinusodale du temps


dont la priodicit dans le temps est la priode :
(17)
un instant donn, la tension est une fonction sinusodale de labscisse x dont la
priodicit dans lespace est la longueur donde :
(18)
Enfin, cette onde se dplace vers les ,v croissants avec la vitesse constante
v = co. (3, appele vitesse de phase . Il suffit en effet pour trouver la vitesse
de londe, dcrire que co/ + <p - pj. = cte.
Soit encore : co d/ - p tbc = 0.
D'o :

dv _

_ co

(19)

t ~ Vp~ p

Ltude du terme vr(x, t) = Vr e <xv e J(G)' +


dence une onde :

mettrait de la mme faon en vi

dont lamplitude diminue exponentiellement mesure que lon se dplace vers


le gnrateur,
de priodicit dans lespace X 2 t i : P et dans le temps T 2n/co,
qui se dplace en sens inverse avec la mme vitesse co/p.
De telles ondes sont appeles des ondes progressives amorties. Leur superposi
tion donne naissance un phnomne dondes semi-stationnaires, qui est le
rgime gnral de fonctionnement dune ligne. Nous ltudierons en dtail dans
le prochain chapitre pour le cas dune ligne sans perle et dans le chapitre VI
pour le cas dune ligne avec pertes.

2.3.3 Le paramtre de propagation

Le paramtre y apparat comme un paramtre de propagation dont :


la partie relle a est un paramtre daffaiblissement exprim en Nepers par
mtre ou en dcibels par mtre (1 dB = 0 ,1151 Np) ;
la partie imaginaire P est un paramtre de phase exprim en radians par mtre
(1 radian = 57,30).
Dans le cas particulier des lignes sans perte (LSP) R { = G, ~ 0
->ot = 0, p = m v'L , C, et vp =

Les ondes se propagent sans affaiblissement et lon peut montrer que leur vitesse
de propagation v est gale la vitesse de propagation des ondes lectromagn
tiques dans le dileelrique dont est constitue la ligne.
Nous allons maintenant tudier les expressions de la tension, du courant et de
limpdance en un point dune ligne de transmission de longueur finie tout
dabord dans le cas gnral et ensuite dans quelques cas particuliers.

2 .4

E x p r e s s io n s

d e l a t e n s io n ,

DU COURANT ET DE L'iMPDAN
Le quantits connues son! V0, / (), y et Zc. Nous voulons calculer V(.v) et / ( a ) en
fonction de ces quantits dans le cas o la ligne est termine sur une charge
quelconque. Daprs ( 11a) et ( 11b) :
Y0 = Y,- + Yr = Z, ( / , - / , )
l = li + lr=

Y ,- Y r
7
'-c

partir de ces relations, nous obtenons :

= Y + z , / 0

Y ( ) -z t. / (,
Y,- =
f

'o
Y0
2 + 2 Z,.

1 =

lu

Jit)

2 _ 2 Z,

( 20 )

( 21)

( 22 )

(23)

hn portant ces valeurs dans ( 11 a) et ( 11 b), nous obtenons :


Y(-v) = Y0 ch ya - Zc / sh yx
/( a) = /

Y
0 ch y v - sh Y-V
A-

Z(x) = Z(.

Z - zc th yx
---- Zc - Z q th y-V

Dans le cas des lignes sans perte, les relations (24), (25) et (26) scrivent
Y( v) = Yy cos p.v - j Zt. Z0 sin Pa

(24)
(25)

(26)

/(a) = /o cos (il - j

si n pA

A
Z(A) = Z (. Z -

(28)
(29)

J Z 1^

Zt--jZ o lg |iv
Enfin, il peut tre intressant de connatre la valeur de l'impdance dentre Z(,
en fonction de limpdance de charge ZR. Daprs (26) :
V,R + Z,, th yf
(30)
Z0 = Z,

Zt- + Z r th yC
Dans le cas des lignes sans pertes :
ZR +J Z , l P
/(n = Z,.
' Z ( + J Z / ,l g >

2. 5.

(31)

C a s p a r t ic u lie r : lig n e t e r m in e
par

ZR

Zc

O n d e s p r o g r e s s iv e s

Si Z.R = Zc, il vient daprs (30) : Z() = Z(..


11 apparat donc que limpdance d entre de la ligne est Z(.. Tout se passe
comme si le gnrateur tait directement ferm sur Z(.. Daprs (26) nous voyons
que dans ces conditions :
Z (a ) = Z(.
(32)
En tout point de la ligne, limpdance est donc limpdance caractristique.
Comme :

z0+ z(

Y/ = *0
r - 0

-,
Z(, ~ ZC
9

Zo , zn)
Li- 0 1+z

I- r.= !.n(, Z\
2

il en rsulte que :
Y, = Y0 ; Y r = 0 et Z,. = / 0 et / , = 0
do:

YC*) = Y0 e _Y'v

et:

/( a)

= Z(, e _Yl

(33)
(34)
(35)

Lorsquune ligne est termine sur son impdance caractristique, il ny a pas


donde rflchie. Le rgime qui stablit sur cette ligne est appel rgime
dondes progressives.

Passons aux valeurs instantanes complexes :


v{x, t) = V0 e -ve J( ' - P-v)

(36)

iix, t) = / 0 e - a 'e

(37)

Nous retrouvons dans ces expressions les caractristiques dune onde progres
sive, dj signales au paragraphe 2.3.2 Caractristiques de ces ondes :
lamplitude dcrot exponentiellement lorsque x augmente ;
la phase se dplace avec une vitesse de phase G), p.
Notons en outre que r et / sont en phase dans le temps et dans lespace : donc
toute la puissance transporte par une onde progressive lest sous forme de puis
sance active. Ce rgime dondes progressives est le rgime de fonctionnement
dune ligne le plus favorable pour le transport de lnergie.

Avec un millivoltmtre H.F. qui mesure directement la valeur efficace de la


tension applique, ce sont les valeurs efficaces de la tension H.F. sur la ligne que
lon obtient :
(38)

Kr=
On observe donc lorsquon dplace la sonde de mesure vers les a

> 0 :

une tension damplitude constante si la ligne est sans perte,


une tension damplitude dcroissant exponentiellement si la ligne a des pertes
(fig- 4).

Figure 4.
Dcroissance
exponentielle de la
tension, le long dune
ligne pertes.

a NiVm

2 .6

x2 ~ Xl

(39)

V2

C a s p a r ticu lier : lig n e en c o u r t - c ir cu it


OU EN CIRCUIT OUVERT

Ltude du rgime dondes stationnaires qui stablit, dans ce cas, sur la ligne est
effectue de faon dtaille au chapitre suivant.

2.6.1 Ligne en court-circuit

{Z R

= 0)

Cela signifie que p o u r* = , Z = 0 et V = 0. crivons donc que pour x


V() = 0. D aprs (24) :
0 = Y0 ch y - Zc / 0 sh y t
(40)
En calculant Vq ou Tq daprs cette relation et en portant la valeur ainsi obtenue
dans les relations (24) ou (25), il vient :
sh y( - Jt)
V{x)= V 0 -
sh-yf
ch y( Jt)

(41)

(42)

ly e

^ \y ( -x )
0 th-yf

(43)

Z0 = Zc lh y t

(44)

sin R ( - x )
VW = Y0
sm p

(45)

m = i 0

(46)

Pour les lignes sans pertes :

cos p

_
^ tg p ( - x )
Z(x) = Zn ------------0 tg p

(47)

Z0 = j Z .tg P

(48)

2.6.2 Ligne en circuit ouvert

(ZR = oo)

Cela signifie que pour x , Z = et / = 0. Ecrivons donc que pour x t,


/() = 0. Daprs (25) :
v0
(49)
0 = 70 ch y t sh yf
z c

Do :
ch y ( - x)

(50)

(51)
thyC
Z(x) = Z()------- ----0 th y ( - JC)

z - ^

thye

(52)

(53)

Pour les lignes sans pertes :


cos R ( - x)
v w = v 0 z cos p C

(54)

sin P>{ - x )
/(x) = / 0 ---- ------
"
sin 3

(55)

tg Pf
= Z n ------------ tg p ( -jr)

(56)

Z (X )

Zc

Z0 =

(57)

jtg p

2 .7

C a s p a r t ic u l ie r : lig n e q u a r t d ' o n d e
- LIGNE DEMI-ONDE

Nous avons vu que dans le cas o une ligne sans pertes est ferme sur une imp
dance terminale
impdance dentre est :
Z r + Z c tg p i
0

c Y,c + j

tg P>

Si Pf = (2n + 1) n/2 cest--dire si E - (2/i + 1) A/4 :


a ..? -

(59)

Donc la ligne X/4 est un inverseur d'impdance :


si ZR est ractif, Z0 le sera aussi mais de signe contraire ;
Cas limites
si ZR est une rsistance, 70 sera aussi une rsistance plus grande ou plus petite Si Zg = *>, Z0 = 0 et
51Zr = 0, Z0 = *>.
que Zt. selon que Z.R est plus petit ou plus grand que Zc.
U. mi a . . .,1i>M
Une application de ce dernier rsultat est que l'on se sert dune ligne Xjd courtcircuile comme isolateur H.F. puisque son impdance dentre est alors infinie.
Si Pf = un cest--dire si - nn/2 :
Une ligne n'est X/ a
ou A/2 que pour une
Une ligne demi-onde a une impdance dentre gale son impdance de frquence. Ce sont
charge. Ceci est utilis pour ramener en un endroit donn une impdance gale donc des dispositifs
trs faible largeur de
une impdance de charge (antenne par exemple) sur laquelle on ne peut nas faire bande.
directement des mesures.
Z= ZR

2 .3

La

(60)

vitesse de groupe

Nous avons vu au paragraphe 2.5 quune onde progressive de pulsation ( se


propage avec une vitesse v = (, P, appele vitesse de phase :
(A .r)

V0 e - cw eJ(C,- P l)

(61)

En fait, les ondes qui se propagent sur les lignes de tlcommunications ne sont
jamais monochromatiques. Elles sont modules par des signaux reprsentant les
informations transmettre et ces ondes modules occupent une bande de frquence
utile dtendue 2Af autour dune frquence centralef 0 : 2Af = (J0 + AJ) - (f0 - AJ).

2.8.1 Cas de deux frquences discrtes


Considrons, dabord, deux ondes sinusodales de pulsations )0 + Aco et C0 Atu pour lesquelles les valeurs du paramtre de phase sont P0 + Ali et P0 - AP :
y(x. t)= V 0 e -U [e j[(co+ At>' - <P+ aP,a| + e j[(% - A,)f " (P0- AP)fl]
y(x, r) = 2V0 e ~ cu cos (tAco - .vAp) e^u; _ Po*w

(62)

Cette expression reprsente une onde sinusodale de pulsation t0 - onde por


teuse e J(tV ~ PolJ - dont lamplitude, qui est module la pulsation Ato, a pour

enveloppe la courbe e

cos (/ Aco - x AP). Nous pouvons donc dfinir :

une vitesse de propagation de la porteuse, daprs :


t 1~ Po x = cte
=> = c0/P o
vitesse de phase,
une vitesse de propagation de lenveloppe, d aprs :
t Aro - x AP = cte => vg = Aco/Ap
vitesse de groupe.

Figure S.
Reprsentation dune
onde porteuse (traits
pleins) dont les ampli
tudes sont modules
(enveloppe en traits
pointills).

La reprsentation graphique (fig. 5) de lexpression (62) apparat comme une


succession de battements (ou groupes ou paquets dondes) qui se rptent prio
diquement :
si nous considrons lun de ces groupes, compris entre les nuds A, et A2,
nous constatons quil se dplace la vitesse Ac/ a P ;
si, par contre, nous concentrons notre attention sur les ondes porteuses (en
traits pleins) qui se trouvent lintrieur de chacun de ces groupes, nous
constatons quelles se dplacent avec leur vitesse de phase propre co/p.
Aux nuds indiqus par A i et A2, v et i (et, par consquent, les champs E et H)
sont nuis. Il semble donc impossible que lnergie lectromagntique soit trans
fre dans lune ou lautre direction travers ces points. Lnergie emmagasine
dans le paquet compris entre A, et A2, doit se dplacer avec la mme vitesse que
lenveloppe : Aco/Ap. La vitesse de groupe apparat donc comme la vitesse de
propagation de l nergie.

2.8.2 Cas d'un spectre continu de frquence


Prenons maintenant lexemple, beaucoup plus raliste en tlcommunications,
d une onde module qui occupe une bande de frquences 2A/autour dune fr
quence centrale / 0.
co0 - A c = co = cy + Aco <=> p0 - A P ^ P p0 + A P

v (x ,/)= V 0 e OJ(

rco0 +Aco
Jo)0 - Aco

A(co) e-'(V Pd dco

(63)

Supposons que dans toute la bande de frquences, P(co) diffre peu de sa valeur
pour co = (0q et que nous puissions crire, avec une bonne approximation :
P -P Q

= P(co0) = p

CO Cq

(64)

Nous avons successivement :


P = Po + (w _ o> Po
co/ - p x = cor p0 x (co - co0) P x = (co - co0)/ - (co - co0) P^x-t- (co0 / - p0 x)

Dans ces conditions, lexpression (63) scrit :


v (x ,t)= V 0 e ~ wc e^o, - Po-d f 0

A(co) e^w-co0)(f-(t0x) j (

(^5)

Jco0- Aco

Le terme hors de lintgrale reprsente une porteuse, de pulsation co0, se


dplaant une vitesse co0/P 0, qui est la vitesse de phase.
Lintgrale reprsente lquation de lenveloppe de la porteuse, qui se dplace
une vitesse dfinie par :
1 (dco\
( 66)
t - P x = cte => v = ---- =
P

U pU

o,

v est la vitesse de groupe.


En fait, lapproximation qui conduit la relation (64f est rarement vrifie dans
toute la bande de frquences occupe par londe module. Il en rsulte que la
vitesse de groupe ne garde pas dans toute cette bande la valeur constante rsul
tant de la relation ci-dessus.
Dans le cas gnral, la vitesse de groupe dpend donc de la frquence :
dco

(67)

Vg~ dp
La variation de vg( f) a des consquences que nous allons tudier maintenant sur
la qualit de la transmission.

2 .8 3 Diagramme de Brillouin - Dispersion


Le diagramme qui, pour une ligne donne, donne les variations de co en fonction
de P (fig. 6) est trs commode. En effet, pour un point M de la courbe co(3) :
la vitesse de phase est la pente de la droite OM ;
la vitesse de groupe est la pente de la tangente en M.

Figure 6.
Diagramme ft)((3)
permettant de
dterminer la vitesse
de phase et la vitesse
de groupe.

L orsque cette courbe


n est pas droite, vp et v
sont diffrentes lune de
l autre et leurs valeurs
varient avec la frquence.
Com m e la vitesse de
groupe est la vitesse de
propagation des diverses
composantes constituant
le spectre de frquences
de londe module, cela
signifie que le temps de
propagation de ces com
po san tes su r une lo n
gueur de ligne donne, ne sera pas la mme. Il en rsultera des distorsions du
signal que reprsente ce spectre de frquences. Ce phnomne, trs gnant en
tlcommunications, est appel la dispersion dune ligne.
P our une ligne T.E.M .
(fg. 7), nous dmontrons
que la vitesse de propaga
tion est indpendante de
la frquence lorsque les
pertes sont nulles, faibles
ou minimises (chap. 2,
2.3 tude des solu
tio n s de l q u atio n de
propagation et chap. 6,
6.1 tude de la
constante de propagation
et de limpdance carac
tristique ) :

Figure 7.
Diagramme to(|3)
dans le cas particulier
dune ligne T.E.M.

( 68)

De telles lignes sont donc exemptes de dispersion et vp =


quil nen est pas de mme pour les guides dondes.

V /. Nous verrons

2.8.4 Temps de propagation de groupe


Pour une ligne de longueur , le temps de propagation de groupe xg est li la
vitesse de groupe vg, par :

Lorsquil y a de la dispersion :
(V m in = VS = (V m ax

CO
*> m

(vP m

(V m

Pour que toutes les composantes du spectre de frquences du signal transmis


arrivent en phase, il faut donc des dispositifs correcteurs du temps de propaga
tion de groupe : ce sont des filtres introduisant des retards AT variables, tels que :
Tg( / ) + A t(/) = c t e V /
(70)
Lorsque cest une impulsion qui se propage, soient (vjk^ et (v )m les vitesses
extrmes du spectre de frquences que lon transmet.
(71)

AT = (t ) - ( t ) =
SM

<Vm

AT est appel llargissement dimpulsion pour une longueur t. Dans les tech
niques de transmission numrique, ce temps doit rester infrieur lintervalle de
temps sparant deux impulsions successives. C est lui qui limite le dbit num
rique dune liaison p )ur une transmission sur une longueur .

Exemnte-

% $&&& - /V " *
SUr Mon Je.,
d^hit numriijue tho
rique mcucimuin sera
de 1U0 Mbit/km, soit
(100/rs) M bit pour
n km.

E X E R C IC E 2 . 4

E X E R C IC E 2 .1
(Paragraphe 2 .2 - quation de propagation)

(Paragraphe 2.4 - Expression de la tension,


du courant et de l impdance)

Dmontrer la relation (12) : V-//-= - Vr/ / r =Zc.

N # 7* E X E R C I C E 2 . 2
(Paragraphe 2.2 - quation de propagation)

Les paramtres primaires dune ligne tlpho


nique dabonn (frquences vocales de lordre
de 1 kHz) sont les suivants :
R x =1 10~3 D /m Lj = 3,1 1CT6 H /m
Gj = 3,8 10" 9 S /m Cx = 5,8 1(T 12 F /m
1) la fquence de 1 kHz, calculer :
R x + j Lj sous la forme P[ e JtfV
G j + j C | ( sous la forme p2 e-'<p2.

Un gnrateur de force lectromotrice 100 volts,


d impdance interne Zc - 50 Q alimente une
ligne dimpdance caractristique Zc 100 D
sur laquelle la longueur donde = 1,5 m. Cette
ligne qui a une longueur = 10 m, est ferme
sur une impdance ZR inconnue. Limpdance
ramene lentre de la ligne (interface lignegnrateur) est Zq = (125 - j 120) L
1) Calculer les amplitudes complexes de la ten
sion V0 et du courant 70, lentre de la
ligne.
2) Calculer les amplitudes complexes de la ten
sion VR et du courant I R, lextrmit de la
ligne.
3) En dduire Z* et vrifier la valeur ainsi
trouve par un calcul direct partir de Z q.

2) En dduire Zc = R + jX et y - a. + j(3.
'< &

E X E R C IC E 2 . 3
(Paragraphe 2.3 - tude des solutions
de l'quation de propagation)

E X E R C IC E 2 . 5
(Paragraphe 2.5 - Ondes progressives)

Les paramtres secondaires dune ligne de lon


gueur = 100 m sont les suivants :

Mme ligne tlphonique qu lexercice 2.2.

z c = 800 - j 200 n et Y= 0,00775 + j 2,55 (m - ').

1) 1 kHz, calculer la longueur donde et la


vitesse de propagation.

Cette ligne est ferm e sur une im pdance


ZR = Zc et elle est alimente par un gnrateur
dont les caractristiques sont / = 100 MHz,
E = 100 volts et ZG = 75 Q.

2) Cette ligne est suppose sans perte. 1 kHz,


calculer la vitesse de propagation et |3.

1) Calculer le courant / 0 et la tension Vf0


lentre de la ligne.
2) En dduire le courant l R et la tension VR sur
la charge.
3) Q uelle est la puissance moyenne active
absorbe par limpdance de charge ZR 1

lentre dun tuner dont limpdance est de


75 2.
1) D term iner l im pdance caractristique
d une ligne dadaptation X/4.
2) On veut effectuer cette adaptation en deux
fois en utilisant deux lignes X/4 en srie,
sachant que limpdance intermdiaire est
de 150 2. Quelle sera limpdance caract
ristique de ces deux lignes ?

EX ER C SC E 2 .6
(Paragraphes 2.2 et 2.6 - quation de propagation
et Cas particulier o Z R = 0 et Z R = j

Une ligne tlphonique de longueur = 100 km


est alimente par un gnrateur de frquence /
= 1 kHz. On mesure son impdance dentre
Zq lorsque son autre extrmit est :
en court-circuit : (Z q)cc = 530 e J 35 ,

E X E R C IC E 2 . S
(Paragraphe 2.8 Lu vitesse de groupe)

La longueur donde Xd sur une ligne dispersive


est relie la longueur donde X de propaga
tion en espace libre par la relation :

en circuit ouvert : (Zq)cO~ 1 070 e ^21 .


1) Calculer les paramtres secondaires de la
ligne : Zc et y = oc + j(3.
2) Dduire des rsultats prcdents les valeurs
des paramtres primaires de la ligne : R v Lj,
Gj, Cj.

Xc est appele longueur donde de coupure de


la ligne considre. Elle ne dpend que des
dimensions de cette ligne. On notera :

(Paragraphe 2.7 - Ligne quart d onde,


ligne demi-onde)

1) Dmontrer que le produit de la vitesse de


phase vp sur cette ligne par sa vitesse de
groupe vg est gal au carr de v, vitesse de
propagation en espace libre.

On veut adapter une antenne FM de type


diple repli dont limpdance est de 300 2

2) Calculer la valeur de vp en fonction de v, X


et Xd. En dduire celle de vg.

E X E R C IC E 2 . 7

Chapitre 3
Etude d e la rfleMion
l'extreit
d'une ligne

3 .1

q u a t io n s c o r r e s p o n d a n t
AUX NOUVELLES HYPOTHSES

Au contraire des hypothses qui avaient t prises au chapitre prcdent, nous


supposerons que les valeurs connues sont la tension et le courant en bout de
ligne, ct charge.
Figure 1.
Ligne de longueur
(origine de laxe
des a: la charge),
G, gnrateur,
ZH, impdance de
charge.

Soit une ligne de longueur , dimpdance caractristique Zc, alimente par un


gnrateur de f.e.m. instantane e = Eq cos Oit et de frquence /, termine sur une
impdance Zr (fig. 1). Nous avons vu que ltat lectrique en un point quel
conque dune telle ligne rsultait de la superposition dune onde incidente et
dune onde rflchie sur le rcepteur. Au chapitre 2, nous nous tions placs

pour ltude du phnomne dans le cas o lorigine tait prise au gnrateur et


laxe des abscisses orient du gnrateur vers le rcepteur. Puisque, dans ce cha
pitre, nous allons tudier le phnomne de rflexion lextrmit de la ligne, il
vaut mieux prendre lorigine au rcepteur et orienter laxe des abscisses du
rcepteur vers le gnrateur.
Pour savoir comment sont modifies les relations crites au chapitre prcdent,
il suffit de remarquer que la variation de x, note prcdemment + dx, est main
tenant - dx. Dans ces conditions, les relations (5) et (6) deviennent :
dZ
(la)
= (R1 + jL l co)l
dx
d/
= (G[ + j Ci () V
dx

db)

Nous aurons toujours :


dZV

a 2/

(2a)

(2b)

dont les solutions gnrales devront tre crites ici :


V(x) = Z i eyx + V r c ~ yx

(3a)

L(x) = Li eY* + l r e ~ yx

(3b)

partir de (la), (1b), (3a) et (3b), on dmontre que lon a toujours :

Soient VR et [R les tension et courant au rcepteur. Nous avons ici :


,=

-R + Z c I r
Z

-Z

(4a)

cI r

(4b)

Les expressions de la tension, du courant et de limpdance en un point quel


conque de la ligne scrivent donc maintenant :
V(x) = VR ch y x + Z cLr sh yx
R

lOd = 1,j ch yx + sh yx
Z(x) = Z

(5)
(6)

ZR + Zc th yx
(7)
z c + z r i

y*

3 .2

C oefficient

de rflexion

Le coefficient de rflexion en un point d abscisse x quelconque est dfini


comme le rapport de lamplitude de londe rflchie lamplitude de londe
incidente en ce point :
V r e~Vx V rr - 2yx
( 8)
EM
TT e
y.i
V,.e^
V

Comme :

Z R -Z ,

~ Z c '- r

Y.R + Z c I r

EM =

il vient :

ZR

ZR ~ Zc

+ z t.

e~ 2yx

Z R +Z c

Pour x = 0, cest--dire sur le rcepteur, nous obtenons :


R ' Zc
R+

(9)

( 10)

Z C

Il y a deux valeurs particulirement intressantes du module du coefficient de


rflexion. Ce sont
= 0 et
= 1.
La premire valeur se produit lorsque Z* = ZC. Il ny a alors pas d onde rfl
chie et nous avons :
Y(x) Y j
et l(x) = /, eY*
Dans ce cas, il stablit sur la ligne un rgime dondes progressives qui a t
tudi au chapitre prcdent.
La deuxime valeur se produit dans les trois cas suivants :
ZR = 0 ; ZR = ; \ZR - Z C\ = \ZR + ZC\
Dans chacun de ces cas, il stablit sur la ligne un rgime dondes station
naires pures. Nous nen ferons ltude dtaille que dans le cas de la ligne
court-circuite, en supposant quil sagit dune ligne sans pertes afin que les
caractristiques du rgime dondes stationnaires ne soient pas masques par
des effets dus aux pertes.

3.3

C as

particulier o

R gime

Tr = 1

d' ondes stationnaires

3.3.1 Ligne en court-circuit


Puisque ZR = 0,

= - 1 et

= 0. Donc :
V(x) = Zc l R sh yx

(H )

Z(a ) = f R c h y x
Z(x) = Zc th y x

(12)

(13)

Dans le cas des lignes sans pertes, nous obtenons :


V(x) = j Zc / R sin Px
L(x) JR co s Px
Z(x) = j z t tg px
-

(14)
(15)
(15)

a) Variations de la tension et du courant


Pour passer des amplitudes complexes aux grandeurs physiques relles, il faut
multiplier ces expressions par e JMt et prendre la partie relle. On obtient ainsi :
v(a , t) = 3i [ V(a ) e J10']
/( a , r) = 9i [/( a ) e j(u'j
fR tant complexe, nous crirons : I R = IR e A Donc :
v(a ,

t) = aJ i [j Zc 1R sin pA e j(w + ^
l) - Ci [!r cos P a e j(<u/ +

/( a ,

t ) - - Z c IR s in P a sin (t + (J))
/( a , r) = IR cos 0 a cos (tnr + <{))

v(a ,

(17)
(18)

Ces calculs ne sont. Ou encore :


v(a , t) = ZL. IR sin p.v sin (oy + <
J>+ 7t)
valables que si Zc es/
7t i . (
K
Ve/, ce qui est trs
/(a , r) = 1R s in p x + I s in I o y + <{>+
souvent le cas dans la
pratique.
j
Nous voyons aisment, d aprs ces deux dernires formules, que v et i sont
constamment en quadrature dans le temps et dans l'espace. La puissance trans
porte sera donc purement ractive. Autrement dit, il n y a pas de transmission
d nergie active. Ce sont l les caractristiques d un rgime d ondes stationnaires.
H Recherchons la priodicit des variations de la tension et du courant dans le
temps et dans lespace, daprs les relations (17) et (18) :
Si nous prenons a comme variable :
i est nul pour : Pa = {2k + 1) 7t/2 => a = (2k + 1) X/4
i est max ou min pour : Pa = k n => x = k X / 2.
La figure 2a reprsente ces variations. Nous y voyons que deux points o i est
nul sont distants de X/2, deux minima ou deux maxima sont distants de X. 11
apparat donc pour i une priodicit dans lespace de X.
Si nous prenons t comme variable :
j est nul pour : (cor + <J>) = {2k + 1) n /2 => t = {2k + 1) 7 /4 - <J)/o>
i est max ou mm pour : (cor -+- <f>) = 7: rc => t = k T / 2 - / (
Les variations de i en f{t) sont reprsentes sur la figure 2c ; nous y voyons
que deux instants o i est nul sont spars par 7 /2 et que deux minima ou
deux maxima sont spars par T. 11 en rsulte que i a une priodicit dans le
temps de T.

Figure 2.
Variations des valeurs
instantanes de la
tension et du courant
sur une ligne en
rgime dondes
stationnaires.

Pour la tension (fig. 2b et 2d) les phnomnes sont identiques, mais :


un instant donn, v est nul aux endroits o / est max ou min et rciproque
Avec les appareils de
ment.
mesure habituels
rnillivoltmtre HF un endroit donn, v est nul aux instants o / est max ou min et rciproque
ou indicateur d ondes
ment.
stationnaires ( voir
3.5 Mesures de
11 ne faut pas oublier que. lors des mesures, ce sont les valeurs efficaces de la
tensions sur une tension et du courant qui sont accessibles, cest--dire V(.v) ^ 2 et /(.v) \ 2 .
ligne ) - la priodi
cit que l on observe
Or :
est A/ 2, puisque ces
V(.\) = Zc IK | sin p.v I
(19a)
appareils mesurent
des valeurs efficaces.
Hx ) = / r |eospc|
(19b)
Remarque

Les valeurs absolues de sin [Iv et de eos (3a expliquent le fait que l'on observe
une priodicit de 2 et non de X.
b) Variations de l'impdance d'entre d'une ligne en court-circuit
Puisque Z(.r) j Z(. tg p.v, les variations de limpdance dentre dune ligne en
court-circuit sont les mmes que celles de tg p.v. Cest pourquoi nous avons
reprsent ces variations en fonction de p.v sur la figure 3.

F ig u re 3.

Variations de la
tg p.v : si tg p.v > 0,
ZL,est inductive ;
si tg p.v < 0,
ZL, est capacitive.

Pour 0 < Px < 7c/2 , Z.e est inductive. Une ligne court-circuite de longueur
comprise entre 0 et A/4 est quivalente une inductance.
Pour Px = jt/2, Z(. = oo. Une ligne court-circuite de longueur X 4 est quiva
lente un circuit rsonnant parallle la rsonance.

Pour k/ 2 < \ix < K, Ze est capacitive. Une ligne court-circuite de longueur
comprise entre X /4 et ./2 est quivalente une capacitance.
Pour P* = n, Ze = 0. Une ligne court-circuite de longueur X/2 est quivalente
un circuit rsonnant srie la rsonance.
Le stub constitue une application directe de ces proprits. Le stub est un l
ment de ligne court-circuite, de longueur variable. Il sert raliser une imp
dance dentre variable simplement en agissant sur sa longueur. Le stub X/4 est
utilis comme isolateur (proprit dj vue au 2.7).

3.3.2 Ligne en circuit ouvert


Puisque ZR = , r w = 1 et I R = 0. Donc :
V(x) = VR ch yx
VR
I(x) = -z, sh j x

( 20)
( 21 )

( 22)

Dans le cas des lignes sans pertes, nous avons :


V(x) = VR cos Px

(23)
(24)

(25)
Comme au paragraphe 3.3.1 Ligne en court-circuit , passons aux valeurs phy
siques relles :
v(x, t) = 3 i [V(x) eJw']
i(x, t) = 3 l [/(x) eJw']
Soit :
(26)
(27)
v(x, t) et i(x, t) tant ici constamment en quadrature, nous observerons sur la
ligne un rgime dondes stationnaires. La seule diffrence avec le cas de la ligne
en court-circuit cest que les variations de la tension et du courant sont ici les
mmes, dans le temps et dans lespace, que celles observes au paragraphe 3.3.1
Ligne en court-circuit respectivement pour le courant et la tension.

Quant aux variations de limpdance dentre, elles sont les mmes que celles de
cotg fk et les valeurs de Z(x)/Zc sont exactement inverses de celles du cas pr
cdent.

3.3.2 Ligne termine sur une ractance


^
.
r. _ J Xr ~
Z r = ) X R et L r - J x ^ T z

Dans ce cas :

Dans les cas o Zc est rel :

-R

Z c- X r
Zc + j X r

T r est donc de la forme TR eJ<^. Nous avons :


r = l

et

XR
<]>= Arg T r = tc2arctan
*-c

(28)

Nous pouvons crire, d aprs les expressions (3a) et (3b) :


V(x) = V, e ^ [1 +

e 2yx]

(29)

L(x) = /, eyx [1 - eJ<1>e 2yx]


Et, dans le cas des lignes sans pertes :
V(jc) = Vf ejpjr [ 1 + e

(30)

2pjr)]

(31)

/(*) = Z, e jPx [1 ej(<!>" 2(k;]


Pour ([) - 2 p .r = 2kn, la tension est max et le courant est nul.
Pour (J>-2Px = (2k + 1) 7t, le courant est max et la tension est nulle.

(32)

La variation de la tension et du courant est donc la mme que dans le cas dune
ligne en court-circuit ou en circuit ouvert, mais les maxima et les minima nuis
sont tous dcals dune longueur identique dpendant de la valeur de la charge.

3 .4

C as gnral des lignes


DE RFLEXION QUELCONQUE

coefficient

De la dfinition du coefficient de rflexion en un point quelconque dune ligne


donne au paragraphe 3.1 quations correspondant aux nouvelles hypothses ,
il rsulte que les formules fondamentales (3a) et (3b) peuvent scrire :
V(x) = Vj e ^ [ l + r fie - 2yx]

(33)

l(x) = Lt eyx [1 - E/? e 2y*]

(34)

l + R e~27x
Z(x) = Zc

d o :

05)

Dans le cas des lignes sans perte et en tenant compte du fait que
V(x) = JA ex [1 + r R e

2^x)]

Z(jc) = / 1 ej(k [ 1 e i(* 2P^]


1 - H ^ e - i^ - 2^
Z(x) = Zc
1- r . e W - w

e-*1 :
(36)
(37)
(38)

Intressons-nous aux valeurs efficaces (que lon observe avec les appareils de
mesures). Il suffit pour les avoir de prendre le module des amplitudes complexes
et de diviser par ^ 2
V(x) = ~ e P* 11 + Tr e-i^ 2liv) I
v2
1
1

(39)

I(x) = JL e JP* 11 - eW ~ 2P*>I


v2
1 + r we j(<!>_2lk)
Z(x) = Zc
1 _ p e j($-2p*)

(40)

(41)

tudions les variations des expressions (36) (38) et (39) (41).


a) Pour cj - 2px = 2/<tt
La tension est maximale
Km = ^ / ( i + r

ou

( vt \
+ r* )

(42)

(i - rR)

(43)

v/2

Le courant est minimal :

l m= l , d - r R)

ou

lm \y/2

Limpdance est maximale :

zM - z,.

1+ rK

i - rD

(44)

b) Pour <J>- 2px = (2k + 1) tt


La tension est minimale :
Vm = V ,(l - T R)

ou

p HJL) d - r R)

(45)

Le courant est maximal :

m
-M - / ( ^

F/?)

(46)

lv /2 j (1 + r /f)

ou

Limpdance est minimale :


Z =7

l ~rR

(47)

l+TR
Deux maxima ou deux minima tant spars par X/2, nous pouvons en conclure
que les variations de toutes ces quantits se font avec une priodicit de X/2.
Pour mettre en vidence quil sagit d une variation sinusodale, il suffit de
considrer lexpression de la tension, par exemple :

Soit :

V(x) =

(48)

cos ((J) - 2px) +

c) Faisons une reprsentation graphique de ces phnomnes (fig. 4) :

Zp

Figure 4.
Variations des valeurs
efficaces de la tension,
et du courant
en rgime dondes
semi-stationnaires.

La tension oscille entre

et Vm.

Le courant oscille entre 7M et 7m.


Limpdance oscille entre

etZ m.

Ces variations se font avec une priodicit de X/2, la tension et le courant tant
en quadrature. Si lon considre, par exemple, les variations de la tension, nous
pouvons dcomposer ce rgime en la superposition :
dun rgime dondes progressives damplitude Vm ;
un rgime dondes stationnaires d amplitude variant de 0
On peut donc parler de rgime semi-stationnaire avec comme cas particuliers :
si T r = 1 le rgime dondes stationnaires (O.S.) pures ;
si TR = 0 le rgime dondes progressives (O.P.) pures.

3.4.3 Rapport d'ondes stationnaires


La quantit v j v m = /M/7m = p est appele le Rapport dOndes Stationnaires
ou R.O.S. ; en anglais : Voltage Standing Waves Ratio ou V.S.W.R. Elle est lie
au module du coefficient de rflexion de la charge par :
J
P

+1*
1 rtf

(49)

Comme 0 ^ Tw 1, la valeur de p est comprise entre 1 et linfini.


Lorsque p est voisin de 1, on peut dire que le rgime qui est tabli sur la ligne
tudie est proche du rgime dondes progressives. Ds que p vaut quelques
units, cest quil y a une dsadaptation ligne-charge assez srieuse. Enfin, des
valeurs de p suprieures 10 caractrisent un rgime qui se rapproche du rgime
dondes stationnaires.
Le R.O.S. est parfois appel T.O.S. (Taux dOndes Stationnaires) ce qui nest
pas une appellation trs judicieuse pour une quantit qui nest pas comprise
entre 0 et 1 .
Notons enfin deux importantes relations :
7

_ - MT
P
-m

V m _ Zc
_
j
'M
p

1c

(50)
(51)

Comme, en gnral, Zc est relle, ZM et Zm sont relles ; nous pouvons donc


noncer ce rsultat trs important :

Lorsquune ligne d impdance caractristique relle est ferme sur une imp
dance quelconque, il y a deux points de cette ligne o l'impdance est relle : ce
sont les points o la tension est maximale et minimale. La valeur de l impdance
en ces points est gale au produit et au quotient de l impdance caractristique
par le R.O.S.

3-5

M e s u r e s d e t e n s io n s s u r u n e lig n e

3.5.1 La ligne de mesures


Cest llment indispensable pour que lon puisse effectuer des mesures sur une
ligne. Elle se compose (fg. 5) :
dun tronon de ligne, identique la ligne que lon veut tudier, la surface
duquel une fente longitudinale a t usine sur une longueur dau moins une
longueur donde ;
d un chariot que lon peut dplacer avec grande prcision le long de ce
tronon de ligne fendue ;
dune sonde trs fine porte par le chariot et qui peut pntrer par la fente
lintrieur de la ligne tudier.
La sonde jouant le rle dantenne pour le champ lectrique sera donc sensible
la tension sur la ligne lendroit o elle se trouve. Cette tension pourra tre
mesure laide dun millivoltmtre (VHF ou UHF selon la frquence laquelle
fonctionne cette ligne) qui en donnera la valeur efficace. Mais les milhvoltmtres sont trs coteux dans ces gammes de frquences et nexistent pas en
hyperfrquences. Aussi prfre-t-on associer la sonde un appareillage de
mesure plus simple, valable de la HF aux hyperfrquences, lindicateur dondes
stationnaires.

Figure S.

t t t t t t t t

Ligne de mesures
fente longitudinale.

3.5.2

Indicateur d'ondes stationnaires

Il est constitu, selon le schma quivalent de la figure 6 :


dun cristal dtecteur choisi pour la gamme de frquences utilises ;
dune capacit C qui peut parfois se rduire la capacit parasite ;

dun milliampremtre ou millivoltmtre continu.

Figure 6.
Schma quivalent
dun indicateur
dondes stationnaires.

Soit v = A cos lt la tension applique au cristal dtecteur par lintermdiaire de


la sonde (cette tension est directement proportionnelle la tension sur la ligne
lendroit o se trouve la sonde). Si v est dans les limites convenables - prcises
par les constructeurs pour chaque type de cristal - le courant quil dlivre est de
la forme : / = j v + a2 v~ + termes dordre suprieur ngligeables.
Soit :

i = n j (A cos lt) + a2 {A cos lt)2


a2A2
a 2 2
i = a \A cos lt +
co s 2 oy H--- n

Les composantes alternatives sont court-circuites par la capacit. Seule parvient


au milliampremtre la composante continue qui est proportionnelle au carr de
lamplitude de la tension applique au cristal.
Le milliampremtre donnera donc une indication proportionnelle au carr de
lamplitude de la tension HF sur la ligne ; on dit que la diode est dtection
quadratique.
Soit
le courant lu sur le milliampremtre lorsque la sonde se trouve en
un maximum (minimum) de tension sur la ligne. Le R.O.S. devra tre calcul
daprs :

3.5.3

Mesure du R.O.S. niveau constant

Une autre mthode de mesure du R.O.S consiste oprer avec un niveau


constant, soit /0, du courant dtect. En un maximum de tension vM. rglons
AM (dB) lattnuation dun attnuateur calibr de prcision pour que le courant
dtect soit iQ. En un minimum de tension Vm, lattnuateur devra tre rgl
Am (dB) pour que le courant dtect soit toujours gal iQ ; nous avons :
201gVM/V m = AM- Am (dB).

Do :

p = 10

20

(5 3 )

Lintrt de cette mthode est que la dtermination du R.O.S. reste prcise,


mme si le courant dtect par la diode nest plus exactement proportionnel au
carr de la tension sur la ligne, ce qui peut se produire avec une diode vieillis
sante ou endommage.

.6

M e s u r e d e l' im p d a n c e d e c h a r g e
d ' u n e lig n e

3.6.1 Principe de la mesure


Il rsulte de la relation ( 10 ) que :
z^

1 + L
zc T T r1
-R

(54)

Le calcul de limpdance de charge dune ligne dimpdance caractristique Zc


connue ncessite donc la dtermination du coefficient de rflexion de la charge :
r * = r* e* .
:4 Cela peut se faire partir de la configuration de la tension le long de la ligne.
Le fa it que Q soit
dfini 2 kn prs Nous avons vu, en effet, que :
i+ rw
implique que la re!a-t
tien (56) peut tre
utilise avec n im
porte quel minimum
de tension. Nous Donc, aprs avoir mesur le R ,O.S., on pourra calculer .
savons, en effet que
p- 1
r = ----deux minima de ten
(55)
* p+1
sion sont spars par
nA/2. Par consquent
2pnk/2 = 2nn : quel D autre part, la position dun minimum de tension est dfinie, partir de la
que soit le minimum charge par : <]>- 2 pAni = (2k + 1 ) 7t.
de tension choisi, la
valeur de <p sera la Donc, la mesure de x m permettra de trouver 2ktt prs :
mme 2ktt prs.
<J) = 2 fk m + n
(56)

3.6.2 Technique de mesure


La mesure du R.O.S. dcoule directement de ce qui a t expos au paragraphe
3.5.2 Indicateur dondes stationnaires . La mesure de x m est aise lorsquil est
possible de connatre avec prcision la distance entre la charge et lun des
minima de tension sur la ligne.

Figure 7.
Cas o lorigine de
laxe des abscisses
ne concide pas
avec limpdance
de charge.
io

Mais, dans certains cas, la graduation porte par la ligne de mesure ne donne pas
la distance du point o se trouve la sonde la charge : soit que cette graduation
nait pas t positionne de telle sorte que son zro concide avec lextrmit de
la ligne de mesure o lon place les charges mesurer, soit que la charge nait
pas t insre en bout de ligne mais plus loin (fig. 7).
La mthode de mesure de jem sera alors la suivante (fig. 8 a et 8 b) :
a) Placer la charge tudie et pointer sur la ligne de mesure la position x l dun
minimum de tension.
b) Remplacer la charge par un court-circuit et pointer la position x 2 d un
minimum nul de tension. Nous savons dans ce cas que : x2 = n k /'l.
c)
sera enfin dtermin par :
x m = x \ ~ x2 + n \

(57)

ce qui donnera toujours 2 krc prs :


([) = 2[3(jtj - x2) + tc

(58)

Zp

charge

Figure 8.
Position
dun minimum
de tension sur
la ligne termine :
a. par la charge
mesurer,
b. par un court-circuit.
court
circuit

E X E R C IC E 3 .1
(Paragraphe 3.1 quations correspondant
aux nouvelles hypothses)

Une ligne de longueur t = 10 m a les caract


ristiques suivantes :
Zc = 500 Q. ; a = 0 ; p = 2,856 rad/m.
Limpdance de charge est :
ZR = 2 0 0 0 + j 1 000 i.
Le courant dans cette charge est :
l R = 0,0341 e --i45166 A.
1) Calculer la tension V e et le courant Le
lentre de la ligne (x = ).
2) En dduire Ze et, pour vrifier le rsultat,
calculer Ze par la formule de transformation
des impdances.
3) Puissance active transporte par la ligne.

E X E R C IC E 3 . 2

2) Sachant que VR = 143,2 V, calculer Vi et Vr.


3) Si la phase de V- est prise pour rfrence,
dterminer V,-, Vr f i et / r.
4) En dduire V R, I R et la puissance active
absorbe par la charge.

E X E R C IC E 3 . 3
(Paragraphe 3.2 - Coefficient de rflexion)

1) Exprimer le module et largument du coeffi


cient de rflexion dune impdance ractive
pure : ZR = j XR (XR > 0 ou < 0) chargeant
une ligne d impdance caractristique Zc
relle.
2) Application au cas o Zc = 75 Q et :
a) ZR = j 100 2 (charge inductive),

b) ZR = - j 50 Q (charge capacitive).

E X E R C IC E 3 . 4

(Paragraphes 3.1 et 3.2 - Coefficient de rflexion)

Une ligne dimpdance caractristique Z C = 7 5 Q .


est term ine par une im pdance de charge
Z R = 125 + j 100 n .
1) Calculer les valeurs de T R en coordonnes
polaires et en cartsiennes.

(Paragraphes 3.2 et 3.4 - Cas get ral des lignes


coefficient de rflexion quelconque)

Une ligne coaxiale dimpdance caractristique


50 Q alimente une antenne dont limpdance
d entre est de 300 Q.. Les pertes sont de
0,3 dB/m et la longueur donde de 0,3125 m.

1) Calculer a (Np/m) et P (rad/m).


2)

une distance =
miner :

E X E R C IC E 2 . S

m de lantenne, dter

a) le coefficient de rflexion,
b) limpdance.

(Paragraphe 3.4 - Cas gnral des lignes


coefficient de rflexion quelconque)

Soit une ligne sans perte transportant une puis


sance active de 100 W. Zc. = 100 O ; R.O.S. :
P = 2.

E X E R C I C E 3 . 5 *12
(Paragraphe 3.4 - Cas gnral des lignes
coefficient de rflexion quelconque)

Une ligne dimpdance caractristique Zc = 50 2


est ferme sur une impdance ZR 115 + j 75 O.
la frquence utilise, le paramtre de phase
P = 3,307 racL/m.
1) Calculer r R et le R.O.S.
2) Dterminer ZM, Zm et les positions
et xrn
des premiers maxima et minima de tension
partir de la charge.

Calculer les valeurs efficaces, maximales et


minimales, de la tension et du courant : VM.
Avi> Air

E X E R C IC E 3 . 7
(Paragraphe 3.6 Mesure de l'impdance de
charge d une ligne)

Mme nonc que pour lexercice 3.6. On sait,


de plus, que le 1 er minimum de tension est
0,75 m de la charge et que X - 10 m.
1) Calculer T R et ZR.
2) Calculer IR et

en modules.

Chapitre 4
Diagramme de Smith

4.1

In tr t

Nous avons vu. au chapitre prcdent, quil y avait une relation simple (formule
35) entre le coefficient de rflexion r(.v) = F w e~ 2^ ' et limpdance Z(x) en un
point dune ligne :

1 + ri.v)
A x )~ Z( i - r(x)

(1)

Dans le cas particulier o lon se place sur limpdance de charge :


1 +!
Z* = Z - T T r 1 R

(2)

En raisonnant en impdances rduites z, qui sont le quotient de limpdance Z


par limpdance caractristique Z(1, nous avons :
2 (.\) =

1+EW
1-EW
1

et

-E *

(3)

(4)

Il est donc possible de calculer Zr daprs le coefficient de rflexion R = ^R eJtp:


la dtermination de
rsulte de la mesure du R.O.S. daprs :

P-

celle de cp rsulte du pointage dun minimum de tension daprs :


cp = 2fixm + 7i

(6 )

De mme, il est possible de calculer z(x) daprs le coefficient de rflexion


E W = r R e ~ 2yx. Mais ces calculs sont longs. Le diagramme de Smith est un
abaque dimpdances qui permet, connaissant T, de dterminer z et inversement.

P rincipe et c o n str u ctio n


DU DIAGRAMME
La valeur du coefficient de rflexion est, dans le cas gnral :
z(x)~ 1
rw =
z(x)+ 1
Nous savons que : r(x ) =

(7)

e 2l/x.

Dans le cas des lignes sans perte : r(x ) = r R e-^<p~


En posant :
la relation (7) peut scrire :

\j/ = cp - 2fix
z- 1
z+ 1

4.2.1 Reprsentation de

(8)

(9)

ejl^

Nous pouvons en faire deux reprsentations dans le plan complexe,


a) En coordonnes polaires (fia. 1a)

Figure la.
Reprsentation
de JT(x) dans le plan
complexe
en coordonnes
polaires :

r w = r,ei.

4'

= 0
x

Dans ce cas, les points


reprsentatifs des diffrentes
valeurs du coefficient de
rflexion sont dtermins de
faon unique partir dun
point O de rfrence et dun
axe Ox de rfrence par :
leur rayon vecteur qui
est proportionnel r* .
leur position angulaire
qui reprsente \p.

Comme dans tous les cas


un cercle de rayon unit comprendra toutes les
valeurs possibles de limpdance.

b) En coordonnes
cartsiennes (fig. 1b)

Imaginaires

Figure Ib.
Reprsentation
de E(x) dans le plan
complexe
en coordonnes
cartsiennes :
(*) = p +}q .

- Rels

Cela est rendu possible puisque :


r R eW = p + iq

( 10)

Dans ce cas, il suffit de rapporter le plan aux deux axes rel et imaginaire. Un
point reprsentatif M est alors repr de faon unique par :
son abscisse complexe p qui est la projection de M sur laxe des rels :
P = r^cosvj;

(11)

son ordonne complexe q qui est la projection de M sur laxe des imaginaires :
9 = r wsinvj/
(12)
c) Compatibilit des deux reprsentations de r R e ^
Dans la mesure o lorigine et lunit de longueur choisies sont les mmes que
dans la reprsentation prcdente et si, en outre, laxe des rels concide avec
laxe Ox, les points reprsentatifs obtenus en coordonnes cartsiennes et en
coordonnes polaires sont confondus.

4.2.2 Reprsentation de z = r + jx
Le coefficient de rflexion est connu (dtermin par les mesures). Nous venons
den indiquer quelles en sont les deux reprsentations ; il sagit maintenant de
trouver limpdance correspondante. Autrement dit, connaissant p et q, dter
miner r et x. Pour cela, nous allons utiliser la relation :
( r - l)+ jx
(13)
p+]q =
(r+ l)+ j*
dduite de (9) et (10) et qui lie le couple de valeurs (p, q) au couple (r, x). En
galant parties relles et parties imaginaires, nous obtenons deux quations
deux inconnues r et x.

liminons x entre ces deux quations ; il vient :

(14)
Si p et q sont connus, il est possible de dterminer r par cette relation.
Mais on peut remarquer que, sous cette forme, cette relation dfinit des cercles de
rayon l/(J + r), centrs au point du plan complexe p = r/(I + r), q = 0. Ce sont
donc des cercles centrs sur l'axe p qui passent tous par un point fixe : p = /,
q = 0. Par consquent, en prenant r comme paramtre, il sera possible de tracer,
dans le plan complexe prcdemment dfini, un abaque dont chacun des cercles
correspondra une valeur de r (fig. 2).

Cas particuliers de ce faisceau de cercles r = cte.


r =0

> Cercle de centre (p = 0, q = 0) de rayon R = I. Cest le grand


cercle du diagramme.

r =

> Cercle de centre (p = 1, q = 0) de rayon R = 0. Cest un cercle


point.

grand cercle
du diagramme
axe

p =1

0,6

x> 0

'0 ,3

point p = 1.

q=0

Figure 2.
Construction des
cercles du diagramme
de Smith.

axe
d es
rels

0,2

l 0,5

ongine
des phases

7-5
x<0

s - 0.3

- 0.6

-1

___

l liminons r entre les deux quations ; il vient :


n2 m
(p -ir+ k -

(15)

Si p et g sont connus, il est possible de dterminer a par cette relation.


Mais, on peut remarquer, comme prcdemment, que cette relation dfinit des
cercles de rayon //x centrs au point du plan complexe p = /, q = ]/x. Ce sont
donc des cercles centrs sur un axe men par p = l, paralllement Taxe imagi
naire. Ils forment un faisceau dont le point fixe est p = /, q = 0. Par consquent,
en prenant x comme paramtre, il est possible de tracer un abaque o, chaque
valeur de x, correspond un cercle (fig. 2).
Cas particuliers de ce faisceau de cercles x - cte.
x = 0 >Cercle de centre (p = 1, q = ) de rayon R =
Cest laxe hori
zontal.
x - >Cercle de centre (p= 1, q = 0) de rayon R = 0. Cest un cercle point.
Les deux faisceaux de cercles ainsi dfinis ont un point fixe commun (p = 1,
q = 0) et ils sont orthogonaux. Leur ensemble forme le diagramme de Smith trs
utilis dans la pratique.

4-3

P r o p r it s du d ia g r a m m e

4.3.1 Positionnement d'un point


Si lon connat Tp et vj/, le point reprsentatif (fig. 3) est lintersection du
cercle ayant pour centre le centre du diagramme et pour rayon TK (se rappeler
que pour le grand cercle du diagramme, r R = l) avec la droite faisant langle
\j/ avec laxe horizontal des rels (rsistances), q/ tant compt comme sur le
cercle trigonomtrique.
Si lon connat r et x, il suffit de positionner ce point par interpolation avec les
faisceaux r = cte et x = cte tracs sur le diagramme.
Dans lun et lautre cas, on voit combien il est ais de passer de TR e jV r + jx
ou rciproquement.

4.3.2 Lieu des points reprsentatifs des diverses


impdances de la ligne (fig. 3)
Daprs la formule (7), les variations sur la ligne de limpdance rduite et du
coefficient de rflexion sont lies. Comme, sur le diagramme de Smith, un point
reprsente z(x) et F(x), rechercher le lieu de z(x) lorsque x varie revient au mme
que de rechercher le lieu de E(x).

_
4 ? c i" *
JDarisice cas, on peut
avoirX > Oou x < 0,
alors 'que dans le pre
mier cas r > 0. J >'

Figure 3.
Positionnement
dun point sur le
diagramme de Smith.
Lieu des points
reprsentatifs de
E(x) et z(x).

Comme
(x) = r K e " 2ou e i(* 2<ir)
nous voyons que lorsque x varie :
sur une ligne avec pertes, ce lieu est une spirale logarithmique ;
sur une ligne sans perte, le lieu de r (x) est un cercle de rayon
centre de diagramme.

(16)

centr au

Donc :
pour une ligne avec pertes, il faut connatre a pour pouvoir tracer la spirale
logarithmique ;
pour une ligne sans perte, il suffira de connatre le point reprsentatif dune de
ses impdances pour pouvoir tracer le cercle sur lequel se trouvent les points
reprsentatifs de toutes les impdances de la ligne.
Remarque
Aux intersections dun tel cercle avec l'axe des rels, on lit le R.O.S. et son inverse.
On sait, en effet, quil y a deux points de la ligne o l impdance est relle : ce sont
les points o l impdance est maximale ou minimale ; en ces points :

ZM~PZc
Zm = -

ZM P

-> Z,ni '

(17)
(18)

le point correspondant au maxima est entre

le point correspondant au minima est entre

et > >p ,
et

> 1 . p.

4 .3 .3 Diagramme d'admittance
Nous avons plac z = r + ]x (r, rsistance, x ractance) et nous voulons trouver
y - g + j> (g, conductance, b, susceptance). 11 suffit pour cela de remarquer que :
1

+r*e->v
(19)
V
( 20)

+ r ffeJV

On passe dune formule lautre en ajoutant n vj/.


Donc, le diagramme d admittance se dduira du diagramme d impdance par
une symtrie par rapport au centre. Il suffira pour reprsenter une admittance de
prendre le symtrique de r + jx.

4 .3 .4 Changem ent de base


Base Zc :

z*
Z = = r+yc

(2 1 )

Base Z. :

, ZR
Zc
z ~ / C ~ z 7 c

(2 2 )

4.3 .5 chelles marginales (f:g. 4)


Une chelle donne les longueurs de ligne divises par pour les dplacements :
vers le gnrateur (en tournant dans le sens trigonomtrique inverse) ;
vers la charge (en tournant dans le sens trigonomtrique direct).
Une autre chelle donne directement les angles de dphasage par rapport lori
gine de 0 + 180.

En dessous du diagramme de Smith, on trouve souvent, sous le titre Radially


Scalled Parameters, des chelles qui ont une utilit pratique. Nous allons donner
la signification des principales en renvoyant le lecteur, pour les autres, aux
ouvrages spcialiss (voir rfrence bibliographique 28).

s,

.Les cercles x = cte


et x - cte devien
nent alors les cercla
. g '}0,cte et b = cte.
Les. signes pom b
pont te s .mmes que
ceux pour x sur
diagramme.

IMPEDANCE OR ADMITTANCE COORDINATES

Figure 4.
Diagramme de Smith. Abaque extraite de Electronic applications of the Smith chart par P.H. Smith.
ditions Mac Graw Hill, New York, USA (1969).

1. Une chelle marque S.W.R. et une autre marque Reflection coefficient don
nent respectivement les valeurs du R.O.S. et du coefficient de rflexion dune
charge correspondant un rayon vecteur OP.
2. Lchelle Return Im s in dB donne le rapport, calcul en dB, entre la puis
sance arrivant sur un plan de dsadaptation dune ligne ou sur une charge et la
puissance rflchie sur ce plan ou sur cette charge. La valeur de ce rapport est
exprime en fonction du coefficient de rflexion T sur ce plan ou sur cette
charge par : - 10 log F2.
3. Lchelle Reflected Loss in dB donne le rapport, calcul en dB, entre la puis
sance arrivant sur un plan de dsadaptation dune ligne ou sur une charge, et
la puissance transmise au-del de ce plan ou de cette charge. La valeur de ce
rapport est exprime en fonction de T par - 10 log ( 1 - F2).

4 .4

D t e r m in a t io n

d e l' im p d a n c e

DE CHARGE D'UNE LIGNE


Nous avons expliqu au chapitre 3.6 comment on pouvait dterminer limp
dance de charge ZK d' une ligne par le calcul en connaissant le R.O.S. et la posi
tion dun minimum de tension sur la ligne. Nous allons voir que lon peut
trouver Z* plus simplement et beaucoup plus rapidement laide du diagramme
de Smith (fig. 5).
Dabord, il faut tracer le cercle R.O.S. = cte sur lequel se trouve limpdance
recherche. Pour cela il suffit de reprer sur laxe horizontal droite, le point M
correspondant au R.O.S. et par lequel passe ce cercle.
Ensuite, il suffit de constater que le point N, diamtralement oppos qui repr
sente limpdance minimale de la ligne, reprsente aussi un minimum de ten
sion.
Soit, enfin xm la distance mesure entre ce minimum de tension et la charge. En
tournant partir de N, vers la change, de xm/X nous obtenons le point P repr
sentatif de limpdance Z* recherche.
Une petite difficult survient lorsque la distance xm doit tre mesure indirecte
ment daprs : xm (jtj - x 2> + h X./2. '
Comme nous lavons expliqu la fin du chapitre prcdent :
si x2 se trouve entr'xj et le plan de la bharge, la rotation de | jcjx2 \/X sur le
diagramme doit bien tre effectue vers la charge ;\
'

>.
mais si x2 se trouve, au contraire, entre Xj et le gnrateur, la rotation de
| Xj - x2 |/L doit alors tre effectue vers le gnrateur.

Figure S.
Dtermination du
point P reprsentatif
de limpdance de
charge dune ligne.

&

(Paragraphes 4.3.1 et 4.3.3 - Positionnement


d un point et Diagramme d admittance)

1) Sur une ligne sans perte dimpdance carac


tristique Zc = 50 fi, une impdance Z* cre
un coefficient d rflexion /( = 0,54 eJ 128 ,
En utilisant labaque de Smith, dterminer :
le R.O.S. p, limpdance ZR et ladmittance
Yr 2) Mmes questions pour VR = 0,4 e ~J 62 .

&

1) Rapport d ondes stationnaires p' et coeffi


cient de rflexion T K.

E X E R C IC E 4 . 1

E X E R C IC E 4 . 2
(Paragraphes 4.3.1 et 4.3.3)

1) Soit une ligne sans perte dimpdance carac


tristique Zc = 50 fi, termine par une imp
dance Z R - 40 + j 65 fi. En u tilisant
labaque de Smith, dterminer : ladmittance
Yr >le R.O.S. p, le coefficient de rflexion
R de la charge.

2) Impdance ramene 11 cm de la charge


3) Position des maxima et minima de tension
sur la ligne et valeur de limpdance en ces points.

&

E X E R C IC E 4 . 4
(Paragraphe 4.3 - Proprits du diagramme
de Smith)

Une ligne sans perte de longueur = 2,25 m et


dimpdance caractristique Zc - 75 fi est ter
mine par une impdance zR <=2,10 + j 1,35 en
valeur rduite. A = 1,9 m.
1) Dterminer son impdance dentre Ze.
2) Rapport dondes stationnaires p et coeffi
cient de rflexion F R.
3) Distance du 1er minimum dimpdance la
charge et valeur de Zm.

2) Mmes questions pour ZR = 30 - j 70 fi.

E X E R C IC E 4 . 5
E X E R C IC E 4 . 3
(Paragraphe 4.3 - Proprits du diagramme
de Smith)

Une ligne sans perte dimpdance caractris


tique Zc = 50 fi est charge par limpdance
Z/j = 30 - j 55 f i . l = 8 cm.

(Paragraphe 4.3 - Proprits du diagramme


de Smith)

1) Calculer la longueur d une ligne courtcircuite dont ladmittance dentre est 2 j


(k = 5 cm).
2) Calculer la longueur dune ligne en circuit
ouvert dont limpdance dentre est 1,5 j
(k = 10 cm).

E X E R C IC E 4 . 6
(Paragraphe 4.3 - Proprits du diagramme
de Smith)

mine par une impdance inconnue ZR. On


mesure un R.O.S. gal 2,7 et la position du
premier minimum de tension partir- de la
charge est jtm 34,8 cm.

Deux tronons de ligne sont mis en parallle :

Dterminer en valeurs rduites :

lun de Zc - 50 O, charg par une imp


dance Z, '= 50 j 75 l, de longueur
= 10 cm (X[ = 30 cm) ;

1) Limpdance de la charge.

lautre de Z( = 75 O, charg par une imp


dance Z2 = (100 + j 75) Q , de longueur
^2 = 6 cm (k2 = 2 0 cm).
Calculer ladmittance Ye ramene entre les
points o ces deux tronons de ligne sont mis
en parallle, sachant que la ligne laquelle ils
se raccordent a une Zc = 50 O.

<#" E X E R C IC E 4 . 7 _____________
(Paragraphe 4.4 - Dtermination de l'impdance
de charge d'une ligne)

Soit une ligne sans perte de-longueur t = 145 cm


sur laquelle . = 10 0 cm. Cette ligne esttr-

2) Limpdance dentre de la ligne.

&

E X E R C IC E 4 . 8

(Paragraphe 4.4 - Dtermination de Pimpdance


de charge d une ligne)

Une ligne sans perte dimpdance caractris


tique Zc = 50 Q. est charge par une impdance
inconnue ZR. On mesure un R.O.S. gal 5 et
une longueur donde A. = 50 cm.
1) Lorsquon court-circuite la ligne, un mini
mum de tension se dplace de 5,25 cm vers
la charge ; dterminer la valeur de ZR.
2) De combien et dans quel .sens se serait
dplac un minimum si ZR avait t rem
place par un circuit ouvert ?

Chapitre 5
Les dispositifs

d'adaptation

5 .i

L e p r o b lm e de l' a d a pta tio n


Figure 1. Gnrateur,
ligne et rcepteur.
Figure 2.
Schma quivalent.

Le problme le plus gnral est schmatis sur la figure 1.


Nous avons :
Un gnrateur de force lectromotrice e = E F(t), dimpdance interne com
plexe Zq .
Une ligne dimpdance caractristique relle Zc.
Un rcepteur (ou charge) dimpdance complexe ZR.
Il sagit de transmettre, par lintermdiaire de cette ligne, le maximum de puis
sance du gnrateur vers le rcepteur.
Le problme se pose, et se rsout, deux niveaux : au niveau du gnrateur et au
niveau du rcepteur. Il faut, en effet, que :
dune part, le gnrateur puisse transmettre la ligne le maximum de puis
sance (puissance disponible),
dautre part, le rcepteur reoive de la ligne le plus possible de cette puis
sance.

;.2

C o n d itio n s d 'a d a pta tio n

5.2.1 Condition d'adaptation du gnrateur


Soit Ze = Re + j Xe limpdance dentre de la ligne. Cela veut dire que tout
se passe comme si le gnrateur tait ferm sur Ze (fig. 2). Calculons quelle
est la puissance active P fournie par le gnrateur, d impdance interne
Zc = R g + j XG, la ligne. Soient Ve et [ e les amplitudes des complexes de la
tension et du courant lentre de la ligne :
P ^ [ y . e C \ =\ ^ e L e C \ ^ R e Q
Or:

Donc :

/ =

ZG + Ze

1
P = n Re

(Rc + Re) +}(XG+Xe)

E2
------------ 2---------------7
(RG + Re)2 + (XG + Xe)2

(1 )
(2 )

(3)

Recherchons les conditions pour que la puissance dlivre soit maximale :


Il faut tout dabord que :

xG+xe= o -> xe=- x G

(4)

Nous avons alors :


(5)

= U faudra donc
utiliser un dispositif
dadaptation gui
transforme l imp
dance dentre de la
ligne en Z*G'

,=> II. faudra donc


placer entre la ligne
|et le rcepteur un dis
positif d adaptation
qui transforme l im
pdance de charge
ZRde la ligne en Zc.

Au dnominateur, nous avons la somme de deux nombres positifs dont le pro


duit est constant. Pour que sa valeur soit minimale, il faut que :
Rc
(6)
= sf* 7 -* R g - Re

Finalement :
Z= ?n

(7)

5.2.2 Condition d'adaptation du rcepteur


Le rcepteur est adapt la ligne lorsque T R = 0, puisqualors il ny a pas donde
rflchie ; nous sommes en rgime dondes progressives et la puissance trans
mise par la ligne est uniquement de la puissance active. La condition
= 0 est
ralise lorsque :
Z* = Zc

(8)

5.2.3 Synthse de ces conditions


Nous venons de dmontrer que, pour adapter le gnrateur dimpdance interne
ZG, au rcepteur dimpdance Z* lorsquils sont relis par une ligne dimp
dance caractristique Zc, il tait ncessaire dutiliser deux dispositifs dadapta
tion (fig. 3) :
Figure 3.
Principe de
ladaptation du
rcepteur la ligne
(dispositif A [) et de la
ligne au gnrateur
(dispositif Ai)-

lun A,, linterface ligne-rcepteur, qui doit transformer limpdance ZR de


la charge en une impdance Zc. Notons que, dans ces conditions, limpdance
dentre de la ligne est : Ze = Zc ;
lautre A2, linterface ligne-gnrateur, qui doit transformer limpdance
Ze ~ Zc en Z*G.
Dans le cas particulier o limpdance interne du gnrateur ZG est relle, il
suffit davoir Zc = Zc pour que ladaptation soit ralise du ct du gnrateur.
Les gnrateurs commercialiss ont, en gnral, des impdances internes de
50 ou 75 Q, voire 300 Q..
En revanche, les dispositifs ltat solide utiliss dans les circuits intgrs ou
semi-intgrs pour micro-ondes prsentent des impdances dentre complexes
et il est donc ncessaire dutiliser un dispositif dadaptation pour raliser la
condition Ze = Z*G.
Les dispositifs dadaptation que nous allons tudier maintenant sont de divers
types :
adaptateurs par ligne quart donde ;
adaptateurs laide dun ou deux stubs qui sont des tronons de ligne
court-circuits ;
adaptateurs par rseau dimpdances et tronon de ligne.

53

A daptatio n par lig n e q u a r t d ' o n d e

Considrons un lment de ligne de longueur X./4, dimpdance caractristique


Z'c ferm sur une impdance Zs (fig. 4). Nous avons vu quil ramne son entre
une impdance :

Z
e

Z 2
Z
As

Une telle ligne peut servir


d adaptateur p u isq u elle
permet deffectuer une trans
formation dimpdances. En
particulier, dans le cas qui
nous intresse, nous avons :
Ze = Zc et Zs = ZR

X
4

Figure 4.
Ligne quart donde.

Z'c

-------------

(9)

do:

Z'C= J Z ^

(1 0 )

-----------------------

5 3 .1 Cas d'une impdance de charge Z R relle


Dans ce cas, ladaptation sera ralise en utilisant une ligne X/4 dimpdance
caractristique relle Z[. = J Z R Zc . Seul le tronon X/4 travaille en rgime
dondes semi-stationnaires ; le reste de la ligne est parcouru par des ondes pro
gressives.

5 3 .2 Cas d'une impdance de charge Z R


complexe
Dans ce cas, si la ligne X/4 est ferme sur ZR, son impdance caractristique
devra tre complexe. Pour avoir Z , relle, il faudrait que la sortie de la ligne X/4
se trouve en un endroit de la ligne o limpdance est relle, cest--dire :
Soit en un maximum de tension, situ une distance xM de la charge, o lim
pdance est maximale ZM =PZcDans ce cas :
Z = Z c yfp

(11)

Soit en un minimum de tension, situ une distance xm de la charge, o lim


pdance est minimale Zm = Z J p.
Dans ce cas :
( 12 )

Afin davoir Z'c rel, une autre possibilit est de placer la sortie de la ligne X/4
directement sur la charge, et de compenser la partie imaginaire de limpdance
de charge en mettant en parallle sur celle-ci un tronon de ligne court-circuit
dont limpdance est imaginaire pure.

5.3.3 Adaptation large bande passante


Les dispositifs dadaptation que nous venons dtudier ne sont valables qu la
frquence pour laquelle la longueur de la ligne est gale 4 : ce sont donc des
dispositifs dadaptation bande troite. Pour obtenir une adaptation large
bande, on peut fractionner l'adaptation en un certain nombre de tronons X 4
(fig. 5) tels que les impdances dentre successives de ces diffrents tronons
soient : Zc > Zi > Z| > ZH, et pour n tronons : Z(. > Zn > ... > Z t > Zj(.

Z2
Z"

1_ Zmc
J

Figure 5.
Adaptation large
bande.

Zr

la limite, on pourrait montrer quil existe une possibilit dadaptation utilisant


des tronons de ligne dont limpdance caractristique varierait de faon
continue : le profil idal serait exponentiel et la largeur de bande importante.
Cest le cas des lignes non-unilormes.

5-4

A daptation

l'aide d ' un stub

Un stub est un tronon de ligne courl-circuil de longueur s que lon branche en


drivation sur la ligne principale une distance de la charge (fig. 6). Son imp
dance dentre tant :
2n
Z(.v)=jZt.tg
v
(13)
A,
nous voyons quil est quivalent une ractance dont on peut faire varier le
signe et la grandeur en faisant varier sa longueur.
Dailleurs, on pourrait aussi bien utiliser un lment localis, capacitif ou
inductif, plac en drivation sur une ligne.
Les quantits connues sont : Z/v Z et X : les inconnues sont : J et s. Nous allons
raisonner :
en admittances parce que nous avons des lments disposs en parallle,
en valeurs rduites pour pouvoir les placer sur le diagramme de Smith.
Pour la charge :
et

y R - Z/j = HR+jbK

(14)

Nous allons calculer successivement les admittances aux divers endroits de la


ligne :
Dans le plan de charge :

yR = gR + j bK

Dans un plan situ la distance d - , c'est--dire juste avant le stub :


(17)

y (d - e) =
1 + J J 7 e tg [5 ( < / - )

Puisque est pris aussi petit que lon veut :


y (d - e) =

v (cl - ) =

y K + J tg fri

8r + ) I r + jlg fri

i + j y r tg fri

1 - hn tg f w+ j s k tg

8k + j (1>K+ 'g P(/)|| ( 1- hk 'g frf ) - i 8k tg_H

(18)

(19)

( 1- bR tg P</)2 + s i tg2 pcl

Cette expression peut scrire sous la forme :


y ( c l - ) = g (d - ) + j b (d - )

( 20)

Dans un plan situ la distance d + , cest--dire juste aprs le stub :


y (d + e) = y (d - e) + y (s) = g (</ - ) + j [b (d - e) - cotg pxj

21 )

Pour que ladaptation soit ralise partir de la distance [d + ), il faut que :


y (d + ) = 1 + j O
(22)

On dduit de cette condition les deux quations qui vont fournir les deux incon
nues cl et s :
gKd + tg 2 (y)
g(cl-E)= 1
(23)

(I ~ bR tg$d)2 +g% tg2 pJ


Cest une quation du second degr en tg Pcl qui fournit deux solutions : cl et cF
2 prs.
bR + (-1 - ^ - ^ ) t g P ^ - = cotg p.s (24)
b (d - 1') = cotg p.v
( l - fc/?tgPr) 2 + g | t g 2 p j

Daprs cette relation, nous voyons quaux deux valeurs d et d'correspondent


les deux valeurs 5 et s'.
Il y a donc deux couples de solutions : (d, s) et (d \ s7). On choisit celui qui cor
respond la plus faible valeur de d afin que le tronon de ligne qui ne fonc
tionne pas en ondes progressives soit le plus rduit possible.

5-5

A daptation

l'aide de deux stubs

Comme prcdemment, nous raisonnerons en admittances et en valeurs rduites.

Figure 7.
Adaptation laide
de deux stubs.

Donnes : Z / M i et d2.
Inconnues : 5 , et s2.
Nous allons calculer successivement les diverses admittances de la ligne.
la distance 0 :

yR = gR + j bR

,,
>/*+jtgPWi-e)
la distance (d, - e) : v ( d . - e) = - :------- - -----'

(26)

l+JjtgpW,-E)

Nous pouvons crire, puisque e est pris aussi petit que lon veut :
,,
, y/t+jtgp^i
y {cl| - e) = ----------------1 + j V/ftgp^i

(27)

Pour la suite, nous poserons :


v (r/, - e) = g

(dl -

(28)

E) + j b (d, - e)

(29)

la distance (</r + e) : v (t/, + e) = y ( d { - e) + v (sj )

(30)

.y (t/| + e) = g (dl - e) + j [b ( d ] - e) - cotg


la distance d - e :

> (d -

y (d, + e) + j tg fk/2
e)

(31)

+ jy (r/, +e)tg(W 2

Dans cette expression, linconnue est s t ; nous crirons :


v (d - e) = g (d - e) + j b (d - e)
la distance (d + e) :
y (d + e) = y (d - e) + y (s2)
y (r/ + e) = g { d - e ) +j [b ( d - e ) - cotg Ps2]
Dans cette expression, les inconnues sont s, et s2.

(32)
(33)
(34)

Pour que ladaptation soit ralise partir de la distance d + e, il faut que


y (d + e) = 1 + j O. Do les conditions :

g (d -e ) = l
(35)
Lexplicitation de cette relation fournit une quation du second degr en cotg Pj'|
partir de laquelle on calcule, si son dterminant est > 0 , deux solutions s\
et s[\
(D

b ( d - ) = cotg Pj'2

(36)

Cest une quation du premier degr en cotg ps2. Donc, aux deux valeurs .v| et
sj de correspondent les deux valeurs .s2 et s2 de s2.
11

y a donc, dans ce cas galement, deux couples de solutions :


fof,s2) et (i|, i 2 )

5.6

U tilisa tio n du d ia g r a m m e d e S m ith

La recherche des inconnues dans les adaptations un et deux stubs est bien
simplifie si lon utilise le diagramme de Smith.

5.6.1 Adaptation un stub (fig. 8)

Figure 8.
'

Adaptation un stub.

Point 1 : Z[ = 2 + j 1,5
Point 2 :

y2 =

0,32 - j 0 ,2 4

Point 3 : >3 = 1 + j 1,3 ; d/A. = 0,212

Point

B :y(s)

= - j 1 ,3 -

s/X = 0,104
= 0,372
s/X = 0,396

Point 3 :yy = 1 - j 1,3 ; d/X


Point

B:y(s) = j

1,3 -

Plaons tout dabord les points 1 et 2 reprsentatifs des impdance et admittance


rduites de la charge. Le cercle R.O.S. = cte passant par ces points est le lieu
des points reprsentatifs de toutes les impdances et admittances aux divers
points de la ligne compris entre la charge et le stub exclu. Cest, en particulier,
un lieu de y [d - e).
Comme dautre part y {d - e) = y {d + e) - y (s) est de la forme 1 - j b, le cercle
g = 1 est un deuxime lieu de y {d - e).

Les solutions du problme sont donc donnes par les deux points dintersection
3 et 3 de ces deux cercles.
a) Point 3

Le stub doit tre plac une distance d/X de la charge qui est lue sur le bord du
diagramme. (Dans lexemple choisi : d/X = 0,042 + 0,170 = 0,212.)
Soit l + j fc3 ladmittance y (d - ) ; le stub doit avoir une longueur s telle que :
y (s) = - j /3 (ici = - j 1,3). Comme le stub est un tronon de ligne court-circuit,
s est dtermin en lisant sur le bord du diagramme de combien il faut tourner
(vers le gnrateur) pour passer du point A (y = ) au point B {y = - j fc3). Ici
s/ X = 0,354 - 0,250 = 0,104.
b) Point 3'

Le stub doit tre plac une distance d'/X. (Ici d '/X = 0,042 + 0,33 = 0,372).
Ladmittance y (d' - e) tant 1 - j 6 3, le stub doit avoir une longueur s telle
que v (.v) = + j i >3 (ici + j 1,3). s est dtermin en lisant sur le bord du dia
gramme de combien il faut tourner (vers le gnrateur) pour passer du point A au
point /?(<=> 0 + j i>3). Ici s'/X = 0,250 + 0,146 = 0,396.

5.6.2 Adaptation deux stubs (fig. 9)


Il faut dabord placer les points 1 et 2 reprsentatifs des impdance et admit
tance rduites de la charge et dduire de 2 , par une rotation de d J X vers le gn
rateur (ici d J X - 0 , 1 2 2 ), le point 3 reprsentatif dey (dl - e).
Nous avons vu que les parties relles de y (d j + e) et y (d j - e) taient les
memes. Un lieu de v ( d x + e) est donc le cercle partie relle constante (ici,
g l (d - e) = 0,3) passant par 3.
De mme, les parties relles de y (d + e) et y (d - e) tant identiques et gales
1, le cercle g = 1 est un lieu de y (d - e). Il est facile den dduire un lieu de
y (r/, + e) car, dans un dplacement le long de la ligne, le point O reste
invariant ; il suffit donc de trouver le point A' qui se dduit de A par une rotation
de d2/X vers la charge (ici d2/X = 0,180) et lon peut alors tracer le cercle de
diamtre OA' qui est un deuxime lieu de y (dt + e).
Les points 4 et 4 qui se trouvent lintersection des deux lieux de y (d{ + e)
vont nous permettre de calculer les longueurs possibles i j et s jdu premier stub.
Nous avons, en effet, puisque :

v(d, + e )= y (r/ 1 -e ) + ;y(s1)


y U'i ) = 3V >3 et y (5i) = >'4 - 3>3
Il suffit de lire sur le bord du diagramme, en tournant vers le gnrateur, quelles
sont les distances s , /X et sJ/A. quil y a entre le point A et les points B' et B"
reprsentatifs de y H ) et y (vj).

Point 1

z, = 0 ,2 6 + j 0,3

Point 2 :

y2 = 1,65 - j 1,9

dy/X = 0,122 Point 3 : y3 = 0 , 3 - j 0,5 ; d2/X = 0,1 8 0 P o in tA .


Point 4 : y4- = 0,3 - j 0,06. Point 4 : y4- = 0,3 + j I
Point B : y (s, ) = j 0,44 > s\/X = 0,3 1 6
Point B : y (.s j) = j 1,5 > s\'/X = 0,406
P o in t5 : yy = I + j 1,28
Point C : y(.s2) = - j 1,28 >s'^X = 0 , 105
Point 5 : y5.. = 1 - j 2,2
Point C : yOtf) = j 2 ,2 -> s2/X = 0,432

Dans cet exemple y ( , ) = j 0,44 s \ / X - 0,250 + 0,066 = 0,316 et


y (*;) = j 1,5 -> ,v|A = 0,250 + 0,156 - 0,406.
Les points 5 et 5 qui se dduisent de 4 et 4 par la rotation de d2A vers le
gnrateur reprsentent les deux valeurs possibles de y (cl - e) et appartiennent
au cercle g = 1 .
y5. =

+ j by et y5" =

+ j b y.

Les longueurs du second slub qui peuvent conduire ladaptation finale sont
donc s2 et .v2 telles que : y (s2 ) = - j by et y (s ) = - j b ^ . (Ici, y (s2 ) = - j 1,28
et y (.s'2 ) = j 2 ,2 ).
A ces valeurs correspondent les points C et C sur les bords du diagramme ; on
en dduit s2 et s"2.
Dans cet exemple y ( s2) = - j 1,28 s 2/ X = 0,355 - 0,250 = 0,105 et
y (.s2 ) - j 2,2 -> .v4,k = 0,250 + 0 ,182 = 0,432.

5-7

A dapta tio n par r s e a u d ' im p d a n c es


ET TRONON DE LIGNE

Figure J0.
Adaptation par rseau
dimpdances
et tronon de ligne.

<
Dans les cas que nous avons traits aux paragraphes prcdents, il sagissait
dadapter une charge dimpdance ZH complexe un gnrateur dimpdance
interne ZG relle et gale l'impdance caractristique de la ligne qui le relie la
charge.
Le cas le plus gnral est celui o lon veut adapter une charge dimpdance Zr
complexe un gnrateur dimpdance interne Zc complexe (Fig. lOy

Pour effectuer cette adaptation, on peut placer en srie avec le gnrateur une
impdance imaginaire pure ZA = j X afin de compenser la partie imaginaire de
limpdance interne du gnrateur. Cette ractance peut tre obtenue en plaant
en srie avec le gnrateur soit un stub soit un composant passif inductif ou
capacitif. Ainsi, limpdance ramene aux bornes du rseau ! est Rc .
Dautre part, on place en parallle une distance de la charge une admittance
imaginaire pure YB = j B qui peut tre obtenue en mettant en parallle sur la
ligne soit un stub soit un composant passif inductif ou capacitif. Le rseau II
constitue lquivalent dun dispositif dadaptation un stub qui doit ramener
ses bornes une impdance gale 7?G-, ce qui ralise ladaptation dsire.
Notons enfin que lorsque la distance entre Z-; et ZR est impose, il est toujours
possible dutiliser, conformment au schma de la figure 3, deux dispositifs
dadaptation lun plac entre la charge et la ligne, lautre plac entre le gnra
teur et la ligne.

si

gueur f 2 = 15 cm et est plac une distance j


de ZK \v = c.

E X E R C IC E 5 . 1
(Paragraphe 5.3 - Adaptation par ligne
quart d'onde)

Une ligne de transmission est compose de


trois tronons comme cela est reprsent sur la
figure 11. Limpdance de charge ZK est consti
tue par une rsistance de 50 2 en srie avecune inductance de 2 10 8 H. Les deux tron
ons extrmes ont pour impdance caractris
tique Zc = 100 2. Le tronon central a une lon

z>2

1) La frquence tant de 500 MHz, dterminer


la plus petite valeur de t conduisant
ladaptation de lensemble et la valeur cor
respondante de Zt..
2) La frquence tant de 600 MHz, dterminer
z2 et le R.O.S. apparent de lensemble, si
lon ne retouche pas aux valeurs de Z[. et de
l trouves prcdemment.

z \ *1

Z 'c

Zc

<2
:
f,
---------------- ----------------- -*-----*
--J------

Zn

Figure JJ.

&

E X E R C IC E 5 . 2
(Paragraphe 5.2 - Conditions d adaptation)

Un gnrateur tiansistor a une impdance


interne Zc = (7,5 + j 11 ) 2 (voir lig. 12). Il est
reli une ligne de longueur et dimpdance

caractristique Zc qui est la moyenne gom


trique entre 50 2 et la partie relle de Zc . La
charge est un dispositif dimpdance ZR = 50 2
avec lequel est mis en srie un condensateur
dadaptation, de capacit C. On demande de
calculer et C pour que ladaptation soit ra
lise la frquence de 2,45 GHz.

Figure 12.

&

E X E R C IC E 5 . 3

&

E X E R C IC E 5 . 5

(Paragraphes 5.4 et 5.6.1 - Adaptation


l'aide d'un stub)

(Paragraphes 5.5 et 5.6.2 - Adaptation


l aide de 2 stubs)

Une ligne dimpdance caractristique Zc - 5 0 (.1


est termine par une impdance ZR - (100 +
j 75) Q. La longueur donde est de 0,5 m.
Dterminer les caractristiques dune adapta
tion par un stub de longueur s, situ une dis
tance d de charge et dimpdance caractris
tique 50 Q.

Une ligne dimpdance caractristique 50 Q.


est termine par une impdance ZR = (100 j 125) Q.. La longueur donde est de 0,5 m. On
veut raliser ladaptation en utilisant deux
stubs en court-circuit dimpdance caractris
tique 50 Q. Le premier est plac 6 cm de la
charge et le second est 15 cm du premier.
Dterminer les longueurs des stubs qui condui
sent ladaptation.

&

E X E R C IC E 5 . 4
(Paragraphes 5.4 et 5.6.1 - Adaptation
l'aide dun stub)
&

Une ligne coaxiale dilectrique air, dimp


dance caractristique Zc = 50 El est termine
par une impdance ZR = (15 - j 42,5) ft. La
frquence est de 1 GHz.

E X E R C IC E 5 . 6

On veut raliser ladaptation en plaant, une


distance d de la charge, un stub en circuitouvert, dimpdance caractristique Z . = 100 Q
et de longueur s.

Une ligne d im pdance caractristique


Zc = 100 Q est termine par une impdance de
charge ZR 150 - j 120. On veut adapter cette
im pdance l aide de deux stubs courtcircuits : le premier en parallle sur la charge
et le deuxime plac X/ 8 en srie avec la
ligne.

Dterminer les caractristiques de cette adapta


tion.

Dterminer les longueurs a' j et s2 de ces deux


stubs qui conduisent ladaptation.

&

transmission de puissance optimale, on utilise


un circuit dadaptation (fig. 13) constitu dune
capacit Cs srie et dune capacit parallle Cp.

E X E R C IC E 5 . 7
(Paragraphe 5.7 - Adaptation par rseau
d impdances et tronon de ligne)

Un gnrateur dimpdance interne Zc = 50 Cl


doit alimenter un amplificateur transistor dont
limpdance dentre est Ze = (4 + j 2) Q, la
frquence de 600 MHz. Afin de permettre une

Entre ce circuit et lamplificateur, il y a une


ligne microbande dimpdance caractristique
Zc = 40 Q, de longueur = 2 cm sur laquelle
X = 25 cm. Trouver les valeurs de Cv et de Cp
pour que ladaptation soit ralise.

------------ ! !

cs
i i
4 + j 2

i i
i i
i i

Figure 13.

Chapitre 6
Lignes awec pertes

E t u d e du p a r a m tr e

6 /1

DE PROPAGATION ET DE L'IMPDANCE
CARACTRISTIQUE

6.1.1 Expression de ol, et Z c


Dans le chapitre Propagation sur une ligne , nous avons montr que lexpres
sion du paramtre de propagation y tait : y - ^ J ( R ] +j L] to) (Gj + j C, oj)
Cest une quantit complexe que nous avons crite sous la forme : y = a + j p.
Il est, ds lors, facile de calculer les expressions de a et P en remarquant que :
dune part :
y2 - a 2 - P2 + 2 j ap
dautre part : ^ = (7?! G, - L , C, (2) + j to (Lj G, + /?, C,).
Ea identifiant :
et2 p2
: , G, - L j C, co2
2 txP =

( (Lj G, + /?] Cj)

do :
a = V I [\A *I + L 1 <2)(C? + C \ (2) +{Rl Gi - L l C l co2)J

(1 )

P = sj~~ [ v ^ + Z-i'co2) (G 2 + C 2 to2) - (Ri C, - L x C, m2)]

(2)

R\
Pour ce qui est de Zc dont lexpression est :
lcrire sous la forme :
/? , C | + L , C [ co

Z,.=
:+

+j

L x co

C j + j C | co

-Hj c o (/-1 G [

nous pouvons

- R x C ])

C\ co2

(3)

Les expressions trouves pour a , (i et Zt. sont gnrales, mais difficilement utili
sables. Elles se simplifient beaucoup dans le cas des lignes faibles pertes qui
est trs important pour les applications, notamment en tlcommunications.

6.1.2 Cas des lignes faibles pertes


n
V
Il est alors possible de ngliger a et /?, G, ; nous avons :
- P 2 = - L , C ] co2
2cxP = co (L , G , +

Do:

Rt C ,) .

P = co^/Ll Cl

(4)
(5)

et :

( 6)

Notons que v est indpendante de la frquence : un signal se propagera donc sur


la ligne sans distorsion (voir 6 .2 .2 )
Daprs (3) et en ngligeant R { G j et G2, nous obtenons :
R,
Zn=\ F 5 - \ /T+7 ------------J ------C( co
L y co

(7)

De plus, si les pertes dans le dilectrique sont ngligeables par rapport aux
pertes dans les conducteurs, on pourra supprimer dans (6 ) et (7) le terme en G|.

6.1.3 Minimisation des pertes


Pour que a soit minimale, il faut que dot2/dLj = 0.

G? + C? co2
Soit :

R \ +L \

co2

L|

co2 C j co2 0

De ceue relation, on dduit la clbre condition de Heaviside :

alors :

L\ C, = Ri C,

( 8)

a = yj R i G,

(9)

3 = < jL x Cx (4)

6.2

et

Importance

de la condition

d 'H eaviside

- M oyens

de la raliser

Voyons combien la ralisation de cette condition est importante propos dun


exemple simple : la transmission dun signal radiolectrique reprsentant la voix
humaine dans une liaison tlphonique par cble.
A

Figure I.
Courbe amplitudefrquence dun signal,
appele spectre de
frquences .

On sait qu un instant donn, ce signal a un spectre de frquences bien dfini ;


cest--dire que si lon trace la courbe amplitude-frquence qui lui correspond,
nous trouvons des frquences/,,/ 2 , / 3 ... des amplitudes qui ont respectivement
pour valeurs A ,,A 2, A3... (fig. 1).

6.2.1 La distorsion d'amplitude


Si lamortissement d la propagation est indpendant de la frquence, nous
avons la rception des amplitudes A \, A2, A3 ... correspondant respectivement
aux frquences/,,/,,/}... et A,/A , = A2/ a 2 = A3 /A 3 = ...

Autrement dit, les amplitudes relatives sont conserves et le spectre de fr


quences du signal la rception est le mme qu lmission.
Si, en revanche, laffaiblissement a dpendait de la frquence, la prcdente
relation ne serait plus vrifie et le spectre de frquences du signal la rception
ne serait plus le mme qu lmission. La consquence pratique en est une
dformation de la voix humaine. Dans un tel cas, on dit quil y a distorsion
damplitude .
Pour combattre la distorsion damplitude, dont aucun cble nest rigoureusement
exempt, il faut disposer larrive des filtres de frquences du type passe-bande
afin de dcouper le signal par bandes de frquences, puis amplifier chacune de
ces bandes de telle sorte que le coefficient damplification A dpende de la fr
quence et vrifie la relation : a (/) A (J) = cte V /.

6.2.2 La distorsion de phase


Le fait que la vitesse de propagation soit indpendante de la frquence implique
que les diffrentes frquences composant un signal mis un instant t seront
toutes reues au mme instant ce qui est la condition ncessaire la reconsti
tution dun spectre de frquences identique celui du signal mis pourvu, bien
sr, quil ny ait pas eu distorsion damplitude ou quelle ait t corrige.
Mais si v dpendait de la frquence, cela signifierait que les composantes du
spectre de frquences reconstitu un instant t' nont pas t mises au mme
instant. Il en rsulterait une impossibilit de reconstituer le spectre de frquences
correspondant au son mis linstant t do une inaudibilit totale de la voix
humaine. Cela sappelle la distorsion de phase . Elle est beaucoup plus grave
que la distorsion damplitude car les moyens mettre en uvre pour la com
penser (disposer dans chaque bande de frquences des lignes retard variables)
sont beaucoup plus onreux.

6.2.3 Moyens de raliser la condition


d'Heaviside
Pour les communications tlphoniques courte, moyenne et longue distance
par cbles urbains, interurbains et intercontinentaux ou transocaniques, les
cbles utiliss sont, en gnral, tels que L l Gj R l Cl . Des procds bien
prouvs sont employs pour satisfaire la condition dHeaviside en augmentant
Lj : ce sont la pupinisation et la krarupisation .
a) La pupinisation

Elle est dun usage courant sur les lignes tlphoniques terrestres, urbaines et
interurbaines. Elle consiste augmenter L l en moyenne en intercalant inter-

valles rguliers des inductances en srie avec la ligne ; en France, lespace est de
955 m ou 1 830 m. Tant que lon napproche pas trop des frquences pour les
quelles X serait de lordre de lespacement d des bobines de charge, tout se passe
comme si lon avait augment L { de manire continue. En fait, la thorie montre
que la condition de validit de ce procd est : X > nd.
b)

La krarupisation

Elle est utilise pour les cbles tlphoniques intercontinentaux ; elle consiste
enrouler autour du ou des conducteurs de la ligne, un ruban de permalloy de 0 ,2
0,3 mm dpaisseur en une spirale aussi serre que possible.

6.3

E xpressions de la tension
DU COURANT ET DE L'iMPDAN

Au chapitre 3, nous avons tabli les expressions de la tension, du courant et de


limpdance en un point dune ligne ferme sur une charge Zr caractrise par
son coefficient de rflexion T R = TR eJ<p. Dans le cas o lorigine est prise sur
cette charge, axe des x orient vers le gnrateur :
V(x) = V( (e ^ + r e - ^ )
(1 0 )
L0Q = L i ( f i y x - L R e - yx)

(H)

1 + E e - 2Y*
Z(x) = Zc -------------
I - r ^ e - 2**

(12)

Limpdance en valeur rduite est donc :


i + r ^ e - 2^
z(x) =

(13)

En prenant un argument complexe 0 = 0 + j 0 tel que :


- e 26 = r e *
1 e 2e 2Y*
2 (x) = -------- =
1 + e _2e e_ 2Y*

th (0 + yx)

(14)
(15)

Nous avons donc mis z(x) sous une forme particulirement simple. Il en est de
mme pour V(x) et I(x) lorsque lon remplace E R par - e
V(x) = Vj (eyx - e " 2e e _YX)

(16)

L(x) = Ij (eyx + e 26 e ~ Yx)

(17)

MlCRO-ONDfS

En mettant e

en facteur, nous obtenons successivement :


V(x) = V,. e 6 (e(e + yx) - e ~ (e + yx))
Z(x) = Z, e " 6 (e(e + ^ ) + e (e + ^>)

Soit finalement en amplitudes complexes :


V(x) = 2 V,- e e sh (6 + yx)
/(x) = 2 /, e 6 ch (6 + yx)

(18)
(19)

(20)
(2 1 )

et en valeurs efficaces :

6 -4

VCx) = v/2 Vf | e | | sh (0 -t- yx) | = A | sh (0 + yx) |

(22)

/(x) = v/2 /; | e _ e | |c h (0 + yx)| = |c h (0 + yx)|

(23)

tude

d e s v a r ia t io n s d e l a t e n s io n

ET DU COURANT
Nous mnerons cette tude partir des expressions (22) et (23) et prendrons
comme exemple pour le calcul, la tension. En nous rappelant que : 0 = G + j G
et y = a + j P, nous avons successivement :
sh (0 + yx) = sh [(G + ou) + j (0 + P*)]
= sh (G + ou) ch j (0 + Px) + ch (G + ou) sh j (0 + Px)
= sh (G + eu) cos (0 + Px) + j ch (G + eu) sin (G + Px)
|sh (G + yx)| = [sh2 (0 + eu) cos2 (G + Px) + ch2 (G + eu) sin2 (0 + px) ] 1 2
= [ {sh2 (G + ou)} {1 - sin2 (0 + Px)} + ch2 (0 + eu) sin2 (G + Px)]1- 2
= [sh2(0 + eu) + sin2 (G + px) ] 1/2
Soit:

De mme :

V(x) - A sJ sh 2 (G + ou) + sin2 (G + px)

/(x)

A-

\J sh 2 (G + ou) + cos2 (G + Px)

(24)

(25)

6.4.1 Cas gnral


Lorsque
est--dire G = G + j 0 est quelconque, ltude des variations de la
tension peut se faire en remarquant que V(x) est la somme :
dune fonction priodique sin2 (0 + Px)
dune fonction monotone sh (0 + ou).

V(x) pourra elle-mme tre considre comme une fonction priodique condi
tion que la variation du sh2 soit assez faible pendant que le sin2 varie entre un
maximum et un minimum. Cela ncessite que la ligne ne prsente pas de trop
grandes pertes. Dans ces conditions, nous pouvons dire avec une bonne approxi
mation (voir fig. 2 ) que :
la priodicit des phnomnes est X/2 ;
les valeurs des maxima et des minima sont obtenues pour les valeurs de x
telles que sin2 (0 + P x ) = 1 ou 0. Soient xM et x m ces valeurs :
VM = A ch (0 + OCjyj)

(26)

Vm = A sh (0 + tm)

(27)
Figure 2.
Courbe des variations
de la tension le long
dune ligne avec
pertes : les enveloppes
des maxima et des
minima sont en
A ch (0 + eu) et
A sh (0 + eu).
a

Pour /(x), le problme est identique. Les seules diffrences dans les rsultats
consistent en ce que :
les amplitudes sont divises par Zc ;
les variations sont en opposition de phase, puisque dans (25), cos2 (0 + px)
remplace le sin (0 + Px) de (24).

6.4.2 Cas particuliers


Nous ne reviendrons pas sur le cas o la ligne est termine par son impdance
caractristique puisque ltude du rgime dondes progressives qui stablit alors
a t effectue au chapitre 2 Propagation sur une ligne en haute frquence en
tenant compte du paramtre daffaiblissement a. Restent donc tudier les cas
o E * = + 1 et
= 1. Nous allons dterminer dans chacun de ces cas les
valeurs de 0, 0, 0 et voir comment scrivent les relations (22), (23), (24), (25).
a) Ligne en court-circuit ZR = 0 => R = - 1
Donc :
Ou:

- e -2e = - l = > e ~ 2e = l .
e - 2 6 = e - -*2te => 0 = j ht.

Comme : 0 = 0 + j 0, il vient : 0 = 0 et 0 = kn.

(22)

(23)
II

'S?
sr

(24)

V(x) A | sh (j kn + yx)| = A | sh yx|

(28)

A
A
Kx) = y - 1Ch (j kn + yx) | = y | c 1h Y * |

(29)

>

(30)

/s h 2 a x + cos2 (kn + (3*) =

(31)

(25)

ii
4 *1*

J sh 2 o x + sin 2 (foc + px) =A \J sh 2 a x + sin 2 px

zc

y j sh 2 a x + cos2 [3x

Daprs ces relations nous avons :


sh ax

(33)

/ s
y sh ax

(35)

(34)

V (x = 0 ) = 0

(36)

Il

(32)

v M -= A ch ax
A u
~ -- ch eux

(37)

b) Ligne en circuit o u v ert Zw = oo => r w = +


e

Donc

26

=> e 26 _ - i .

e " 2e = e -j(2/r+1)71^0 = j (2k + 1 ) tc/ 2

Ou :
Comme :

= 0 + j 6 , il vient 0 = 0 et 0 = (2k+ 1 ) tc/ 2 .


TC
V(x) = A sh j (2 /c + 1 ) 2 + 7 - = A | ch yx

(2 2 )

(23)

>

(24)

(38)

ch j ( 2 * + l ) | 4 yx = y |s h y x |
TC

V(x)=A\ / sh2 ax + sin 2 (2k + 1 ) j + Px

S)| S
+
?

j sh 2 ax + cos2 (2k

(40)
+

>

4*|*>

^sh 2 OX+ cos2 px


(25)

(39)

(41)

^sh 2 ax + sin2 px
Daprs ces relations nous avons
Vm = A'ch ax
A K
^M= yz c. ch ax

(32)

sh ax

(33)

(34)

(35)

V (x = 0) = A

(42)

/ =A
y sh ax
^C
1 (x = 0 ) S 0

(43)

Figure 3.
Courbe des variations
de la tension le long
dune ligne avec
pertes court-circuite.

Figure 4.
Courbe dej variations
de la tension le long
dune ligne avec
pertes en circuit
ouvert.

Les courbes ci-dessus montrent les variations des amplitudes de la tension pour
une ligne en court-circuit (fig. 3) et pour une ligne en circuit ouvert (fig. 4).
Dans les deux cas, les variations de la tension sont comprises entre des courbes
en ch eue et sh ouc. Pour ce qui est des variations du courant, la figure 3 corres
pondrait au cas dun circuit ouvert et la figure 4 au cas dun court-circuit.

6.5

V a r ia tio n s d e l' im p d a n c e
ET DU COEFFICIENT DE RFLEXION

6.5.1 tude de l'impdance


Daprs (20) et (21 ) :

Z{x) =

V(x)
IM

sh (0 + va)
=zr
r
ch (0 +yx)

V(x)
\/s h 2 (0 + eu) + sin 2 (0 + Rx)
Daprs (24) et (25) : Z(x) = ~ T = ZC
------- U --------------/w
\J sh 2 (0 + ou) + cos2 (0 + p.v)

(44)

En faisant la mme approximation quau paragraphe 6.4.1 nous pouvons remar


quer que :
* (45) est maximale lorsque sin2 (0 + Px) = 1 (et alors cos2 (0 + Px) = 0)
do

v/T + s h 2 (0 + oa)
ZM = Zc. -j=~---------- = Zc coth (0 + ocx)
\J sh 2 (0 +oex)

(46)

(45) est minimale lorsque sin2 (0 + px) = 0 (et alors cos2 (0 + Px) = 1 )
do

x/sh 2 (0 + ocx)
Z _ Zc X =
------ = Z th (0 + ocx)
m
^ / l + s h ^ +orx)

(47)

Les variations de limpdance seffectuent donc, comme le montre la figure 5, entre


les courbes reprsentatives de Z( th (0 + ou) et de Zc coth (0 + ocx). Notons qu
mesure que x augmente th (0 + ocx) tend vers coth (0 + ocx) ; il en rsulte que
Z(x) >Zt.. Le rle des pertes semble donc, grande distance, d adapter la ligne ;
c est videment un procd dadaptation inadquat.

Figure 5.
Courbe des variations
de limpdance le long
dune ligne avec
pertes : les enveloppes
des maxima et des
minima sont en
Z(. coth (0 + ou)
et Zc th (0 + eu).

6.5.2 tude du coefficient de rflexion


Nous avons vu que :

V,. e ~ 7-v
.
T(x) = -----------= P e J<Pe
v ^ x

Soit :
le module de T(x) est :
largument de E(x) est :

E (x) = T* e 2cu e ** ~ 2Pv)


e ~~2cu
tj) - 2Px

(48)

La courbe reprsenta
tive des variations de
E(x) dans le plan com
plexe (fig. 6) est donc
une spirale logarith
mique qui coupe tous
les rayons vecteurs
sous le mme angle et
s enroule asym ptoti
quem ent autour de
lorigine lorsque x aug
mente indfiniment.

6 .6

Figure 6.
Courbe des variations
du coefficient de
rflexion T(x) dans le
plan complexe.

P u is s a n c e t r a n s p o r t e par u n e lig n e

6.6.1 Relation donnant

P (x )

Nous savons quen un point quelconque dune ligne, dans le cas o lorigine des
abscisses est prise au rcepteur et laxe orient du rcepteur vers le gnrateur :

VU)= y (. ( e ^ + r ^ e - ^ )
Z(x) = Z(- (e ^ - r * e - ^ )
Par dfinition, la puissance active en ce point est donne par :

n * ) = ^ 9 U Y ( * ) - /V ) ]

P(x) = 2 Z

Soit :

9t (eY* + r R e - r*) (eY* -

e - **)*

(49)

Le terme entre crochets scrit successivement :


[ea( e J

e - a( e M ~ PA>] [e eJ Px - r R e ~ c e W " PA)]*

[eav eJ P'v + T R e~ -x e** - PA)] [ea( e


PA- TR e a( e " M " Pa)]
e2cu + T* e j(<>- 2Pa) - r ^ e ' j (<t>' 2PaJ - T%e ~ 2cu

dont la partie relle est :


Do :

e2ou- TRe ~ 2ax


V2
P{X)=2V (e2aX- r e_20W)

(50)

6.6.2 tude de la perte de puissance


par transmission
lentre de la ligne o * = t , la puissance fournie par le gnrateur est :
P((') = Y z (e2ai - T l e - 2at)
lextrmit charge o v= 0 . la puissance absorbe par la charge est :
V-2
Pi0) = 2
Do la perte de puissance par transmission
Ar=m =
{ ~ vr
P (0 e2e - rAi e

e -2ae

l - r ^ e ~ 4trf

(51)

Ltude de K est dlicate faire car on ne peut sparer compltement linfluence de


la ligne et linfluence de la dsadaptation. En effet, K est le produit de trois termes :
lun, e _ 2cxt qui reprsente uniquement linfluence de la ligne ;
lautre, 1 - r jq u i reprsente uniquement linfluence de la dsadaptation ;
le dernier,------ --------- dans lequel les deux influences sont lies.
l - r ^ e - 4^
Les trois cas considrer sont les suivants :
1. Si a. 0 r K l .
Cela signifie que la puissance est la mme en tous les points de la ligne
comme le montre dailleurs la relation : P(x) = (V~ j l Z ^ (1 - r ^ ) qui est
indpendante de x et ne dpend que de r*.
Rem arque

Attention K est un
rapport de puissances
et a est en Np/m.

2. Si r R = 0-> K = e ~ 2(xe.
Il y a sur la ligne une perte qui est exprime :
- en Nepers par :
(1/2) In K = a
(52)
- en Dcibels par :
10 log K 8 ,6 8 a f
(53)
* 0, il est intressant de remarquer, par
3. Dans le cas gnral o a 0 et
rapport au cas prcdent, que linfluence de la dsadaptation se traduit par
lapparition dans K dun facteur :
l -r*
T

l ft c

4otf

(54)

Les pertes totales, dans ce cas, sont donc la somme :

des pertes propres la ligne, dues linfluence de a seul, qui apparaissent


dans le facteur e ~u< de la relation (51 ),
des pertes causes par la dsadaptation, dues laction conjugue de a et r *
qui apparaissent dans le facteur (54) suscit.

6.6.3 tude de la perte de puissance


par dsadaptation
Lexpression (54) reprsente laugmentation de la perte due une mauvaise
adaptation de la ligne. Labaque de la figure 7 permet de connatre directement,
en dcibels, cette perte par dsadaptation. Le R.O.S. p, associ la charge, est
port en abscisses et les pertes propres la ligne en ordonnes. Laugmentation
de perte due la dsadaptation de la ligne se lit sur les courbes cotes.
Par exemple, nous allons calculer la perte totale dune ligne de 20 m, daffaiblis
sement linique a = 0,05 dB/m, dimpdance caractristique 320 2, ferme sur
une rsistance de 40 2. Nous avons oc = 0,05 20 = 1 dB et p = 320/40 = 8 . Le
point dabscisse 8 et dordonne 1 est situ sur la courbe 2. Cela signifie que la
dsadaptation de la ligne fait perdre 2 dB. Comme les pertes propres la ligne
sont de 1 dB (ocf? = 1 dB), il en rsulte que la perte totale sera de 3 dB.

Figure 7.
Abaque donnant
la perte par
dsadaptation dune
ligne pertes
dsadapte ; a f (dB)
est la perte propre de
cette ligne et p est le
R.O.S. de la ligne.
D aprs Roubine 125]
Lignes et antennes
p. 136, Editions de la
Revue d'Optique,
Paris (1954).

&

E X E R C IC E 6 .1
(Paragraphe 6.1.1 - tude du paramtre
de propagation)

Dmonstration des expressions (1) et (2) de a


et (3 dans le cas gnral o lon ne fait aucune
approximation.

&

E X E R C IC E 6 . 2
(Paragraphe 6.1.2 - Cas des lignes
faibles pertes)

1) Dans lhypothse des lignes faibles pertes,


dmontrer lexpression de Zt. en fonction de

\>j, Gj, Cj.


2) Si de plus G, = 0, montrer quil est possible
dexprimer Zt en fonction de LJt C, et a,
P-

&

E X E R C I C E 6 . 3 __________ ___

(Paragraphe 6.6.2 - tude de la perte de puissance


par transmission)

Pour une ligne avec pertes adapte, de longueur


, la perte relative de puissance par transmis
sion est : K = P(0)/P() = e ~ 2at.
Exprimer K en Nepers et en Dcibels.
Dmontrer la relation de passage des Nepers
aux Dcibels.

E X E R C I C E 6 .-4
(Paragraphe 6.6.3 - tude de la perte de puissance
par dsadaptation)

Calculer la perte totale d une ligne de lon


gueur = 10 m, daffaiblissement linique a =
0,1 dB/m, d'impdance caractristique Zc =
300 D, ferme sur une antenne dimpdance
dentre ZR = 75 Q.
Mme question si Zc = 15 Q. et ZR - 300 2.

Chapitre 7
Lignes
en rgim e impulsionnel

7 /1

I n t r o d u c t io n

Dans les prcdents chapitres, nous avons toujours fait ltude des lignes en
rgime sinusodal. Dans ce chapitre, nous allons nous intresser aux phno
mnes qui se produisent sur une ligne avec ou sans pertes en rgime impul
sionnel, cest--dire lorsque la tension applique lentre de la ligne varie de
faon quasi instantane de 0 une valeur constante E, avant de revenir aprs un
intervalle de temps T, galement de faon quasi instantane, la valeur 0. Si X
est nettement plus petit que le temps de propagation sur la ligne tudie, nous
dirons quil sagit dune impulsion de tension et, dans le cas contraire, dun
chelon de tension.
Cette tude est trs utile car lemploi des impulsions en hyperfrquences, en UHF
et en VHF permet beaucoup dapplications pratiques dont les principales sont :
la radionavigation, avec la plupart des radars ;
- les tlcommunications o lon utilise de plus en plus des systmes de modu
lation par impulsion ;
les mthodes de mesure des caractristiques dune ligne.
Cette tude est galement trs intressante dun point de vue pdagogique. La
comprhension physique des phnomnes est, en effet, beaucoup plus aise en
rgime impulsionnel o lon peut distinguer (et au besoin visualiser) facilement
les ondes incidentes et les ondes rflchies quen rgime sinusodal o lon ne
peut sparer Fonde incidente de Fonde rflchie que par la pense.

7 -2

t u d e en r g im e d ' im p u lsio n
d e t e n s io n

Figure 1.
Impulsion le tension
d amplitude E,
de dure T.

Limpulsion peut se dfinir


comme un signal (lectrique,
dans cette tude des lignes)
ayant la forme indique la
figure 1 et dont la dure x est
trs faible vis--vis des temps
de propagation considrs.
Cest ainsi que pour une ligne
de 10 0 m de long sur laquelle
la vitesse de propagation est
de lordre de 2 0 0 0 0 0 km/s, un
signal lectrique ne pourra tre
vraiment considr comme une impulsion que si : x < T = 100/2 108 = 0,5 fis.

Figure 2.
Gnrateur
d im pulsions, ligne de
longueur et
im pdance de charge.

Considrons une ligne dimpdance caractristique Zc, de longueur , termine


par une charge dimpdance ZR (fig. 2). Nous supposons que cette ligne est ali
mente par un gnrateur dimpulsions de force lectromotrice E et dimpdance
interne ZG. Nous allons tudier deux cas fondamentaux.

7.2.1 Gnrateur adapt la ligne -

ZR

relle

Cest le cas o Zc = Zc ; le coefficient de rflexion la jonction ligne-gnrateur


ZR - Z C
est donc nul. En revanche, au niveau de la charge : r,, = ^ 0 puisquen
Zd+
gnral : ZR ^ Z( .

Figure 3.
R seau quivalent
linstant / = 0.

La force lectrom otrice du gnrateur


d impulsion tant fixe, l amplitude des
impulsions quil va dlivrer lentre de la
ligne dpend de limpdance qui charge ce
gnrateur. Or, linstant t = 0 o une
impulsion arrive lentre de la ligne, on
peut considrer que limpdance de charge
du gnrateur est Zc (fig. 3). De 0 2T,
instant o limpulsion rflchie sur la charge ( linstant T) reviendra sur le

gnrateur, tout se passe comme si ce dernier tait ferm sur une ligne infiniment
longue dont limpdance en tous points peut tre considre, nous lavons vu,
comme gale Zc.
linstant t 0 , nous avons donc, lentre de la ligne, une impulsion de ten
sion damplitude :
Z t.

V = E

( 1)

z,. + zr

Puisque nous tudions le cas o Z(. = ZG-, Ve E/2. Cette impulsion va se rfl
chir sur la charge linstant T t j v et retourne vers le gnrateur, quelle atteint
linstant 27, avec une amplitude r A>E/2. Elle ne peut se rflchir lentre de
la ligne puisque r G = 0 .
En branchant un oscilloscope aux bornes dentre de la ligne, nous observerons
limpulsion aller (t - 0) damplitude E/2 et limpulsion retour (t = 2T) dampli
tude TK E/2, de polarit positive ou ngative selon que 1^ est positif ou ngatif
(lg. 4).

Figure 4.
Phnomnes
observables avec un
oscilloscope lentre
de la ligne dans le cas
o le gnrateur est
adapt.

Rem arque

Les cas particuliers intressants sont ceux o :


a) ZR =
b) ZR = 0
c) ZR = Z(.

Alors,
= + 1. Limpulsion rflchie est positive, de mme
amplitude que limpulsion incidente.
Alors, VR = - 1. Limpulsion rflchie est ngative, de mme
amplitude que limpulsion incidente.
Alors, r R = 0. Il n'y a pas dimpulsion rflchie.

7.2.2 Gnrateur dsadapt -

ZR

relle

Dans ce cas une impulsion se propageant sur la ligne pourra tre rflchie tant
(
Z -Z ,.
au niveau de la charge :
*0
R ZR + Z,

Lorsque l on a une
ligne avec pertes, les
impulsions rflchies
dans les cas a) et b)
ont une amplitude Vr
plus petite que celle
Vj de limpulsion inci
dente. Cela permet de
dterminer les pertes
de la lign d'aprs r
aifdB) = 20 log fV,/
VrJ ou a(Np) ln
fVArJ-

que du gnrateur :

Z -Z r
zG+ zc

linstant t = 0 , nous avons lentre de la ligne une impulsion de tension


damplitude :

zt.

V = E ----- = E
e
Z + Z,

( 2)

est ici diffrent de E /2 puisque ZG est diffrent de Zc.. Si ZG > Zc, E' < E/2 et
si ZG < z ,, e: > E / 2. Cette impulsion se rflchit sur la charge linstant t - T t t
revient lentre de la ligne avec une amplitude Tr E' ; linstant t - 2T, elle se
rflchit sur le gnrateur et repart vers la charge avec une amplitude r * r G fr
et ainsi de suite...
Il y a donc lentre de la ligne une succession dimpulsions damplitudes E,
VR r G E \ T \ T2g E' ... respectivement aux instants 0, 2T, AT... ; il y a de mme
au niveau de la charge une succession dimpulsions damplitudes e r* . e r 2* r 6,
E
r 2c ... respectivement aux instants T, 3T, 5T...
Comme r * r c est infrieur 1 en module, les variations de la tension Ve len
tre de la ligne reprsentent une succession dimpulsions damplitudes progres
sivement dcroissantes, toutes positives ou alternativement positives et nga
tives selon que r * r G est positif ou ngatif (fig. 5 et 6 ).
Remarque
Dans le cas o Ton a une ligne avec pertes, la dcroissance est plus rapide
puisque le facteur multiplicatif qui permet de passer d'une impulsion la sui
vante n 'est plus r R Tq mais f R CG e ~ 2a^.

Figure 5.
Phnomnes
observables avec un
oscilloscope lentre
de la ligne dans le cas
o le gnrateur est
dsadapt

<r*rc >o).

Kk
^flrG> o (- 0 ,8 )

2T
4T

7.2.3 Phnomnes observables en pratique


La description qui vient dtre faite des variations de la tension lentre de la
ligne repose sur deux hypothses :
a) Les impulsions utilises sont infiniment courtes en dure.
b) La mesure de Ve se fait exactement lendroit o seffectue la rflexion la
jonction ligne-gnrateur.
Or, ces deux conditions ne peuvent tre exactement ralises dans la pratique :
on utilise des impulsions qui, si courtes soient-elles, ont une dure x non ngli
geable ;
la mesure de Ve se fera le plus souvent une petite distance de la jonction
ligne-gnrateur, ct ligne.
La consquence en est quune partie de limpulsion incidente (celle qui vient
de la charge) et de limpulsion rflchie (celle qui repart vers la charge) peu
vent se superposer.

Figure 6.
Phnomnes
observables avec un
oscilloscope lentre
de la ligne dans le cas
o le gnrateur est
dsadapt

(r*rc <oj.

Figure 7.
Dcalage, au niveau
du point de mesure,
entre limpulsion
incidente et
limpulsion rflchie
sur le gnrateur.

Le schma de la figure 7 montre, en effet, que limpul


sion rflchie arrive au point de mesure alors que
limpulsion incidente nest pas encore termine. Il
suffit, pour cela, que la dure T de limpulsion inci
dente soit suprieure au temps t que met le front
avant de celte impulsion effectuer laller-retour entre
le point de mesure et le point o seffectue la rflexion.
Dans ces conditions, nous observerons loscillo
scope :
t = 0,
une impulsion simple damplitude ".
k t-2 T ,

f= 0
v

A t = AT

une impulsion double d amplitude


E r /f ( 1 + r c ), rsultant de la superposi
tion de limpulsion incidente dampli
tude E" r * et de limpulsion rflchie
damplitude E ER TC.

une impulsion double damplitude E'

r c ( I + r c ) ; etc.

Or, si nous avons toujours, en module : E'


(1 + r c ) > E F g Tc (1 + r c ) > ...
puisque lon passe dune impulsion sa suivante en la multipliant par r /t r O
nous navons pas forcment : E' > E TR (1 + r c ) car r R (1 + r c ) peut tre plus
grand que 1 , en module.

Figures 8 11.

Phnomnes
observables avec un
oscilloscope lentre
de la ligne dans
diffrents cas :
r c etr^>(H fig. 8),
rc > 0 et
r*<o (fig. 9),
. rc < 0 et

r^>0 (fig. io),

r c et r /e< 0( f i g . il).

Cela dpend des valeurs de VR et de r c . Il ne faudra donc pas stonner si la


premire impulsion observe est plus petite que la suivante. Les diffrents cas
possibles sont reprsents aux figures 8 1 1 .

7 -3

t u d e en r g im e d ' c h e lo n
DE TENSION

Un chelon de tension peut tre considr comme une impulsion dont la dure
est beaucoup plus grande que le temps de propagation aller-retour sur la ligne de
longueur f laquelle elle est applique (fig. 1 2 ).
e
E

Figure 12.
chelon de tension,
damplitude E.

t
0

Cest un signal particulirement intressant car la plupart des phnomnes


impulsionnels peuvent se ramener une superposition dchelons de tension :
cest ainsi, par exemple, quune impulsion damplitude E peut tre envisage
comme la superposition de deux chelons de tension dcals de x dans le temps,
le premier damplitude + E, le second damplitude - E.
Figure 13.
Gnrateur dchelons
de tension, ligne de
longueur t et
impdance de charge.

Considrons, comme au paragraphe 7.2 tude en rgime dimpulsion de ten


sion , une ligne dimpdance caractristique Zt. de longueur f, termine par une
charge dimpdance Z* (fig. 13). Nous supposons, cette fois-ci, quun gnra
teur de signaux, de force lectromotrice E, envoie cette ligne un chelon de
tension. Nous allons tudier trois cas fondamentaux.

7.3.1 Gnrateur adapt la ligne -

ZR

relle

Soit t 0, linstant o lchelon de tension est


appliqu lentre de la ligne. Durant linter
valle de temps 2T que le front de lchelon de
tension mettra aller sur la charge, sy rfl
chir et revenir lentre de la ligne, tout se
passe comme si le gnrateur avait entre ses
bornes une impdance Zc (fig. 14). La tension
lentre de la ligne sera donc gale :
V. = E

Zc
Z.. + Zn

E
2

(3)

Cest, par consquent, un front donde damplitude E /2 qui se propage vers la


charge o il se rflchit linstant t = T (fig. 15). Alors prend naissance une
onde rflchie damplitude
E 2 tandis quil existe toujours une onde inci
dente damplitude E< 2 .
Si ^ > 0 , les amplitudes des deux ondes se superposeront ; lamplitude
rsultante sera : (1 + | TR |) E j2 (fig. 16).
Si

< 0, les amplitudes des deux ondes se retrancheront ; lamplitude rsul


tante sera : (1 - | r A, |) E /2 (fig. 16).

Figure 15.
Propagation de
lchelon de tension
sur la ligne pour
0< t< T.
Figure 16.

Propagation de
lchelon de tension
sur la ligne pour
T <t < 2T.
Figure 17.
Phnomnes
observables avec un
oscilloscope lentre
de la ligne dans le cas
o le gnrateur est
adapt.
Arrive lentre de la ligne, londe rflchie ne pourra pas, son tour, se rfl
chir puisque r G = 0. partir de linstant 2T, la ligne sera donc charge la ten
sion (fig. 17) :
-+- 1r E
2

Les cas particuliers intressants sont ceux o :


a) ZR = Alors
= 1 : londe rflchie double la charge prise sur la ligne.
b) ZR = 0

Alors

= - 1 : londe rflchie annule la charge prise par la

li g n e .

c) ZR = ZC

Alors
= 0 : il ny a pas donde rflchie. La ligne est tout
entire charge la valeur E /2 ds que / = T.

7.3.2 Gnrateur adapt la ligne Z R imaginaire pure


Loriginalit de ce cas, par rapport au prcdent, tient ce que r * est fonction
du temps ; il faudra donc se demander quel est le rgime transitoire sur limp
dance de charge.
a) Charge capacitive : ZR = 1/jCw

Figures 18 et 19.
Rseau quivalent et
variation de la tension
en fonction du temps
aux bornes dune
charge capacitive
soumise un chelon
de tension.

Lorsquau bout de i = T, lchelon de tension damplitude E/2 arrive sur C, tout


se passe comme si, cet instant prcis, on constituait en bout de ligne le circuit
de la figure 18. Le condensateur va donc se charger exponentiellement la
valeur E /2 avec une constante de temps T = C Z(. (fig. 19).
lextrmit de la ligne, il y a, t = T, un courant instantan de charge, de
valeur IR = E /2 Z.c et, comme la charge commence juste, VR = 0. Donc :
r R ( T ) = - 1. Tout se passe comme sil y avait un court-circuit en bout de
ligne. En fin de charge, 1R = 0 et VR - E/2 donc ZR = : le condensateur est
quivalent un circuit ouvert. Alors
= 1. Dans le rgime transitoire, le coeffi
cient de rflexion passe donc de - 1 + 1 au fur et mesure que la charge sef
fectue.
lentre de la ligne, entre 0 et 2T, la tension est E/2 (fig. 20).

ve k
E
Figure 20.
Phnomnes
observables avec un
oscilloscope lentre
dune ligne pour un
gnrateur adapt et
une charge capacitive.

E
2

linstant 27, il arrive une onde damplitude - E /2, ce qui fait que la tension
sannule comme si la ligne tait court-circuite. Mais, au fur et mesure que t
augmente, VR passe de - I + 1 avec une constante de temps gale celle du
condensateur. Dans ces conditions, lamplitude de londe rflchie crot avec la
mme constante de temps de - E j2 + /2 donc Ve crot de 0 + E.
b) Charge inductive : Z R = jito

Figures 21 et 22.
Rseau quivalent et
variation de la tension
en fonction du temps
aux bornes dune
charge capacitive
soumise un chelon
de tension.
Lorsquau bout de t = T, lchelon de tension damplitude E /2 arrive sur L. tout
se passe comme si, cet instant prcis, on constituait en bout de ligne le circuit
(fig. 21). Il va donc apparatre aux bornes de L une force lectromotrice dinduc
tion propre qui va sopposer ltablissement dun courant dans L. Pour t = T,
elle a pour valeur E/2 et elle va dcrotre ensuite exponentiellement pour san
nuler avec la constante de temps t = L /Z c (fig. 2 2 ).
A lextrmit de la ligne, nous avons pour t T : IR = 0, VR = E /2 do ZR .
Tout se passe comme si lon avait une ligne ouverte. Donc :
(7) = + 1. En fin
de charge, au contraire, VR = 0, donc Z* = 0 et r * = - i . Tout se passe comme si
lon avait une ligne court-circuite. Dans le rgime transitoire, le coefficient de
rflexion passe donc de + 1 - 1 .
lentre de la ligne, la tension est gale E /2 entre 0 et 2T (fig. 23). lins
tant 2T arrive une onde damplitude E/2 : la tension double comme si la ligne
tait ouverte. Mais, au fur et mesure que t augmente, TR passe de + 1 - 1

avec une constante de temps gale celle de linductance. Alors lamplitude


rflchie diminue de E/2 - /2 et la tension lentre de la ligne dcrot de E
0.

Figure 23.
Phnomnes
observables avec un
oscilloscope lentre
dune ligne pour un
gnrateur adapt et
une charge inductive.

7.3.3 Gnrateur dsadapt -

ZR

relle

partir de linstant (soit t 0 ), o lchelon de tension est appliqu la ligne,


nous avons vu quune onde incidente initiale damplitude E' = E Z(./(Z(. + Zc ) se
propageait du gnrateur vers la charge. La charge ayant un coefficient de
rflexion r K, il y prend naissance, r = T, une onde rflchie damplitude r R E
qui se propage vers le gnrateur. En arrivant sur le gnrateur, t = 2T. cette
onde donne son tour naissance une seconde onde incidente damplitude
r R Tc E' et ainsi de suite.
La diffrence entre ce rgime et celui dimpulsions de tension est que nous nous
intressons ici la propagation dondes qui se superposent les unes aux autres
alors que nous avions faire prcdemment des impulsions qui taient bien
spares les unes des autres. Pour faire une tude mathmatique du phnomne,
appelons en les affectant dun numro dordre :
OI, les ondes qui se dplacent vers la charge,
OR, les ondes qui se dplacent vers lentre de la ligne.
II

O I2 -

O R \ =
r R r G

t r

E '

o r 2 = r% r c

O f3 = ( r H r c ) 2 E '

O K 3 = r * T * E '

0/n = (r* r G ) " - 1

OR = r ;

- 1

Faisons la somme de toutes ces ondes :


011+ o r 1 = e ' ( i + r p
012 + o r 2 = E ' (i + rK) r R r G
013 + o r 3 =e {\ +rR) (rR rc )2

ln+ ORn=E(\+rR) (VRrc )"~1


H1
- r u + i v Z (rs rc )'

C est une srie gom trique de raison q =


= (! + r R) dont la somme lorsque n > est :

r G et de term e gnral

i + r
5=

(5)
rR rC

N
l

N
"

Z,.
Z( + ZG

O r:

\-q

Z R + Zc

et

F - ZC" ^
G Zc + Zt-

Nous trouvons :
S =E

Zr
Z r + Zq

( 6)

Tout se passe donc, en fin de compte, comme si le gnrateur tait directement


ferm sur ZR Physiquement, cela correspond au fait que lorsque / > les
rflexions aux extrmits de la ligne deviennent ngligeables ; le rgime transi
toire est alors termin et lon se trouve en rgime permanent continu pour lequel
la ligne tudie sert tout simplement connecter le gnrateur limpdance ZR.
En principe, le rgime transitoire dure un temps infiniment long puisque, tous
les 2T, il y a un accroissement de tension supplmentaire. En fait, le rgime per
manent stablit assez vite sur la ligne car, dune part les accroissements sont de
plus en plus faibles, dautre part les intervalles de temps TT sont trs courts (de
lordre de 1 ps pour une ligne de 1 00 m).
Avec un oscilloscope, nous observerions aux instants 2T, 47, 67... les variations
de la tension lentre de la ligne rsultant de la superposition la tension dj
existante dune onde de tension venant de la charge et de Tonde rflchie len
tre de la ligne laquelle elle donne naissance. Nous aurons ainsi successive
ment (fig. 24) :
deO 2 7 - :
de 27 4 7 - e : E' + TR E' + VR Tc = + r ^ d + Tc )
d e 4 7 6 7 - e : + r ^ ( l + Tc ) + T ^ r G + r ^ r 2G
= ET + r R (1 + r c ) + r 2 r G (1 + r G)

= + r* (i + r G) * [i + r R r c i
etc.

Figure 24.
Phnomnes
observables avec un
oscilloscope lentre
dune ligne pour un
gnrateur dsadapt
dans le cas o TG et
r* > o .

La tension la fin du rgime transitoire est la somme de et de la srie :

r*(i + rc) [i + rA>rc + r^r^+...j


dont le terme gnral est : r A( 1 + FG) , et la raison : r * r c .
La tension lentre de la ligne lorsque le rgime permanent est tabli est donc :
r,, ( i + rr )
- r r> + r ..+ rA>rr
1 + r A,
------ S - = E ----- ? - ---- *-_ = E ------- (7)

' r ro

' rro

' - rro

Cest bien l le rsultat qui avait t trouv, en sommant dune faon diffrente
la formule (5).
Dautres cas de figure que celui reprsent ci-dessus (qui correspond r R et
rG> 0 ) sont possibles selon que FA>et Tc sont tous deux < 0 ou lun > 0 et
lautre < 0. Il est vivement conseill au lecteur de tracer, pour chacun deux, les
graphiques Ve (t) titre dexercice.
Remarque
Tous les raisonnements de ce paragraphe ont t faits en supposant que l on avait
une ligne sans perte. Dans le cas d une ligne avec pertes, ils restent qualitative
ment valables ; dans les relations crites, on tiendra compte des pertes en ajou
tant un facteur multiplicatif e n< chaque fois qu'une longueur de ligne ( a t
parcourue par une des ondes considres.

Chapitre 8
Lignes bifilaires
et casa!es

8 .1

Pa r a m t r e s p r im a ir es

Nous avons expliqu au chapitre Propagation sur une ligne en haute


frquence - Modlisation de la ligne , limportance et le rle des paramtres
dits primaires /?j, L x, G 1 et C, dans la modlisation dune ligne. Pour les lignes
T.E.M. comme les lignes bifilaires et coaxiales - o les notions de tension et de
-

t
courant conservent un sens, car les champs E et H sont transversaux - ces
paramtres ont une signification physique et peuvent tre dtermins par des for
mules analytiques simples.
La dtermination de L { et Cj peut tre effectue partir des lois fondamentales
de llectromagntisme dans lapproximation des tats quasi-stationnaires ; en
effet, pour les lignes T.E.M., la rpartition du champ dans un plan transversal est
la mme que lon soit en rgime stationnaire ou en rgime variable.
Pour ce qui est de Aj et Gj, on doit tenir compte des phnomnes lis aux hautes
frquences : pour R {, il sagit de la localisation superficielle des courants dans
les conducteurs (effet de peau) et pour G x des pertes de nature conductrice dans
le dilectrique.
Divers ouvrages [22, 24, 26, 32] dtaillent les mthodes de dtermination de ces
paramtres et lexercice 8. 1 traite le cas important de la ligne coaxiale.

8.1.1 La ligne coaxiale

Figure J.
Ligne coaxiale :
gomtrie et lments
constitutifs.

Soient d } le diamtre du conducteur intrieur de conductivit Cj, d2 le diamtre


intrieur du conducteur extrieur de conductivit 0 2, r la constante dilectrique
relative et tg 5 le facteur de pertes du dilectrique.
Les valeurs par unit de longueur de la rsistance, de linductance, de la conduc
tance et de la capacit sont respectivement :
1

* ='

d2

= 0,632 10- y / "

(Q/m)
2

li 0 d
L , = ln - - = 0,2 10 6 ln - - (H/m;
d,
2 71

( 2)

Ef tg
n e ,/ 6
Cj = A%2 J = 0,349 10 9 ----- (S/m)
do
In
ln

C, =

= 0,055 10 " 9
In

U)

(F/m)

(3)

(4)

ln

8.1.2 La ligne bifiliaire


I
Figure 2.
Ligne bifilaire :
gomtrie.

_____________

Soient d le diamtre
des conducteurs de
la ligne et D leur
espacement daxe
axe.

Les valeurs par unit de longueur de la rsistance, de linductance, de la conduc


tance et de la capacit sont respectivement :

= 1,26- 10

(Q/m) (5)

jr)
Ho, 2D
L, = ln
= 0,40 10 _6 ln r (H/m)
n

( 6)

e rf tg 8
9 ------- (S/m)
c ' = 2 ,t2 Z l r = 0-1 7 5 1 0
2d
ln r
ln d
d
ne
= 0,028 10 - 9
(F/m)
C, =
2D
1 , 2D
ln
ln

3.2

Pa ra m tres

(7)

( 8)

secon daires

DE LA LIGNE COAXIALE
8.2.1 Affaiblissement
Nous nous plaons dans-le cas des lignes faibles pertes qui est videmment le
seul intressant en pratique. Laffaiblissement doit alors se calculer daprs la
formule (5) du chapitre 6 Lignes avec pertes et lon obtient :
a (Np/m) = y je fn (

n f tg

(9)

d2 \T < h J In
V d \j
Dans cette formule, il est facile de reconnatre que le premier terme reprsente
les pertes 0.c. dans les conducteurs et que le second reprsente les pertes O.j dans
le dilectrique. En explicitant numriquement e et v = 1/ \fe \x , nous obtenons :
a f (dB/m) = 45,8 10

1
U

i
n/ O,

( 1
do
d2 \ j G 2 j ln - l d\/

a d (dB/m) = 9 1 -1 0 ~9^ jT r f tg 8

( 10)

( 11)

8.2.2 Paramtre de phase


Pour le paramtre de phase [3, nous savons daprs la relation (4) du chapitre 6
Lignes avec pertes que : p = co

C[ .

En tenant compte des relations (2) et (4), nous avons :


( 12)

Comme, par dfinition, [3 = 2n/X, il en rsulte que la longueur donde sur la


ligne X = Xfy/\J~r est la mme quen espace libre dilectrique.
Dautre part, la vitesse de phase :

J .__

P P \ / Ll C1
est aussi la mme quen espace libre.
Enfin, puisque vp est indpendante de la frquence, nous avons pour la vitesse
de groupe : vg = d)/d(3 = co/[3 = v .
La ligne coaxiale est donc exempte de distorsion de phase.
Toutes ces proprits sont normales puisque le mode de propagation est un mode
T.E.M., comme en espace libre.

8.2.3 Impdance caractristique


En se plaant toujours dans lhypothse des lignes faibles pertes, limpdance
caractristique est donne par la formule (7) du chapitre Lignes avec pertes .
En VHF et aux frquences suprieures, il est possible de ngliger les termes en
^?l/2Lj) et G|/2CiC (voir exercice 8.3) do : Zc = L x/C\
d2
log (LJ)
a\

et

Figure 3.
UllllOUC <x
isolant semi-aere.

i
y.

(14)

I
i

Afin de rduire les pertes, on a intrt utiliser lair comme dilectrique, do


lemploi disolants dits semi-ars (notamment dans les lignes rigides) o le
centrage des conducteurs est obtenu au moyen de disques de permittivit er et
dpaisseur e espacs dune priodicit Z (flg. 3) ; alors :

60
zc=
1 + ( e - 1)

8 .3

D imension

dl
=
e ln drj

(15)

optim ale

d ' u n e ligne co a xia le

Pour rendre les pertes minimales, on a videmment intrt utiliser un dilec


trique et des conducteurs de trs bonne qualit. Ceci tant, nous constatons
daprs la relation (9) que a dpend de la frquence et des dimensions du
coaxial. une frquence donne, seul le terme a c reprsentant les pertes dans
les conducteurs, dpend des dimensions du coaxial :

( 1 ^
1
dlp
ln
V di)

Dans le cas particulier, important en pratique, o a , = a 2 = a, nous avons :

a=K
^j= f"2+il ( 1d j \
U J ln -p
Si d2 est fix, nous voyons que le minimum de a c, donc de et, correspond au
minimum de la fonction : (1 + x)/\n x avec x = d2j d ,.
Sa drive sannule pour ln x: = 1 + \ / x soit pour x 3,59.
Pour quune ligne coaxiale prsente des pertes minimales, il faut donc que
d2/dy = 3,59. Daprs (14), la valeur correspondante de limpdance caractris
tique est : Zc = 76,7/\J^crIl
Il est intressant de
noter q u au voisi
nage du minimum,
otc varie re la tiv e
ment peu en fonc
tion de d2/ d i (fig.
4). Ceci laisse donc
une m arge pour
choisir d2/d y afin
que la valeur corres
2
3
4
pondante de Zc soit
adapte au problme traiter ou la normalisation tablie.

Figure 4.
Variation de
laffaiblissement
relatif dune ligne
coaxiale en fonction
du rapport d2j dy
cfe
6 ^

Exemples
1. Avec du tflon dont er = 2,Z C= 50 2, si :
d2 _ 50 v/2

1,178

'<*1

d2
di

3,25

2. Avec du polythylne dont zr = 2,25, Zc = 50 2 si :


, d2
n d{

50

1,5
= 1,25
60

d2
- r = 3,5
di

3. Avec du polythylne dont er = 2,25, Zc = 75 2 si :


, d2
n cf,

B .4 -

7,5 1,5
60

d2
1,875 <=> - r = 6,52

P u is s a n c e t r a n s p o r t a b l e
PAR UNE LIGNE COAXIALE
Soit un point M(p, <)>) de la section droite
dun coaxial dont la distance radiale laxe
est OM = p. Les champs E et H en ce
point sont dfinis respectivement par leurs
composantes radiale et tangentielle (fig. 5).

Figure 5.
Champs E et H en un
point M du dilec
trique dune ligne
coaxiale. OM = p ;
Pi et p2 sont les rayons
des conducteurs
intrieur et extrieur.

A est une constante quil est judicieux de dterminer en fonction de la valeur du


champ lectrique E i la surface du conducteur intrieur puisque cest cet
endroit que le champ lectrique est le plus lev.
Nous avons, en module : j = (2n/X) ( A / )
do:

(18)

Daprs le vecteur de Poynting : P = (1/2) E a H , la densit de puissance


transporte na quune composante, parallle laxe du coaxial, qui a pour
expression en M :

do :
La puissance moyenne active transporte par le coaxial est donne par le flux de
P , travers la section droite :

En appelant EG la rigidit du dilectrique utilis, nous obtenons lexpression de


la puissance maximale transportable par le coaxial :
s ] r

) o

^inax j 20 ^0 P 1

P2
~

(21)

Nous voyons que cette puissance dpend des dimensions du coaxial. Pour un
coaxial de diamtre extrieur d2 donn, nous voyons que P max dpend de la
fonction ( l / x 2) In x avec x = J 2A V Cette fonction passe par un maximum
pour x = \[ = 1,65.

S .S

Pa r a m t r e s

s e c o n d a ir e s

DE LA LIGNE BIFILAIRE

8.5.1 L'affaiblissement
Dans lhypothse des lignes faibles pertes, nous savons que laffaiblissement
doit se calculer daprs la formule (5) du chapitre 6 Lignes avec pertes .
Compte tenu des valeurs des paramtres primaires donns au paragraphe 8.1.2,
nous obtenons :
n f tg

a (Np/m) 2D
d In r
d

(22)

Comme pour la ligne coaxiale, le premier terme reprsente les pertes a c dans les
conducteurs et le second les pertes a d dans le dilectrique. En explicitant numriquement et v = 1/
il vient :
1
~2
6 \ // ---rf - ) 1
a c (dB/m) = 45,8 - 10 ~6
(23)
20
d..ln
d
(dB/m) = 91 10~9

f tgS

(24)

Laffaiblissement le plus important est celui d aux conducteurs. Nous voyons


que, pour un dilectrique et des conducteurs donns, il dpend en premire
approximation des dimensions par la fonction \ / d ln (2D/cl). Pour un carte
ment des fils D = cte, cette fonction passe par un minimum pour 2D /d = 2,72.
En pratique, cette condition nest pas respecte car cela obligerait prendre des
fils de diamtre trop important (do augmentation du poids et du cot).
Heureusement, la variation de a c avec le diamtre des fils est trs lente puisque
la valeur du minimum est seulement multiplie par deux pour 2D 10d.

8.5.2 Le paramtre de phase


Pour le paramtre de phase P, nous trouvons comme pour la ligne coaxiale :
P = to y jL | Cj = co y/ep do les mmes conclusions concernant , vp et vg.

8.5.3 L'impdance caractristique


Elle pourra tre calcule, dans les mmes conditions que pour la ligne coaxiale,
daprs :

Sil sagit dune ligne bifilaire sous cran (fig. 6), limpdance caractristique
est donne par :
(26)

Enfin, pour une ligne bifilaire dilectrique air dont lcartement entre les deux
conducteurs en cuivre est maintenu constant au moyen dentretoises isolantes de
permittivit r dpaisseur e et despacement f (fig. 7) :

276
zc=

. 2D
iog

(27)

1 + ( - 1)-7
gaine mtallique

isolant de constante

Figure 6.

Ligne bifilaire sous


cran en coupe
transversale :
gomtrie et lments
constitutifs

Figure 7.
Ligne bifilaire avec
entretoises isolantes.

(chlorure de polyvinyle)

8 .6

Pa r a m t r e s

s e c o n d a ir e s d e s l ig n e s

UTILISES EN BASSE FRQUENCE


Les paramtres secondaires dtermins au paragraphe 8.2 pour la ligne coaxiale
et au paragraphe 8.5 pour la ligne bifilaire lont t dans lapproximation des
hautes frquences qui suppose que /?,
L,to et G,
C,t. Pour les lignes
coaxiales, cette approximation est, en gnral, valable car elles ne sont pas utili
ses en tlcommunications des frquences infrieures 100 KHz. Les lignes
bifilaires, en revanche, sont utilises aussi bien pour acheminer des signaux mul
tiplex HF (comprenant de 12 120 voies tlphoniques) que pour les liaisons
entre centraux et abonns o elles fonctionnent alors aux frquences vocales,
cest--dire des basses frquences comprises entre 0 et 4 KHz. A ces fr
quences, alors que la condition G,
Cjto est toujours satisfaite, la condition
R j < Lyio ne lest plus et lon a, au contraire, R\ > Ljt ( moins que LjC ne soit
relev artificiellement par le procd de pupinisation, expliqu au chapitre 6).
De plus, en BF, linductance Ly est fonction de la frquence ; elle est, en effet, la
somme :
dune inductance externe Le, due aux lignes de force du champ magntique
qui ne pntrent pas dans les conducteurs. Le est indpendante de la frquence
et se calcule par les formules (2) ou (6) ;

dune inductance interne Z.,-, due aux lignes de force qui se ferment dans les
conducteurs. L, dpend de la frquence ; elle vaut 0,1 mH/km aux frquences
vocales, elle dcrot quand la frquence augmente cause de leffet de peau et
tend vers zro aux hautes frquences.
Voyons maintenant quelles formules doivent tre utilises en BF pour le calcul
des paramtres secondaires des lignes, compte tenu de ce que : R l S> Ljto et
G, =0.
y= \f(R i

Do :

+ jL , to)(G, + jC , to)

s y f j R l C l ( = i j R l C l (

/ Rl Ci t
= P= V
co

et :

2
/

2co
* iC ,

+^

(28)
(29)

Laffaiblissement et la vitesse de propagation augmentent comme la racine


carre de la frquence ; il y a donc de la distorsion damplitude et de phase.
/? l+ jL ,to
/ Ri
-jZ = x / ----- ---- = \ / r ---- = % / ----- e 4
Gj + j C] tu
y j Cj co Y C j to

(30)

Le module de Zc. varie donc en raison inverse de ^ f f .

8 -7

Exem ples

d e l ig n e s u t il is e s

EN TLCOMMUNICATIONS

8.7.1 Cbles tlphoniques


pour frquences vocales
Figure 8.
a. Ligne bifilaire
appele paire .
b. Quarte constitue
par deux lignes
bifilaires.
c. Quarte dite en
toile .
Ces cbles sont constitus de plusieurs lignes bifilaires ou paires , en gnral
groupes par deux pour former une quarte (fig. 8). Ils servent relier les
abonns aux centraux tlphoniques.

Les conducteurs sont isols par du papier sec ou du plastique (er s 2) ; la dis
tance entre axes des conducteurs est denviron 1,5 fois leur diamtre (2D /d = 3) ;
dans lexemple qui suit d = 0,4 mm.
a) Paramtres primaires (fig. 9, d'aprs [33 p. 223-224])
La rsistance /?j a la mme valeur en BF quen courant continu, soit de lordre
de 100 Q/km. A partir de 100 KHz, elle augmente proportionnellement \ f f
(relation (5)) sous linfluence de leffet de peau.
La conductance G i peut tre nglige V /.
Linductance
natteint sa valeur de 0,44 mH/km donne par (6) quen HF ;
lorsque la frquence diminue, elle augmente pour atteindre une valeur de
lordre de 0,6 mH/km aux BF ; cette augmentation est due ladjonction de
linductance interne Li dont nous avons parl au paragraphe 8.6.
Enfin, la capacit Ci est daprs (8) : 0,051 pF/km.
b) Paramtres secondaires
En BF, par exemple 1 KHz, o Ljto h 4 Q/km < R { ces paramtres doivent se
calculer laide des formules (28), (29) et (30).
avp
A-

R i Cj to

= 0,1265 Np/km s 1 dB/km

2co
= 4,962 104 km/s
/?7c7
Ri
_ i*
e J 4 = 5 5 8 Q Z -4 5
to

En HF, par exemple 1 MHz, o L,to s 3 1 4 0 Q s> /?,, ces paramtres doivent
se calculer laide des formules (23), (13) et (25).
/
____
2D
a = 45,8 \ / yJTviHz d ln

1
dB/km

En prenant r = 2, a = 5,65 107, d = 0,4 mm et 2D /d = 3 : a = 26 dB/km.


> v = - y = L = = 198 000 km/s (avec L ! = 0,50 mH/km)
-H> Zr = - ^ L l n ^ = 93,22
'p
d
Les figures 10, 11 et 12 daprs [33 p. 224-225] montrent respectivement les
variations de Zc, a et vp dans la gamme de frquences 1 KHz-10 MHz.

R, (fi km-1)

L, (mH km-1)

IZCI (Ci)

ZZ

Figure 9.

Variation frquentielle
des paramtres
primaires Ri et
dune ligne bifilaire
utilise en
tlcommunications
Figure 10.
Variation frquentielle
de | Zc | et arg Z(. de la
ligne bifilaire caract
rise la figure 9.

f (MHz)

Figures 11 et 12.
Variation frquentielle
des paramtres a
et vp de la ligne
bifilaire caractrise
la figure 9.

f (MHz)
Les figures 9 12 sont extraites du livre de K.F. Sonder et C.A.L. Reed
Transmission and propagation of electromagnetic waves (Chap. VI, p. 223
225). diteur Cambridge University Press ( 1978).

8.7.2 Cbles tlphoniques circuits coaxiaux


pour multiplex
Ces cbles sont constitus par le regroupement de plusieurs lignes coaxiales
dont les diamtres des conducteurs intrieur et extrieur sont 2,6 et 9,5 mm
dans les ralisations les plus courantes. Ils servent aux liaisons interurbaines
entre centraux tlphoniques et permettent lacheminement de 960 voies
(A/ = 4 MHz), 2 700 voies (A /= 12 MHz) ou 10 800 voies (A /= 60 MHz). Il
existe aussi des cbles du type 1,2 4,4 mm qui achem inent 300 voies
(Af - 1,3 MHz), 1 200 voies (Af = 6 MHz) ou 2 700 voies (Af = 12 MHz). Enfin
les cbles 3,7/13,5 mm sont utiliss pour 10 800 voies (A/= 60 MHz).
Nous allons nous intresser aux cbles 2,6/9,5 mm dont le dilectrique est de lair
sauf pour des rondelles de polythylne rgulirement espaces qui assurent le cen
trage des conducteurs. La constante dilectrique relative quivalente est r = 1,09.
a) Paramtres primaires
Daprs (1), (2), (3) et (4), nous avons :

R { =41 yjf QJ km

(/e n MHz)

Lj = 0,26 mH/kin
G,

= 0 car, 10 MHz, tg = l,5 - 1 0 ~ 4 => G ,< 1 0 - 3 S/km

Cj = 0,046 pF/ km
b) Paramtres secondaires
Ils se dterminent daprs (10), (13) et (14) :
sfr <1d\

a = 19,9 10-6

AJ
d2

log

^2
d.

Avec : er = 1,09 ; a = 5,65 107, d y - 2,6 mm et d2 = 9,5 mm :


a = 2,42 J fu H i dB/km
-h>

= 287 356 km/s


s f^ C ,
60
d2
-> Z . = j=* ln - = 74,4 Q
'F
d '1
,
"

8 .8

b a q u e d ' im p d a n c e s c a r a c t r is t iq u e s
800 Ci

700Q.

600 2

500 n

400 a

Figure 13.
Abaque donnant la
variation de
limpdance
caractristique
(0 < Zc < 400 2) et
(400 < Zc < 800 2) en
fonction des dimen
sions gomtriques
(rapport D/d) de
lignes bifilaires et
coaxiales
dilectrique air
(, = 1) et
polythylne
(er = 2,26).
Daprs Liot (L) :
Lignes de transmis
sion et filtres pour trs
hautes frquences ,
Dunod (1959).

Le diagramme de la figure 13, d aprs [27], est trs intressant parce quil
donne les valeurs des impdances caractristiques des lignes bifilaires et
coaxiales dont nous venons dtudier la technologie, en fonction des dimensions
transversales de ces lignes et pour er, constante dilectrique relative de lisolant,
gale 1 (air) ou 2,26 (polythylne).

&

E X E R C IC E 8 .1

pertes est donne par :


Cl 1
1, - j. -------- + j ---------2 Z.jt
2 C LcoJ

(Paragraphe 8.1 - Paramtres primaires


d une ligne coaxiale)

Dmontrer les formules (1) (4) donnant les


paramtres primaires dune ligne coaxiale, en
appliquant les lois fondamentales de llectro
magntisme des rgimes stationnaires pour L x
et C, et des rgimes variables pour /?, et G j.
On tiendra compte de ce que, dans le dilec
trique dun coaxial :
les lignes de champ lectrique sont radiales,
les lignes de champ magntique sont des
cercles concentriques.
On notera r ( et r2 (d^ et c/2) les rayons (dia
mtres) des conducteurs intrieur et extrieur.

&

E X E R C IC E 8 . 2
(Paragraphe 8.2.1 - Affaiblissement
d une ligne coaxiale)

Le paramtre daffaiblissement a dune ligne


coaxiale est la somme de deux termes a (. et o.(!
qui reprsentent respectivement les pertes dans
les conducteurs et dans le dilectrique.
Dmontrer les expressions (9), (10) et (11) de
tt(Np/m), a c(dB/m) et a^fdB/m) dans lap
proximation des lignes faibles pertes.

&

E X E R C IC E 8 . 3
(Paragraphe 8.2.3 - Impdance caractristique
d'une ligne coaxiale)

Limpdance caractristique dune ligne faibles

(exercice 6.2).
Soit une ligne coaxiale utilise 100 MHz
pour laquelle : J 2 = 1 cm, d 2/r/j = 3,6,
1 = 2 ~ 5,8 107 S/m, er = 2,25 et
ig 8 = 10~3
1) Dmonter que les termes en R l/ 2 L l( et
G j/^C jto peuvent tre ngligs.
2) tablir, dans ces conditions, lexpression de
Zc et calculer sa valeur numrique.

&

E X E R C IC E 8 . 4
(Paragraphe 8.4 - Puissance transportable
sur une ligne coaxiale)

La puissance transporte par un cble coaxial


est donne par la formule (20) o : E l est le
champ lectrique la surface du conducteur
intrieur, de rayon Pj ; p2 est le rayon du
conducteur extrieur ; r est la permittivit
relative du dilectrique.
1) Quelle est la valeur optimale du rapport
p2/p j = x pour que la puissance transporte
soit maximale pour E j et p2 donns ?
2) On donne : E i = 103 V/cm, p2 = 1 cm et
er~ !
Calculer la puissance maximale transpor
table.

Chapitre 9
Les lignes

iandes

et fentes

9 .1

L es pr in c ipa u x t y p e s d e lig n es

Il y a une grande varit de lignes structure bidimensionnelle, appeles parfois


lignes planaires, qui ont t tudies et ont fait lobjet dapplications pour la ra
lisation des circuits passifs et aussi des circuits actifs hyperfrquences. Nous les
avons classifies en deux catgories : les lignes bandes et les lignes fente.

9.1.1 Les principaux types de lignes bandes


a) La ligne microbande (en anglais microstrip)
Elle comporte un substrat en dilectrique, compltement mtallis sur lune de
ses faces et couvert dune bande mtallique sur lautre (fig. 1).
b) La ligne triplaque (stripline)
Elle est constitue par deux plaques mtalliques spares par un substrat de
dilectrique au sein duquel se trouve une bande mtallique (fig. 2).
Cest, en quelque sorte, une ligne microbande rendue symtrique par ladjonc
tion dun plan mtallique suprieur, image du plan mtallique infrieur par rap
port au plan o se trouve la bande centrale.

//////SS/S//////////////.'* :. Y//////
Figure 1.
Ligne microbande.

" S

Ef

Figure 2.
Ligne triplaque.

er
/zw/7//////////////////////;////;//////////

/7//y/sz77////z//////y///////////////////////

c) La ligne substrat suspendu (suspended substrate line)


Elle est constitue par un substrat de
dilectrique supportant une bande mtal
lique (fig. 3), le tout tant enferm dans
le plan H dun guide d ondes qui se
trouve la coupure pour la bande de fr
quences utilise. Ce guide dondes joue
uniquement le rle de botier empchant
le rayonnement de la ligne qui deviendrait non ngligeable au-del de 10 20 GHz.

9.1.2

Les principaux types de lignes fente

a) La ligne fente (s/ot Une)


_____ i......."i______i
Figure 4.
Ligne fente.

i_____

er

Les deux conducteurs formant la ligne


sont dposs sur la mme face du substrat dilectrique (fig. 4).

b) Le guide coplanaire (coplanar wave guide)


I .

., 1------- ___________I . I _____

Figure 5.
Guide coplanaire.

er

Il prsente 3 bandes mtal


liques spares par deux
fentes d un mme ct du
substrat (fig. 5).

c) La ligne ailettes (fin line)


Figure 6.
Lignes ailettes.
a. unilatrale,
h. bilatrale.
a)

b)

Cest une ligne fente unilatrale (fig. 6a) ou bilatrale (fig. 6b) enferme (pour
viter des pertes par rayonnement) dans le plan E dun guide dondes la cou
pure pour la bande de frquences utilise.
La ligne la plus utilise est, sans conteste, la ligne microbande, en raison de sa
simplicit de fabrication et des applications quelle permet tant en circuits pas
sifs quen circuits actifs. Cest donc elle que nous tudierons le plus longuement.
Nous nous intresserons aussi la ligne triplaque qui a t trs utilise dans les
annes 1960 et 70 et dont il existe encore beaucoup de ralisations. La ligne
fente, qui est complmentaire de la microbande, et la ligne ailettes, qui en est
la version utiliser au-del de 20 GHz pour viter les pertes par rayonnement,
seront galement tudies : la premire dans ce chapitre et la seconde la fin du
chapitre 12 Les guides dondes rectangulaires . Pour les autres types de lignes
et leurs variantes, nous renvoyons le lecteur aux ouvrages spcialiss, notam
ment la rfrence [34].

9 -2

P e r m it t iv it

e f f e c t iv e d e s l ig n e s

MICROBANDE - LONGUEUR D'ONDE


e t v i t e s s e d e p r o p a g a t io n

Figure 7.
Paramtres
caractristiques de la
ligne microbande.

Les paramtres caractrisant la microbande sont (figure 7) :


pour le substrat, son paisseur h et sa constante dilectrique relative qui est
souvent leve (~ 10) afin dy concentrer le champ lectromagntique et de
rduire ainsi les pertes par rayonnement ;
pour la bande, sa largeur w qui est, en gnral, de lordre de grandeur de
h (0,1 w/h 10) et son paisseur b, presque toujours petite (b/h
1).
La difficult de ltude de la propagation dans une ligne microbande vient de ce
que cette propagation seffectue dans le substrat, de permittivit er, et dans lair,
de permittivit 1, comme le montre la forme des lignes de champ lectrique sur
la figure 8.

Figure 8.
Lignes de champ
lectrique de la ligne
microbande.

Il sagit donc dune propaga


tion par modes hybrides ayant
les six composantes du champ
lectromagntique non nulles.
En fait, les composantes longi
tudinales Ez et H sont trs
faibles et le mode de propa
gation dom inant peut tre
considr comme quasi T.E.M. Mais, mme dans ce cas, il est difficile de dfinir
une vitesse de propagation. On sait, en effet, que pour un mode T.E.M. :
Z, = Z0/ \ f r , er tant la constante dilectrique relative du milieu o seffectue
la propagation. Or, dans notre cas, la propagation seffectue dans deux milieux
de r nettement diffrents.

Figure 9.
a. Microbande relle.
b. Microbande
quivalente.
b)
Le problme serait beaucoup plus simple si lon avait un dilectrique homogne
et illimit entourant la bande. La vitesse de propagation serait alors dfinie sans
ambigut puisque la propagation serait purement T.E.M. Cest pourquoi une des
mthodes dtude de la microbande relle (fig. 9a) consiste en rechercher une
modlisation quivalente par une ligne microbande dilectrique homogne illi
mit, que nous appellerons dsormais ligne microbande quivalente (fig. 9b). La
cl du problme rside dans la dtermination de la constante dilectrique effec
tive e de ce modle en fonction de er, h et w.
Pendant 25 ans, depuis Assadourian et Rimai (1952) jusqu Hammerstad (1975
[35]) sans oublier la contribution majeure de Wheeler [36] les efforts des cher
cheurs ont port sur la dtermination la mieux approche de E^, pour les diverses
valeurs de w/h.
Une formule explicite de e a t donne par Hammerstad.
Pour les bandes telles que w /h> 1 :
e ,= -2V+ 1 > 4 V - ' > ( 1 +

l2 P

1
(i)

Pour les bandes telles que w/h < 1 :


ee = ^ ( e r + 0 + ^ (e , - D

1 + 12 ] 2 +
W
H

'- x )3

( 2)

Ces relations donnent une approximation meilleure que 1 % lorsque 0,05 < w/h < 20
et Zr < 16. Le graphique de la figure 10 permet de calculer, d aprs ces rela
tions,
Z en fonction de w/h pour diverses valeurs de r.
g : a

Figure 10.
Abaque pour le calcul
de t e en fonction de
w/h pour diverses
valeurs de er Daprs
Gardiol [37, p. 83J.

i------->--------
10
20 w/h

0 -0,1

Pour une meilleure approximation sur ze et un domaine de validit plus tendu


concernant w /h et Zr, on se reportera aux expressions beaucoup plus compli
ques de Bhartia et Bahl [38, p. 277],
Notons que, de cette permittivit effective, lon dduit :
la longueur donde Xm sur la ligne microbande, daprs :
X,.

avec

X - -f
K0~

2tc 2ji .Vf z ~e


le paramtre de phase p, daprs : p =
Xq
' K ~

(3)

(4)

.
la vitesse de propagation daprs : vp =

II

(5)
U)

9 -3

Im p d a n c e

c a r a c t r is t iq u e

DES LIGNES MICROBANDE

9.3.1 Rsultats de Wheeler pour b = 0


Pour les bandes telles que w/h > 2 :

120n w

-1

e +1

er ~ l

, + 0,883 + ---- ln + 0,94) + 1,451 + 0,165- V


2h
h
ne.

( 6)

Pour les bandes telles que w/h <2 :


Z =
F

60 ^ 2
8/j \ 1 l w i?
ln
+
w j 8 \2h)
+1

e r 1
2 (e + 1)

TC

ln - +

ln

er

(7)

tc

La figure 11 permet dobtenir limpdance caractristique Zm de la ligne micro


bande daprs :
Z =

( 8)

Figure 11.
Abaque pour le calcul
fonction de w/h pour
diverses valeurs de r
Daprs Gupta

134. p . 12] .

Zq est limpdance caractristique quaurait la mme ligne microbande avec uni


quement de lair pour dilectrique. Ze est la permittivit effective de la micro-

bande quivalente. Pour un rapport w /h donn, Z0 est obtenu daprs la courbe


en tirets et se lit en ordonnes gauche, tandis que \J ^ e est obtenu daprs lune
des courbes en traits pleins (chacune delles correspond une valeur intressante
de r) et se lit en ordonnes droite.

9.3.2 Rsultats de Hammerstad pour b = 0


Pour les bandes telles que w/h > 1 :
S7 i

120jt

j + 1,393 + 0,667 ln

+ 1,444

(9)

e est donn par la formule (1).


Pour les bandes telles que w/h < 1 :
60
8/j
ln
z =
vt

v/

w
Ah

( 10)

est donn par la formule (2)


La figure 12 donne les variations de limpdance caractristique Zm en fonction
de w /h pour diffrentes valeurs de r Cette figure est intressante deux points
de vue : dune part elle permet une estimation directe de la valeur approche de
Zm en fonction de w/h et r ; dautre part, elle montre que les impdances carac
tristiques des lignes microbandes sont comprises entre une dizaine dohms pour
les plus faibles, qui correspondent des valeurs leves de w/h (w/h ~ 10) et
de r (r > 10), et 200 El environ pour les plus fortes qui correspondent des
valeurs faibles de w /h (w/h 0,1) et de r (r 2).

Figure 12.
Abaque pour le calcul
de Zm en fonction de
w/h pour diverses
valeurs de r. Daprs
Gardiol f37, p. 83].

9.3.3 Facteurs de correction


Bien que trs faible, lpaisseur de la bande nest pas nulle. On peut en tenir
compte, dans les relations prcdentes, en substituant la largeur relle w du
ruban, une largeur quivalente we un peu plus grande, donne par :

avec : x = h si vv > h] 2k et x = 2 n w si h/2 n > w > 2b.


Par ailleurs, les formules (8), (9) et (10) sont indpendantes de la frquence. Il
est possible den tenir compte de faon approche en remplaant eg dans ces formules
par :

/ / ) = r

r ~ e
, ,2

(12)

l + c f)

107
S d ~ 8tc ' h

avec :

(13)

G = 0,6 + 0,009 Zm

et :

(14)

Si / < f j , il nest pas ncessaire deffectuer la correction. Dautres expressions


sont donnes dans [38, p. 279-280],

3 .4 Dimensions d'une ligne microbande


en fonction de Z m
Des relations dues Wheeler [36] permettent de trouver le rapport w /h en fonc
tion de limpdance caractristique Zm de la ligne relle (avec Z0 = 1207T) :
Pour le cas o w/h < 2 :
w

avec :

(15)

eA e A

- 1
A - n J l (e + 1)
+ r f 0,23
v
z o e +1

o.i n

(16)

r y

Pour le cas o w/h > 2 :


1
0,61
W
+ - [ 5 - 1 - I n ( 2 5 - 1)]
In (JB- 1) + 0,39
77
7C

(17)

avec :

B=

9 .4

(18)

f f a ib l is s e m e n t

d ' u n e l ig n e m i c r o b a n d e

Il y a trois types de pertes dans les lignes microbandes : dans les conducteurs,
dans le dilectrique et par rayonnement. Le calcul tant assez long et complexe,
nous ne donnons ici que les rsultats, daprs j34j.

9.4.1 Pertes dans les conducteurs


Si w/h Si 1 :

Rs
a (dB/m) = 1,38 A t - ~

32-

32 +

Si w /h 1 : a (dB/m) = 6,1 10 5z\

RS Zm

(19)

w*

v
-667T
h + re

( 20)

+ 1 ,4 4 4

avec :

A = 1+

et :

w,,

1+

1,25 , 2B\
lnT j

( 21 )

B = h si vv/h = l / 1 k et B 2n w si w /h 1/ 27t.
Rs =

f o p, rsistivit des conducteurs.

Zm, impdance caractristique de la microbande.


we, largeur quivalente du ruban.
Ces valeurs thoriques sont valables pour des conducteurs dont la surface est
plane. Mais si la surface prsente des rugosits, laffaiblissement est nettement
plus grand. En pratique, on obtient une limite suprieure de a (. en utilisant la for
mule approche suivante :
Rs
a (dB/m) = 8,686 -c
VI /.

9.4.2 Pertes dans le dilectrique


(23)

a d (dB/m) = 4,34

ou :

e
e - 1 te S
ot . (dB/m) = 27,3----------- - j =
\ f ze

er-1

^0

(24)

a d = co 0 r tg 8 est la conductivit du dilectrique, tg 8 est la tangente de


langle de pertes.
Sauf si le substrat est en semi-conducteur (Silicium ou AsGa), o.d < a c comme
le montrent les courbes traces sur la figure 13.

Figure 13.
Graphique donnant les
pertes conductrices ac
et dilectriques ad
pour diffrents
substrats.
D aprs Gupta

[34, p. 75].

9.4.3 Pertes par rayonnement


Le rayonnement dune ligne microbande est le fait des modes dordre suprieur
qui sont engendrs par les discontinuits. Daprs Hammerstad ces pertes sont
proportionnelles (hf )2/ p o u r une ligne de 50 Q.
La frquence limite au-dessus de laquelle plus de 1 % de la puissance est
rayonne lextrmit dune ligne ouverte est donne par :

(e /f M (GHz) = 2 , 1 4 ^

(25)

Par exemple [37, p. 85], pour un substrat de h 1 m m ,/M = 3 GHz si er = 2,5 et


_/M = 4 GHz si er = 10.

9.5

La

l ig n e t r i p l a q u e

La ligne triplaque (fig. 14) est constitue par trois conducteurs - une bande cen
trale et deux plans de masse - et elle est remplie par un dilectrique homogne.
Elle fonctionne donc en mode T.E.M. tant que sa hauteur b est suffisamment
petite pour quil ny apparaisse pas de modes dordre suprieur, cest--dire tant
que : b < k j 2 ^ V r .

Figure 14.
Coupe transversale
dune ligne triplaque
avec les lignes de
champ E (-...... )
et de champ
H ( ------ )-

Dans ces conditions, les champs E et H sont contenus dans un plan de section
transversale (fig. 14) o ils sont concentrs autour de la bande centrale si bien
quil est possible de limiter la largeur des deux plaques infrieure et suprieure
5w.
Pour le cas limite o t 0 (bande centrale dpaisseur nulle) limpdance carac
tristique Zt est donne par la relation :
= 37C

Z,

K{u)
KW)

(26)

K(u) est une intgrale elliptique complte du premier ordre.

J2
TC

K(u) =

u = ch 1 (nw/2b)

sJ 1 w2 sin (p dtp
= \J 1 w2 = th (n w/2b)

(27)

Si

w /b <

0,5 (0,5 n2 1)
<

<

Z, J
Si w /b > 0,5

-301nf2 1+V^

(28)

v i-V

(0 < u2 < 0,5)


Z v

r =:3 0 tc2

(29)
1- y R

Pour les cas o lpaisseur de la bande centrale nest pas ngligeable, Wheeler
[40] a calcul les formules suivantes :
t
b -t 8 b -t
8
+ 6,27
(30)
w TC w
7C
w = w + Aw

ou :
av ec.

Aw
x
,

b - t n (l-x )

'4 '"

in = 2
et :

(31)

m
* f | ( 0,0796* ^
2 -x)
T +
\b
J

(32)

- I
(33)
(34)

Pour une dtermination plus rapide de limpdance caractristique Z ,, il est pos


sible dutiliser un graphique d S.B. Cohn [daprs 42] qui donne la valeur de
Z,
en fonction du rapport w /b et pour diverses valeurs du rapport t/b
(fig. 15).
Pour la conception de circuits micro-ondes utilisant des lignes triplaques, il est
essentiel de pouvoir calculer les dimensions de ces lignes permettant dobtenir
une certaine impdance caractristique. Les courbes prcdentes permettent de
dterminer directement ou par interpolation les valeurs de w /b et t/b correspon
dant la valeur dsire de \J r
Pour plus de prcision, il est possible dutiliser des formules de synthse
dduites des formules prcdentes ; on les trouvera explicites dans la rfrence
[43, p. 58-59], La mme rfrence fournit les formules donnant lattnuation
dans les conducteurs et dans le dilectrique de la ligne triplaque.
Notons enfin que tous ces rsultats restent valables avec une trs bonne prci
sion (au moins un pour cent) tant que la ligne fonctionne en mode T.E.M. La fr
quence limite au-dessus de laquelle apparat le premier mode suprieur est
donne par :
0,15
1
/ (GHz) =
(35)
W
TC
b \[ %
J* 4

Figure 15.
Abaque pour le calcul
de >JTr Z, dune

L es lignes bandes et fentes

ligne triplaque en
fonction de w/b pour
diverses valeurs de
t b. D 'aprs Saad
(T. S) [42. p. 117],

9 -6

La

l ig n e f e n t e

Dans la ligne fente


(slot line) les deux
conducteurs formant
la ligne de transmis
sion sont dposs
sur la mme face du
substrat dilectrique
(fig. 16). Les rsul
tats donns dans ce
paragraphe sont
tirs de la mthode
d analyse due
Cohn [44] qui est la
plus gnralem ent
utilise.

9.6.1 Le champ lectromagntique


En tudiant la ligne fente comme un guide d ondes lectromagntique, il est
possible de dterminer les six composantes du champ lectromagntique. Dans
le cas o la largeur w de la fente est trs infrieure la longueur donde Xq en
espace libre, la fente peut tre considre comme une source linaire de courant
magntique. En zone de champ lointain, cest--dire des distances r
w, le
champ produit par la fente a trois composantes :
Hz = A H $ H k c r)
Hr = -

k}

(36)
H \'H kc r)

dr
1-

(37)

W
<X0J

jtoji dHz
kl

(38)

dr

yz, est le paramtre de propagation selon laxe de la fente (axe des z).
Xs, est la longueur donde selon laxe de la fente.
[y \ 2
1-

(39)
\ XSI

Pour que la ligne fente ait peu de pertes par rayonnement, il convient que ces
champs soient concentrs son voisinage et diminuent donc rapidement avec la

. aa^'M
H ftjfM
%%
w ry rtih ir

&

distance r. 11 faut, pour cela, que largument kc r des fonctions de Hankel inter
venant dans les expressions de //,, / / (.et
soit imaginaire En effet, le compor
tement asymptotique de telles fonctions est donn par :
(a grand et > )
V J tA

(40)

Si donc kc r = j | kc r |, tous les champs sont affects dun affaiblissement e L- r.


Ceci se produit si : 1 - (A.0 X^)~ < 0 => L() > Xs .
Or, selon une approximation grossire concernant une fente sur un substrat sup
pos infini,
"Ks y (er + 1), 2 . On a donc intrt prendre Er > 1 et aussi
grand que possible. Le choix de substrats haute permittivit dilectrique (en
pratique, e,. > 9) permet de concentrer les champs au voisinage de la fente.

9.6.2 Longueur d'onde


et impdance caractristique
Les travaux de Cohn [44] ont permis de prsenter sous forme graphique les
rsultats concernant la longueur donde Xs et limpdance caractristique Zs de
la ligne fente pour plusieurs cas particulirement significatifs (voir par
exemple Gupta et al. [34. p. 204 207]).
Il a t possible de dduire de ces rsultats graphiques une formulation empi
rique qui souffre dune erreur infrieure 2 % lorsque 9,7 e,. 20 pour b = 0.
Pour 0,02 w/h < 0,2.

X,

vv
w
h
= 0,923 - 0,448 log Er + 0,2 " - [0,29 " + 0,047 log
10vv

Zs = 72,62 -3 5 ,1 9 log e^ + 50

- 0,02

(41)

, - 0,1

vv
h

-t-Iog K 10 2 (44,28- 19,58 log er)- 0,32 log e -0,11


+ -- ( 1,07 log er + 1,44) Il 1 ,4 -6,07 loge 1

71 102)

(42)

/ = 3 GHz
. V(r)
= 0,038
HO)
<<K

Notons enfin que, daprs la formule asymptotique de


*(j.v) donne ci-dessus,
les deux composantes en Hr et IL du champ magntique sont en quadrature de
phase. Il en rsulte que ce champ est polaris elliptiquement. Cette proprit
pourra tre utilise pour raliser des dispositifs non rciproques, en plaant au
voisinage de la ligne fente des barreaux de ferrite convenablement polariss.

Exemple
er = 16 et

Pour 0,2 ^ w/h ^ 1


= 0,987 - 0,483 log er + (0,111 - 0,0022 e r)
A.

- Jo, 121 + 0,094 ~ - 0,0032

erj N

log ( 102

(43)

W
Z5 = 113,19 - 53,55 log e r + 1,25 - ( 114,59 - 51,88 log e#r)

+2 ( ? - a2)('-)
- 0, 15 + 0,23 log e r + j (- 0,79 + 2,07 log e r )
10,25 - 5 log e r + ^ (2,1 - 1,42 log er ) -

10:

(44)

E X E R C IC E 9 .1

&

E X E R C IC E 9 . 4

(Paragraphe 9.2 - Permittivit effective des lignes


microbande et 9.3- Abaque de Wheeler)

(Paragraphes 9.3.3 et 9.3.4 - Dimension dune ligne


microbande enfonction de Zm)

Une ligne microbande a pour dimensions go


m triques h = 0,635 mm et w = h / 2. Son
dilectrique a pour caractristiques : er = 9 et
tg 6 = 10 ~ 3.
1) Calculer sa permittivit quivalente ze et
comparer cette valeur avec le rsultat trouv
graphiquement.
2) Trouver Zm daprs Labaque de Wheeler et
comparer cette valeur avec le rsultat cal
cul partir des formules.

On veut obtenir une impdance caractristique


de 50 f i avec une ligne microbande dont le
substrat a pour permittivit er = 10. Calculer la
valeur du rapport w/h.
Si h = 0,635 mm et b = 0,01 mm (paisseur de
la mtallisation), calculer la largeur we du
ruban.

E X E R C I C E 9 . 5 *123
&

E X E R C IC E 9 . 2
(Paragraphe 9.3.1 - Rsultat de Wheeler)

Dmonstration de la formule (8) :


~ Z/ \ / % -

E X E R C IC E 9 . 3
(Paragraphe 9.3.3 - Facteurs de correction)
Les donnes sont les mmes qu lexercice
9.1 et Zm = 70 fi.
Dterminer la frquence partir de laquelle on
peut considrer que le terme de correction ffquentielle a une importance infrieure 1 %
dans la formule donnant e (J).

(Paragraphe 9.4 - Affaiblissement dune ligne


microbande)
Une ligne microbande a pour dimensions go
mtriques : h = 0,635 mm et w = h. Son dilec
trique a pour caractristiq u es : e r = 9 et
tg = 10 ~ 3.
1) Quelle est la frquence en dessous de laquelle
on peut considrer que les pertes par rayon
nement sont ngligeables ?
2) Pour / = 1 GHz, faire le calcul des pertes
dans les conducteurs, sachant que la
rsistivit de la mtallisation est p = 0,18
10 7 fi m et son paisseur b = 0,01 mm.
3) Calculer enfin les pertes dans le dilec
trique.

<#" E X E R C IC E 9 . 6

de 50 Q avec une ligne triplaque dont le


dilectrique a une permittivit er = 2,26

(Paragraphe 9.5 - La ligne triplaque)

1) Calculer les rapports w /b et t/b (fig. 15)


pour obtenir une impdance caractristique

2) Dans le cas o w /b = 0,6 et b = 1 cm, quelle


est la frquence limite pour un fonctionne
ment en mode T.E.M. ?

Chapitre 10
Ralisation
impdances
e t des
circuits rsonnants

En rgime alternatif, jusqu des frquences de lordre de 10 MHz, les compo


sants passifs tels que les rsistances, inductances ou condensateurs classiques
prsentent des caractristiques (R, L, C) qui sont indpendantes de la frquence et
les tronons de lignes connectant ces composants (dans les circuits non intgrs)
ont un comportement neutre . On dit quil sagit dlments constantes
localises . Pour des frquences suprieures (hautes frquences), au contraire,
les composants passifs classiques ont des caractristiques {JR, L, C) qui ne cor
respondent plus aux valeurs nominales donnes par les constructeurs et varient
avec la frquence, tandis que les lignes de connexion se comportent comme des
impdances dont linductance ou la capacit change avec la frquence. On dit
alors quil sagit dlments constantes rparties ou distribues .
Dans une premire partie de ce chapitre, nous allons voir comment il est possible
de tirer parti du comportement de tronons de ligne pour raliser des inductances
et des condensateurs ( 10.1 et 10.2) ou des circuits rsonnants ( 10.3 et
10.4). Ces tronons de ligne, qui ont des dimensions trs petites devant la lon
gueur donde, peuvent tre considrs comme des lments constantes semilocalises puisque leurs caractristiques sont bande relativement troite.
Cest pourquoi toute une technologie - rendue possible par le dveloppement
des techniques photolithographiques de miniaturisation - sest dveloppe pour

laborer jusqu des frquences de plus en plus leves (30 GHz en 1996) des
composants dont les caractristiques restent indpendantes de la frquence et
mritent donc le label constantes localises . Le paragraphe 10.5 expose les
ralisations correspondantes.

10-1

q u iv a l e n c e en tr e un t r o n o n
DE LIGNE ET UNE INDUCTANCE
OU UN CONDENSATEUR

Nous savons quun tronon de ligne dimpdance caractristique Zc, ferm sur
une impdance ZR, prsente, une distance x de cette dernire (fig. 1), une
impdance Z(x) donne par :
ZR + '}Zc \.g$x
Z(x) = Zc
( 1)
Zc + jZ R tgpc
X-

Figure J.
Notations et
conventions utilises
pour un tronon
de ligne.

Z(x)

X-

XO
Les tronons utiliss ont une longueur x telle que : tg Pjc P*, 10 % prs
=> P* < 7t/ 6 ou x < X / 1 2 .
Dans ces conditions, (1) peut scrire :
Z(x) = Zc

z R + j Z c frx

(2)

Z c + } Z R $x

a) Cas o ZR < Zc px
La ralisation technologique de cette condition peut se faire avec une ligne dont
limpdance caractristique Zc est trs grande devant son impdance de charge Z-RPour avoir Zc leve avec une ligne microbande, nous voyons, daprs la figure
6 (chap. 9), quil faut un rapport w /h infrieur 1 et une permittivit er faible.
Remarquons que, puisque P* < n / 6 , la condition ZR < Zc px implique numri
quement que ZR Px < Zc ; nous avons alors, daprs (2) :
co
Z(x) jZ c p * = jZ c x

Un tel tronon de ligne se comporte donc comme une impdance inductive jLco
et lon peut dire quil est quivalent linductance
L = ZCcv

(3)

Un cas particulier de la condition ZR < Z CP* est Le cas o la ligne est en courtcircuit (ZR = 0). Dans ce cas, Z(jc) = j Zc tg px et nous savons (chap. 3, 3.3.1.b)
que cette impdance est inductive pour 0 < x < %/A.
b) Cas o ZR px > Zc
La ralisation technologique de cette condition peut se faire avec une ligne dont
limpdance caractristique Zc est trs petite devant son impdance de charge ZR.
Pour avoir Zc faible avec une ligne microbande, nous voyons toujours daprs la
figure 6 (chap. 9), quil faut un rapport w /h suprieur 1 et une permittivit zr
leve ; en fait, au-del de w/h = 6 et t r = 6 , la diminution de Zc est trs lente.
Remarquons aussi que, puisque Px < 7t / 6 , il en rsulte numriquement que
Z r ^ Zc px ; dans ces conditions, nous obtenons daprs (2 ) :

Z(x) - j

Un tel tronon de ligne se comporte donc comme une impdance capacitive


- j/Ceo et il est quivalent la capacit :
J_ x
(4)
C
Zc 'v
Un cas particulier de la condition Z* Px > Zc est le cas o la ligne est en circuit
ouvert (ZR = <). Dans ce cas, Z(x) = - j Zc cotg Px, impdance qui est capacitive
pour 0 < x < A./4.
Il serait possible darriver aux mmes conclusions partir de la reprsentation
quadripolaire classique dun tronon de ligne de longueur unit (fig. 2 a)S
i

Si nous nous plaons dans lapproximation des lignes sans perte (qui est utilise
dans le raisonnement prcdent), le quadriple se rduit celui reprsent sur la
figure 2b. Pour un tel quadriple, nous pouvons crire, daprs la thorie des lignes :
P - t \JL \ c \

ou

(
V

= \ 7 ( L , ) ( C ! o)

Figure 2.
Reprsentation
quadripolaire dun
tronon de ligne.
a. Avec pertes.
b. Sans perte.

/ L, <o
Dautre part :
Do :

V
L,

(0

CO

= Z
V

et

C l
CO

C i (0 = 1

Donc, pour un tronon de ligne de longueur :


cof
Lco = - Z et
v

La reprsentation quadripolaire dun tronon de ligne ntant valable que pour


des longueurs de ligne t < X (en pratique < X / 10), nous pouvons dire que,
dans ces conditions :
Pour modliser une inductance srie, il faut prendre un tronon de ligne
forte impdance caractristique car alors Lco Ceo. Cette inductance a pour
valeur : L = Zc. C/v. Cest la relation (3).
Pour modliser une capacit parallle, il faut prendre un tronon de ligne
faible impdance caractristique car alors Lco
Ceo. Cette capacit a pour
valeur : C = ( 1/Z c.) ( /v). Cest la relation (4).

1 0 .2

R a lis a t io n d ' in d u c t a n c es
ET CONDENSATEURS

De tout ce qui vient dtre expliqu dcoule la ralisation dinductances et


condensateurs avec des tronons de ligne. Cette ralisation est particulirement
aise en technologie microbande puisque limpdance caractristique dune telle
ligne est inversement proportionnelle la largeur de la bande.
Une inductance srie (fig. 3) sobtient par un fort rtrcissement de la
bande mtallique ; en effet, le tronon de faible largeur, qui prsente donc une
forte impdance caractristique, se trouve charg ses extrmits par des
lignes dont limpdance caractristique est plus faible (condition du cas o
Z * Z CPa).
Figure 3.
Ralisation dune
inductance srie en
ligne microbande.

-------------w w ------------
X= L(

-------------

---------------Z c >
^ zcm
Z C 1.2
zc

Z Cy

(_______________

Z C2

----------------------------------
Une inductance parallle (fig. 4) sobtient en plaant en drivation sur la ligne
principale un tronon de ligne court-circuit. Ceci peut dailleurs se faire aussi
bien en technologie microbande (fig. 4 b) que coaxiale (fig. 4 c).

Figure 4.
.Ralisation dune
inductance parallle.
b. En ligne
microbande.
c. En ligne coaxiale.

Une capacit parallle (fig. 5) sobtient par un largissement important de la


bande mtallique ; ce tronon, qui prsente une faible impdance caractris
tique, se trouve charg ses extrmit par des lignes dont limpdance carac
tristique est plus forte (condition du cas o ZR [Lr > Zc).
----------------- >------------------

2c,
3
O
il
CO
IL
----------------- - >------------------

2c* ^C,,2
e

Une capacit srie est plus dlicate raliser car elle ncessite de couper la
ligne sur une trs petite longueur (quelques microns ou dizaines de microns).
Quantitativement, la valeur de la capacit ainsi obtenue ne peut se calculer
quavec une approximation grossire et qualitativement, le schma quivalent
dune telle discontinuit comporte non seulement une capacit en srie, mais
aussi des capacits parasites en parallle. Aussi prfre-t-on utiliser des capa
cits sous forme dlments localiss telles que celles qui sont dcrites au
paragraphe 10.5.3 Ralisation des condensateurs .

10.2.1 Discontinuit dans la largeur


d'une ligne microbande
Les ralisations qui viennent dtre tudies prsentent des discontinuits dans la
largeur w de la ligne microbande. La figure 6 a montre une telle discontinuit et
la figure 6 b reprsente sa modlisation par une cellule en T comprenant deux
inductances sries L x, L2 et une capacit parallle Cs. La rfrence [52, p. 130
et sq/ tudie, parmi dautres, ce type de discontinuit.

Figure 5.
Ralisation dune
capacit parallle en
ligne microbande.

Figure 6.
a. Discontinuit dans
la largeur dune ligne
microbande.
b. Modlisation.

Les inductances L1 et L2 sont donnes par :

et

La valeur approche de Ls =

K ,l :
, = ___Z
1
L, + L w2 Ls

(5 )

^w2
L l = Lw
w
V
V
j + l w2

( 6)

est la suivante :

w,
tVi
\
Wi
L s (nU) = h 40,5 I - - 1 - 75 log + 0,2
w2
'w 2
w2

(7)

Cette relation est approche mieux que 5 % pour wi/w 2 Si 5 et w-Jh = 1.


Lw est linductance par unit de longueur dune microbande de largeur w, 2,
dimpdance caractristique Zm et de permittivit effective e, sa valeur est :
L w (H/m) =

z m \je e

(8)

3 108

Pour la capacit Cs, nous avons :


Wi
Cs (pF) = sJ w l w2 (10,1 loge + 2,33)------12,6 lo g e - 3 ,1 7

(9)

w2

Cette expression approche donne une prcision meilleure que 10 % pour :


r 10 et 1,5 w J/ vv2 = 3,5.
Dans le cas dun substrat dalumine (e,. = 9,6), Cs est fourni avec une prcision
de lordre de 0,5 % par :
w\
( w l)
pour 1,5 gi g 10
Cs (pF) =
w2 130 log| - 4 4
( 10)
w2

10.2.2 Cas particulier d'un circuit ouvert


Si lon raisonne selon les concepts des circuits constantes distribues, cette
capacit Cs a le mme effet quune augmentation AL de la longueur de la ligne
la plus large (fig. 6 a), compense par une diminution de la mme longueur de la
ligne la plus troite. Une expression approche de AL est :
A<? = A,0( l - ^ ) ,

(11)

Si w2 = 0, la discontinuit est un circuit ouvert et A i = Ai co ; Ai co est donc


laugmentation de longueur correspondant la capacit qui modliserait leffet
de bout d un circuit ouvert :
w
e , + 0,3 - + 0,264
( 12)
Aico - 0,412/t --------**
e -0,258

10.3

R a l is a t io n

d e c ir c u it s r s o n n a n t s

Nous avons signal au paragraphe prcdent la difficult quil y avait raliser des
capacits srie avec des tronons de ligne. Il en rsulte que les seuls circuits rson
nants que lon puisse raliser en lignes microbandes sont des circuits srie (fig. 7)
ou des circuits parallle (fig. 8 ) placs en drivation sur la ligne principale.

Figure 7.
Ralisation dun
circuit rsonnant srie
mis en drivation sur
une ligne microbande.

Z'

-Cl
Z

Figue 8.
Ralisation dun
circuit rsonnant
parallle mis en
drivation sur une
ligne microbande.

Pour ce qui est des circuits rsonnants srie ou parallle placs en srie sur la
ligne principale, ils ne peuvent tre obtenus, partir des prcdents, quen utili
sant la proprit quont les lignes quart donde dinverser les impdances. Nous
savons, en effet, que limpdance dentre Ze dune ligne X/4, dimpdance
caractristique Zt charge par une impdance Zs est : Ze = Z^/Zs.
Xl4

X/4

Figure 9.
Rseau quivalent
un circuit rsonnant
srie (Z,, C) mis en
srie sur une ligne.

C'
-IF

Z= 0

Ainsi, un circuit rsonnant parallle mis en drivation entr deux lignes quartdonde est quivalent un circuit rsonnant srie mis sur la ligne principale
(ftg. 9). Pour que les deux circuits soient quivalents, il faut que les impdances
vues leurs bornes A et B soient les mmes pour co = to0 (Z = 0) et pour t ^ 0)0.
Les conditions dquivalence (exercice 10.2) sont :13
Zz
(13)
Y1
(14)

et
0

Zc est limpdance caractristique de la ligne /4 ; Yc = 1/Z c.


De mme, un circuit rsonnant srie mis en drivation entre deux lignes quan
tionde est quivalent un circuit rsonnant parallle mis en srie sur la ligne
principale (fg. 10 ).
X/4
Figure JO.
Rseau quivalent
un circuit rsonnant
parallle (L\ C) mis
en srie sur une ligne.

Xl 4

L'

<=>

A
-

-y r a s NC

il
Z=.

Les conditions dquivalence sont exactement les mmes que dans le cas prc
dent.

Rseaux inverseurs d'impdance ou d'admittance


Il serait galement possible dutiliser dautres types de rseaux inverseurs que la
ligne X/A : rseaux quadripolaires ou longueurs de ligne, avec des lments
ractifs en srie ou en parallle (fg. 1la et 11b) [45, 52],

Figure 11.
a. Rseau inverseur,
dimpdances
b. Rseau inverseur
dadmittances.

Le rseau de la figure lia est un rseau inverseur d impdances : une rac


tance X = Lto ou X = - l/C t est mise en parallle sur une ligne dimpdance
caractristique Zc et de longueur . Les impdances dentre Ze et de sortie Zs
sont relies par un coefficient de transformation dimpdances K tel que :
j/ l
z e = -(15)
Le coefficient K se calcule daprs :
(16)

K = ZCtg
2n

avec
0 est li X par :

= - arctg

(17)
2X
Z.

(18)

Si X > 0 0 < 0 et i < 0. Il sagit dune longueur de ligne fictive qui devra
tre dduite des longueurs des autres lignes se trouvant de part et dautre de ce
rseau.
Si X < 0 0 > 0 et t > 0. Il sagit ici dune longueur de ligne relle.
Le rseau de la figure 11b est un rseau inverseur d admittances : une suscep
tance B = - 1/L( ou B = Cto est mise en srie sur une ligne dadmittance carac
tristique Yc et de longueur i. Les admittances dentre Ye et de sortie Ys, sont
relies par un coefficient de transformation dadmittance J tel que :
J2
Ye ~ ~
.S

(19)

Le coefficient J se calcule daprs :


' tg
avec : 0 = 27t/X (17).

0
2

(2 0 )

0 est li B par lu relation :

* 1 0 .4

2B
= - arctg -y-

q u iv a l e n c e

(21)

e n t r e u n e l ig n e a /4 o u

/2 e t u n c ir c u it r s o n n a n t

Des lignes de longueur X/A o u X/2 court-circuites ou en circuit ouvert sont


quivalentes, selon les cas, des circuits rsonnants srie (Z = 0) ou parallle
(Z = o). Mais, pour tablir cette quivalence, nous devons nous assurer que le
comportement dune telle ligne et du circuit rsonnant correspondant sont les
mmes lorsque la frquence varie autour de la rsonance.
a) Considrons le circuit rsonnant parallle de la
figure 12, G tant la conductance de pertes et Q le
coefficient de surtension en charge.
Figure 12.
Circuit rsonnant
parallle avec
conductance de pertes.

La susceptance de ce circuit, pour une variation A)


autour de 0o, est :

B=2

Aco
w

= 2C cu0

Ac
co

Le taux de variation de la susceptance au voisinage de )0 est :


(IB
2G
= 2C
dcu Q<o0

( 22)

(23)

Figure 13.
Variation de la
susceptance B en
fonction de la
pulsation <x>.

La variation exacte de B est donne par la courbe en pointills de la figure 13.

/
Pour une Jigne en courtcircuit de longueur ( =
(2n + 1 ) Xq/ 4 la pulsa
tion cOq, avec une conduc
tance de charge G (fg.
14), on dmontre [45]
que :

et :

Figure 14.
Ligne quart-donde
court-circuite avec
conductance de pertes.

Jt Aco
B =YC- ----2 c0

(24)

dB _
dto 4 / 0

(25)

Nous voyons quau voisinage de la frquence daccord, une ligne quart donde
en court-circuit prsente la mme variation de susceptance quun circuit rson
nant parallle condition que son admittance caractristique satisfasse la
relation :
Pt. = 8 C / 0

(26)

Notons toutefois que cette variation est une courbe en cotangente (en trait gras
sur la figure 13) alors quavec un circuit rsonnant il sagissait de la courbe en
pointills. La reprsentation dun circuit rsonnant parallle par une ligne
quart-donde en court-circuit ne sera donc valable quautant que les courbes
de variation de la susceptance peuvent tre confondues avec leur tangente
commune, soit en pratique dans une bande relative de quelques pour cent.
b) Considrons maintenant le circuit
rsonnant srie de la figure 15,
dont R est la rsistance de pertes et
Q le coefficient de surtension en
charge.

R
B -

La ractance de ce circuit pour une variation Aco autour de co0 est :


X = 2QR

Aco
co

= 2 /. coo

Aco
co

(27)

Le taux de variation de la ractance, au voisinage de co0, est :

M = 2QR = 2l
dco co0

(28)

Pour une ligne en court-circuit de longueur n X.0/ 2 la pulsation co0, en srie


avec une rsistance de charge R (fig. 16), on dmontre (exercice 10.4) que :

Figure 15.
Circuit rsonnant srie
avec rsistance de
pertes.

nXo
Figure 16.
Ligne demi-onde
court-circuite avec
rsistance de pertes.
e.
b

Comme au cas prc


dent, on remarquerait
que cette quivalence
n est valable que
dans une bande de
frquence rduite
quelques pour cent.

AAAAA
R

Aco
X = Zc n
Cy

(29)

=
doo 2 / 0

(30)

Une ligne demi-onde en court circuit se comporte donc comme un circuit rson
nant srie, condition que son impdance caractristique satisfasse la relation :
Zc = 4 L f0

iO - S

(31)

R alisation d e s im pdances p a r
DES LMENTS CONSTANTES LOCALISES

Aux frquences infrieures 3 GHz, les lments passifs constantes semilocalises que nous venons dtudier, ncessitent beaucoup plus de place que les
composants actifs. Il est donc normal de chercher laborer des composants
passifs constantes localises dont les dimensions doivent tre beaucoup plus
petites que la longueur donde (< X/10) afin de ne prsenter que des variations
de phase ngligeables. Vers les frquences suprieures, la limite dutilisation des
lments localiss dpend des possibilits offertes par les techniques de miniatu
risation. En 1996, la frontire semble stablir vers les 30 GHz.
La bibliographie relative aux lments localiss (lumped lments en anglais) est
assez disperse. Nous nous sommes rfrs, pour notre part, au chapitre de syn
thse rdig par Caulton [47]. Il fournit, par ailleurs, une importante liste de
rfrences o se trouve la dmonstration ou la justification des formules appro
ches ou semi-empiriques que nous donnons dans ce qui suit.

10.5.1 Ralisation des rsistances


La rsistance dune bande mtallique de longueur 6 , de largeur w et dpaisseur
t, est donne par :
(p, rsistivit)
Si lon considre une bande carre (w = 6 ), on notera sa rsistance :

(32)S
i

Dans le cas dune bande de cuivre on calcule que :


Rs = 2,6l 10 ~ V /

(34)

La rsistance dun carr est donc gale sa rsistivit p divise par son pais
seur t ; elle est indpendante des dimensions du carr. Nous exprimerons donc
Rs en ohms par carr. Pour calculer la rsistance totale dune bande, nous la
considrerons comme une succession de n carrs et sa rsistance vaudra donc n
fois la rsistance dun carr. Pour raliser des rsistances assez grandes, on peut
replier la bande mtallique en mandres ; le problme est alors de dterminer le
nombre de carrs quivalents une telle structure.
En fait, une bande mtallique nest quivalente une rsistance pure que si elle
se trouve dans un circuit de polarisation, par exemple, dcouple par une capa
cit et donc parcourue par un courant continu. En revanche, si cette bande est
utilise comme rsistance de charge dun amplificateur ou dun oscillateur, elle
se comportera comme une ligne constantes rparties. Cela signifie quelle pr
sentera non seulement une partie relle rsistive dont la valeur est donne par
(32) mais aussi une partie imaginaire inductive dont le comportement est tudi
ci-aprs ( 10.5.2.a)

10.5.2 Ralisation des inductances


a) Ruban mtallique

Figure 17.
Inductance ruban.

Figure 18.
Variation du facteur
de correction K de la
formule (36).

En espace libre, un ruban mtallique de largeur w, de longueur et dpaisseur t


(fg. 17) prsente une inductance par unit de longueur :
L (nH/cm) = 2 |ln

+ 1,19+ 0,22

(35)

En hyperfrquences, cette valeur doit tre rduite denviron 10 % pour tenir


compte de leffet de peau. La rsistance par unit de longueur de ce ruban est :

IVlCRO-ONDES

_K R s_
2(w + 1)

(36)

Rs est la rsistance superficielle en D /carr ; K est un facteur de correction qui


tien compte de la coupure des ligner de courant sur les bords du ruban. Il dpend
dont de w /te t ses variations sont reprsentes sur la figure 18.
Dans ces conditions, le coefficient de surtension prsent par un ruban de cuivre
(pour lequel Rs = 2 ,6 1 10 ~ 7 J J ) e s t.
(37)

Qcu = 4.81 102 -Jf L (nH/cm) ~

Pour un mtal de rsistivit p et en normalisant les frquences 2 GHz, on


dmontre que le coefficient de surtension est :
0 = 2,15 103 L (nH/cm)

p (Cu)

/(G H z)l

(38)

b) Inductance boucle
Linductance boucle, qui prsente un
encombrement plus rduit que la bande,
a la forme dun omga majuscule. Soit
a = (i + a2) / l son rayon moyen, w sa
largeur et t son paisseur.

Figure 19.
Inductance boucle.

L (nH/cm) = 2

__

vv + 1

est la longueur de la circonfrence de la boucle.


Lorsquelle est presque referme, nous prendrons : = 2na.
c) Inductance spirale

Figure 20.
Inductance spirale.

substrat

Figure 21.
Variation du facteur
de correction K
de la formule (41).

(w+ s )/w

(39)

Les inductances obtenues avec une bande ou une boucle sont de lordre du
nanoHenry et leurs coefficients de surtension peuvent tre de lordre de 100, ou
plus, pour des rapports /w < 10. Pour avoir des inductances plus importantes, il
faut prendre des longueurs plus grandes, ce qui conduit utiliser des spirales
comportant un nombre n de spires.
Pour une spirale n spires, dont les paramtres gomtriques sont indiqus sur
la figure 2 0 , une expression approche de linductance est :
L (nH) = 393 - v r - v r 8a + 11c
avec :

(40)

dD+ d;
da - dt
a = - (cm) et c = - (cm)

Cette expression est valable dans lapproximation BF et si : da > 1,2 dt ; n > 1 et


t > trois paisseurs de peau.
La rsistance de la spirale est :
J?c
R = K 'n n a
w

(41)

Rs est la rsistance superficielle en Cl/carr ; K' est un facteur de correction per


mettant de tenir compte des couplages entre deux spires voisines. La figure 2 1
montre la variation de ce facteur en fonction du rapport (s + w)/w.
Dans ces conditions, le coefficient de surtension de la spirale est :
Qs = 2 109f L

\L (8 a + U c ) \ ~ l

(42)

Une autre approximation, due Bryan [48] et valable si n 10 est :


L (nH) = 21,5a n 1,5 ln
c

(a est en cm)

(43)

Il est possible dobtenir des inductances plus grandes sur une mme surface en
utilisant des spirales carres. Bryan a donn une expression approche de
leur inductance :
5/, 8 b
L (nH) = 0,24 b cm n /3 ln
(4 4 )
o b (qui remplace le a des formules de la
spirale circu laire) est la dim ension
moyenne dun demi-ct du carr.
Cette relation peut aussi tre utilise sous
une autre forme, en faisant intervenir la sur
face S de la spirale carre :

Figure 22.
Inductance
spirale carre

d) Domaine de validit des formules des inductances


Toutes les valeurs de L qui ont t donnes dans ce paragraphe ne sont valables
que si les mtallisations formant les inductances peuvent tre considres
comme isoles dans lespace. Or, ces mtallisations sont ralises sur des lignes
microbandes comportant un plan de masse sur la face du dilectrique oppose
la mtallisation. On dmontre [47, p. 159-160] que la condition prcite nest
remplie que si le produit de \j~^r par limpdance caractristique de la ligne a
une valeur au moins gale 300 fi.

10.5.3 Ralisation des condensateurs


a) Condensateurs en pav
Figure 23.
Reprsentations
tridimensionnelle
et en coupe dun
condensateur en pav.
Ces condensateurs se prsentent sous la forme dun paralllpipde rectangle en
dilectrique dont les faces suprieure et infrieure sont recouvertes de mtallisa
tion. Pour un pav de surface S, dpaisseur d et de constante dilectrique rela
tive er, la capacit est ;
C (pF) = 8,84

(46)

La rsistance des plaques mtalliques du condensateur est :


2
Rs -C .

(Rs, rsistance superficielle en fi/carr)

Il lui correspond un coefficient de surtension :


_3
1
w
C, " 2 ' t o C R S ' i
La conductance due aux pertes dans le dilectrique est :
J_
1
Qd Ce

(47)

(48)

(49)

o Q j = l/tg 6 est le coefficient de surtension dilectrique et 5 est langle de


pertes du dilectrique. Notons que tg 8 est peu prs constant pour les dilec
triques utiliss en hyperfrquences.

Le coefficient de surtension global du condensateur est donc :


_J_
Q 0,
1

l
Qd

(50)

Exemple
De tels condensateurs utilisent souvent du dilectrique en silice (Si 0 2) dont la
constante dilectrique relative er vaut 2,25. Pour S - 1 mm2 et d = 10 p., nous
obtenons une capacit de 2 pF.
b) Condensateurs structure interdigitale
Ces capacits interdigitales sont une structure plane (fig. 24) de N minces
bandes conductrices parallles, de longueur , relies alternativement lune ou
lautre des deux bandes, de longueur w, qui leur sont perpendiculaires. Le tout
est dpos sur la surface dun substrat qui est souvent de lalumine (er = 10 ).
Alley / 49] a calcul sa capacit :

e +1
C (pF) = ---- |( N - 3 ) A ,+ A 2|

(51)

Aj (pF) = 8,85 10 2 w (cm) et A2 (pF) = 9,92 10 2 w (cm).

Figure 24.
Condensateur
structure interdigitale.

Les valeurs couramment obtenues sont de lordre de 0,1 15 pF.


MB

Ces formules ne sont valables que si l paisseur du dilectrique est suffisamment


importante (> I mm) par rapport l espacement des doigts (qui est de l ordre de
10 100 pm). Quant l paisseur de la mtallisation, elle na quune impor
tance secondaire.
La rsistance dun tel condensateur est :

Son coefficient de surtension est donn par :


1

Qc =

cCR

(53)

Des coefficients de plusieurs centaines peuvent tre obtenus des frquences de


quelques GHz pour des condensateurs de quelques pF.

2e partie
GUIDES D'QiyDE;
ET

Chapitre 11
Rflexion et: rfraction
des ondes
lect 'magntiques

11/1

I n t r o d u c t io n

Dans ce chapitre, nous allons tudier la rflexion dune onde lectromagntique


(O.E.M.) sur une plan conducteur ainsi que la rflexion et la rfraction dune
O.E.M. sur une interface entre deux milieux dilectriques. Ces phnomnes phy
siques sont, en effet, la base des tudes que nous effectuerons dans les cha
pitres suivants au sujet de la propagation des ondes en guides dondes mtal
liques et dilectriques.
Nous demandons au lecteur dadmettre les quelques proprits suivantes dune
O.E.M., qui seront dmontres dans le chapitre 6 Propagation des O.E.M. en
espace libre du second volume de cet ouvrage.
-*

1) Une O.E.M. est constitue dun champ lectrique E et dun champ magntique H qui forment un tridre direct avec la direction de propagation ; soit
u le vecteur unitaire de cette direction, nous avons :

(i)
ou

( 2)

et p. sont la permittivit et la permabilit magntique du milieu o


seffectue la propagation. Dans le cas de lair ou du vide :

= e =
0

36t 109

(F/m)

p = p 0 = 4t 10

et

(H/m)

Dautre part, et H forment un plan perpendiculaire la direction de propa


gation que lon appelle le plan donde.
2) Lquation de propagation en espace libre dilectrique, selon une direction
Oz, des champs lectrique et magntique, pris en valeurs instantanes complexes e(z, t) et h(z, 0 est :
d.. 2 e-* ou hT

32e ou h
dr

=0

(3)

En rgime sinusodal, ces quations admettent des solutions de la forme :


I(z, t) = E(z) e ^

(4)

E(z) est lamplitude complexe :


E(z) = 0 e ~J40 y = E0 e ~j*z
1

(5)
( 6)

est la vitesse de propagation de londe


2n
v

(7)

est le paramtre de phase de londe.


h(z, t) = H(z) eiy

( 8)

K(z) = HQe "* l = H0 e ~ikz

(9)

De mme :
avec :

Le rapport des modules de E et H sexprime en ohms :


E _ Eq _
H ~H o~Q

( 10)

Il est appel impdance donde et a pour valeur :


( 11)

Dans lair ou dans le vide :


=o=
3) Les conditions aux limites linterface dilectrique-conducteur sont :
-9
- la composante tangentielle du champ E est nulle :

( 12)

et

=0

(13)

- la composante normale du champ Fi est nulle :


Hn = 0
(14)
Les conditions aux limites linterface entre deux milieux dilectriques
parfaits 1 et 2 sont :
- continuit des composantes tangentielles des champs E et H , soit :
ET , - Er 2
H t | Hr 9

et

(15)
(16)

- continuit des composantes normales des vecteurs D = e E et B = p.H ,


soit :
ei ^

et

11.2

l = e 2 E tf' 2

(1 7 )

M-i Hn , l P-2 ^ n , 2

(18)

R f l e x io n

n,

s u r un pla n c o n d u c t e u r

SOUS INCIDENCE NORMALE

Figure 1.
Rflexion sous
incidence normale.

11.2.1 Rflexion la surface d'un conducteur


parfait
Le milieu (1) est un dilectrique caractris par e et p dans lequel se propagent

* *
t
londe plane incidente (Ei , Hi ) et londe plane rflchie {Er , Hr ). Le milieu (2)

est un conducteur parfait lintrieur duquel le champ lectromagntique est


nul. X est linterface entre ces deux milieux (fg. 1).
La direction de propagation de londe incidente tant normale la surface X, un
plan donde quelconque de cette onde est parallle X et il en est de mme pour
m
Ej et Ht . Si nous prenons un axe de rfrence Oz parallle la direction de pro>

>

>

>

pagation (fg. 1), . et Ht sont donns, en valeurs instantanes complexes par :


( z , i ) = ? -e

(19)

( z ,r ) = ^ e j (tf- ta)

(20)

impdance donde du milieu (1)

Cette onde incidente donne naissance une onde rflchie (Er , Hr ) qui se pro
page perpendiculairement X- Lexpression des champs Er et Hr est donne, en
valeurs instantanes complexes, par :
r (z, t) = Er e >to' +kz)

(21)

t r (z,t) = Hr e'iia,+kz)

(22)

Le tridrc form par les vecteurs (, //, z ) ainsi que celui form par les vec
teurs (iEr , Hr, - z) doivent tre directs. Il en rsulte donc que :

(E j, Hj ) et ( Er , Hr ) sont aussi lis par les conditions aux limites imposes par
le plan conducteur X :
= 0 et HN = 0. Les champs incidents et rflchis tant
parallles X, il ny a pas de composantes normales et la seule condition que

* >
lon doit crire est : Ej + Er = 0
soit :
II rsulte enfin de (23) et (24) que :

Er = - E j

(24)

H=Hj

(25)

Si lon se donne le champ E j, lorientation de // rsulte de lapplication de la


rgle du tridre direct et celles de Er et Hr rsultent des relations (24) et (25).
Ltat lectromagntique en un point d abscisse z du dilectrique rsulte de la
superposition des champs de londe incidente et de londe rflchie :

\ (z , t) = E j eJ

= Ei (e

+ E r e.i

- e

e ^ = - 2j f sin kz e 'tof

(26)

h(z,D = Hi ej (y fo) + /7 ,e i(oy+fa)


= //, (e ~ + e j*z) e - ^ = 2 H-t cos &z e ^

(27)

Comme (26) peut encore scrire :


e (z,t) = 2 Ei cos

|te j

- - e J(Cf~

(28)

nous constatons que les champs lectrique et magntique sont constamment en


quadrature dans le temps et dans lespace. Cela signifie quil existe dans le
milieu dilectrique un rgime dondes stationnaires :
dans le plan X (z = 0) et dans tous les plans situs n X/2 de X. le champ
lectrique est nul et le champ magntique est maximal ou minimal ;
dans tous les plans situs (2n + 1) ./4, cest le contraire : le champ lec
trique est maximal ou minimal et le champ magntique est nul.
Remarquons enfin que les problmes de polarisation ne prsentent pas de diffi
cult dans ce cas de lincidence normale puisque les champs lectrique et
magntique gardent une polarisation uniforme en tous points de lespace.

11.2.2 Rflexion la surface


d'un conducteur imparfait
-9

Daprs le paragraphe prcdent, les amplitudes complexes des champs ( E HOx

t
et tlHOy) le* onde* ineidenle et u'ikvluo se piopageniu dans le dilectrique,
sont donnes par :
(29)
(30)
(31)

E rx
j =

(32)

p /e est limpdance donde du dilectrique.

Puisque le conducteur nest pas parfait, la rflexion nest pas totale et il y a une
onde transmise de la forme :
E
F e - Yz
IX= t

(34)
c. est limpdance donde du conducteur imparfait.
Comme la rflexion et la transmission seffectuent sur une interface avec un
milieu imparfait, nous avons pris f, Er et Et complexes.
Nous dmontrons au chapitre 6 du second volume Propagation des O.E.M. en
espace libre que le paramtre de propagation y = a + jp dans un milieu
conducteur imparfait est tel que :
(35)

Dautre part, limpdance donde de ce milieu est donne par :


(36)

Nous pouvons donc crire :


(37)
(38)

Les quations de continuit des composantes tangentielles de E et H pour z = 0,


nous donnent :
(39)
, + r = ,
( ,- ,) = , ; 1

(40)

do:

(41)

et :

(42)

Le coefficient de rflexion complexe est donc donn par :


K- 1

1*1-3

R fle x io n s u r un plan c o n d u c t e u r
SOUS INCIDENCE OBLIQUE

Figure 2.
Rflexion sous
incidence oblique
Gomtrie du
problme.

11.3.1 Position du problme


Pour traiter cette question, nous choisirons un tridre de rfrence Oxyz tel que
yOz concide avec le plan dincidence et xOz avec le plan conducteur. Le plan
dincidence de londe est le plan dfini par la direction dincidence de londe et
la normale X (fig. 2).
Considrons une onde incidente plane dont la direction de propagation fait
langle xj/ avec la surface plane X du milieu conducteur. Cette onde va donner
naissance une onde rflchie dont la direction de propagation fait aussi langle
\|/ avec X.
Ltat lectromagntique en un point quelconque du dilectrique rsulte de la
superposition de londe incidente (O.I.) et de londe rflchie (O.R.). Pour
trouver cet tat, il nous faut dterminer lexpression mathmatique des champs
correspondant aux ondes incidente et rflchie :
pour ltude de londe incidente, nous rapporterons la figure un systme
daxes Oxy'z' dans lequel Oz est parallle la direction de cette onde ;
pour ltude de londe rflchie, nous rapporterons la figure un systme
daxes Oxy"z dans lequel Oz" est parallle la direction de cette onde.
Nous nous ramnerons ensuite au systme daxes Oxyz grce aux formules de
changement daxes :
z = - y sin \|/ + z cos \|/ ; z = y sin \|/ + z cos xj/
Dans ces conditions :

lamplitude complexe de lune des composantes de lO.I. est de la forme :


A t = A e = A e iky sin V e -->fa cos V
(44)
lamplitude complexe de lune des composantes de lO.R. est de la forme :
A r = A c - = A c "J ^ sin Ve - j fc cos V
(45)

A reprsente lamplitude lorigine de la composante du champ E ou H consi


dre.
Ces expressions ne nous donnent que lamplitude et la phase des champs : reste
le problme de la polarisation. Il est particulirement intressant de traiter les
deux cas fondamentaux de polarisation suivants :
champ lectrique perpendiculaire au plan dincidence (et alors le champ
magntique se trouve dans le plan dincidence) cest le cas T.E. ;
champ magntique perpendiculaire au plan dincidence (et alors le champ
lectrique se trouve dans le plan dincidence) cest le cas T.M.
Il y deux raisons cela :
dune part, un champ lectrique (ou magntique) ayant une polarisation quel
conque peut toujours se dcomposer en une composante perpendiculaire au
plan dincidence et une composante dans le plan dincidence ;
dautre part, si le champ incident Ei (ou Hi ) est perpendiculaire au plan din

- >

cidence, le champ rflchi Er (ou Hr) est lui aussi perpendiculaire au plan
dincidence. Le champ total lectrique (ou magntique) a donc une polarisa
tion uniforme dans tout lespace.
Cest en fait une dmarche qui est trs gnrale pour traiter tous les problmes
de rflexion, rfraction ou diffraction des ondes : plutt que de rsoudre ces pro
blmes dans un cas de polarisation quelconque, ce qui ncessiterait des calculs
trs lourds et dont la signification physique serait difficile saisir, on prfre les
rsoudre dans les deux cas fondamentaux de polarisation. Un cas de polarisation
quelconque peut ensuite tre trait trs facilement en superposant de faon
approprie, les rsultats obtenus dans les deux cas fondamentaux de polarisation.

11.3.2

Figure 3.-*
Champ E
perpendiculaire au
plan de la figure .
dirig de
larrire vers lavant ;
dirig de
lavant vers larrire.

est perpendiculaire
au plan d'incidence (cas TE)
E

a) Onde incidente
Prenons ( dirig darrire en avant du plan de la figure 3. Comme le tridre
form par Et , Ht et la direction de propagation doit tre direct, H; est dirig vers
le bas ; il a donc :
une composante normale la surface X : HiN
une composante tangentielle la surface X : Hir
Les expressions de ces champs en amplitudes complexes sont :
4

= - e ^ sint 'e 'jfecosV

(46)

H iy = - H cos \|/ e*ky sin V e _ c o s v

(47)

H iz = - H sin V e & sin ^ - j fc cos V

(48)

E et H reprsentent les amplitudes prises lorigine du champ lectrique et du


champ magntique de londe incidente. Leur rapport E /H est limpdance
donde du milieu dilectrique.
Si le milieu est un dilectrique sans perte, les champs E et H sont en phase.
-* Dans ces conditions, les amplitudes E et H dans les relations (46) (48) sont
relles et l'impdance donde est une quantit relle. Cest le cas que nous consi
drons dans ce paragraphe.
Mais si le milieu tait un dilectrique avec pertes, son impdance donde serait
complexe et il faudrait alors considrer que les amplitudes E et H sont des quan
>
ths complexes s i bien que les champs correspondants E et H ne sont pas en
.

phase.

b) Onde rflchie
Dans londe rflchie, compte tenu des conditions de continuit, le champ lec
trique reste perpendiculaire au plan dincidence yOz et il na donc quune seule
composante selon x ; le champ magntique reste dans le plan dincidence o il a
deux composantes, lune selon > et lautre selon z. Avant de donner les ampli
tudes complexes E rx, H iy et H n de ces composantes, nous devons dterminer
lorientation des champs rflchis.
Celle du champ Er sobtient en crivant la condition aux limites qui est impose
sur X au champ lectrique tangentiel Er = 0, soit ici : Ei + Er = 0, puisque les
champs Et et Er sont parallles X. Er est donc dirig de lavant vers larrire
de la figure 3 et puisque le tridre form par Er , Hr et la direction de propaga
tion, doit tre direct, Hr est dirig vers le haut.
Les amplitudes complexes des composantes du champ rflchi scrivent donc :
E rx = E e ~*ky sin V e _j*z cos v
U = H cos y e ~'*ysin V e - Jfe cos V
H rz = - H sin y e ~iky sin V e -

cos V

(49)
(50)
(51)

c) Champ total
En un point quelconque du dilectrique, ltat lectromagntique rsulte de la
superposition de ces deux ondes incidente et rflchie. Les amplitudes com
plexes des composantes des champs rsultants sont :
Kx = - 2j E sin (ky sin y ) c~ 'kzcos V

(52)

H y = - 2j H cos y sin (ky sin y ) e ~]kz cos ^

(53)

H l = - 2 H sin y cos (ky sin y ) e ~ikz cos v

(54)

Il est maintenant possible de savoir dans quel sens se propage lnergie, en cal
*

>

culant les trois composantes du vecteur de Poynting : = ~ A


Px = \ l ( E y H*z - E z H*y) = 0

(55)

Il ny a pas de puissance transporte selon la direction Ox.


Py =^

z K*x - E x H l) = - ^ E x H l

P y = - 2j EH sin y sin (ky sin y ) cos (ky sin y )

(56)
(57)

Cette expression est imaginaire pure : la puissance qui se propage selon la direc
tion Oy est de la puissance ractive.
P z = l2 (Ex f i ; - E y H*x) = ~ E x H ;

(58)

P = 2 EH cos y sin2 (ky sin y )

(59)

Cette expression est relle : la puissance qui se propage selon la direction Oz est
de la puissance active.
La propagation est donc caractrise par lexistence dun rgime dondes sta
tionnaires pures dans une direction perpendiculaire X et dun rgime dondes
progressives dans la direction Oz. Dans une direction quelconque, on observerait
un rgime dondes semi-stationnaires.
Enfin, il est intressant de dterminer limpdance donde dans la direction Oz de
propagation de la puissance active. Il sagit de la composante ,z de cette imp
dance qui est le rapport des composantes de et H perpendiculaires Oz :
(60)

z tL y

H cos y

ce que nous crirons :

t e - cos y

(61)

11.3.3

est perpendiculaire
au plan d'incidence (cas TM)
H

Figure 4.
Champ H
perpendiculaire au
plan de la Figure :
dirig de
larrire vers lavant
& de lavant vers
larrire.

a) Onde incidente
Prenons Hi dirig darrire en avant du plan de la figure 4. Ei , qui est dans le
plan de la figure, est donc dirig vers le haut et il a :

une composante normale la surface X : EiN


une composante tangentielle la surface X : ,r
Les expressions de ces champs, en amplitudes complexe, sont :
ljx - H e jA>'sin V e _

cos v

(62)

E iy = E cos Ig e & sin V e " *kz cos v

(63)

E iz = E sin y e & sin * e ~ikz cs V

(64)

Mme remarque pour E e tH quau paragraphe 11.3.2.a.


b) Onde rflchie
Dans londe rflchie, compte tenu des conditions de continuit, le champ
magntique reste perpendiculaire au plan dincidence et le champ lectrique
reste dans le plan dincidence o il a deux composantes, lune selon y et lautre
selon z. Avant de donner les amplitudes complexes H rx, E i y e t E rzde ces com
posantes, nous devons dterminer lorientation des champs rflchis.
Hj et Hr tant parallles la surface X, nous ne pouvons rien dire sur lorientation
i)

->
de Hr, par rapport celle de Hi , puisque la condition de continuit du champ
magntique total la surface de X ( HN = 0) porte sur la composante normale. Ce
que nous savons, en revanche, cest que : EiT + ErT = 0 sur X-

Cette condition nous montre que ErT = - jT et permet de dterminer Er qui


doit tre perpendiculaire la direction de Fonde rflchie, dune faon unique :
Er est dirig vers le haut de la figure. Il en rsulte donc que Hr = //,. Par cons
quent, les amplitudes complexes des champs rflchis scrivent :
/ / _ _ yy e - jity sin y e - j kz cos y
E ^ = E cos q/ e ~sky sin v e ~kz cos V
E rz = - E sin v e ~iky sin v e -->*zcos V

(65)
(66)
(67)

c) Champ total
Les amplitudes complexes des champs rsultant de la superposition des compo
santes de ces deux ondes sont :
H X= - 2 H cos (ky sin y) e ~*kz cos v

(68)

E V= 2E cos y cos (ky sin \|f) e ~*kz cos v

(69)

E . = 2j E sin vj/ sin (ky sin y ) e ~>kz cos v

(7U)

Par un calcul tout fait analogue celui effectu dans le cas prcdent, partir
des composantes du vecteur de Poynting complexe, nous dmontrerions que
P x est nulle, P y est imaginaire pure et P , relle. Cela signifie quil ny a propa
gation de puissance active que selon Oz et amne aux mmes conclusions que
dans le cas T.E. quant la nature des rgimes dondes stablissant dans le
dilectrique.
Limpdance donde dans la direction Oz est donne par le quotient de E.y par
H x ; comme lordre des oprations est effectu en sens inverse de la permutation
circulaire directe sur (x, y, z) nous devons affecter ce quotient dun signe moins :
Ky E
Wx = H C ^

(71)

ce que nous crirons :


^TM - cos V

R flexion et
l' interface

(72)

transmission
de d eu x dilectriques

Nous nous proposons de dterminer les coefficients de rflexion R et de trans


mission T la surface de sparation de deux milieux dilectriques. Comme pour
ltude de la rflexion sur un plan conducteur, nous nous placerons dans les deux
cas fondamentaux de polarisation : les cas T.E. (T.M.) dans lesquels le champ
E(H) est perpendiculaire au plan dincidence.

Nous ne ferons aucune hypothse prliminaire sur lorientation relative des


champs incidents, rflchis et transmis. En effet, si nous avons toujours le droit de
donner une orientation Ei (et, par consquent, H;), nous ne savons rien quant
celles de Er , Hr, E, et Ht , puisque ce sont prcisment les signes de R et T
qui vont nous les donner.

11.4.1 Le champ est perpendiculaire


au plan d'incidence ( C a s TE)

Figure S.
Les champs E si
perpendiculaires
plan de la figure.

Nous allons crire les conditions de continuit des champs E et H la surface


de sparation de deux milieux dilectriques. Comme elles sappliquent aux com
posantes tangentielles des champs, il est important de remarquer que, dans ce
cas de polarisation (fig. 5) :
* t a illillll^ pta'-U'HjW Mil hWiiPillcU il V
les composantes tangentielles des champs magntiques sont en cos 0.
En passant immdiatement aux amplitudes complexes, nous avons :
jE i +E r =E t

(73)

U ^ / + ^ r) cose i = /f ,c o s e 2

(74)

Er
Comme :

E,
Si-

Zi
^ E = E t
il vient :

1E r E = E t

(75)
Cl

cos02

2 cos 0 1

(76)

(75)/,.

1 + R F T F

-+

(77)

'

(75)+ (76) -+

.cosG ,'
1
+
------2 Ei = E t
2 c o s e ij
V

(78)

2 co sG j

et :

E. = 2 E ; '
1 j cos 02 + 2 cos 0 1
2 2 cos 0 j

do :

Te =
x 2 cos 0 j + j cos 02

Daprs (77) :

(79)

RF = TE
ci -1
2 cos 0 j j COS02

do :

Rp =
'X

2 COS0! + ( COS02

(80)

Remarque

Puisque (Hr/H j) = - (E r/ E ,), il suffit de changer de signe le coefficient de


rflexion relatif au champ E pour obtenir le coefficient de rflexion relatif au
champ H ;
Rh - - Re
(81 )
En ce qui concerne le coefficient de transmission,
i = E, ^
H, e , c2

Donc :

En gnral, p, = p2 = Hq. Donc : j =

(82)

et 2 =

- Les for

mules (80) et (79) deviennent alors, aprs multiplication du numrateur et du


dnominateur par J
e2 , puis simplifications :

Nous voyons tout de suite que si TE est toujours positif, par contre RE peut,
selon les valeurs respectives de y j 8j cos0j et
z 2 c o s 2 tre Positif ou
ngatif. Do les deux cas de figure 6 et 7, en prenant pour hypothse que est
dirig de larrire vers lavant.
Figure 6.
Cas o Ej > 2
alors 0j < 0 2
=> co s 0j > co s 0 2.

Figure 7.
Cas o E] < e2
alors 0 , > 0 2
=> co s 0[ < co s 0 2.

Champ dirig de
1auiiv \ers Tavant
{'avant vers l'am ne.

11.4.2 Le champ H est perpendiculaire


au plan d'incidence (Cas TM)
Remarquons tout dabord que, dans ce cas de polarisation (fig. 8) :
les champs magntiques sont tangentiels E,
les composantes tangentielles des champs lectriques sont en cos 0.
En passant immdiatement aux amplitudes complexes, les conditions de conti
nuit des composantes tangentielles scrivent :
( a , +H r d ,
K , + r)co s0 | =,co:S02

Compte tenu de :

Ei
tU

(85)
(86)

El
=f et de = r
Sl
H
, s2
tr
,
Er E = -rE t

(87)

)
cos02
E i+E rE,
I '
r cos 0 j 1

(88)

(87) /Ej

-*

l
l ~ R En

(89)

^2
COS02

(87) + (88)

->

cosQj

(90)

, COS 0 J + ., COS02

et :

2 E = E
'
'

do :

2 cos0j

2 2 cos 0
TP =
" , c o s 0 ,+ 2 cos02

(91)

2 cos02 - , cos0 i
RF =
" , c o s0 ,+ 2 cos02

(92)

Figure 8.

>
Les champs H sont
perpendiculaires au
plan de la figure.

Daprs (89) :

do :

Remarque

Nous avons, comme dans le cas prcdent :

(93)
et

Dans le cas gnral o


obtient enfin :

(94)

JJ.[ = |X2 = M-o* i = \ J M-o/e i 2 = \ J lJ-(/e2 et ^ on

(95)

(96)
Dans ce cas galement, TE^ est constamment positif alors que RE^ peut tre positif
ou ngatif selon les valeurs respectives de

Ej cos02 et

e2 cos 0j.

Pour chacun des deux cas de figure 9 et 10, nous avons suppos que Hi est dirig
de larrire vers lavant.
Figure 9.
cos 02
cos 0,
Figure 10.
cos 02
cos 0j
Champ dirig de
larrire vers lavant
champ dirig de
lavant vers larrire

11.4.3 tude des variations des coefficients


de rflexion
Ltude de ces variations est particulirement intressante car elle va nous per
mettre de prciser dans quels cas ils peuvent tre positifs ou ngatifs et dans
quelles conditions il peut y avoir rflexion totale ou, au contraire, rflexion
nulle. Nous allons effectuer cette tude en utilisant les coefficients relatifs au
champ lectrique que nous noterons dsormais Rx et R/f pour plus de simplicit.
Nous voyons immdiatement daprs la loi de Descartes
^/^ sin 0, = ^ / j sin 02
que si Ej > e2, 0! < 02 et que si Ej < e2, 0j > 02.
a) Cas o e-, < e2 (fig. 11)
Les intervalles de variation de 0[ et 02 sont : 0 < 0j < rc/2 et 0 < 02 < Q2l-

(97)

Voyons quelles sont les valeurs de V?x et R pour les valeurs extrmes de 0j :

Les deux coefficients de rflexion sont gaux et ngatifs.


Si 0j = rt/2, 02 = 02l R = - 1 et R// = + 1.
11 y a rflexion totale avec changement de signe lorsque Ei est perpendiculaire
au plan dincidence et sans changement de signe lorsque Ei est dans le plan
dincidence.

b) Cas o e 1 > e2 (fig. 12)

Les intervalles de variation de 0j et 02 sont : 0 < 0j <, 0 ^ et 0 S 02 ^ ft/2.


Langle 0j limite est donn par :
sin 0i L = \f^ 2

(loi)

(102)

Donc :

Figure 12.
Variation des
coefficients R// et R
si e, > en

voyons quelles sont les valeurs de R et R// pour les valeurs extrmes de 0j :

Les deux coefficients de rflexion sont gaux et positifs.


Si 0j = 0j et 02 = 7t/2 /? = 1 et R,f = - 1.
Il y a rflexion totale sans changement de signe lorsque ( est perpendiculaire
au plan dincidence et avec changement de signe lorsque Ei est dans le plan
dincidence.
c) Rfraction totale
Dans lun et lautre cas qui viennent dtre tudis, R reste ngatif (si Ej < e2)
ou positif (si Ej > e2) tandis que Rf/ change de signe et sannule pour une valeur
0B de 0] qui est calcule (si Ej < e2 ou E| > e2) daprs :
\J^2 COS

= \J~7 cos 02

\J~\ sin 0fi =

sin

En faisant le produit de ces deux quations, nous trouvons que sin 2QB = sin 202,
ce qui ne peut tre vrifi ici que si QB = (7t/2) - 02. Dans ces conditions :
sin 0B =

cos

(104)

do

Cet angle 0B est appel angle de Brewster et lincidence correspondante, inci


dence brewsterienne . Si e, < e2,
> 45 (fig. 11) et si Ej > e2, 0e < 45
(fig. 12). Il y a alors rfraction totale, en polarisation parallle.
d) Rflexion totale
Dans un systme daxes de rfrence Oxyz ayant laxe des x parallle linter
face X et laxe des z perpendiculaire X dirig vers le bas (fig. 13), les champs
lectriques de londe incidente dans le milieu 1 et de londe transmise dans le
milieu 2 scrivent, en amplitudes complexes :
( = 'e -j* ixsmei e -j* izcosei

(105)

E, = T E e ~ *2*sin e2 e - J** cos e2

(106)

Figure 13.
Interface entre deux
milieux de
permittivits E, > E2.
- J*- Cas o 0, < 0lL
Cas o 0, > 01Z_

Daprs la loi de Descartes : sin 02 = J E j/e2 sin 0j . Si E, > e2, nous avons vu
quil y avait rflexion totale pour 0 1L = arcsin

e2/ ej

Si 0j > &lL, sin 02 > 1. Dans ces conditions, cos 02 est imaginaire pur :
cos 02 = j y j sin2 02 - 1

Lexpression (106) de londe transmise scrit alors :


E = T E e k2Z' / iiin2

[ -j*2Jfsin02

(108)

Seul le signe moins correspond une onde ayant une signification physique : il
sagit dune onde qui se propage le long de linterface S entre les deux milieux
(fig. 13), puisque la variation de phase na lieu que selon laxe des x, et dont
lamplitude diminue exponentiellement lorsque lon scarte de linterface selon
laxe des z. L onde qui existe dans le milieu 2 est donc une onde vanescente.
Compte tenu de la valeur de cos 02 = - j ^^sin2 0? - 1 les coefficients de
rflexion R et R//y calculs d aprs (83) et (95) sont complexes. Le champ
rflchi a donc un dphasage par rapport au champ incident, qui dpend de la
polarisation utilise. 11 en rsulte quune onde incidente polarisation rectiligne,
non situe dans le plan dincidence, va se transformer, aprs rflexion totale, en
une onde rflchie polarisation elliptique puisque les composantes parallle et
perpendiculaire de cette onde, sont dphases lune par rapport lautre.

11.4.4 Rfraction d'une onde


dans un dilectrique pertes
Nous nous intressons au cas o londe provenant dun milieu sans perte, de per
mittivit j, se rfracte dans un milieu pertes, de permittivit e2 complexe.
Dans ce cas, nous crirons la loi de Descartes sous la forme :
sin 0 f = s j z r sin 02
avec:

e r = = Er - j e
ei

(109)
( 110)

sin 02 est une quantit complexe et 02 ne reprsente pas langle de rfraction 02,
cest pourquoi nous lavons not 02 .
Dans le cas o le milieu 2 tait sans perte, nous avons dj vu que lexpression
de londe rfracte pouvait scrire, en amplitude complexe :
p _ ^ e - j L ri sin 0 i+ z tos ^2>
avec :

^2 = ^0 \ / er2 =

\ J Er

( 111)

( 112)
(113)

Dans le cas o le milieu 2 a des pertes, nous pouvons donc crire lexpression du
champ rfract sous la forme :
p

e - j * i y M * sin 0'2 + j cos e2)

Dans cette expression, . / er sin 02 = sin 0 } est rel, et :


r c o s O ^ y ^ ^ / l - s i n 2 ^ = ^ /E r - s i n 2 j

(115)

est complexe, de la forme p - j q , avec :


P=

\J(er - sin2 0 j)2 + e2 + (er - sin2 0 L)j2

(116)

q=

\J(er - s i n 2 0 ])2 + e2- (er - sin2 0j)]2

(117)

Avec ces notations, lexpression du champ rfract dans le milieu 2 peut


scrire :
E t = A e ~

J* i* sinei e - j * i z ( p - j 4 )

E t =A e~ k

e - j ki(.x sin

+pz)

Les plans quiphases de londe rfracte sont dfinis par :


x sin 0, + pz = cte

(118)

(119)

Figure 14.
Rfraction dune onde
dans un dilectrique
pertes.

Ces plans font avec la surface de sparation entre les deux milieux, un angle 02
identique celui de la direction de londe rfracte avec la normale linterface
(fig. 14).
Pour calculer cet angle, diffrentions la relation (119) : (sin 0 t) dx + p dz = 0.
dz
x

sin 0j

Langle de rfraction tant ainsi dtermin, il est possible de dfinir une permit
tivit relle quivalente | q, pour le milieu pertes, telle que :
( 121)

Cette permittivit est celle dun milieu sans perte qui produirait la mme rfrac
tion que le milieu pertes considr. Lanalogie doit sarrter l car, dans un
milieu sans perte, les surfaces quiphases sont galement quiamplitudes tandis
que dans le milieu pertes, la relation (118) montre quil y a une variation de
lamplitude en e " *1 qi.

E X E R C IC E 1 1 . 1
(Paragraphe 11.2 2 - Rflexion la surface
d'un conducteur imparfait)

Leau de mer est caractrise par une conducti


vit a = 3 (S m " *12) et une permittivit relative
er = 70.
1) Lorsque a/co 0 f > 10, on peut considrer
que leau de mer se comporte comme un
conducteur imparfait. partir de quelle fr
quence cette proprit est-elle vraie ?
2) la frquence de 10 MHz, calculer la
valeur du coefficient de rflexion linter
face air-conducteur dans le cas dune inci
dence normale.

permittivit relative r = 15, cest--dire par


une permittivit complexe = - j a/CO avec
= q r.
1) Lorsque o/co 0 r < l/lO , on peut consi
drer que le sol se comporte comme un
dilectrique pertes. partir de quelle fr
quence cette proprit est-elle vraie ?
2) la frquence de 1 GHz, calculez le coeffi
cient de rflexion linterface air (milieu 1)
- sol (milieu 2) pour un angle dincidence
de 60 par rapport la normale, dans les
deux cas de polarisation.

&

E X E R C IC E 1 1 . 4
(Paragraphe 11.4 - Rflexion et transmission
l'interface de deux dilectriques)

E X E R C IC E 1 1 . 2
(Paragraphe 11.3 - Reflexion sur un plan
conducteur sous incidence oblique)

Soit une onde qui se rflchit sur un plan


conducteur sous une incidence de 45. Le
milieu dilectrique tant de lair, calculer les
impdances donde dans les cas T.E. et T.M.

De part et dautre dune interface X entre deux


milieux dilectriques, nous avons :
-
>

une onde incidente ( ,, Hs) selon la direc


tion ul ;
une onde rflchie ( Er , Hr) selon la direc
tion ur ;
une onde transmise ( Et , Ht ) selon la direc
tion ur

&

E X E R C IC E 1 1 . 3
(Paragraphe 1 1 4 - Rflexion et transmission
l'interface de deux dilectriques)

1) partir des vecteurs de Poynting de cha


cune de ces ondes, crire la relation qui tra
duit la conservation de la puissance.

Le sol moyennement sec est caractris par


une conductivit a = 10 ~ 3 (S m " *) et une

2) Par projection sur un axe des z perpendicu


laire linterface Z, en dduire la relation

quil y a entre les modules des coefficients


de rflexion et de transmission.

3) Vrifier que (Rx , Tx ) dune part et (R/,, T)


dautre part satisfont bien cette relation.

Chapitre 12
~L.es guides d'on des
rectangulaires

*12-1!

L es

d iv e r s t y p e s d e g u i d e s d ' o n d e s

TUDIS
Dans ce chapitre, nous tudierons dabord le guide dondes rectangulaire
parois lisses qui est trs utilis en micro-ondes, notamment en ondes centim
triques et millimtriques, tant pour les techniques de laboratoire que pour le
radar ou pour les applications industrielles.
Notre approche de la propagation en guide rectangulaire sera trs physique :
nous partirons des rsultats tablis au chapitre prcdent pour la rflexion dune
onde sur une plan conducteur et puis nous verrons quelles conditions, il est
possible dadjoindre un plan parallle ( 12.2) et deux autres plans perpendicu
laires afin de constituer un guide dondes rectangulaire ( 12.3). Nous mettrons
aussi bien en vidence les caractristiques de la propagation qui seffectue par
rflexions successives sur les parois du guide que les longueurs d onde de pro
pagation guide et de coupure ( 12.4).
Nous tudierons les modes de propagation de types TEmu et TEon, notamment le
mode fondamental pour m = 1 ou n = 1 ( 12.5) ainsi que les modes suprieurs
( 12.6), en ce qui concerne la rpartition transversale du champ, la puissance
transporte, le calcul des dimensions du guide, sa bande passante et son attnua
tion ( 12.7 et 8). Nous rserverons ltude des modes TEm/1 et TMmn au cha
pitre suivant.

Aprs avoir parl des guides rectangulaires surdimensionns ( 12.9) qui sont
utiliss en ondes millimtriques, nous nous intresserons enfin aux guides rec
tangulaires nervure ( 12.10, fig. 11) et aux guides rectangulaires ailettes
( 12.11, fig. 13). Ils permettent dobtenir des bandes passantes plus larges par
abaissement de la frquence de coupure du mode fondamental et leur structure
est moins sensible que celle du guide rectangulaire parois lisses, aux imperfec
tions de fabrication.

1 2 .2

P ropagation
ENTRE DEUX PLANS PARALLLES

12.2.1 Rflexion sur un plan conducteur


tant donne une onde lectromagntique (O.E.M.) arrivant obliquement sur un
plan conducteur X, le champ lectromagntique a t dtermin au chapitre 11
Rflexion et rfraction des ondes lectromagntiques dans les deux cas fon
damentaux de polarisation. Dans les expressions des champs donnes par les
relations (52, 53, 54, 68, 69, 70) E et H ne sont dtermins qu une constante
prs puisque leur valeur numrique exacte dpend de la puissance dmission de
la source qui les rayonne. Lexplicitation du facteur 2 nest donc pas indispen
sable linterprtation de ces relations et nous pouvons remplacer 2E par E et
2H par H.
Dans ces conditions, les expressions que nous utiliserons dans ce chapitre sont
les suivantes (rappelons que k = 2n/X).

a) Si le champ E est perpendiculaire au plan d'incidence

E x = - j E sin (ky sin \j/) e ~ikz cos v


H y - - j H cos sin (ky sin \g) e ~ikz cos v
H_z = - JVsin y cos (ky sin vj/) e ~ikz cos v

(1)
(2)
(3)

Un tel rgime, dans lequel le champ lectrique est perpendiculaire la direction


de propagation Oz, est appel un mode Transverse lectrique (TE) ou encore
mode H, pour rendre compte du fait que seul le champ magntique a une com
posante dans la direction de propagation.

b) Si le champ H est perpendiculaire au plan d'incidence


Hx = - H cos (ky sin y j e ~ kz cos v
E y = E cos vj/ cos (ky sin y ) e ~ikz cos v
E z = j E sin \j/ sin (ky sin \|/) e
cos v

(4)
(5)
(6)

Ici cest le champ magntiqu qui est perpendiculaire la direction de propaga


tion et seul le champ lectrique a une composante dans cette direction ; cest
pourquoi on parlera de mode Transverse Magntique (TM) ou mode E.

12.2.2 Condition de propagation


entre deux pians parallles

Figure 1.
Notations utilises
pour la propagation
entre deux plans
parallles.

Nous ne pourrons introduire un plan X parallle X (cest--dire xOz daprs


les notations de la figure 1) sans interrompre la propagation que si les conditions
de continuit sont vrifies sur ce plan, savoir :
E j 0 Ex = 0 et Ez = 0,
Hn = 0 Hy = 0 V x et z.
En se reportant aux relations qui donnent E x, E z et H y, dans les deux cas fonda
mentaux de polarisation, ces conditions imposent que : sin (ky sin y ) = 0 soit
ky sin y = rat.
Si langle dincidence y 0 est donn, nous avons :
nk
y = ---------2 sin y Q

(n, entier * 0)

(7)

Cette relation nous permet de dterminer les distances auxquelles on doit placer
le plan X parallle X : X/2 sin y 0 ou ./sin y 0 ou 3./2 sin y 0, etc.
Si la distance entre X et X est fixe une valeur b, il ne pourra y avoir propa
gation entre X et X que si :
sm y =
n devant tre tel que : 0 < tik/2b < 1.

(8)

12.2.3 Longueurs d'onde et vitesses de phase

Figure 2.
Propagation par
rflexions successives
sur X et X-

La propagation entre deux plans parallles X et X est caractrise par des


rflexions successives sur lun et lautre plan. En un point M de lespace dilec
trique, ltat lectromagntique rsulte de la superposition de deux ondes planes
rflchies galement inclines par rapport Oz (fig. 2). Les expressions des
champs en M restent donnes par les relations (1) (6) pour les deux cas fonda
mentaux de polarisation.
Rappelons q u partir de ces expressions, nous avons dmontr ( 11.3
Rflexion sur un plan conducteur sous incidence oblique ) que les axes Oy et
Oz correspondent respectivement aux directions de propagation de la puissance
ractive et de la puissance active.
Daprs la figure 2, nous voyons que les axes de rfrence yOz ne concident pas
avec la direction de propagation de lune ou lautre onde rflchie. Il en rsulte
que, selon ces axes, elle semble se propager avec une longueur donde et une
vitesse diffrentes de celles quelle a suivant la direction de propagation.

Considrons, par exemple (fig. 3) londe qui se propage de X vers X- Nous pou
vons reprsenter, dans le plan yOz, les traces de deux plans donde successifs
dphass de 2t ; ce sont deux droites perpendiculaires la direction de propaga
tion et dont la distance est :
PP' = X, longueur donde selon la direction de propagation
X
MM = X = ------ ,
y sin y

longueur donde selon Oy

(9)

NN

longueur donde selon Oz

( 10 )

Or, de mme quil y a un dphasage de 2n entre P et P , il y a aussi un dpha


sage de 2tc entre M et M' ou N et N . Cest pourquoi XY et Xz mritent bien le
nom de longueurs donde, mais ce sont des longueurs donde apparentes
selon des directions diffrentes de la direction de propagation de londe.
Dautre part, comme le temps mis par londe pour aller de P en P est le mme
que celui quelle met pour aller de M en M ou de N en N', elle semble se pro
pager plus rapidement selon les axes Oy et Oz que selon sa propre direction de
propagation ; sa vitesse apparente sur les axes Oy et Oz sera respectivement :
v
( 11)
v>= sin vj;
v

( 12)

COS \[/

ces longueurs donde et vitesses de propagation apparentes , on a donn le


nom de longueurs donde de phase et de vitesses de phase.

1 2 -3

P r o p a g a t io n d a n s un g u id e d ' o n d e s
RECTANGULAIRE

12.3.1 Condition de propagation


a) Nous venons de voir quil tait possible, sous certaines conditions, dad
joindre un plan X parallle X et dobtenir une propagation guide entre ces
deux plans qui sont, rappelons-le, perpendiculaires au plan dincidence (fig.
4). Voyons maintenant sil est possible dadjoindre des plans tc et n' parallles
yOz et perpendiculaires X et X sans perturber cette propagation. Les
conditions de continuit leur surface sexpriment par :
Ey = 0, Ez = 0, Hx = 0

\ / y et z.

Figure 4.
Propagation
en mode TEon.

Dans le cas o le champ lectrique de londe incidente est perpendiculaire au


plan dincidence (onde TE), ces conditions sont toujours vrifies puisque
nous avons : Ev = 0, Ez = 0 et Hx = 0.
Il est donc possible dadjoindre deux plans 7t et 7t perpendiculaires X et X
et parallles au plan dincidence ; la seule condition est que les deux plans X
et X soient distants de
2 sin \j/

(13)

Par contre, dans le cas o le champ lectrique de londe incidente est dans le
plan dincidence (onde TM), il nest pas possible que les conditions de conti
nuit soient satisfaites sur les deux plans 7t et tc puisque Ey, Ez et Hx ne peu
vent tre nulles V y et z. Il semble donc que la propagation dune onde TM
soit impossible lintrieur dun guide dondes rectangulaire (voir, en fait, la
remarque du 12.3.3)
b) Nous avons considr jusquici une propagation qui se ferait par rflexions
successives sur les plans X et X, ce qui est dailleurs logique puisque nous
avons pris n et 7t parallles au plan dincidence et quil ne saurait y avoir,
dans ces conditions, de rflexions sur 7t et 7t.

Figure 5.
Propagation
en mode TEmo.

Lautre cas de propagation (fig. 5) est celui qui se ferait par rflexions succes
sives sur n et n ; il faudrait, pour cela, que le plan dincidence soit parallle
S et S et que les deux plans n et 7t soient distants de
rrik
* = ---(14)
2 sin V
Dans ce cas, nous dmontrerions aussi que la propagation nest possible que
si le champ lectrique de londe incidente est perpendiculaire au plan dinci
dence (onde TE).
Il est dailleurs remarquer, notations mises part, que les figures 4 et 5 se
dduisent lune de lautre par une simple rotation de 90 autour de laxe du
guide. Aussi nest-il pas tonnant que l tude de ces deux cas de figure
conduise au mme rsultat. La conclusion que lon peut en tirer est la sui
vante : La propagation d une O.E.M. l intrieur d un guide d ondes rectan
gulaire sera possible si le champ lectrique de l onde incidente est parallle
aux faces sur lesquelles cette onde va se rflchir successivement.

12.3.2 Dfinition des modes TE^q et TEon


Dans le premier cas de figure envisag (fig. 4), la propagation de londe sef
fectue par rflexions successives sur les faces perpendiculaires Oy qui sont dis
tantes de b = nk/ 2 sin y . Nous dirons, par convention, quil sagit dune onde
ou mode TEon. Le champ lectrique tant horizontal, lexcitation de cette onde
seffectue laide dune petite antenne rectiligne (ou sonde), couple au gnra
teur, qui pntre dans le guide horizontalement.
Les expressions des champs pour un tel mode TEon sont donnes par les rela
tions (1), (2) et (3).
Dans le second cas de figure envisag (fig. 5), la propagation de londe sef
fectue par rflexions successives sur les faces perpendiculaires Ox qui sont dis
tantes de a m k/2 sin y . Par convention, il sagit dun mode TEwo. Le champ
lectrique tant vertical, lexcitation de cette onde seffectue laide dune
sonde qui pntre dans le guide verticalement.
Les expressions des champs pour ce mode TEmo se dduisent des relations (1),
(2), (3) du mode TEon en remplaant x par y et y par - x pour les variables et en
remplaant x par y et y par - x pour les vecteurs unitaires des axes. Nous
obtenons ainsi :
Ey = j sin (fcc sin \j/) e ~*kz cos v
(15)
H x = - j H cos Vj/ sin {kx sin tj/) e ~ikz cos v
(16)
H z = - H sin Vf/ cos (fcc sin \|/) e ~ikz cos v
(17)
Si m = 1 ou si n = 1, la propagation dans le guide dondes seffectue selon le
mode fondamental TE10 ou TE0I, les deux appellations tant quivalentes puis
quelles ne dpendent que de la convention prise au dpart.

12.3.3 Remarque sur les modes

T M m n

et TEmn

Nous avons vu quil ne paraissait pas possible de faire propager dans un guide
d ondes rectangulaire une onde dont le champ lectrique se trouverait dans le
plan dincidence et qui se propagerait par rflexions successives sur E et E'
(mode TMon) ou sur n et n (mode TMmo). Or, une tude thorique complte de
la propagation guide (chapitre 13) montre la possibilit de propagation dondes
TM mn (avec m et n 0). Cela tient au fait suivant : alors que pour les modes
TMon (TMmo) les conditions de continuit ne peuvent tre satisfaites sur les faces
K et n (E et X), au contraire, pour les modes TM/Jm (avec m et n =*Q), ces condi
tions de continuit peuvent tre satisfaites.
Il existe aussi des modes TEmn avec m et n ^ 0 dont la prsence sera rvle par
ltude gnrale de la propagation en guides dondes mtalliques (chapitre 13).
Dans la suite de ce chapitre, nous tudierons uniquement les modes TEmo (ou
TEon) et notamment le mode fondamental TE10 (ou TE01).

Lo n g u eu rs

d ' o n d e d e p r o p a g a t io n

GUIDE ET DE COUPURE

12.4.1 Relation fondamentale


de la propagation guide
En guide dondes rectangulaire, comme dans le cas de la propagation entre deux
plans parallles, nous pouvons dfinir des longueurs donde de phase.
Dans le cas (fig. 4) o la propagation seffectue par rflexions sur E et E (mode
TEon), nous avons :
une longueur donde de phase Xz = X/cos \|/, correspondant la propagation de
puissance active selon Oz,
une longueur donde de phase Xy = X/sin y avec sin \p = n k/2b correspondant
la propagation de puissance ractive selon Oy.
D o la relation :

(1 )

2
+

'x \

(18a)

Dans le cas (fig. 5) o la propagation seffectue par rflexions sur 7t et 7t (modes


TEmo), nous avons :
une longueur donde de phase Xz = X/cos t|/, correspondant la propagation de
puissance active selon Oz,

une longueur donde de phase Xx = X/sin y avec sin y = mX/2a correspondant


la propagation de puissance ractive selon Ox.
( V 2 f X)
+y
W

Do la relation :

(18b)

1 Ni
1^

II

N?

Dans chacun de ces cas, la relation fondamentale de la propagation guide


scrit :
m 2 1 )2 1
ModeTEwo:
(19a)
+
IV

il

m2 f

Mode TEC :

11

1 12

[ \i

(19b)

+k

A est la longueur d oncle de propagation en espace libre dans le dilectrique


constituant le guide ; elle est relie la vitesse de propagation en espace libre
par la relation A = v/f.
Az est la longueur d onde apparente dans la direction de propagation de la
puissance active : on l appelle longueur d onde de propagation guide et on la
note Ag. Elle est relie la vitesse de phase vz selon Oz par Az = \ J f. C est la
longueur d onde qui intervient dans le terme de phase e ~i kz cos v des relations
donnant les champs. En effet :
i cos y/=
2n z cos y/= 2n
, z
kz
A

( 20)

La signification physique de Ax et Ay est beaucoup plus dlicate saisir : en


effet, il ne s agit pas, proprement parler, d une longueur d onde de propagation
dans la direction Ox ou Oy puisqu'il y a, selon cette direction, un rgime d ondes
stationnaires. Nous allons approfondir cette question en prenant l exemple de Ax
(mode TEmo).

12.4.2 Longueur d'onde


et frquence de coupure
Voyons d abord quelle est la valeur de Xx :
XX

X
-----sin y

mX
et sin w =
2a

(21)

Xx est donc fix par les dimensions du guide. Pour comprendre sa signification,
il est bon de rcrire la relation fondamentale (19a) en faisant intervenir la fr
quence / de lO.E.M. se propageant dans le guide :

* Si f / v > m /lu , cette relation peut tre satisfaite en prenant Xz rel. Dans ces
conditions, lexponentielle e - *
intervenant dans les relations qui don
nent les champs est bien un terme de phase (dont le module est gal 1) et il y
a propagation de londe sans affaiblissement autre, bien sr, que laffaiblisse
ment dans les parois que nous navons pas envisag dans cette tude puis
quelles sont supposes infiniment conductrices.
* Si f / v < mj2a, cette relation ne peut tre satisfaite quen prenant Xz imaginaire
pur puisque l/X? < 0. Donc tous les termes e ~J 2nz/ \ , sont de la forme e
A tant rel positif. Cela signifie que les champs saffaiblissent exponentielle
ment lorsque z augmente. Dans ces conditions, londe ne se propage pas plus
loin que quelques longueurs donde.
La frquence f c = mv/2a qui vient dtre mise en vidence et la longueur d'onde
correspondante Xc = 2a/m , qui nest autre que Xx, sont appeles la frquence de
coupure et la longueur d onde de coupure du guide d ondes considr, pour le
mode TEnio. En effet :
S i / > (fc)m0 ou X < (Xc)mo il pourra y avoir propagation de T E ^ ,
Si/ < (fc)mo ou X > (kc)mo il ny aura pas propagation de TE ' .ilFigure 6.
Directions de
propagation des ondes
planes dans le guide.
a. Prs de la coupure.
b. Loin de la coupure.
Les figures 6a et 6b reprsentent les directions de propagation des ondes planes
dans le guide dondes, prs et loin de la coupure ; elles aident comprendre les
phnomnes : la coupure, X = Xx = 2a/m, donc y = tl/2 ; au fur et mesure
que lon sloigne de la coupure, langle \j/ diminue.

12.4.3 Relations concernant

\ c et \ g

Dornavant, nous crirons toujours X, = Xg et Xx ou Xv = Xc. Dans ces condi


tions, les relations (19a) et (19b) qui expriment la relation fondamentale de la
propagation guide scrivent :
( 22)

X est la longueur donde de propagation en espace libre dans le dilectrique qui


remplit le guide dondes ; elle se calcule daprs X = v / f avec v = c / e r et
c - 3 108 m/s. er est la permittivit relative du dilectrique du guide dondes.
Xc = 2a/m en mode TEmo ; Xc = 2b/n en mode TEon

Si X < Xc, il y a propagation du mode considr ; la longueur donde guide se


calcule alors daprs (22).
Si X > Xc, il ny a pas propagation du mode considr.

*1 2 -5

t u d e du m o d e f o n d a m e n t a l

Les modes TE10 (ou TE0[) sont ceux qui se propagent lintrieur des guides
dondes standard les plus utiliss dans la pratique et dont on dit quils fonction
nent selon le mode fondamental. Il est intressant, loccasion de ltude tho
rique de ce mode fondamental, de mettre en vidence certaines proprits carac
tristiques des guides dondes.
Dans ce paragraphe, noiis allons prendre pour exemple le mode TE]0 qui se pro
page par rflexions successives sur les faces n et n distantes de a = X/2 sin \|/,
avec un champ lectrique polaris verticalement (fig. 5).

12.5.1 Expressions des champs


Les champs du mode TEm sont donns par les relations (15), (16) et (17). Pour
le mode fondamental TE10, nous avons :
X

(23)

(24)

do lon dduit :
2tc

nx

2n X
kz cos y = z

* \
Dans ces conditions, les expressions des champs deviennent :
(25)

h z=

-h |

cos e -j2t
J z/X'x

(27)

Remarquons que le champ magntique, qui a deux composantes orthogonales et


dphases de 90, est polarisation elliptique dans un plan perpendiculaire la
direction du champ lectrique.

12.5.2 Variation transversafe


du champ lectrique
Dans un plan de section transversale du guide, tel que ABCD (fig. 7), le champ
lectrique reste parallle aux cts AC et BD puisquil na quune composante
Ey. En dplaant paralllement Ox une sonde de mesure associe un cristal
dtecteur et un galvanomtre, nous allons observer les variations de la valeur
efficace du champ lectrique :
-y
E
(>:)e f ' / 2 = / 2

JVC

sin

(28)

Figure 7.
Variation transversale
de Ey pour le mode
fondamental.

Comme 0 < x < a, nous obtenons une demi-sinusode avec minima nuis pour
x = 0 et x = a et maximum pour x = a/2.
Cest la coupe transversale du champ E caractristique du mode fondamental.

12.5.3 Lignes de courant


sur les faces du guide d'ondes
Les lignes de courant se calculent partir de lexpression de la densit surfacique de courant lectrique : J = n a H o n est la normale une face du guide

Leur rpartition sur les faces du guide est reprsente sur la figure 8 dans
laquelle le ct alIOx est plac horizontalement tandis que le ct blIOy est ver
tical.

Figure 8.
Lignes de courant sur
les faces dun guide
rectangulaire.

Il est important de noter que ces lignes sont :


verticales sur les faces latrales,
parallles laxe des z uniquement au milieu des faces suprieure et infrieure.
On ne pourra donc usiner des fentes dans le guide, sans en perturber gravement
le fonctionnement, que selon ces deux directions ; do les lignes de mesure
fente longitudinale. Ds que des fentes coupent les lignes de courant, elles
rayonnent une partie de lnergie qui se propage dans le guide : cela est
dailleurs utilis pour certains types d antennes (volume II, chapitre 14
Antennes rseaux ).

12.5.4 Puissance active transporte


par le mode fondamental
Cest la puissance transporte selon la direction Oz. La densit de puissance
selon cette direction nous est donne par la relation :
P z = ~ ^ E y H*x
Soit :

En tenant compte de ce que EH = E~

(W/m2)

(29)

9 KX

sin
a

et

4 a2

Cette expression reprsente la puissance qui traverse lunit de surface perpendi


culaire Oz ; elle dpend du point o lon se place dans la section transversale
du guide dondes. La puissance moyenne active traversant un plan transversal du
guide dondes nous est donne par :

Connaissant la puissance dalimentation du guide, cette expression permet de


calculer E, valeur maximale du champ lectrique au centre du guide (x = a/2).
Rciproquement, connaissant E, on peut en dduire P. En particulier, la puis
sance maximale transportable est donne par la valeur, note Pnax, de lexpres
sion prcdente lorsque le champ lectrique au centre du guide correspond au
champ de claquage Emax (champ pour lequel il y a amorce dun arc lec
trique). Lair sec est le dilectrique qui donne les meilleurs rsultats, notamment
quand sa pression est leve. En pratique, les guides dondes rectangulaires per
mettent de transporter des puissances moyennes de lordre du Mgawatt
2,45 GHz (frquence dutilisation industrielle des micro-ondes) et de lordre de
la centaine de Kilowatts 10 GHz, ce qui est remarquable.

12.6

tude

d es m o d es

TEmo ou TEon

12.6.1 Coupes transversales

Pour les modes TEmo, le module de Ey dans un plan de section transversale, tel
que le plan ABCD de la figure 7 est donn par | sin (m itx/a) |. Pour les modes
TEon, le module de Ex est donn par | sin (nny/b) I.

La figure 9 montre quelles sont les variations, selon Ox, de lamplitude du


champ lectrique parallle Oy pour les modes TE20 et TE30. Nous aurions les
mmes variations selon Oy pour le champ lectrique parallle Ox des modes
TE02 et TE03.

12.6.2 Longueur d'onde de coupure


Nous avons toujours Xc = X/sin V|/ ; mais ici :
mX
a - -------2 sin v|/
>>
n
!

Pour un mode TEmo :


do :
Pour un mode TEon :

(32)

nX
b = --------2 sin v)/
,
2b
X =
c n

do:

(33)

Pour m = l (mode TE10), on retrouve bien Xc = 2a.


Pour m = 2 (mode TE20), Xc = a
Pour m 3 (mode TE30), Xc. = 2a/3, etc.
Pour un mode TEmn (metri^O):
(34)

12.6.3 Longueur d'onde de propagation guide


Cest Xo = X^ ; daprs (22), nous avons :

( x\

1
1

>>

X)

=1

vX 8 j

(35)

do :

Pour un mode TEmo :

1-

(36)
m 2 X2
4 a2

(37)

Pour un mode TE()/| :

12.7

D im e n s io n s

et ba n d e passa n te

d ' u n g u i d e d ' o n d e s r e c t a n g u l a ir e

Le problme se pose, en gnral, de la faon suivante :


le guide dondes doit fonctionner en modes TEmo jusqu lordre p et trans
mettre une bande de frquences comprises entre/j e t/2 (J\ > /2 auxquelles cor
respondent X! < Xf) ;
calculer les dimensions a (parallle Ox) et b (parallle Oy) du guide
d ondes.
Voici la solution de ce problme :
1) Pour que les modes TE/no puissent se propager jusqu lordre p, il faut que
X < (Xc)p0, soit : X2 < 2afp. Do une premire condition sur a :
X2
a > p -pr-

(38)

2) Pour que les modes TEnm dordre suprieur p ne puissent pas se propager, il
faut que X > (X ^ , + , 0 > (Xc)p + 2, o > Do une deuxime condition sur a :
*1
(39)
a < (p + l )~2
3) La dimension b, distance entre les deux plans X et X, ninflue pas, nous
lavons vu, sur les rgimes TEmo qui admettent les plans 7t et 7t (parallles
yOz) comme plans de rflexion. Elle ne peut cependant pas tre arbitraire ;
elle doit, en particulier, tre suffisamment faible pour quaucun rgime
parasite TEon, correspondant des rflexions sur les plans X et X (parallles
xOz) ne puisse exister. Il faut donc que :
^1 > i \ . ) m = 2 b > (\.)o2 = b > ...
(40)

D o la condition sur b :
Finalement les conditions fixant les dimensions du guide sont :
Xj
X2
Xj
b<~Y

et

p ~ Y < a < ( p + 1)~y

Le problme inverse est celui de la dtermination de la bande passante, selon un


mode TEmo, d un guide dondes de dimensions a et b (a > b).
En rsolvant en
et X2 les ingalits (38), (39) et (40), nous trouvons que les
longueurs donde suprieure
et infrieure A.j, de la bande passante du guide,
sont :
2a
(42)
X2 m
2a
m+1

= Val sup

(43)

Nous voyons que la bande passante d un guide peut atteindre une octave
(k2 = 2^[) en mode fondamental (m - 1) si a = 2b.

12.8

A t t n u a t io n

d a n s l e s g u id e s d 'o n d e s

RECTANGULAIRES
Nous avons suppos jusquici que les parois du guide taient constitues par des
conducteurs parfaits, ce qui est videmment un cas idal auquel correspond une
propagation sans perte. En fait, lexistence de courants dans des parois de
conductivit finie implique quil y ait une dissipation dnergie par effet Joule
et, par consquent, une propagation avec pertes. Les champs donns par les
expressions (l) et (3) pour un mode T E ^ et (15) (17) pour un mode TEol| doi
vent tre affects dun facteur e ~ az si lon appelle a le paramtre de pertes.
Dans ces conditions, la variation de puissance le long de laxe du guide est de la
forme :
(44)
P(z) = / >0 e ~2ctz

et par consquent :

dP
l
dz
2 'P ( z )

(45)

Si lon suppose que la puissance perdue dans le dilectrique est ngligeable :


P(z) est la puissance transmise dans le dilectrique et (dP jdz) reprsente la
puissance perdue dans les parois du guide.
La puissance transmise dans le dilectrique est donne par :
P(z) = ^JJ ( i

H *) z dS

(46)

La puissance perdue, par unit de largeur, sur les parois du guide [53, p. 55-57]
est :

La puissance totale perdue dans les parois doit tre value sur le primtre de la
section transversale du guide, soit :
=

(47)

Rs est la rsistance superficielle de la paroi, qui est le quotient de sa rsistivit p


par la profondeur de pntration 8 due leffet de peau. Comme 8 = 1 / \ j npcjf
et p = i / o (a, conductivit) :

p. est la permabilit magntique des parois (en gnral p = p0) ; Js est la densit
surfacique de courant lectrique, donne par :
J s =n^H _= H t

(49)

n est la normale la paroi et H est le champ magntique dans le guide au voisi


nage de la paroi.
Compte tenu des relations (45) (49), le coefficient dattnuation est obtenu
daprs :
1

Rs | H} d t

JIS
(

(50)

H ) zdLS

Finalement, ce coefficient a sexprime (voir exercice 12.8) pour le mode fonda


mental par :

a (Np/m) = -

2ne v
3/2

1
'fc\2 _a f ) 2
f j + 2b \fc

(51)

es

Pour des guides gomtriquement semblables (a/b fix) et construits avec les
mmes matriaux (e, es donns), si lon maintient constant le rapport de la lon
gueur donde aux dimensions du guide (X/a et f / f c constants) lattnuation
varie en raison inverse de A.3/ 2. Donc, toutes choses gales dailleurs, a aug
mente si / augmente.
Pour un guide dondes donn (a, b, e, et a fixs), a si/ /c ou si/ .
Il doit donc y avoir une v a le u r^ de f ( f c < fm < ) pour laquelle lattnuation
est minimale. Ltude des variations de :

\2

A =-

fl

. j l l \2
2b{fcj

(52)

1en fonction de la frquence/, montre en effet que A passe par une valeur mini
male Am pour la valeur f m de/ dfinie par :
b
a

f J 2 Jfm )2
-3
fc
fcj

(53)

Dans le cas, courant dans la pratique, dun guide en cuivre rempli dair : p. = u0,
a = 5,8 107 et ev = e0 c = 1/1 20k , le coefficient dattnuation sexprime par :

a (dB/m) =

1,47 - 10 4
1

(54)

A a (d B / m )

Figure 10.
Courbe dattnuation
du guide dondes
R 100, de la bande X,
pour lequel
f c = 6,56 GHz.

La variation de a (dB/m) en fonction de f / f c est reprsente par la courbe de la


figure 10 dans le cas du guide dondes standard R 100 de la bande X, pour
lequel : a = 22,86 mm et b = 10,16 mm fJfc = 2,34 et Am = 2,2.
La valeur minimale de A est Am = 2,2 obtenue pour f m/ f c = 2,34. 11 lui corres
pond une valeur de a = 9,4 - 10 2 dB/m. Nous voyons donc quun tel guide
dondes, qui prsente des pertes ngligeables pour les faibles longueurs (de
lordre du mtre) utilises en laboratoire, aurait, en revanche, des pertes prohibi
tives sur les longueurs, de lordre de la dizaine de kilomtres sans rpteurs
ncessaires pour les tlcommunications.
Notons enfin que cette attnuation de 0,1 dB/m ne pourrait tre obtenue que
pour un guide dont les parois seraient parfaitement polies et recouvertes lectrolytiquement dun dpt dor afin de prserver le cuivre de la corrosion par lair
ambiant. Pour des guides dondes grossirement usins et non recouverts de
couches protectrices, lattnuation peut augmenter de 50 % (0,15 dB/m).

12-9

G u id e s r e c t a n g u l a ir e s
SURD1MENSIONNS

Au-del de 40 GHz, les dimensions des guides dondes standard deviennent trs
faibles (a < 5 mm et b < 2,5 mm), ce qui les rend trs coteux ; les attnuations
deviennent suprieures 1 dB/m, atteignant 3 dB/m vers 90 GHz pour des
guides dont les parois doivent tre parfaitement polies. Par ailleurs, cette rduc
tion des dimensions saccompagne dune diminution de la puissance maximale
transmissible. Ainsi, 75 GHz, est-on limit des puissances moyennes de
lordre du kilowatt.
Pour pallier ces inconvnients, on peut utiliser en ondes millimtriques, des
guides standard des bandes centimtriques qui sont largement surdimensionns
pour les bandes millimtriques, do leur nom de guides surdimensionns.
Lattnuation, en mode fondamental, dans un guide surdimensionn en cuivre
pour lequel a = 2b, est dduite de la formule (54) compte tenu de ce que f > f c \
elle est donne par :
a (dB/m) = 3,8 10 - 4 ^
a

(55)

Dans cette formule/ est en GHz et a en mtres. Avec un guide dondes standard
de la bande X, cette attnuation nest que de 0,14 dB/m 70 GHz et 0,2 dB/m
140 GHz.
Quant aux puissances transmissibles, elles se calculent toujours par la formule
(31) et lon voit quelles auraient plutt tendance augmenter en guides surdi

mensionns puisque, X ayant diminu, le facteur 1 - X2/4 a 2 se rapproche de sa


valeur maximale qui est 1.
Les performances qui viennent dtre indiques ne peuvent tre atteintes que si
le mode tondamental se propage seul. Or, toute discontinuit du guide surdimen
sionn (changement de dimensions ou de direction) provoque la transformation
dun pourcentage important du mode fondamental en modes dordre suprieur. Il
faudra donc prendre de trs grandes prcautions :
dune part pour lexcitation des guides surdimensionns partir des guides
standard, par des transitions dont les dimensions varient trs progressivement
et en gardant aux dimensions le mme rapport dhomothtie ;
dautre part, pour les liaisons du gnrateur au dispositif dutilisation, qui doi
vent tre exemptes de toute discontinuit.

12.10

L es g u id e s d ' o n d e s n e r v u r e
a

Figure 11.
Guides dondes
a. simple nervure
b. double nervure.

a)

b)

Il existe deux sortes de guides d ondes nervure (en anglais, ridged wave
guide) : les guides simple nervure (fig. lia ) et les guides double nervure
(fig. 11b). Dans la section transversale du guide, la discontinuit due la
nervure se traduit par une charge capacitive qui a pour effet par rapport au
guide rectangulaire quivalent, de dimensions a et b - dabaisser la frquence de
coupure du mode fondamental TE10 et de laisser peu prs inchange celle du
premier mode dordre suprieur TE20. En effet, cette nervure se trouve au centre
du guide, l o le champ lectrique du mode TE10 est maximal alors que celui
du mode TE20 y est nul. Il en rsulte une nette augmentation de la bande pas
sante du mode fondamental par rapport celle que l on aurait en guide
rectangulaire. Des largeurs de bande avec des rapports / max/ / m;n. de lordre de
trois ont pu tre obtenues, ce qui est deux fois meilleur quavec les guides rec
tangulaires.
Cette importante diminution de la frquence de coupure du mode fondamental
permet dutiliser un guide nervur de mmes dimensions a et b quun guide rec-

tangulaire, des frquences nettement plus basses, ce qui rduit beaucoup len
combrement. Cette proprit est particulirement intressante dans la gamme
des UHF (300 - 3 000 MHz) o les dimensions des guides rectangulaires
deviennent prohibitives.
Le calcul de la longueur donde de coupure du mode fondamental en fonction de
la largeur s de la nervure, en prenant pour paramtre le rapport d /b , a t fait par
Hopfer [54] dans le cas de guides simple et double nervure pour lesquels
b /a = 0,45.
Les rsultats sont montrs sur les figures 12a et b.

Figure 12.
\ de guides
a. simple nervure,
b. double nervure.

Ce calcul a t gnralis par Pyle [55] pour des rapports b /a quelconques. Il a,


en effet, tabli une relation entre la "kc dans le cas gnral et dans le cas particu
lier :

a
Cette relation fait intervenir un facteur F dont Pyle a donn des abaques en fonc
tion de b/a, en faisant varier les paramtres s/a et d /b par diximes entre G,1 et
0,9.
D aprs Thourel [46], des bandes de fonctionnement avec un rapport f^ /fn , = 3,6
ont pu tre obtenues avec les dimensions suivantes :
Pour le guide asymtrique : b /a = 0,45 : d /b = 0,128 ; s/a = 0,170.
Pour le guide symtrique : b ja = 0,45 ; d /b = 0,195 ; s /a = 0,25.
Par ailleurs, le calcul de limpdance donde du mode fondamental daprs :
te = /cos V o = / |l/e et cos y = \ J

- Q ./ 'k J 1 montre que limpdance

donde dun guide nervure est plus petite que celle dun guide rectangulaire.
Cette proprit peut tre utilise pour raliser des transitions large bande entre
lignes coaxiales dont limpdance caractristique est plus faible et les guides
rectangulaires dont limpdance donde du fondamental est plus leve [46, p.
245].

Cependant, les guides nervure ne peuvent pas transporter des puissances aussi
importantes que les guides rectangulaires. Ltude des champs lectromagn
tiques montre, en effet, quils sont concentrs au voisinage de la nervure o le
champ de claquage est rduit dans le rapport d jb par rapport au guide rectangu
laire. Comme la puissance maximale transportable est proportionnelle au carr
du champ de claquage, il en rsulte une trs forte diminution de celle-ci.
Notons enfin que ce guide a un affaiblissement plus grand que le guide rectangu
laire, mais que ses caractristiques sont moins sensibles aux imperfections de
fabrication.

H2.H1!

La

l ig n e a i l e t t e s

12.11.1 Intrt de cette ligne


La ligne ailettes, propose par Meier en
1972, est un guide dondes qui contient,
dans le plan de symtrie lectrique, un
substrat dilectrique sur lequel sont dpo
ses des bandes conductrices spares par
une fente (fig. 13) dans une configuration
bilatrale ou unilatrale.
Par rapport au guide dondes non charg
de mmes dim ensions, les principaux
avantages sont les suivants :

a)
Figure 13.
Ligne ailettes
a. bilatrale
b. unilatrale.
**)

la frquence de coupure du mode fondamental (qui est un mode hybride) est


abaisse et sa bande passante est largie (environ une octave),
les paramtres caractristiques de la propagation (a et p) sont assez peu sen
sibles la prcision dusinage et la qualit des conducteurs constituant les
parois (ce qui permet dutiliser des matriaux lgers et bon march comme
laluminium),
une structure plane facile fabriquer - grce aux mthodes bien au point pour
les lignes microbandes - et pouvant se prter aux techniques dintgration.
Par rapport la ligne microbande, notons que :
les pertes de propagation sont un peu plus faibles (de 0,05 0,1 dB/A,) et il ny
a pas de perte par rayonnement,
le couplage un guide dondes est beaucoup plus simple.

12.11.2 Paramtres caractristiques


Les paramtres caractristiques de
cette ligne, calculs par Meier [56],
sont la longueur donde guide et
limpdance caractristique. Pour
cela, M eier a identifi la ligne
ailettes (fig. 14a) un guide nervur
(fig. 14b) dont la fente, de largeur c
et de hauteur d, contiendrait un
dilectrique de mme permittivit
er que celui qui sert de substrat dans
la ligne ailettes.

Figure 14.
Dimensions de la
ligne ailettes (a) et
du guide nervur (b)
quivalents.

Cette approximation suppose que


le champ dans le dilectrique situ
entre les ailettes est nul ; elle est donc dautant meilleure que les rapports cja et
(b - d )/b sont plus petits. Dans ces conditions, on peut utiliser les formules des
guides nervurs [54].
La longueur donde guide (do lon dduit le paramtre de phase (3 = 2v./'kg)
est donne par la formule gnrale de la propagation guide :

Dans le modle quivalent la ligne ailettes, il faut tenir compte de leffet du


dilectrique qui charge la fente. Pour cela, Meier a introduit une permittivit
quivalente ee la place du 1 correspondant au cas de lair ; do
(57)

Xc est identique. la longueur donde de coupure dun guide nervur, de dimen


sions identiques la ligne ailettes.
Limpdance caractristique Za est donne par :
7 = '^a
7 oo e* -

/X
(58)
\K j

Dans cette formule, 7aoo est limpdance caractristique du guide dondes ner
vur de dimensions identiques, dans la limite o X 0 ( f ).

Le calcul de "K et Za passe donc par la dtermination de e, Xc et Zfloo. Des for


mules empiriques ont t tablies par Sharma et Hoefer [60] ; seule lexpression
de Xc est simple :
b_
= 0,245
(59)
K

tr3

Les expressions pour ze et ZOOsont trop longues pour que nous les donnions ici.
On pourra les trouver, in extenso, dans [38, p. 293-296] pour divers types de
lignes ailettes unilatrales ou bilatrales, substrat dilectrique de er = 2,22 et
er = 3.

&

E X E R C IC E 1 2 .1

2) En dduire la direction des lignes de courant :


a) au milieu de la face suprieure du guide
dondes,

(Paragraphe 12.4 Longueur d onde


de propagation guide et de coupure)

1) Dterminer la fonction qui donne laffaiblis


sement en dcibels dun guide dondes rec
tangulaire si X > Xc, longueur donde de
coupure du mode fondamental.
2) Affaiblissement sur une distance z = X dun
guide dondes utilis une frquence telle
que k = y/2 XcMme question si X = 1,2 Xc.

&

E X E R C IC E 1 2 . 2
(Paragraphe 12.5.1 - Expression des champs
du mode fondamental)

1) Dmontrer que dans un guide rectangulaire


fonctionnant en mode fondamental TE10,
il existe deux plans x = cte o le champ
magntique est polarisation circulaire droite
ou gauche.
2) Quelle relation y a-t-il entre ces deux valeurs
de x ? Application au cas o f= f. \[2.

b) sur les faces latrales du guide.

&

E X E R C IC E 1 2 . 4 _____________

(Paragraphe 12.5.4 - Puissance active transporte


par le mode fondamental)

Le guide dondes standard de la bande X (8,212,4 GHz) a pour dimensions a = 22,86 mm


et b = 10,16 mm ; il est utilis une frquence
/ = 10 GHz.
1) Ce guide d ondes tant aliment par une
puissance de 20 mW, en dduire la valeur du
champ lectrique au milieu du guide.
2) Sachant que la valeur maximale du champ
lectrique dans lair qui remplit le guide est
de 15 000 V/cm, calculer la puissance maxi
male transportable par ce guide.

E X E R C IC E 1 2 . 5
(Paragraphe 12.7 - Dimension et bande passante
d un guide d ondes rectangulaire)

&

E X E R C IC E 1 2 . 3
(Paragraphe 12.5.3 - Lignes de courant sur les
faces du guide d'ondes)

1) Calculer les composantes de la densit surfa


cique de courant J sur les faces dun guide
dondes fonctionnant en mode fondamental.

1) Calculer les conditions auxquelles doivent


satisfaire les dim ensions a et b (a > b)
des cts dun guide dondes rectangulaire
rempli d air pour que, la frquence de
10 GHz, il ne puisse sy propager que le
mode fondamental TE,0.

2) Mme question dans la bande de frquences


8-12 GHz.

laires aux faces orthogonales du guide qui est


rempli dair.
1)

E X E R C IC E 1 2 . 6

(Paragraphe 12.7 Dimension et bande passante


d un guide d ondes rectangulaire)

1) Ls dimensions du guide dondes standard


R 48 sont : a = 47,55 mm et b = 22,15 mm ;
il est rempli d air. C alculer dans quelle
bande de frquences il ne pourra se pro
pager que le mode fondamental TE10.
2) Mme question si le guide est rempli dun
dilectrique de permittivit er = 2,25.

crire la condition de propagation des modes


fondamentaux TE 10 et TE01.

2) Trouver le premier mode suprieur et crire


la con d itio n de non propagation de ce
mode.
3) En dduire la bande passante de ce guide
carr en modes fondamentaux.

E X E R C IC E 1 2 . 8
(Paragraphe 12.8 Attnuation dans les guides
d ondes rectangulaires)

&

E X E R C IC E 1 2 . 7

(Paragraphe 12.7 Dimension et bande passante


d un guide d ondes)

Un guide dondes carr 2,4 cm de ct. Il est


excit par deux petites antennes perpendicu

Dmontrer la formule (51) donnant le coeffi


cient dattnuation du mode fondamental dans
un guide dondes rectangulaire, de dimensions
a et b, rempli dun dilectrique caractris par
(e, p 0) et dont les parois sont caractrises par
(o, p).

Chapitre 13
tssde g n r a le
d e la p ro p a g a t^ s
er (^Mldes d'oraees
m

1 3 .1

t a lliq u e s

In t r o d u c t io n

Un ensemble de conducteurs mtalliques dont la structure reste invariante quand


on effectue une translation, le long de lune des lignes de coordonnes, forme un
guide dondes. Les schmas de la figure l en donnent quelques exemples.
Figure 1.
Divers types de guides
dondes.
a. Guide rectangulaire
b. Guide cylindrique
c. Guide de forme
quelconque.
d. Ruban mtallique.
Daprs Gabillard,
Vibrations et
phnomnes de
propagations, Dunod
157, p. 179]

Figure 1 (suite).

e)

D ivers types de gu id es
d ondes.
e. L ign e co a x ia le.
f L ign e bifilaire
b lin de.
g. L igne bifilaire.
h. L igne bande:,
D aprs

[57, p. 179],
g)

QUATIONS DE

P R O P A G A T IO N

DES ONDES GUIDES


*3 .2 . A

Principes gnraux

Dans un milieu dilectrique de permittivit lectrique e, de permabilit magn


tique p et de conductivit a = 0, les quations de Maxwell scrivent, en rgime
sinusodal (vol. 2, chap. 5 Equations de Maxwell ) :
_t
(quation de Maxwell - Faraday)
rot/T = -jcop H
(-)

(quation de Maxwell - Ampre)

r o t// =jcoeE
div = 0 et d iv / / = 0

(quations complmentaires)

(2 )
(3)

On dmontre (vol. 2, chap. 6 Propagation des O.E.M. en espace libre ) que


lquation de propagation des champs est de la forme
A ( | ou H) + co2 ep ( I ou / / ) 0

(4)

Dans tous les cas de propagation guide - par rapport un systme de coordon
nes curvilignes orthogonales dsignes par q j, q2, q 3 - la solution de cette

quation de propagation, pour une onde qui se propage le long dun guide dont
laxe est la coordonne q2, est de la forme :

e (q\, c/2 (li<


h (qi, c/2

C*/i

<l2 >e~yn

c jtiy

0 = FLLqi </2)e yq- e J',V

(5 a )

(5 b )

e et h sont les valeurs instantanes complexes des champs ; E et / / sont leurs


amplitudes complexes dans un plan transversal, perpendiculaire laxe longitu
dinal de la propagation.
y est le paramtre caractristique de
la propagation guide.
On choisira le systme de coordon
nes le mieux adapt la gomtrie
du cas considr.

Figure 2.
C oord on n es
rectangulaires
et cylin d riq u es.
Laxe z est
perpendiculaire
au plan de la fig

Les deux systmes les plus utiliss


(fig. 2 ) sont :
les coordonnes rectangulaires o :

q\ =x, q2~ <73= 2


les coordonnes cylindriques o :
q\ = P ; <?2 = <p ; = z
Ltude de la propagation guide revient dterminer :
a) le paramtre de propagation y qui, selon les cas, pourra tre :
imaginaire si la propagation est sans perte, soit y j(3,
rel sil ny a pas propagation, soit y d,
complexe si la propagation est avec pertes, soit y = a + jB ;
>

b) les deux fonctions vectorielles E (qt, q->) et


six composantes scalaires des champs :

>

(q j, q2) qui mettent en jeu les

les composantes longitudinales E^ = E . e t / / 3 = ELZ,


les composantes transversales E j. E 2 et t L L L 2En fait, il suffira de dterminer les expressions des deux composantes longitudi
nales, compatibles avec les conditions aux limites, pour que le problme soit
rsolu.
En effet, nous allons montrer que lon peut dduire toutes les composantes trans
versales des deux composantes longitudinales grce aux quations de Maxwell
( 1 ) et ( 2 ) ; c est pourquoi E et tL z sont appeles fonctions gnratrices .
Nous ferons cette dmonstration en utilisant dabord les coordonnes rectangu
laires et puis les coordonnes cylindriques.

1 3 .2 .2 Dterm ination des com posantes


transversales en x et y
En coordonnes rectangulaires, en remarquant que d /d , = - y et en sous-enten
dant les facteurs e
e Jtu' les quations de Maxwell scrivent, sous forme dif
frentielle :
d,
dy

+ y v= - M i ( x

(6 )

dE,

-y e 3,

- m u t,

(7)

dEx
jwji H 7

(8 )

+ yK v = ) u>e E x

(9)

dy

dH
~dy

d /fz
(1 0 )
y

*tLx . r
=jcoe / u
dy

(H )

Les liminations de H entre (7) et (9) et de H x entre (6 ) et (10) permettent


d obtenir E a. et E y en fonction de E <, et H c De mme, les liminations de E y
entre (6 ) et (10) et de E x entre (7) et (9) permettent dobtenir H x et H_y en fonc
tion de E , et / / ,. En posant :
y 2 + (2 ep = ft2
(1 2 )
nous obtenons :
,

dEz

dfz
(13)

-,
dfy z
kc E y = - y - ^ f + w
dEz
dy

d^ z

^Z
^ dx

(15)

dEz
kc d y = - jwe

dH z
-y -^
dx
dy

(14)

(16)

Remarquons tout de suite que les composantes transversales ne peuvent tre calcules en fonction des composantes longitudinales que si I ^ 0 .

1 3 .2.3 Dterm ination des com posantes


transversales en p et tp
Dans un guide d ondes prsentant la symtrie de rvolution, il est judicieux
d utiliser les coordonnes cylindriques (p, tp, z). Comme les champs doivent se
reproduire identiques eux-mmes chaque fois que <p augmente de 2n ou de
2n /n , nous pouvons supposer quils dpendent de cp par une fonction e
o n
est un entier ; donc d/dcp - - jn. Nous nous rappellerons, par ailleurs, que les
champs varient en 3 comme e ~ ^ : donc d d; - Y crivons maintenant les
composantes des quations de Maxwell tU et v.2) compte tenu de ces rsultats et
en sous-entendant, de plus, e ~ e JoM.
- J-p' tL z + y a tp=)0tEEp

(17)

d //,
- y H r = jcoe E
r-p
^
-P

(18)

jn
+ / / p = jc z
dp
p
p
z
<p

- ^ E z + y E lp = - k o p H p

- ye

,+
p P

_ = jw p / /
dp
<p

p
<p

dp

jn

E = jwn H z
p

(19)
(2 0 )

(2 1 )

(2 2 )

Les liminations de H ^ entre (17) et (21) et de H p entre (18) et (20) permettent


dobtenir 7P et E.. en fonction de , et H C De mme, les liminations de E...
entre (18) et (20) et de E entre (17) et (21) permettent dobtenir f0 et H en
fonction de E z et H ,. En posant
+ o r ep, il vient :
s.

'f '

tOLin

(23)

dH z
K m= jHi

dp

_ yn
+ J

E,

(24)

dtLz

,,
*?=

<x>en
Ez

(25)

. Y"
- d &z
kc (p=J
Z z jcoe
V
P
p

(26)

9p

1 3 .2 .4 Cas particulier
des ondes T.E..VI.
D aprs les relations (13) (16) ou (23) (26), nous voyons que les compo
santes transversales ne peuvent tre calcules en fonction des composantes Iongitudinales que si kf. ^ . Par contre, si k~ - 0, les relations suscites ne peuvent
tre satisfaites que si E z et t L z sont nulles. Cela se produit pour des guides tels
que la ligne bifilaire ou coaxiale o les champs sont purement transversaux, ce
qui correspond des modes de propagation du type T.E.M. (Transverse lec
trique et Magntique).
C est dailleurs en raison de cette proprit que ltude de ces types de lignes a
pu tre effectue (voir l re partie] partir des concepts tension et courant pour
lesquels on peut alors trouver en H.F. une signification physique. Notons enfin
que puisque k 1^. = y + (~ ep = 0, la vitesse de propagation sur une ligne T.E.M.
sans perte : v = oo/p = J/ \J q . est la mme quen espace libre.

1 3 .2 .5 quation de propagation
des com posantes longitudinales
Considrant la relation (4). Le laplacien peut tre mis sous la forme :
a2
A = A ,+ y
' dz2
sj

sj

sy

sy

sy

sj

Comme d " / d f =
et
= y + <*> |-t, nous pouvons crire les quations de
propagation des composantes longitudinales :
A ,E z + k2 E z = 0

(27a)

A, z + k2 H z = 0

(27b)

Ces quations nous permettent, compte tenu des conditions aux limites imposes
E z et / / de calculer E et H , en tout point du dilectrique du guide.

1 3 .3

L es

diffrents types d ' ondes

DE PROPAGATION GUIDE
Ltude que nous venons de faire nous montre que le rgime le plus gnral pou
vant exister dans un guide dondes est form des six composantes des champs.
Ce rgime peut toujours tre considr comme rsultant de la superposition de
deux ondes :
a) Une onde pour laquelle , = 0, dite Onde Transverse lectrique TE ou onde
H.
b) Une onde pour laquelle H z - 0, dite Onde Transverse Magntique TM ou
onde E.
Une telle dcom position na rien d artificiel. Elle correspond lexistence
simultane des deux types dondes que nous avons dj tudis au chapitre 11
Rflexion et rfraction des O.E.M. :

p
une onde plane dont le champ E est perpendiculaire au plan dincidence pour
laquelle E_z = 0 ( H est alors dans le plan dincidence et H , T 0 ) ;

une onde plane dont le champ H est perpendiculaire au plan dincidence pour
laquelle H_z = 0 ( E est alors dans le plan d incidence et E T 0 ).
Pour les ondes TE, la fonction gnratrice H z dont dcoulent les quatre autres
est dtermine partir de : A, / / , + A2 H , = (27b) en tenant compte de la
condition impose H, sur les parois mtalliques du guide* : d H /du 0.
Pour les ondes TM, la fonction gnratrice E , est dterm ine partir de
Af E , + A2 = 0 (27a) en tenant compte de la condition impose Ez sur les
parois du guide* : E. = 0.
On dmontre, en mathmatiques, que ces quations nadmettent de solutions,
compte tenu des conditions imposes aux limites, que pour une suite de valeurs
discrtes de kc, note & , appeles valeurs propres. Les ondes correspondant
ces valeurs de kc sont appeles modes de propagation .
Le paramtre de propagation y est donn par la relation :
!
Y=(At2 - w 2 ep )2

(28)

Dans cette relation CD2 8 |a est positif pour un milieu dilectrique comme Pair
dont les pertes sont ngligeables tandis que A~ est une constante qui peut tre
positive ou ngative. Nous allons donc distinguer les deux cas.
* Les parois du guide tant parallles laxe des z :
* la condition . = 0 se dduit directement de E , = 0,
* la condition H.Jn = 0 se dmontre partir de rot H = jwe E en tenant compte de ce que Er = 0

et n N = o.

1 3 3 .1 Ondes TE ou TM telles que

> 0

Ce cas se produit pour des guides dits section simplement connexe consti
tus par un conducteur creux lintrieur duquel il ny a que du dilectrique
(fig. la, b, c).
a) Si k 2 > co2 ep, c est--dire si f < k c/ 2
rel. Par consquent :

Y2 est positif, donc y = a est

- en onde TM :

ez - E ,

- en onde TE :

h = K z (a , > ou p, cp) e ~ 02 e ^

(a ,

y ou p, cp) e - az e

(29a)
(29b)

C est une onde dont lamplitude diminue exponentiellement et pour laquelle


on ne peut mettre en vidence ni priodicit dans lespace ni vitesse de propa
gation puisque le terme de phase ne dpend que du temps. Une telle onde est
appele onde vanescente. Au sens classique que lon donne la propagation,
on peut dire quil ny a pas propagation.
b) Si k 2 < CO2 ep, c est--dire si f > k c/2 n ^ /e p , T est ngatif, donc y = jP est
imaginaire et par consquent :
- en onde TM :

e_z = ,( a, y ou p, cp) e

- en onde TE :

h , = ! i z(x, y ou p, cp) ej (w'

(30a)
(30b)

Ces expressions caractrisent une onde qui se propage sans affaiblissement le


long de laxe des z- Si, de plus, lon tenait compte des pertes dans le dilec
trique (paragraphe 13.3.3) et dans les parois, la propagation se ferait avec un
affaiblissement. Un tel phnomne est reprsent par un paramtre de propa
gation complexe y = a + jP et les expressions des champs doivent tre multi
plies par un terme en e ~ az.
Vitesse de phase et vitesse de groupe

Dans le cas sans perte, y = j(3 ; P est le paramtre de phase de la propagation qui
est reli la vitesse de phase de londe par :
co

vp

(31)

et la longueur donde de propagation guide par :


2n

(32)

Xg= P
Comme nous traitons un cas pour lequel A'2 = co2 ep - p 2 est positif, nous avons :
9
O
0 3"
03"
---- y- >0 d o: v > v
y-

y*"
P

Il sagit donc dune propagation par ondes dites rapides .


Lorsquune onde est module par des signaux reprsentant une information, on
peut dfinir une vitesse de propagation de la modulation (chap. 2 Propagation
sur une ligne de haute frquence ) qui est la vitesse de groupe et se calcule
d aprs :
dco
(31)
Vg~ dp
En drivant la relation / = (tt>2/ v2) - (32, on trouve : 0 = 2(C/v2) dco - 2 PdB
do lon dduit :
X

y v

*
et

(35)

Xg

(36)

VP vg = v2

Puisque \ p > v, il en rsulte que vg < v.

Longueur d'onde et frquence de coupure

Si f> kc/2 n \ / p , il y a propagation,


Si / < kc/2 n y f p", il ny a pas propagation.
La frquence :
(37)

fc = 2 k J ep

apparat donc comme une frquence de coupure pour les guides dondes sec
tion simplement connexe.
La longueur donde associe :
_v __ 1

_ 2

(38)

apparat galement comme une longueur d onde de coupure telle que :


si X < Xc, il y a propagation,
si X > Xc, il ny a pas propagation.
Il est important de noter que ce concept de longueur donde de coupure est plus
gnral que celui de frquence de coupure. En effet, le calcul de Xc ne fait inter
venir que kc, valeur propre de lquation de propagation dans le guide. Or, kc ne
dpend que de la forme et des dimensions du guide tudi et elle est indpen
dante du dilectrique dont ce guide ect rempli.

Relation entre X. Xc et \ g - Paramtre de phase


La relation k~ = CO2 p - p2, peut scrire :
!2n)?

(2k \ :

2 jt'

yK j

y )

?i..i

(39)

do la relation fondamentale de la propagation guide :


n

i 32

(40)

Kl +\K
Le paramtre de phase (3 = 2n Xo, sexprime d aprs la formule :

P=

(41)

qui nous montre que le guide dondes est un milieu dispersif. En effet, la courre
reprsentative des variations de (3 en fonction de oo n est pas une droite, au
contraire des lignes T.E.M., et il en rsulte que la vitesse de propagation vp
dpend de la frquence.

1 3 3 .2 Ondes TE ou TIV! tees

que

<

Figure 3.
Reprsentation
schmatique dun
guide dondes charg.

Ce cas se produit uniquement pour des guides structure priodique. Pour la


plupart de ces structures, la priodicit rsulte dune simple translation, comme
cela est reprsent schmatiquement pour le guide dondes charg de la figure 3.
Les figures 4 et 5 donnent quelques exemples d obstacles pouvant charger un tel
guide dondes
Pour de tels guides, & 2 est ngatif ; comme ce2 ep est positif, il en rsulte que
y 2 - 2 - o r ep est ngatif et donc que y = j[3. Il y a donc une propagation sans
affaiblissement qui est rgie par les mmes relations que celles du paragraphe
13.3.1b.
Comme nous traitons un cas pour lequel I = m2 ep - p 2 est ngatif, nous avons :
o

00

t
^
-----5 < 0

d ou :

vp < v

Figure 4.
Obstacles capacitifs et
inductifs pouvant
charger un guide
dondes circulaire et
un guide dondes
rectangulaire.

Figure 5.
Autres types
dobstacles pour
guides rectangulaires
et circulaires.

Il sagit donc d une propagation par ondes lentes et il ny a pas de frquence


de coupure. Ce type de propagation qui nexiste que dans les guides d ondes
chargs a t tudi en dtail par L. Brillouin (58] ainsi que par Collin [59],

13.3.3 Cas des dilectriques pertes


Nous ne reviendrons pas sur le cas des pertes dans les conducteurs, dont la for
mulation a dj t dveloppe au chapitre 12, paragraphe 8 . Nous traiterons
donc, ici, le cas o la propagation seffectue dans un dilectrique pertes carac
tris par une permittivit complexe e = e je et une permabilit relle
p = p0. Il rsulte alors de la relation (28) que y est complexe, de la forme a + jp,
et nous pouvons lcrire : y 2 = k2 - co2 e p 0 = k2 - o r e0 e r p 0
S oit:
Do :
ou :

y2 = l -

er

( c t+ jp )2 = k2 - k ^ (e; - j e;)

-P '

l-j

= k~ - k 2 l - j

(43)
(44)
(45)

271
k =
K

271

avec :

2n
k=
X

et

2n

-----^0

La relation fondamentale de la propagation guide scrit donc, lorsque lon


tient compte des pertes dilectriques :
n 2(

E
-j
\
Er

^2

( 1 )2
+ ---

J iV l

f1 - j
l
P

(46)

Nous allons calculer maintenant les expressions de a. et fi. Pour cela, nous spa
rons parties relle et imaginaire dans (44) :
o r - \ i 2 - k 2 - k ^ e r
2 a P = Aq

(4 7 )

(4 8 )
1

d o :

+ p

= [(a

- p

2) 2

+ (2 ap

(4 9 )

) 2] 2

Nous devons prendre la solution avec le signe moins afin que lorsque r et a. sont
nuis, on puisse retrouver la relation du cas sans perte : (JP)2 = ! - k^ e)..
D onc:

o 2 + p2

= -[(*2

-kl

Er ) 2

+ (/cg e ) 2 |2

(50)

Finalement, en faisant la somme et la diffrence de (47) et (50), nous pouvons


calculer :
(51a)
+ (*0 EP
(51b)
k et e; / e> tg S, ces expressions permettent d obtenir :

En posant :

_i_
2

_ k
p ~72

k2

tg 2

(52a)
(52b)

*<2 - * 2

Nous pouvons galement faire apparatre les longueurs donde dans ces expres
sions puisque k = 2n /X et kc - 2 n / \ c :
a __1_

2n f

P v/ 2 '

2\

( B S V

_1_

1
2

(53a)
(53b)

Si les pertes dilectriques sont faibles, ce qui est souvent le cas, et si X nest pas
proche de Xc, de telle sorte que Ig 6 / ( 1 - X~/X~.) soit petit devant 1 , nous
allions :
n
tg 6
(54)
a =
X
2\
1

2\ 2
1

2
O

L es

I
tg 2
l + 8 '

(l

g u id es d ' o n d es

(55)

x2
__

r e c t a n g u l a ir e s

Le calcul de la solution gnrale de la propagation en guide d ondes rectangu


laire se ramne la dtermination des modes TE ou TM pouvant exister dans ce
guide.

13.4.1 Dtermination des modes TM


Dans ce cas. H , (.v, v) = 0 et il faut trouver une fonction gnratrice E ,(jt, y) :
qui soit solution de lquation :
3 2E ,

d 2E

y + ~ + k2 E =0
3 x~ y~

(56)

qui satisfasse la condition aux limites E z - 0 sur les parois du guide (fig. 6 ),
soit :
y

n
mtal

dilectrique

Figure 6.
Notations utilises
pour le guide
rectangulaire.

z = 0 sur les plans x = 0 et x = a


, = 0 sur les plans y = 0 et y = b

(57)
(58)

a) Recherche de la solution de (56)


Nous allons supposer que la fonction inconnue i(x, v) est le produit dune
fonction de x seul par une fonction de y seul :
,(-*, V) = X{x) Y (y)
(59)
(56) devient alors :

X "Y + Y"X + k* X Y = 0

En divisant par XY, nous obtenons :

(60)

Y"
-y
- + k~ = 0

Soit aussi :

(61)

Le premier membre est une fonction de x seul, le deuxime membre est une
fonction de y seul ; ils ne peuvent tre gaux qu une mme constante qui devra
tre de mme degr et de mme signe que !.
X

Posons donc :
11

vient :

Posons :
avec :

- ~

- k f . = - kx

- k x
ou

(62)

^ y = kx - kx.

^ y = - /

(63)

k* + kx = k}.

(64)

Dans ces conditions, la solution de (56) est de la forme :


,(-*, y) = (A cos k xx + B sin kx x) (C cos ky y + D sin ky y)
b)

(65)

Les constantes sont dtermines d'aprs les conditions


aux limites

= 0 pour y = 0 et y = b \/ x
0 = (A cos kx x + B sin kx x) C do C = 0
0 = (A cos kx x + B sin kx x ) D sin k v b
D ne peut tre nul, sinon E,(x, y) = 0 V x et y.
Donc :

sin k b = 0
v

do :

Ez = 0 pour .v = 0 et x = a V y
0 = A (D sin kv y) comme D ^ 0, A = 0

kv=
y b

(6 6 )

0 = (B sin kx a) (D sin kv y)
mn

d o

(67)

kx =

Par consquent, la fonction gnratrice est donne par :


mnx
nny
E (x, y) - E0 s in ----- sin -

( 68)

'mn 2 f7t\
+ I t J = k}.
l- J

(69)

et :

n et m sont deux entiers positifs quelconques.


Les composantes transversales du champ sont ensuite obtenues au moyen des
formules (13), (14), (15), (16) qui se simplifient du fait que H = 0.
y rmt
mnx
nny
E , (-v, y) = - E() --------- c o s ------ sin , v
u pi a
a
b

(70)

mnx
nn
nny
- sin ----- COS
a
b
~b~
*I

(71)

mnx
nny
jcoe nn
- sin ----- cos
b
a
~b~

(72)

E v 0c,y) =

H x (x,y) =

j(0E

mn

p i

H v (x, y) = - E 0 T
~ v

mnx
nnv
c o s ----- sin
a

(73)

Dans les quations (72) et (73), il est possible de remplacer e par k \J e/] 1 , avec
k = 2n/X. Il apparat ainsi que :

y
H,

(74)

13.4.2 Dtermination des modes TE


Les composantes du champ pour les modes TE (pour lesquels E z(x, y) = 0)
seront obtenues de la mme manire, en recherchant par la mthode de spara
tion des variables une fonction z(x, y) satisfaisant lquation :
d 2Z z

d 2H z
+ - 2 + kc tL z -Q
d xz
dy
2

et les conditions aux limites : dH_z/d n =

sur les parois du guide.

(75)

Dans tous les calculs


effectus dans ce
paragraphe.
les
termes exvonentiels
e ~ V" e -m, qui affec
tent tous les champs
ont t sous entendus f
E0 est une constante'
irbitraire dont la
valeur ne pourrait
tre fixe q u en
connaissant la puis- s
sance transportepa
le guide d ondes.

La fonction gnratrice que lon obtient ainsi est :


nmx
nnv
H Oc, v) = H,s cos
cos ,
z
u
a
b

(76)

En portant cette expression de / / , dans les formules (13), (14), (15), (16), et en
tenant compte de /?, = 0 , on obtient les composantes transversales du champ des
ondes TE.
E x ix, y) = H()

JOOLl nn
- 2-
b

nmx . nnv
a
b

nny
jp nvji _ mjix
E y f r y ) = - H 0 - - f ----s in ------ cos r-^a
a
b
kc.
Hx

(a ,

y) = H0

H y (a , y) = H0

(77)

(78)

mn
nmx
nnv
cos r
sin
U
b
kc a

(79)

un
nmx
nnv
r- c o s ----- sin

b
a
b
kc

(80)

Mme remarque pour H0 que pour E0 au paragraphe 13.4.1, ainsi que pour les
termes exponentiels qui sont sous-entendus. Par ailleurs, dans les quations (77)
et (78), il est possible de remplacer top par k J p / e avec k = 2njX. Il apparat
ainsi que :
k
H.

( 81)

1 3 .4 .3 Lignes de champs
Pour les ondes TE et TM les composantes des champs lectrique et magntique
dpendent de deux entiers arbitraires, m et n. A tout couple de ces deux entiers
correspond une onde du type TE et une onde du type TM que lon appelle mode
TE,,,,, ou TM,.

Figure 7.
Coupes transversales
des champs des
premiers modes TEmn
partir de la coupe
transversale du champ
du mode TE| f.

Pour m = n= 1, les composantes tangentielles du champ lectrique ne peuvent san


nuler que sur les parois du guide. Par contre, si m et (ou) n > 1, ces composantes
peuvent aussi sannuler sur des plans mridiens, comme le montre la figure 7.
Dans chaque rectangle dlimit par ces plans, la configuration du champ est ana
logue celle du cas m n 1. Par exemple, pour m n = 2, nous retrouvons la
juxtaposition de 4 motifs lmentaires correspondant au cas m = n 1. Les
configurations des champs sont dautant plus compliques que les indices m et n
ont des valeurs plus leves.

Figure S.
Lignes de champs de
quelques modes du
guide dondes
rectangulaire.
Figure extraite
de Rigal et Voge, Les
Hyperfrquences,
itions Eyrolles
(1963), p. 121.

La figure 8 reprsente les lignes du champ correspondant quelques modes du


guide rectangulaire, dans divers plans transversaux et longitudinaux.
Remarquons que s il existe un mode TE()/ ou TE/0 (mode fondamental du guide
rectangulaire), il ne peut pus exister de mode TMq] ou TMjqpuisqu 'alors Bf x, y)
- qui est un produit de sinus - serait nul et comme Hz Test dj pur hypothse,
toutes les autres composantes seraient galement nulles.

13.4.4 Longueur d'onde et frquence


de coupure
Nous avons vu que :

nrtr 2+im\

i J lT

Comme kc - 2n/Xc, il en rsulte que la longueur donde et la frquence de cou


pure sont donnes respectivement par :

*0 .5

L es g u id e s d ' o n d e s c ir c u l a ir e s
A
axe
i de rotation

Figure 9.
Joint tournant.

Pendant longtemps, les guides dondes cir


culaires nont servi quen tant qulments
de dispositifs tels que attnuateurs, dphaseurs (vol. 2, chap 2 Circuits passifs rci
proques ), joints tournants. Pour ces der
niers (fig. 9) ils jouent le rle dlments de
connexion entre deux guides G et G, per
mettant lun deux de tourner par rapport
lautre.
Au dbut des annes 1970, on a envisag de
les utiliser en tant que lignes de tlcommu
nications, dune part parce quun de leurs
modes de propagation prsente des pertes
qui diminuent quand la frquence augmente,
dautre part parce que lon sait liminer les
autres modes pour ne conserver que celui-l.

13.5.1 Bases thoriques


Nous utiliserons les coordonnes cylindriques (fig. 2) selon lesquelles les A, des
32
d
relations (27a et b) ont pour expressions : ------- 1- ----- 1___ . ------ .
d p 2 P d p p 2 0 (p2
Conformment ce qui a t expliqu au paragraphe 13.2.3 Dtermination des
composantes transversales en p et cp , la fonction angulaire /(cp) est prise de la
forme e "Jmp et nous recherchons donc pour \|/(p, cp) = E , ou / / , des solutions de
la forme : \|/(p, cp) E(p) e
Les relations (27a et b) nous donnent donc :
dl F

---- -

dp

dF

---- + kf.

9p

y
O F =0
p J

(84)

et admettent pour solutions :


F ( p ) ^ A J ri(ki p) + BY(kr p)

(85)

A et B sont les constantes d intgration dont les valeurs seront dtermines


d aprs les conditions aux limites.
J et Yn sont les fonctions de Bessel d ordre n, respectivement de premire et
deuxime espce.
Notons ds maintenant que \p doit garder une valeur finie sur laxe du guide
(p = 0). Comme Yn(x) quand x > 0, la constante B doit tre nulle et la
solution pour \p doit tre de la forme :
\|/(p, cp) = A Jn(kc p) e

(8 6 )

13.5.2 Dtermination des ondes TM


Dans ce cas Hz = 0 et daprs ( 8 6 ) :
E z = E0 Jn(kr p ) e - j ',tP

(87)

La condition aux limites sur la paroi du guide impose que E = 0 pour p = a,


soit : Jn(kc a) = 0. Or la figure 10 montre que la fonction Jn(u) sannule pour une
suite de valeurs de u qui sont les racines de Jn. chacune de ces racines corres
pond un mode de propagation. Le mode correspondant la me racine d une
fonction dordre n est appel mode
Le tableau de la figure 11 donne les
valeurs discrtes unm de ces racines. Pour ces modes TM :

Figure 10.
Courbes de variation
des fonctions Jp, J(,
K0 et YfX)

En portant la fonction gnratrice E , donne par la relation (87) dans les rela
tions (23) (26), nous obtenons les composantes transversales :
R e m a rq u e
Dans toutes ces rela
tions, le facteur e
e,m est sous-entendu.
Quant E, cest une
constante arbitraire
dont la valeur ne
pourrait tre fixe
qu'en connaissant la
puissarce transporte
par le guide dondes.

(89)
P

kc

yn
^ ,p = J ^ 0 73 Jn (kc p) e Jn<P
kL. p

(90)

coen
tL p = E0 72 Jn (^c p) e _ ^
kc p

(91)

tL(p = ~} E q ~j~ Jn (kc p) e jn<P


c

(92)

En remplaant i par k V nous avons :


Z l =^ -P = -

Kp

H tp

[ V

JV

(93)

13.5.3 Dtermination des ondes TE


Dans ce cas, Er - 0 et daprs ( 8 6 ) :
H z = H0 J(kc p ) e - W

(94)

La condition aux limites sur la paroi du guide impose que d H jd n = dH ,jdp = 0


pour p - a, soit J'n{kc a) = 0. La fonction Jn{u) prsentant une suite de maxima et

de minima, cest pour les valeurs de , notes u]mi , correspondant ces maxima
et minima que J'n(u) = U. chacune de ces racines de J'n correspond un mode de
propagation appel TE/W|. Le tableau de la ligure 11 donne les valeurs discrtes
unn de ces racines. Pour ces modes TE W#J1 :
kC

(95)

Cl

En portant la fonction gnratrice / / , donne par la relation (94) dans les rela
tions (23) (26), nous obtenons les composantes transversales :
cou n
(96)
L = -ff0 7 2 -^ rP )e -W
K
k,COU

p ) e J"P

fip = - o ^ ^ r P ) e -
Yn
U <p= j

Cy D

<kc p ) e

-JKtp

(97)

(98)

(99)

Mme remarque quau paragraphe 13.5.2 concernant //0 et les termes exponen
tiels qui sont sous-entendus. Par ailleurs, en remplaant cop par k ^ /p /e - nous
avons :
k

( ? 00)

13.5.4 tude des divers modes de propagation


Nous venons de mettre en vidence que, comme dans les guides rectangulaires,
la propagation dans les guides circulaires seffectue selon diffrents modes TE et
TM. Ces modes sont caractriss par :
leur configuration transversale ;
leur longueur donde de coupure.
Les configurations transversales de quelques-uns de ces modes sont reprsentes
sur la figure 1 1 .
Les longueurs donde de coupure des modes TEwm sont donnes par :

CTEnm
a est le rayon intrieur du guide.

in a
Unm

Les longueurs donde de coupure des modes TM,im sont donnes pur :
2 na
K
=
n,n u nm

(10 2)

Les valeurs de unm et utvn sont donnes dans le tableau joint la figure 11.
- 0.1
Xc = 1,640a

mm mm

! H!l I M:!!;

"Ml! !--!!!!!!
m
i 1 1 : il
rr
1
11 ;

1 t t i 1 r
i,i 1 il

1 1

1-

Figure 11.

Lignes de champs
et de courants
de quelques modes
des guides dondes
circulaires.

TM?

Figure extraite de
Microwave Theory
and Applications par
S.F. Adam, diteur
Hewlett-Packard
(1969), p. 6?.

Xc = 1,224a
( 3)

Lgende :

r ri 1 i T- S ri Pi

-lzz-zz
. -LLLU-

m r n rn n

- E
H

I
u 01 = 2,405
Uf

-j 3,832

u 2.i = 5,136

u 0.2 = 5,520
2 -

7,016
uo,3 = 8,654

Ml

IL

' Vv\Ui

coupe transversale
coupe longitudinale selon le plan /-/
rpartition sur la surface selon S -S
u ' 0-1

= 3,832
u,,, =1,841
u'z ,1 = 3,054
u 3i1 = 4,201

u Q2 7,016
u' 1,2 = 5,332
u'z _2 = 6,706
u 3 2 = 8,013

Le mode qui apparat le premier est celui dont la longueur donde de coupure esi
la plus grande ; c est donc T E n (kc = 3,41) qui est le mode fondamental des
guides d ondes circulaires. Apparaissent ensuite, successivement, lorsque la fr
quence augmente :
TM 01 (Xc = 2,6 lo)
TE 21 (kc = 2,05 a)
TE 01 et TM, j (kc = l,64ci)...

Les modes TM 01 et TE 01 prsentent une parfaite symtrie de rvolution autour


de laxe du guide et leur emploi est donc particulirement indiqu lorsque lon
doit faire tourner lune par rapport lautre deux sections de guides circulaires
sans perturber la propagation, comme cela est le cas dans les joints tournants
pour les metteurs radar. Le mode TM0I a lavantage de pouvoir tre excit sim
plement par un lment dantenne plac selon laxe du guide.

1 3 . 5 .S tude du mode fondam ental T E^


La structure de ce mode qui est caractrise, dans un plan de section trans
verse, par une polarisation uniforme selon un des diamtres correspond la
structure du mode fondamental des guides d ondes rectangulaires. D ailleurs,
lorsque par une transition gomtrique
ment progressive (fig. 1 2 ) on transforme
un guide d ondes rectangulaire en un
guide d ondes circulaire, c est le mode
T E (I qui est excit dans le guide dondes
circulaire si le guide d ondes rectangu
laire fonctionne selon le mode fonda
mental T E jq, et rciproquement.

Figure 12.
Transition guide
rectangulaire - guide
circulaire.

Lennui est qu cause de la symtrie de rvolution du guide circulaire, la direc


tion de polarisation uniforme dont nous avons parl plus haut peut tourner car
il ny a aucune direction privilgie. Cette direction de polarisation dpend, en
fait, de la polarisation du champ dlivr
par la sonde excitatrice, laquelle elle est
i
i
parallle.
C est pourquoi on prfre souvent (notam
ment pour les circuits d mission et de
rc e p tio n des fa isc e a u x h e rtz ie n s)
employer des guides dondes elliptiques
pour lesquels la direction de polarisation
est fixe, grce lasymtrie du systme,
selon la direction du petit axe ou du grand
axe de lellipse (fig. 13). En gnral, c est
la premire disposition qui est utilise.

Figure 13.
Guide dondes
elliptique.
ligne de champ E
(excitation II au petit axe)
ligne de champ E
(excitation II au grand axe)

1 3 .5 .S Attnuation
en guide d'ondes circulaire
Les attnuations (en dB/m) sont donnes, pour les trois modes les plus impor
tants, par :

Mode TE H :

a-

Z )-5 +4 J Z V
2,38 /c

5,5-10-5 l/J

a 3/2

(103)

/V -

7c
a passe par un minimum pourf / f c = 4,5.

ir
Mode TM0| :

a =

6,3 10 - 5
,3 /2

2-

(104)

fc

a passe par un minimum pour f / f c = v^3".


Mode TEqj :

a=

8 10 - 5

(105)

,3 /2

Z 12
fc

- '

Ici, a diminue constamment quand/ augmente.


Les variations de a en fonction de la frquence sont reprsentes, pour ces trois
modes, sur la figure 14. Elle met bien en vidence que, pour le mode TE0,, les
pertes diminuent lorsque la frquence augmente. Il est le seul parmi tous les
autres modes des guides circulaires possder cette intressante proprit.

Figure 14.
Courbes des variations
de lattnuation
linique en fonction
de la frquence
pour les 3 modes
principaux du guide
dondes circulaire.

Physiquement, cela provient de ce quil est le seul dont les lignes de courant
dans la paroi du guide soient situes dans des plans de section droite alors que,
pour tous les autres, les lignes de courant sont longitudinales. Cette proprit
permet d ailleurs d liminer les autres modes en ne conservant que le mode
TE q| : il suffit, pour cela, que le revtement intrieur du guide, au lieu dtre
lisse, soit constitu par des spires en cuivre jointives, recouvertes d un vernis qui
les isole les unes des autres ; tous les modes dont les lignes de courant sont lon
gitudinales peuvent ainsi tre stopps tandis que le mode TE 01 n est pas affect.
C est selon ce principe quavaient t dvelopps, lors des annes 1970, des pro
jets de transmission moyenne et grande distance par guides circulaires hli
codaux . La solution retenue en France tait un guide de 50 mm de diamtre
prsentant des pertes de 3 dB/km 35 GHz. Mais laugmentation du cot de la
matire premire (cuivre) et les promesses apportes par les transmissions sur
fibres optiques dont la matire premire (silice) est bon march ont entran
labandon de ces projets.

H 3LS

tu de

g n r a l e de la r flex io n

DES ONDES GUIDES


13.6.1 Dfinition d'une onde de rfrence
Nous allons exprimer, par rfrence une onde unit (onde transportant une
puissance de 1/2 Watt), les rsultats obtenus dans ltude des lignes T.E.M.
(chap. 2 Propagation sur une ligne en haute frquence et chap. 3 Etude de
la rflexion lextrmit dune ligne ) partir des concepts tension et courant
aussi bien que dans ltude des guides dondes (chap. 12 Les guides d ondes
rectangulaires et chap. 13) partir des concepts champs lectrique et magn
tique.
Nous montrerons ainsi que, dans le cas gnral du rgime dondes semi-stationnaires, les expressions de V(z) et I(z) pour les lignes T.E.M. ainsi que celles de
E t (x, y, z) et H t (x, y, z) pour les guides d ondes, peuvent tre crites sous fe
forme du produit de deux facteurs :
lun, qui est relatif londe unit crite en fonction de V, I ou de E r jt selon
le cas considr,
lautre, qui est identique dans les deux cas et qui ne dpend que du paramtre
de propagation y et des caractristiques de la charge de la ligne ou du guide
dondes.

".htJrtwwf X

M icro - ondes

13.6.2 Caractrisation de !a rflexion


en lignes T.E.VI.
G : gnrateur

R : charge

Figure J5.

Conventions de choix
de lorigine
et de lorientation
de laxe des

a) L'onde progressive unit


Si laxe des abscisses est orient du gnrateur vers la charge (fig. 15) :
Vfe) = Y 0 e - > 2
et

Zu) = Z

(106)
d

Il est possible de dfinir une onde unit :


et
telle que:

V"(z) = J /tfe " *

(108)

Lu(z) = L ^e ~ yz

(109)

~ V{J (/g)* = ^ Watt

(110)

b) Le rgime d'ondes semi-stationnaires


Un tel rgime peut toujours tre considr comme la superposition dune onde
progressive incidente, se dplaant du gnrateur vers la charge, et dune onde
progressive rflchie, se dplaant en sens inverse :
si laxe des abscisses est orient du gnrateur vers la charge (fig. 15) :

avec:

JZ(z) = Vi e-'*z + Vr e*z

(1 1 1 )

Kz) = l i e ~ yz + l r eyz

(1 1 2 )

V,V,
------I-i
I-r

(113)

si laxe des abscisses est orient de la charge vers le gnrateur :


V{z) = Y i e ^ + Vr e - ^ 7

(114)

Lit) = lj eY; + Zr e -z

(115)

Exprimons, dans ce dernier cas, les ondes progressives incidente et rflchie par
rfrence londe progressive unit :
pour londe incidente, il est possible dintroduire un facteur A tel que :
V, eYZ= A Y," e *

(116)

Lt eYz = A Zq e7z

(117)

pour londe rflchie, il est possible dintroduire un facteur B tel que :


Vr e ~ yz = B V f i e ~ yz

(118)

Lr e ~ yz = - B V ^ e ~ yz

(119)

NB
Nous avons affect cette dernire expression d un signe moins puisque :
Y r/ I r = - ; / ! ; .

Dans ces conditions, londe semi-stationnaire scrit :


V(z) = V " (A e yz + B e ~ yz)

( 120 )

I(z) = Zq (A

( 121)

^ e myz)

1 3 .6.3 Caractrisation de la rflexion


en guides d'ondes
a)

L'onde progressive unit

Les champs constituant londe lectromagntique (axe des z orient du gnra


teur vers la charge) ont :
pour le modes TE et TM, des composantes transversales de la forme :
, = ,(<:/,, c/2)e -Y z
K , = H , { q x,cl2) ^ - yz

(122)
(123)

pour le mode TE, une composante longitudinale du champ H de la forme :


H z = H z (qi , q2) e - yz
(124)
pour le mode TM, une composante longitudinale du champ E de la forme :
E z = E z {qA, q 2) z - yz
(125)
Nous avons tabli quil y avait propagation selon laxe Oz dun rgime dondes
progressives. La densit de puissance active (W/m2) transporte est :
Pz = ~ E f / s H * = ~ E t N * z

(126)

Cette relation est vraie quelle que soit la base de dcomposition choisie pour Et ,

Ht ( j = x, q2 = y en cartsiennes ou q j = p, q2 = tp en polaires).

Remarque

Puisque les compo


santes longitudinales
n interviennent pas
pour dfinir la puis
sance active, nous ne
nous intresserons
dsormais quaux
composantes transver
sales de ces champs.

travers la section droite du guide, la puissance active est :


1
/> = -2 | )s E , H , d S = - y - ] |v E ,
cJ2) H , (</

q 2)

cLS

(127)

11 existe des valeurs


et tL de /?, et H_, telles que, pour clz = 0, cette puissance
active transporte est gale 1/2 Watt. Londe ainsi dfinie est londe unit. Elle
est constitue par les champs :
E = E i; { q l,q 2) e - K

(128)

tLut =Li.qx,q2) e ~ r'

(129)

b) Le rgime d'ondes semi-stationnaires

Nous crirons les ondes incidente et rflchie composant ce rgime par rfrence
londe unit, mais nous orienterons maintenant laxe des abscisses de la charge
vers le gnrateur (fig. 15).
Pour londe incidente, nous introduirons un facteur A tel que .
F\ = A E e7;
et

H \ = A H eyz

(130)
(131)

Pour londe rflchie, nous introduirons un facteur B tel que :


E r, = B E ul e ~ 1<z
et

(132)
(133)

nie
*

>

(E[, H[, - c) et (E,r, H[, /) doivent former des tridres directs, par consquent :
e:

(134)

h;

Les champs de londe semi-stationnaire scriront enfin :


E, =

'/ (</,, q 2) [A e^ + f i e ^ ]

(135)
(136)

13.6.4 Dfinition d'un coefficient de rflexion


gnralis
Grce au concept donde unit que nous venons dintroduire, il a t possible
dcrire :
les tensions et courants pour les lignes T.E.M (relations (120) et ( 12 1)),
les champs E f et H_t pour les guides dondes (relations (135 et ( 136)), comme
tant le produit de deux facteurs dont lun ne dpend pas du tout de la charge

alors que lautre contient les termes A et B qui vont permettre de la caract
riser.
Pour faire ltude de la rflexion, nous nous intresserons donc, dans tous les
cas, aux facteurs :
c * +lic ~ v =U

(137)

A e^z B c ~ ^ = J

(138)

Nous aurons toujours lesprit la signification de U et J :


pour les lignes T.E.M. :
VY

U=
~
Vu
Lo

et,

(139)

J, =/(z)

,U
0

(140)

Z,
J =-

(142)

dans ce cas : y = a. + jp avec fi = 2 n /X ;


pour les guides dondes :
E.
U =-

(141)

et

dans ce cas : y = a. + jp avec P - 2nf X .


Le rapport :

---------- = -7 e
A eVz

-y- = rxz)

(143)

quotient de londe rflchie du type U par londe incidente de mme type, appa
rat comme le coefficient de rflexion dans un plan dabscisse z. Ce rapport a
pour valeur B /A dans le cas particulier o z = . C est le coefficient de rflexion
de la charge F(0) = JT^.
Comme r * est un nombre complexe de la forme r * e J * :
r(z) = r w e - 2az e M - 2PZ)

(144)

Remarque

Cest par convention que le coefficient de rflexion a t dfini punir d une


onde de type U, c est--dire de tension ou de champ lectrique. S'il avait t
Ri dfini partir d une onde de type J, c est--dire de courant ou de champ magntique, son signe aurait t chang.

13.7

L es
d

c o n c e p t s

' im p d a n c e

ET

d
d

' im p d a n c e ,

'o n d e

D'IMPDANCE

RD U ,t E

13.7.1 L'impdance en lignes T.E.M.


Dans ce cas, les expressions de U e t J sont quivalentes :
V{z) = Vfi A e?z( 1 + T *>e 2yz)

(145)

Kz) = l " A e yzO - r ^ e - 2^ )

(146)

1 + E * e -2YZ
Z(z) = Z -----------------

(147)

dont le quotient est :

Z(z) est limpdance en un point de la ligne,


Zc est limpdance caractristique de la ligne,
zk - zc

(148)

j_ r
R

est limpdance de charge pour 2 = 0.


Rappelons la formule de transformation des impdances :
Z(z) = Z.

ZR + ZCthyz

(149)

ZC+ ZR thyz

Pour les lignes T.E.M., ces notions dimpdance ont une signification puisquil
est toujours possible de dfinir :
le courant en un point dabscisse z dun conducteur,
la tension dans un plan dabscisse z de la ligne comme la tension existant entre
les deux conducteurs de la ligne bifilaire ou entre le conducteur intrieur et le
conducteur extrieur de la ligne coaxiale (dans ce dernier cas, quelque soit le
couple de points choisi, le rsultat est toujours le mme).

13.7.2 L'impdance d'onde en guides d'ondes


Dans ce cas, les expressions de U et J sont quivalentes :
E ,(z) = E A *z ( l + H* e - 2yz)
H .

,(z) =

t L u,

A e ^ l l - r ^ e '

2E )

(150)
(151)
(152)

dont le quotient est :


TE

(z) est limpdance donde en un point de la ligne,


est limpdance donde propre au mode TE ou TM,
TM

TM

l+ E
i_r
R

(153)

est limpdance donde au niveau de charge (z = 0).


partir de (152) et (153), on dmontre la formule de transformation des imp
dances donde :
C
+ STE
C th yz
ZR
'
TM

(154)

TM ^ T E + ^ t h 'E
TM

La comparaison des formules (152.) et (147) ou celle des formules (154) et


(149), nous montre quc// guides d ondes, c est l impdcmce d oncle qui joue le
mme rle que l impdance pour les lignes T.E.M. En effet, le concept dimp
dance au sens classique (rapport tension/courant) na pas de signification phy
sique en guides dondes : si le courant en un point dabscisse z peut tre dfini
comme la circulation du champ magntique le long du contour du guide, par
contre la tension dans un plan de section transverse ne peut pas tre dfinie
dune faon unique, en raison de lambigut sur le choix du couple de points
entre lesquels elle doit tre dtermine.
En guide dondes, on peut donc utiliser le formalisme tabli en lignes T.E.M.,
condition dy remplacer limpdance par limpdance donde. En particulier,
limpdance donde TE ou TM doit tre substitue limpdance caractristique
dune ligne. Cependant, alors que limpdance caractristique dune ligne
T.E.M. est propre cette ligne, limpdance donde dun guide est propre un
type de propagation TE ou TM dans ce guide et, plus spcifiquement encore,
lordre mn du mode qui sy propage. Il n est donc pas possible, en toute rigueur,
de parler d impdance caractristique d un guide d ondes.

13.7.3 L'impdance rduite


La seul quantit qui ait la mme signification physique car directement et uni
quement lie au coefficient de rflexion aussi bien pour les lignes T.E.M. que
pour les guides dondes, est :
1 + E w e ' 2^

pour les lignes T.E.M. elle est le quotient Z(z)/Zc,


pour les guides dondes, elle est le quotient (z)/ j e ou tm .

Elle est appele impdance rduite. Son nom, qui esi li la faon dont elle est
gnralement introduite, ne doit pas faire oublier quil sagit d un nombre sans
dimension et dun concept qui na rien voir avec le concept classique dimp
dance.
Nous noterons cette impdance rduite par un z minuscule soulign dune part
parce que c est, en gnral, une quantit complexe, dautre part afin dviter
toute confusion avec la coordonne z qui repre un point de la ligne ou du guide.
Comme

e 2^z = E(z), la relation (155) scrit :


1 + E(z)
l-E (z )

(156)

Si nous nous plaons en z = 0, sur la charge, nous avons :


1 + E-R
,
Zr ~
On peut aussi exprimer E ou
(156)

(157)

en fonction de z :

Efe) =

l~ 1
Z+1

(158)

1
0 5 7 )-*

Zr + 1

(159)

Rappelons que ces relations sont la base du diagramme de Smith qui a t


tudi au chapitre 4 Diagramme de Smith .

&

EXERCICE 13.1
(Paragraphe 13.3 I - Ondes TE ou TM
telles que > 0)

Une onde de frquence/ = 3 GHz, traversant une


section de guide dondes de longueur L - 2 cm,
rempli dun dilectrique de permittivit relative
er subit un dphasage de 288. On sait, de plus,
que la frquence de coupure du guide dondes
vide est de 9 GHz. Quelle est la valeur de , 7

EXERCICE 13.2
(Paragraphe 13.4 - Les guides d ondes
rectangulaires)
Le guide dondes standard franais de la bande
X (8,2-12,4 GHz) avait pour dim ensions :
a - 20 mm et b = 10 mm. Supposons que ce
guide soit rempli dun dilectrique de permitti
vit relative er = 2,56.
1) Trouver par le calcul les modes TE propagatifs jusqu 15 GHz.

2) Retrouver ce rsultat par une mthode gra


phique, en portant en abscisses (ordonnes)
les points reprsentatifs du rapport m ja (n /b )
et en traant les droites passant par ces points.

&

EXERCICE 13.3
(Paragraphe 13 4 - Les guides d'ondes
rectangulaires)

Quelle est la bande passante maximale dun


guide rectangulaire en mode fondamental,

sachant que ses dimensions ne peuvent tre


infrieures k / l . Pour avoir une telle bande,
quelle relation doit-il y avoir entre les dimen
sions a et b du guide ?

&

EXERCICE 13.42
1
*
(Paragraphe 13.4 - Les guides d'ondes
rectangulaires)

On considre un guide d ondes rectangulaire


dont les dimensions a et b sont trs voisines et
telles que : a
b. Ce guide est excit simulta
nment en polarisations horizontale et verticale.
1) a) Entre quelles limites doivent tre com
prises les dimensions a et b de ce guide
pour quune onde occupant la bande de
frquences ( /j ,/^ > f \ ) puisse sy pro
pager selon les deux modes fondamen
taux TE10 et TE01, lexclusion de tous
les modes dordre suprieur.
b) Quelle relation doit-il y avoir entre /[ et
f 2 pour que le problme soit possible ?
Montrer que le choix de a = 1,8 cm et
h = 1,7 cm est bien justifi si lon prend
/ , = 9 GHz et f 2 = 11 GHz.
2) a) une frquence /, quel dphasage un
tronon de ce guide, de longueur L, intro
duit-il entre les deux composantes des
champs E qui se propagent ?
b) Quelle doit tre sa longueur L0 pour qu
une frquence / 0 = 10 GHz, il produise
une onde polarisation circulaire ?

&

EXERCICE *13.5

(Paragraphe 13.5 - Les guides d'ondes circulaires)


1) Rayon a dun guide dondes circulaire pour
quil ne propage que le mode fondamental
TE,, de 16 18 GHz.
2) On prend a = 0,6 cm. Quels sont les modes
qui peuvent se propager jusqu 35 GH/. ?

&

EXERCICE 133.6

(Paragraphe 13.5 - Les guides d ondes circulaires)


Un guide dondes circulaire est excit de telle
sorte que seuls des modes TM peuvent sy pro
pager.
1) 10 GHz, dterminer entre quelles limites
doit tre compris le rayon a de ce guide pour
quil ne sy propage que le mode TM0].
2) On choisit a = 1,5 cm. Quelle est la bande
passante de ce mode TM0, ?

Dans une premire exprience, ce guide est


rempli d air et lon pointe la position d un
minimum nul de champ lectrique une dis
tance zm du court-circuit pris pour origine.
Dans une seconde exprience, on place contre
le court-circuit un morceau de dilectrique sans
perte qui remplit compltement le guide sur
une longueur f? = 1,5 cm. Soient E = q, r et
p = p0 les paramtres caractristiques de ce
dilectrique. On retrouve un minimum nul de
champ lectrique en se dplaant de d = 0,97 cm
vers le gnrateur partir du prcdent mini
mum.
1) Quelles sont les expressions des impdances
donde du mode fondamental TE ,0 dans la
partie o le guide est rempli dair (soit "0)
et dans la partie o il est rempli de dilec
trique (soit jj), en fonction de p0, p, Xq, t r
X et Xc.
2) Dans le cas o le guide est totalement rempli
dair, dterminer (z) et la distance zm.
3) Dans le cas o le guide est partiellement
rempli de dilectrique :

&

EXERCCE 13.7
(Paragraphe 13.7 - Le concept d impdance,
d'impdance d onde et d'impdance rduite)

Le guide d ondes standard de la bande X


(a - 22,86 mm, b - 10,16 mm) est aliment par
un gnrateur de frquence 10 GH/, et est ter
min par un court-circuit.

a) Dterminer ,(() linterface dilectriqueair.


b) Dterminer (.v + (?) une distance s de
cette interface dans le tronon rempli dair.
4) crire la condition qui est impose (s + (!)
en un minimum nul de champ lectrique. En
dduire la valeur de r

Chapitre 14
Les gu ides i'ossies
d itecfr ici ns

14.11

Introduction

De mme quil existe des guides dondes mtalliques o les ondes lectro
magntiques (O.E.M.) se propagent lintrieur dun dilectrique dlimit par
une enveloppe mtallique, il existe aussi des guides dondes dilectriques o
les O.E.M. se propagent lintrieur dun dilectrique dlimit par une enve
loppe constitue par un autre dilectrique (fig. 1).

E2
Figure I.
Propagation dans
un guide dilectrique.

Les guides dilectriques sont, en gnral, constitus


par des cylindres de dilectrique (fig. 2) dont la per
mittivit Ej est suprieure la permittivit e2 du milieu
extrieur. La partie centrale est appele le cur (core,
en anglais) tandis que la partie situe autour est appele
la gaine (en anglais, cladditig).

Figure 2.
Coupe transversale.

Rappelons que la permittivit absolue et la permittivit relative Er dun milieu


dilectrique sont lies par la relation e = 0 Er dans laquelle q (F/m) = 1 / (36k 10y)
est la permittivit absolue de lair ou du vide.
Dautre part, la racine carre dC la permittivit relative dun milieu est son indice
de rfraction n :
(i)
Dans ce chapitre, nous utiliserons indiffremment lun ou lautre.
Rappelons aussi que la longueur donde dans un dilectrique de permittivit Er
se calcule d aprs :
( 2)

(3)
est la longueur donde dans lair ou le vide. Pour quil ny ait pas dambigut,
nous la noterons toujours
; nous rserverons la notation X la longueur
d onde dans un dilectrique autre que lair.
Dans ce chapitre, nous tudierons la propagation lintrieur des guides dondes
dilectriques en utilisant soit le concept de rayons ( 14.2, 14.4 et 14.5) soit le
concept dondes ( 14.3, 14.6 et 14.7). Nous verrons quil sagit dune propaga
tion par modes, comme pour la propagation en guides dondes mtalliques.
La frquence de fonctionnement optimale se situant dans le domaine des fr
quences optiques, on a donn aux ralisations technologiques de ces guides
dilectriques, le nom de fibres optiques.

4 -2

Propagapon
STRUCTURE

en g u id es d 'lec tr iq u es
plan e

14.2.1 Rflexion sur une interface plane


entre deux dilectriques
Considrons deux milieux dilectriques de permittivits j et e2, spars par une
surface plane X (fig. 3). Soient 0[ et 02 les angles que font les directions de pro
pagation des ondes, dans chacun de ces milieux, par rapport la normale XNous rsumons ci-aprs les rsultats dmontrs au paragraphe 11.4 Rflexion
et transmission linterface de deux dilectriques dans les deux cas fondamen
taux de polarisation :

-S
i*

Cas TE o E est perpendiculaire (et H parallle) au plan d incidence.


Cas TM o H est perpendiculaire (et E parallle) au plan dincidence.

Figure 3.
Rfraction et rflexion
linterface de deux
milieux dilectriques.

Figure 4.
Variations de R et R/t
pour 0 < 0 , <

Le coefficient de rflexion du champ F est donn respectivement par :


y j | cos 0 (
*L =

dans le cas TE

(4)

dans le cas TM

(5)

E j co s 0 J + ^ / e 2 COS 0 2
E , COS 0 2 - ^ / E

COS

e,

R =
Ej cos e 2 + \ /^2 cos 0 1
Daprs la loi de Descartes :
\f~, sin 0 L

2 sin e 2

( 6)

Si j > 2, cela implique que 0 ( < 02. Les limites de variation de 0, et 02 sont
donc : 0 < 0j < 0j^ et 0 < 02 < 7t/2.

Langle Q{L est donn par :

Si 0, = 02 = 0,

sin ^

R =R/i =

(7)

(8)

Si 0j = 0 1Z^et 02 = n / 2, R , = + 1 et R/f - - 1.
La figure 4 montre les variations de R et R,/ quand 0, varie entre 0 et
Nous
notons quil y a une valeur de 0 ( pour laquelle R// = 0 : c est lincidence
Brewsterienne correspondant 0,/j = arctg y j e2/j Dautre part, pour langle

(9)

il y a rflexion totale dans les deux polarisations.


Considrons (fig. 1) une couche de dilectrique de pennittivit Ej, entoure par
deux autres couches de pennittivit e2 < j. Si 0 t > Gj^, nous pouvons donc
avoir propagation par rflexions successives linterface entre les deux couches
de dilectrique. Cest le principe des guides dondes dilectriques.

14.2.2 Les modes de propagation


La condition de propagation guide 0 l ^ 0 1L est ncessaire, mais elle nest pas
suffisante.
B

Figure 5.

Plans donde 7t et jt
correspondant
aux directions
de propagation
AF et CD.

Considrons, en effet, deux directions de propagation parallles (fig. 5) telles


que AB et CD et deux plans donde 7t et 7t, relatifs ces deux directions, qui
sont confondus. Pour que les champs correspondant ces plans donde saddi
tionnent et donnent donc lieu propagation, il faut quils soient dphass de
2mn. Or, le dphasage d au parcours PBCP est :
271
4iia
(2a cos 9) = -----cos 0
<P:
A,,
A,.

( 10)

Soit tp le dphasage d chacune des rflexions en B et C. Ce dphasage dpend


de langle d incidence 0, mais aussi du cas de polarisation tudi : cas TE
-*
P
(TM) si E(H) est perpendiculaire au plan de la figure.
Le dphasage total est : tp + 2tp. Il doit tre gal 2mn. Il y a donc une suite de
valeurs de 0 possibles correspondant a la suite des valeurs de ni : soient (0t , me)
dans le cas TE et (6/r mh) dans le cas TM. Comme certaines de ces valeurs sont
diffrentes alors que dautres sont identiques, il en rsulte que la propagation en
guides dilectriques se fait par modes TE et TM spars ou par modes TE et TM

superposs que l on appelle m odes hybrides et qui sont nots HE (pour


TE + TM si le mode TE est prpondrant) ou EH (pour TM + TE si le mode TM
est prpondrant).
Le nombre M de modes (TE ou TM) pouvant se propager dans un guide dilec
trique structure plane, dont la couche centrale a une paisseur a, peut se cal
culer par la formule :
M = 2 y E l r - 2r 5^

14*3

P ro pagatio n

(11)

en g u id es d ilec t r iq u es

STRUCTURE CYLINDRIQUE
Ces guides dilectriques (fig. 2) sont constitus dune partie centrale, de rayon a,
appele le cur dont la permittivit relative est e Ir et dune enveloppe appele la
gaine dont la permittivit relative est e2r La propagation dans ces guides se fait
galement par modes TE, TM, EH et HE ; elle sera tudie en dtail au para
graphe 14.6 Thorie lectromagntique des guides dilectriques .

1A.3.1 Le mode fondam ental


Le mode fondamental est le mode hybride HE (, dont la frquence de coupure
est thoriquement nulle. Les premiers modes dordre suprieur sont les modes
TE0l et TM0l dont la frquence de coupure est :
yOi _
2,405c
( 12 )
'2 r

Pour que le mode fondamental se propage seul, il faut q u e /c / 1, do lon tire :


si a est donn

/<

2,405c

(13)

2tC \ J Z \ r ~ E 2r

si f est donne :

a<

____2,405c

2nf \ [ E1r

(14)
2r

Il est galement possible de raisonner en longueurs dondes. La longueur donde


de coupure est :
1

D o :

(16)

soit :

(17)

est la diffrence relative de permittivit entre le cur et la gaine.


Pour que le mode fondamental se propage seul, il faut que
tire :
^
2.JZ
>vl > 2^5 V A

si ci est donn :

2,405 X !

a<

si X est donne :

> A^01, do lon

(18)

2n

(19)

Remarque
Im longueur d onde Ay intervenant dans ces formules est celle dans le dilectrique
qui constitue le cur. Elle est relie la longueur d onde A0 = c / f dans le vide
(ou l air) par A ; = A
Si l'on veut exprimer ces conditions par rapport

J\J

Ay, il faut donc crire :


2k a
<>> 2,405
2,405 Av
a < ---------2k

ou

( 20 )

( 21 )

Nous voyons que ces conditions seront dautant plus faciles tenir que la diff
rence de permittivit entre le cur et la gaine sera plus petite ; do le choix de
permittivits trs voisines pour le cur et la gaine.
E x e m p le
ny =

1,456 (silice) et n2 = 1,450 (silicone)

y elr - e2t = yj 1,456 - 1.4502 =0,132


2a<^ O j 3 2 o - ^ ^
Le diamtre du cur doit donc tre infrieur 5,8 X0.
Mais pour que la propagation du fondamental seffectue en quasi-totalit lin
trieur du cur - avec le minimum de pertes - on a intrt ce que les dimen
sions du cur soient aussi grandes que possible. C est pourquoi on utilise, dans
la pratique, des curs don' le diamtre correspond la limite suprieure auto
rise soit, dans cet exemple, 5,8 A0.

14.3.2 Choix d'une frquence optimale


Pour le mode fondamental, laffaiblissement de propagation est donn par :
a (dB/m) - 9 1 F (GHz) ^ 7 ^ tg 8

(22)

Or, alors que e ]r ne dpend pas de la frquence, en revanche tg 8 en dpend. Par


consquent, la propagation en guides dilectriques ne pourra tre envisage que
dans la mesure o le produit F tg 8 aura une valeur suffisamment faible.
Aux frquences radiolectriques, tg 8 reste peu prs constant ou crot trs
lgrement avec la frquence. C est ainsi que pour le polythylne 10 GHz,
tg 8 = 0,35 O " 3 et e lr = 2,25. Dans ces conditions :
a = 91 10 72725 - 0,35 10 ~ 3 - 0,4 dB/m.
Une telle attnuation est prohibitive pour les tlcommunications (400 dB/km).
Il y aurait la solution d oprer des frquences cent fois plus faibles
(100 MHz o. - 4 dB/km) mais alors les dimensions du guide deviendraient
beaucoup trop grandes puisque nous avons vu quelles sont de quelques lon
gueurs donde.
Aux frquences optiques, c est--dire des frquences de lordre du THz
(1014 Hz) o la longueur donde est de lordre du micron, plusieurs matriaux
ont la proprit remarquable de prsenter, dans certaines gammes de fr
quences, des fentres transparentes o leurs pertes sont trs faibles. Ainsi,
500 THz (kG- 0,6 pm) la silice a un tg 8 = 15 10 " 11 et lr = 2,25.
Nous avons alors : a = 91 500 103 1,5 15 10 11 = 0,01 dB/m.
Soit une perte de 10 dB/km acceptable pour les tlcommunications.
Avec des silices spciales, des affaiblissements de 1 dB/km, et mme moins, ont
pu tre obtenus. Cest pourquoi les guides dondes dilectriques sont utiliss aux
frquences optiques o ils sont, en fait, connus sous le nom de libres optiques.

L es

fibres o pt iq u es

Il y a deux types de fibres :


les fibres monomodes, dont le diamtre est de quelques longueurs donde,
les fibres multimodes, dont le diamtre est de plusieurs dizaines de X.

14.4.1 Les fibres monomodes


Les fibres monomodes ne propagent que le mode fondamental H En . Ceci est
trs intressant car, puisque lO.E.M. na quun seul mode de propagation, elle

na quune seule vitesse de phase et son parcours (fig. 6), par rflexions succes
sives lintrieur du cur de la fibre, est unique et bien dfini : il ny a donc pas
de distorsion de phase et les signaux sont transmis sans dformation.
Figure 6.
Propagation dans
une fibre monomode
saut dindice.

Ces fibres conviennent donc trs bien pour la transmission de signaux analo
giques ou de signaux numriques occupant une arge bande de frquences
(1 GHz) et sur de trs grandes distances (50 100 km) sans rpteur. Comme la
dimension du cur est de lordre de quelques longueurs donde dans le cur
(cest--dire de 4 8 pm), ces fibres monomodes sont trs difficiles raliser et
le raccordement de deux tronons de libres pose de dlicats problmes de
connectique. Par exemple, si le rapport des surfaces en regard, aprs connexion,
est de 0,8, seule 80 % de la puissance sera transmise, ce qui correspond une
perte de - 1 dB.
C est pourquoi, dans bien des applications o il est possible daccepter une cer
taine distorsion des signaux transmis notamment pour la transmission num
rique par impulsions, o celles-ci sont rgnres la rception - lon prfre
utiliser des fibres de plusieurs dizaines de microns, qui sont multimodes.

14.4.2 Les fibres multimodes

Figure 7.
Propagation dans les
fibres multimodes.
a. fibre saut
dindice,
b. fibre gradient
dindice.

Les fibres m u lti


m odes propagent
donc
p lu sie u rs
modes qui ont des
vitesses de phase
diffrentes et des
parcours diffrents,
do distorsion de
phase pour le signal
transmis.
C et in co n v n ien t
est particulirement
sensible dans les fibres multimodes saut dindice (fig. 7a) dans lesquelles les
temps de parcours sont directement proportionnels aux longueurs des trajets
puisque la propagation seffectue dans un milieu indice constant.
Cet inconvnient peut tre limit dans les fibres multimodes gradient dindice
o lindice du cur diminue progressivement depuis laxe de la fibre o il a une
valeur n l jusqu la priphrie du cur o il a une valeur n2 gale celle de la

gaine (fig. 7b). Il en rsulte que les parcours caractristiques des divers modes
sont incurvs de telle sorte que :
pour les modes dont le parcours est le plus long, les vitesses de phase augmen
tent au fur et mesure quils pntrent dans des zones dont lindice diminue,
pour les modes dont le parcours est le plus court, les vitesses de phase sont
plus faibles puisquils se situent dans des zones o lindice reste relativement
lev.
Ainsi, en dterminant convenablement la loi de variation de lindice, il est pos
sible dobtenir des temps de parcours qui sont peu prs les mmes pour les
divers modes, ce qui permet de limiter la distorsion de phase (galement appele
dispersion modale).
n 1k
,---- =
n,
y
\ \
X / \\ \ \
a =2
/ /
/
\ Y'
/
\ V'
a 1
/ /
/ /
/ /
\
1/
\ \
1/
\ \
1/
\\
(X

n 2

OO

Figure 8.
Profils dindice pour
quelques valeurs du
paramtre a.

--------------;---------------- ------- r

La variation de lindice dans le cur (fig. 8) peut tre reprsente par une loi du
type :
^
,
n{r) - r i \

1 - 2A |

A=

I ~
aj

o 0

< r < a

i n 2
n

(23)
(24)

est la diffrence relative dindice.


r est la distance radiale compte depuis laxe de la fibre.
a est un paramtre caractristique du profil.
Pour a = , n(r) = /?, pour r < a et n(r) ~ n9 pour r = a. Cest le cas limite de la
fibre saut dindice. Pour a = 2, le profil dindice suit une loi parabolique ; c est
cette loi qui permet de limiter le mieux la distorsion de phase.
Le nombre de modes M transports par une fibre multimodes se calcule partir
du carr du paramtre
2na
V=
y /n \-n \
'o
appel la frquence normalise de la fibre ;

pour une fibre saut dindice :


(26)

pour une fibre gradient dindice et profil parabolique :


(27)

pour une fibre gradient dindice et profil de paramtre a quelconque :

Les fibres multimodes saut dindice ne sont utilises que pour des liaisons
courte distance (de lordre du km) ; elles ont des diamtres de cur de lordre de
100 a 200 pm. Les fibres multimodes gradient dindice sont utilises dans les
liaisons terrestres large bande passante (100 MHz) et sur des distances dune
dizaine de kilomtres sans rpteur. Le diamtre de leur cur est de lordre de
50 pm et celui de leur gaine de 125 pm.

-5

C a r a c t r is t iq u e s

d es r b r e s o p tiq u es

14.5.1 L'ouverture numrique


Il y a une relation entre langle de rflexion linterface entre les deux dilec
triques, et langle dincidence de londe produite par le dispositif dmission, sur
la face dentre du guide dondes dilectrique. Traitons le cas (fig. 9) dun guide
dilectrique dont le cur et la gaine sont respectivement caractriss par des
et dont le face dentre se trouve plonge
indices n ] =
dans un milieu extrieur dindice ne.

Figure 9.
Trajets dans une fibre
optique dans les
conditions de
rflexion totale.

Loi de Descartes en M :

ne sin 0 = j sin 0

Loi de Descartes en N :
Dans le triangle MON :

/?! sin $ = 2 sin


0 + (]) = t/2.

Pour quil y ait rflexion totale en N, il faut que $ soit suprieur ou gal langle
limite
tel que arcsin tpm = d/ [. Nous avons donc > <|)m => 0 < BM et
0 < 6m. 11 y a donc un angle dincidence maximal des ondes, QM, qui dfinit en
M un cne lintrieur duquel doivent se trouver les rayons incidents si lon
veut que la rflexion totale se produise en N.
Par analogie avec les instruments d optique, on dfinit un paramtre, appel
ouverture numrique de la fibre (numerieal aperture, en anglais) et not N.A.,
qui est li BM par :
N.A. = ne sin 6M
(29)
d o:

= arcsin A

(30)

ne

Calculons ce paramtre :
En N : j sin tpw = rt2.
En M : ne sin 0M = n, sin Q'M = rt, cos tpm.
D o :

N .A .= iy / l - |^ y j

(31)

Si : j = n2 J = o + Art, avec An petit.


Donc :

N.A. =

2n j An

(32)

Pour une fibre gradient dindice la formule de louverture numrique est la


mme que (31) ou (32) si lon remplace rtt par lindice n(. selon laxe du cur de
la fibre et An = /r j rt2 par nc n2Rendement nergtique du couplage metteur-fibre
La connaissance de louverture numrique d une fibre et du diagramme de
rayonnement de la source mettrice est essentielle pour dterminer le rendement
nergtique du couplage metteur-fibre.
Soit :

Pp(B) = PQcos"' 0 (w > 1)

(33)

le diagramme de rayonnement en puissance de la source mettrice dans un plan


passant par la direction de rayonnement maximal 0 = 0. Pour simplifier, nous
supposons que ce diagramme est symtrie de rvolution autour de cette direc
tion de rayonnement maximal.
Dans ces conditions, la puissance Pe rayonne dans lespace lintrieur dun
cne de demi-angle au sommet 0O, est donne par :
ffln
Pp sin 0 d0 = 2k Pq J cos'" 0 sin 0 d0

^ (e

Lintgrale / est facile calculer :


rcosG
rcos

J,

1 - cos"'+ 1 0
cos"' 0 d(cos 0) =

1 - cos'" + 1 0.
D o :

P'e (Gy) ~ 2 n Pq

m+1

(34)

La puissance totale rayonne par la source metlrice est obtenue en prenant


6 = K 2
7t
l 9
do :

-n
m~+7 P

pe (6()) = ^ ( ? ) ( 1 cos" '+ e o)

(35)
(36)

Si Bm est langle maximal dadmission des ondes par la libre optique, le rende
ment nergtique du couplage de la source mettrice la fibre est donc :
(37)
1 - cos'Ml +- 1 M
Une autre cause de pertes est prendre en compte pour le calcul du couplage
metteur-fibre : lindice ne du milieu o est mise la lumire est diffrent de lin
dice n | de la fibre ; souvent ce milieu est de lair et ne = 1. 11 en rsulte, en pre
mire approximation, un coefficient de rflexion dont le module est donn par :
" \ - e
a
(38)
n, + n.
Le pourcentage de puissance transmise dans la fibre est donc donn par :
1-^= 1-

n \ ~ n e\
n i + il el

(39)

La perte correspondante est appele perte de Fresnel ; elle est, en gnral,


trs faible devant la perte de couplage.

La bande passante
Dans une fibre optique utilise pour les tlcommunications, les informations
sont transmises sous forme de signaux numriques c est--dire d impulsions
spares par des zros. Aprs transmission, il faut que ces impulsions puissent
tre clairement reconnues, afin de reconstituer linformation avec un minimum
d altrations. Si les.impulsions transmises sont dformes et largies en cours de
propagation, elles peuvent se chevaucher larrive et il devient difficile, voire
impossible de les sparer et donc de reconstituer le signal transmis.
Pour dterminer la bande passante dune fibre, nous allons donc calculer llar
gissement, en cours de propagation, d une impulsion dont la largeur lmission
serait nulle. Si llargissement est AT, le nombre dimpu'sions transmissibles par
seconde et l/A T (bil/s) et la bande passante est l/A T (Hz).
Ce phnomne provient de ce que, dans une fibre multimodes (fig. 7), il y a plu
sieurs chemins possibles pour aller dun point de laxe de la fibre un autre. En

effet, les premiers modes se propagent quasiment en ligne droite tandis que les
modes d ordre suprieur se propagent avec un grand nombre de rflexions
totales linterface des deux dilectriques.
a) Prenons d'abord l'exemple d'une fibre saut d'indice
Pour aller de M P (fig. 9 ) :
le trajet minimal est MP = L,
le trajet maximal est MNNP = 2MV L j sin tpm.
La diffrence des trajets est : AL = h/sin (pm - L
La diffrence des temps de parcours est : A T = n x/ c (L/sin <pm - L)
d o :

A7= t ( ^ - 1J L = ^ ( b * - " 2) 7

(4C)

Cette formule permet de calculer llargissement AT dune impulsion pour un


parcours L dans la fibre.
Si a j et ai2 sont voisins, la formule approche est
AT = An

a v ec

A a

= a , ai 2

(4 1 )

Llargissement dune impulsion se propageant sur une fibre dpend donc de la


longueur de la fibre et de la diffrence relative des ses indices. Le dbit num
rique maximal d une fibre sera l/ A T (bit/s) et sa bande passante maximale
1/ AT (hertz).
E x e m p le
ny = 1,456 (silice) et n2 = 1,4JO (silicone).
PourU = 1 km >AT = 0,046 1/(3 IC) = 0,15 10~ 16 s~ 1.
Le dbit numrique maximal est : 6,666 Mbit/s.
Im bande passante maximale est : 6,666 MHz.
b) Voyons maintenant le cas d'une fibre gradient d'indice
Soit nc lindice de laxe du cur et An - nc - n2.
Llargissement dimpulsion dpend beaucoup du profil dindice adopt, cest-d<re de la valeur de lexposant a de la loi (23) de variation de cet indice ; daprs
[63J:
AT - p(a.)

(42)

Le facteur p(a ) est minimal pour un profil quasi parabolique. Plus prcisment,
lon a p(a) = 0,12^ pour a = 2 (1 - A n /n L).
Llargissement d impulsion dune fibre gradient dindice optimis est donc :

(An)2 L
AToptg

nc

(43)

Par rapport au cas de la fibre saut dindice, A T est multipli par un facteur
A n/8nc ; il en rsulte une bande passante multiplie par 8nc/A n. Ainsi, avec lin
dice nc = 1,456 et le An = 0,046 de lexemple prcdent du a), nous avons
8t./A h = 253 et le dbit numrique passerait donc 1 688 Mbit/s pour 1 km.
En pratique, il est trs difficile dobtenir exactement cette valeur optimale de a
et p(a.) varie trs rapidement en fonction de a . C est ainsi que :
pour a = 2 (profil parabolique) p (a) - 0,5.
pour a = 2 (1 + A n /n c) >p{oc) = 1.
Une valeur assez raliste de AT lorsque lon na pas pris toutes les prcautions
technologiques coteuses pour optimiser le gradient dindice est :
.9
(Ah)' L
(44)
AT =
c

T h o rie le c t r o m a g n t iq u e
DES GUIDES DILECTRIQUES
Nous allons traiter le cas du guide dilectrique saut dindice. Le manque de
place nous oblige ne pas donner le dtail de certaines dmonstrations et ne
pas faire tous les commentaires utiles. Au lecteur dsireux dapprofondir cette
question, nous recommandons la lecture de Marcuse (65, p. 289-305].

14.G.1 Mise en quations


Le dbut de la mise en quations est
identique celle qui a t faite au
paragraphe 13.5.1 pour les guides
d ondes circulaires. Nous recher
chons donc des solutions de la forme
y (p , (p) = F (p )e ~ jmp
(45)
satisfaisant :
dune part lquation diffrentielle
d2F 1 d f
..2 \
kKC
2 - 0 F = 0 (46)
d p 2 P dp
P
ou :

k^. = o rep . + y 2

(47)

soit (si y = j(3) :


I k2 - p2
(48)
d autre part les conditions aux limites suivantes :
1) F ( p ) fini pour p = 0 et F(p) 0 quand p
2) Continuit des composantes tangenlielles des champs la surface de spa
ration des deux dilectriques.
Daprs [65, p. 292], les solutions quil convient de choisir sont les suivantes :

P our < a, F{p) est une fonction de Bessel de premire espce dordre n et
dareument p avec :
w2 = k\ - |32
(49)
e t:

k [ = o r , p

(50)

Les composantes longitudinales des champs sont donc :


E Z = AJ ( i/p ) e - j,MP

(51)

H z = B J n(u p ) e J,,<P

(52)

Les composantes transversales E p, E ^ H p, H_ ^ se calculent daprs les rela


tions (23) (26) du chapitre 13 tude gnrale de la propagation .

NB
Four quil y ait pro
pagation l intrieur
du cur, il faut que u
soit rel positif,
d o :kj2 .p .

P our p > a, F(p) est une fonction de Bessel modifie de premire espce1
d ordre n et dargument ivp avec :
w2 = p2 - k }
(53)
et :
k l eu2 2 p
(54)
Les composantes longitudinales des champs sont donc :
E z = C K n(w p ) e
H z = D K n( w p ) e - W

(55)
(56)

Quant aux composantes transversales, elles se calculent toujours d aprs les


relations (23) et (26) du chapitre 13.
Pour dterminer compltement les champs, nous devons trouver A, B, C, D ainsi
que u et w cest--dire p. De plus, notons que p doit se trouver dans lintervalle :
A'2 < p <
(57)
La dtermination de ces cinq inconnues seffectue en crivant que, pour p = a,
les composantes tangentielles des champs E E <p, K z et LL ^ doivent tre conti
nues, ce qui fournit dj 4 quations dont le dterminant doit tre nul do une
cinquime relation dite quation caractristique ou de dispersion :
7,, ( u a )
u J (ua)

(yva)

w K n (w a)

k \

7,

(ua)

u J n (ua)

k l K

vr K

n
(w a)
n (w a)

= 2

P2

f 1 + 1 1
O
Q \
l u~
w~J

(58)

Au lieu d'utiliser les fonctions de Bessel modifies de premire espce Kn(x), certains auteurs
165]prfrent utiliser les fonctions de Hankel de premire espce H (n" (J.v) qui leur sont relies

par :
Kn(x)-

Jt
iV

n" (i-9

(59)

IVB
Pour les grandes
valeurs de l ' a r g u
ment : K(wp) o wp.
Pour que Ez et Hz >
0 quand p >
il
faut donc que w soit
positif, d o : []> k2.

Nous nallons pas entreprendre la rsolution fastidieuse, et dailleurs pas trs


heuristique, de ce systme dquations. Ce qui va nous intresser maintenant,
c est la dtermination des modes de propagation possibles, de leur frquence de
coupure et du mode fondamental.

14.6.2 Modes de propagation


Lorsque n = 0, lquation (58) a deux solutions :
- lannulation du premier crochet nous donne :
l 7, {ua) ] A', (vva)
=0
u y() (ua) vv Kq (wo )

(60)

On en dduit que A ~ C = 0, soit E, = 0. La propagation se fait donc selon des


modes du type TE ;
lannulation du second crochet nous donne :
k\
u

( )

J\ u a

Aj

J q ( u ) + vv

(wa)

Kq {wa)

On en dduit que B = D = 0, soit H, = 0. La propagation se fait donc selon des


modes du type TM.
* Lorsque n * 0, lquation (58) ne peut tre spare en deux. Physiquement
ceci correspond au fait que les modes qui se propagent ont des composantes
longitudinales E, et LA diffrentes de zro De tels modes sont appels des
modes hybrides et nots EH ou HE selon que la composante prpondrante est
E, ou /A.

14.6.3 Frquences de coupure


Nous avons vu que la variation des champs en fonction de p lintrieur de la
gaine (p > a) est rgie par une fonction de Bessel modifie A(wp) qui a une
dcroissance du type exponentiel Dans le cas o w est grand, les champs dimi
nuent trs vite et la quasi-totalit de lnergie est concentre lintrieur du
cur. Mais, si w nest pas grand, les champs dans la gaine pntrent dautant
plus loin que w est petit. la limite (vv 0), il ny a plus de diminution des
champs dans la gaine, londe qui se propage a la forme dune onde plane et elle
nest plus guide lintrieur du cur.
La condition w2 = p2 - k } = 0, qui conduit une telle propagation, est appele
une condition de coupure. Elle va nous permettre de calculer la frquence de
coupure daprs u ~ \ j k f - P~ qui peut scrire dans ce cas :
uc = \ f k 2\ ~ k 2 = \f ? c

p -

(>l e2 p = 2n

f c

^J (e, - e 2)p

(62)

D o :

fc =

(63)

2n y /( e , e 2) p.

La dtermination des valeurs de u correspondant la coupure est effectue dans


[65, p. 297-301]. Elle donne les rsultats suivants :
les modes TEorn ou TMom (n 0) sont obtenus pour les valeurs uma qui sont
racines de J0{u) = 0 ;
les modes
avec u a * 0 ;

(V n ^ 0) sont obtenus pour les racines uma de : Jn(ua) 0

les modes HE)im (V ^ 0 et 1) sont obtenus pour les racines uma de :


[(,/ e2) + 1] Jn _ ,(a) = \{ua/n) - 1] Jn{ua) ;
les modes H E 1/7I (n = 1) sont obtenus pour les racines uma de : y, (a) = 0.
Le tableau ci-dessous donne les valeurs de uma pour les trois premires racines
de quelques-uns de ces modes. Pour les modes HEnw (avec n ^ 0 et 1) elles cor
respondent au cas o Ej /e 2 = 1,1.
m

n 0
1
1
2
2

2,405

5,520
3,832
7,016
5,538

8,654
7,016
10,173
8,665

8,417

11,620

0,000
3,832
2,445
5,136

M odes i; i TE, TM
HE
EF
HE
EH

1 4 .S .4 !V?ode fondam ental


Parmi tous ces modes, le seul pour lequel um = 0, soit f c - 0, est le mode HEj,
qui est donc le mode fondamental des guides dondes circulaires. Les premiers
modes dordre suprieur sont TE01 et TM01 dont la frquence de coupure corres
pond la premire racine de JJjua) = 0, obtenue pour u ja = 2,405. Pour quils
ne se propagent pas, il faut que ua < 2,405, soit :
2Ttaf
Si a est donn :

,
/<

S i/e s t donne :

Cl

(e, - e2) p. < 2,405


2,405
/------------- 2ka ^ (e , - e 2) p

2,405
< --------...........

(64)
(65)

(66)

* -t. jtggwtyi%x

Dans les relationi


(62)(66):
ei = eQelT,
f Z =

2n

u ~ lh

H 4--7-

C a r a c t r ist iq u e s

de

la propagation

14.7.1 L'attnuation
Les causes des pertes sont au nombre de quatre :
pertes par absorption dans les atomes du dilectrique, les plus gnantes tant
celles dues lion OH qui prsente des raies d absorption 0,875 et
0,950 jim ;
pertes par diffusion, dues aux inhomognits du dilectrique ( ~ 1/X 4) ;
pertes par rayonnement car la gaine a des dimensions radiales limites et est
entoure par une enveloppe dissipative ;
pertes par couplage de modes, dues aux dfauts mcaniques et aux courbures.
Ceci tant, les pertes dpendent de lordre du mode considr. Soient a , et i les
constantes de pertes dans le cur et la gaine ; le coefficient de pertes global a imi
pour un mode dordre nm sera :
a

p(!)
p(2)
n m + a.

(67)

a ... =

P,

P^},\ et pl-} sont respectivement les puissances dans le cur et la gaine ; Pt est la
puissance totale transporte par ce mode.
Gloge [66] a montr que :
rP(2)
nm _ j
Pt
avec :
d o : (n * 1)

J 2 (u)
Jn
J + i (l0 / _ i (u)

(69)

V2 = (u2 + w2)
U2

( 68)

->

2---

a = a , + (<x2 - a l) ~2 (w~ + n 4)

(70)

Signalons que loin de la coupure, Marcuse a trouv :


(71)
o uma est la ni1 racine de Jn{ua) = 0.
La perte totale est enfin obtenue en sommant les pertes pour chaque mode avec
un facteur de pondration gal son importance relative.
lheure actuelle (1996), les grands constructeurs mondiaux annoncent des
pertes, en laboratoire, infrieures 0,5 dB/km. Cela conduit, sur le terrain, des
pertes infrieures 1 dB/km. Notons qutant donn la faible bande de fr
quences de la porteuse occupe par la modulation, ces pertes sont indpendantes
de la frquence du signal transmis

14.7 .2 Dispersion - Bande passante

Figure IL.
Courbes de dispersion.

Les signaux numriques qui se propagent sur une fibre ont un certain spectre de
frquences. Il est donc trs important de connatre la loi de variation du paramtre
de phase (3 avec la frquence. Au lieu de tracer le graphique |3(/), dont les chelles
ne sont pas commodes, on prfre reprsenter (fig. 11) (3/7c2 dont lintervalle de
variation est compris entre 1 et n j n ^ - en fonction de V= 2v.(a!X) \ j 0r
variable proportionnelle /.
Il est souvent plus instructif de tracer le diagramme to - (3 car :
la pente de 0Q reprsente la vitesse de phase

= t/(3 ;

la pente de la tangente en Q reprsente la vitesse de groupe v = dco/d-3.


La figure 12 prsente ce diagramme pour le mode HEn . La courbe a un point
dinflexion au voisinage duquel v reste peu prs constante, do lintrt
dutiliser la bande de frquences correspondante. Les asymptotes la courbe
donnent les
extrmes.

Figure 12.
Diagramme t - (3.

Nous avons dj indiqu paragraphe 14.5.2 que la bande passante dune fibre
sexprime en Hertz par le mme nombre que linverse de llargissement dim
pulsion 1/ AT. Pour une longueur L de fibre :
172)
Pour les fibres monomodes, llargissement dimpulsion ne peut pas provenir de
lexistence de plusieurs modes ayant des trajets et donc des temps de parcours
diffrents. La seule cause d'largissement est la dispersion chromatique qui est
lie aux matriaux constituant le cur de la fibre :
AL t d 2
A7= (73)
~c i r2
c ~
l
AL/L est la largeur de bande relative du signal ; L2(d2n/dL 2) est la dispersion du
dilectrique.
Avec les silices spciales actuellement utilises dans les fibres faibles pertes,
on obtient des AT /L = 1 ns/km pour AX /X = 1 % X = 0,82 pm, ce qui corres
pond une bande passante de 1 GHz.
Pour les fibres multimodes, la dispersion chromatique est infrieure dun facteur
10 la dispersion modale. Llargissement dimpulsion provenant de cette der
nire a t calcul directement par une mthode de rayons au paragraphe 14.5.2.

&

EXERCICE 14.1
(Paragraphe 14.2 - Propagation en guide
dilectrique a structure plane)

Calculer le nombre de modes TE ou TM qui


peuvent se propager dans un guide structure
plane dpaisseur a - 50 |im o se propage une
onde dont la longueur donde dans lair ou le
vide est :
= 1 pm ; n t = 1,5 et n2 = 1,4.
1) Faire ce calcul partir de la formule (10) en
crivant que tp + 2cp = 2mn et en se plaant
dans le cas o 0 = 0 IZ/

2) Retrouver le rsultat en appliquant la for


mule (11).

&

EXERCICE 14-2
(Paragraphe 14.3 - Propagation en guides
dilectriques structure cylindrique)

tant donn un guide dilectrique pour lequel


lr = 2,25 et e 2r = 2, trouver la dim ension
m axim ale du cur, exprim e en longueurs
d onde, pour que seul le mode fondamental
puisse se propager. Calculer cette dimension
pour/ = 230 1012 Hz.

1) Calculer le nombre de modes qui peuvent se


propager X0 = 1,3 ]um dans une fibre saut
d indice et dans une fibre gradient din
dice avec un profil dindice parabolique
2) En dduire le nom bre de m odes X0 =
0,83 [un.

&

EXERCICE 1 4 .4
(Paragraphe 14.5.1 - L ouverture numrique)

1) Une fibre optique a un cur dindice n, = 1,5


et une gaine d indice n2 1,4. Quelle est
son ouverture numrique ?
En dduire langle maximal dadmission des
ondes si le milieu entre la source et la fibre
est de lair.
2) La source mettrice est une diode lectrolu
minescente (DEL) dont le diagramme de
rayonnement en puissance est de la forme
F(0) = P G c o s 2 0. Calculer en dB la perte du
couplage metteur-fibre.
3) Calculer aussi en dB la perte de Fresnel.

&
&

EXERCICE 14.3 *5
0
(Paragraphe 14.4 - Les fibres optiques)

Une fibre multimodes a un cur de diamtre


50 |im. Lindice du cur est n, = 1,45 ; celui
de la gaine n2 = 1,4.

EXERCICE 14.5 *1
(Paragraphe 14.5.2 - La bande passante)

1) Calculer le dbit numrique maximal et la


bande passante maximale, sur une distance
de 1 km, dune fibre multimodes saut din
dice dont les indices du cur et de la gaine
sont : /i j = 1,5 et n2 = 1,4.

2) Mmes questions pour une fibre optique


gradient d indice optim is pour laquelle
nc = 1,5 et /h = 1,4.

<#" EXERCICE 14.6


(Paragraphe 14.5 2 - lui bande passante)

Quelle est la condition pour que ta bande pas


sante d une fibre gradient dindice optimis
soit 1 000 fois plus grande que celle d une
fibre saut dindice ?
Application au cas o n l = nc = 1,5. En dduire
77-,.

EXERCICE 14.7
(Paragraphe 14.7.2 Dispersion Bande passante)
Une diode laser met une longueur d onde
Xq = 1,3 |im avec une largeur spectrale AX =
5 10-y m. La fibre optique dope qui est uti
lise prsente une dispersion chrom atique
d12/i/d?w2 = 0,5 10l m - 2 .
1) Calculer llargissement dimpulsion pour
une propagation monomode sur une lon
gueur de fibre L = 1 km.
2) En dduire le Jbit numrique maximal et la
bande passante maximale.

Chapitre 15
leciriaspri!tIgK!s

HS-11

INTRODUCTION

Une cavit lectromagntique est un volume vide ou rempli de dilectrique,


limit par des murs de type lectrique (interface avec un conducteur) ou de type
magntique (interface avec un dilectrique haute permittivit). Dans le premier
cas, il sagit d une cavit mtallique et dans le second cas d un rsonateur
dilectrique.
Les cavits peuvent, en principe, avoir des formes gomtriques quelconques
mais, en pratique, elles ont une forme cylindrique section rectangulaire ou cir
culaire. Les cavits mtalliques peuvent ainsi tre obtenues en fermant des
guides dondes rectangulaires ou circulaires par des plaques mtalliques perpen
diculaires laxe longitudinal du guide
Si, par un moyen appropri (une antenne du type lectrique ou magntique), un
champ lectromagntique est excit lintrieur dune cavit, il peut sy pro
duire des ondes stationnaires lorsque les conditions de rsonance existent. La
cavit emmagasine alors une certaine quantit dnergie lectromagntique.
Ltude des conditions de rsonance de la cavit ( 15.2) permet de dfinir les
modes propres ainsi que les longueurs et les frquences propres dune cavit.
Nous tudierons ces paramtres caractristiques pour les principaux types de
cavits et de rsonateurs dilectriques ( 15.3).
Lorsque la cavit a des pertes - dans ses parois, dans le dilectrique, par le ou
les couplages avec les circuits extrieurs - il est possible, comme dans un circuit

rsonnant classique, de dfinir des coefficient de surtension dont nous donne


rons les expressions ( 15.4).
La modlisation dune cavit par un circuit rsonnant srie ou pamllle permet
d en calculer limpdance dentre dans un plan de rfrence sur la voie de cou
plage de la cavit avec lextrieur ( 15.S). Cette tude est trs importante car
elle tablit les conditions d quivalence entre le circuit rsonnant constantes
rparties quest une cavit et les circuits rsonnants constantes localises (/?, L,
C) classiquement utiliss aux frquences infrieures.
Lorsque la cavit est couple par un seul accs ( 15.6) elle fonctionne en
absorption ou en rflexion (voire en mission dans les dispositifs radiomtriques) tandis que si elle est couple par deux accs ( 15.7) elle fonctionne en
transmission. Dans le premier cas, nous tablirons les expressions de la puis
sance absorbe ou rflchie et dans le second cas, celles des puissances trans
mise et rflchie. Dans chacun de ces cas, nous analyserons le fonctionnement
de la cavit dans et hors le domaine de rsonance.
Limportance des cavits va bien au-del de leur utilisation en tant qulment
rsonnant ou en tant que frquencemtre dans les circuits hyperfrquences. Elles
sont, en effet, utilises pour la dtermination de la permittivit complexe de gaz
ou dchantillons de matriaux de petites dimensions. En ondes millimtriques,
on utilise des cavits ouvertes analogues aux rsonateurs employs en optique
dans les interfromtres de Fabry-Perot ( 15.8). Notons enfin lutilisation des
cavits mtalliques et des rsonateurs dilectriques pour Fixer la frquence de
rsonance des oscillateurs tubes (tels que le klystron) ou ltat solide (tels
que les transistors effet de champ lArsniure de Gallium).

15.2

TUDE DES CONDITIONS DE RSONANCE


D'UNE CAVIT

15.2.1 Modes de rsonance propre^ d'une cavit


Figure I.
Schma dune cavit
dcoupe dans
un guide dondes
cylindrique.
(Figure extraite de
[681, p. 31)

Considrons un guide dondes cylindrique suppos sans perte daxe Oz dont la


section droite a une lorme quelconque (fig. 1). Soit une onde TE ou TM qui se

propage dans ce guide vers les z > 0 une frquence/. Ses champs dpendent de
par lintermdiaire dun facteur de propagation e _JT \ Si nous plaons, en une
section transversale du guide, une plaque de court-circuit, il y a propagation vers
les z < 0 dune onde rflchie dont les champs dpendent de z par le facteur e-^2.
En vertu des conditions aux limites / = 0 et H = 0, le coefficient de flexion
sur la plaque mtallique est gal :
+ 1, pour les composantes longitudinale de E et transversale de H ,
- 1, pour les composantes transversale de E et longitudinale de H.
Par consquent, les champs rsultant de la superposition des ondes incidente et
rflchie seront affects :
dun facteur e
dant R + 1.

+ e-^2 = 2 cos pz pour les composantes Ez et Hr correspon

dun facteur e ~ - e
pondant R = - 1.

= - 2j sin Pz pour les composantes Hz et ET corres

Si nous dsirons placer une autre plaque de court-circuit en une autre section
transversale, de faon avoir des rflexions successives sur les deux plaques, il
faudra que les conditions aux limites // = 0 et H = 0 y soient satisfaites. Ces
conditions ne concernent que ET et H, qui varient en sin Pz. Comme deux
minima nuis de sin Pz correspondent Pz = pn, il faudra donc que :

Les modes de rsonance possibles de la cavit sont appels modes propres et


nots TEmnp et TMmn/r Les indices ni et n dpendent de lordre du mode TE ou
TM considr. Ce sont deux entiers positifs ou nuis, mais ils ne peuvent tre
nuis simultanment. En ce qui concerne /;, la dmonstration ci-dessus qui
conduit des valeurs de p strictement positives - doit tre complte.
En effet, dans le cas de modes TM la coupure, le champ E est uniquement
f

longitudinal alors que le champ H est transversal . les conditions aux limites sur
des plaques mtalliques transversales sont donc satisfaites quelle que soit la dis
tance sparant ces deux plaques. Ces modes sont nots TMff|JIt) puisque p ninter
vient dans aucune condition.
Nous trouverons donc, dans les cavits, les modes TE suivants :
T E ,0/).... TF
TE01.... .... TE mp
TE ,,,,.... .... T E i,.p
T E lip .... IEiip
et les mmes modes TM, mais avec p pouvant tre nul.
Notons que, pour les cavits paralllpipdiques, il est exclu, en mode TM,
davoir m ou n nul.

15.2.2 Longueurs et frquences de rsonance


propres d'une cavit
Si la frquence f, c'est--dire g> est impose, la distance entre les deux
plaques de court-circuit (hormis le cas des modes
doit tre :

( 2)

Nous savons qu une frquence donne, la longueur donde guide dpend, par
lintermdiaire de la longueur donde de coupure, de lordre (ni, n) du mode
TEmn ou , clu>se propage.
Comme :

(3)

v
/
il en rsulte que les longueurs possibles de la cavit sont :

avec :

A.=

(4)

(5)

Il y a donc une suite discrte de longueurs Cmnp possibles de la cavit correspon


dant dune part la suite des modes TEfjm et TMm/I, dautre part, pour chacun
deux, la suite des nombres entiers p. On dit que la cavit oscille en mode
Enmp
Si la longueur /d e la ca\ it est impose, la condition porte alors sur X^, c'est-dire sur la longueur donde de fonctionnement, daprs :
ie
XK
( 6)
p

D o :

fmnp

v 11]

( 8)

|
avec :

v=

(9)
\f r

Il y a donc une suite discrte de frquences possibles de fonctionnement de la


cavit, correspondant dune part la suite des nombres entiers p, d autre part
la suite des modes TEW/| et TMwm. Ces frquences sont appeles les frquences
propres de la cavit.
Parmi les frquences propres aux modes TE et TM, la plus basse est appele fr
quence fondamentale et correspond au mode propre dominant de la cavit. Les
frquences propres suprieures ne sont pas des multiples entiers de la frquence
fondamentale et leurs frquences se rapprochent lorsque leur ordre crot. Dans la
mesure o la cavit est sans perte, la largeur de bande est nulle autour des
diverses frquences propres ; elles forment un spectre discret de frquences
monochromatiques. Bien sr, il nen est pas vraiment ainsi, en raison des pertes,
et il y a une courbe de rsonance autour de chaque frquence propre (fig. 2).
En gnral, un seul mode propre est associ une frquence propre, mais il
arrive que plusieurs modes propres correspondent la mme frquence, on dit
alors quil y a dgnrescence.
Ah
Figure 2.
Courbe de rponse
spectrale dune cavit
ayant des pertes.
(Figure extraite
de [681, P70)

1 5 .3

P r in c ip a u x t y p e s d e c a v it s
RSONNANTES

15.3.1 Cavit cylindrique section rectangulaire


Les quations des champs sont obtenues partir de celles des champs du guide
dondes rectangulaire, en multipliant :
les composantes E et H_T par 2 cos fiz,
les composantes H_z et E T par 2j sin \iz.
Conformment la relation (8) les frquences propres sont donnes par :

Figure J.
Lignes de champs
E (----- ) et
// ( -)
dans une cavit
cylindrique section
rectangulaire.
Modes TE101,
TMno- TE,,,.

E
H

(Figure extraite de
168/. p. 42)

La figure 3 permet de visualiser les lignes de champs lectrique et magntique


pour ds coupes transversales (au milieu) et longitudinales ( droite) d une
cavit paralllp.pdique fonctionnant en modes T E 10], T M |10 et T E jjj .
La frquence fondamentale sobtient en donnant aux indices ni, n, p leurs plus
basses valeurs (0, 1, 1 ou 1,0. 1) et en faisant correspondre lindice 0 la plus
petite dimension. Dans ces conditions, on trouve que la frquence fondamentale
est soit celle de l'un des modes T E I01 ou TE011 soit celle du mode TM ^g.
{Exemple

A = 3 cm
V = 100 dm3
N 9 mvdes/MHz

Exem ple

K m = 3 C'm
V = 1 dm-> N = 330

Une cavit rectangulaire o peuvent coexister un nombre lev de modes est dite
surJimensionnee. Le nombre de modes possibles dans une bande de frquences
L/0, / 0 + A/J est :
., . r
V AJ
8k rr (11)
A3 h
o V est le volume de la cavit.
Par ailleurs, le nombre de modes correspondant des longueurs donde propres
suprieures une longueur donde minimale est :
N' =

8k

( 12 )

15.3.2 Cavit cylindrique section circulaire


Les quations des champs sont obtenues partir de celles des champs du guide
dondes circulaire, en multipliant :
les composantes _ et H r par 2 cos pc,
les composantes U. et E t par - 2j sin P^.
Les frquences propres sont donnes pour les moues TEm
cavit de rayon R et de hauteur h, par :

et

dune

modes TE mnp

modes TM mnp

Iu mn

fm n p

fm n p

''

'2+

[2nRj

]2
\2 /t j

(P

2
l mn) + \ p 2
[2h\
{2nR)

(13)

(14)

Le tableau I donne les valeurs de umn et u nm pour les premiers modes TMm;l et
TEnm respectivement.

Tableau I

Le mode fondamental correspond la plus petite valeur de p et de umn ou u nm.


Pour les modes TM, il sagit du mode TM0)q pour lequel w01 = 2,405
do:

010 = 2,612 R

(15)

et

/ 010 = 0 ,3 8 3 -

(16)

Pour les modes TE, il s agit du mode T E jjj pour lequel mjj = 1,841, do :

^111

(0,086 0,25
R2 + h 2

(17)

Les configurations des lignes de champs et H sur les parois mtalliques de


la cavit ainsi quen un plan de section transversale mdiane sont donnes la
figure 4 pour ces modes fondamentaux et pour les modes TE0] j et TM01 j.

Figure 4.
Lignes de champs
E (----- ) et
//( - )
dans une cavit
cylindrique section
circulaire.
Modes TEn l, TEon,
TM0I0 et TM011.
(Figure extraite
de 168], p. 51)

ESemarejuc
Afin d'avoir une notation homogne pour toutes les cavits cylindriques, les
modes des cavits section circulaire ont t nots TEmn et TMnwp alors que
les modes des guides d ondes circulaires avaient t nots TEnm et TMnm au cha
pitre 13.
Il serait possible, sans ambigut, d'adapter la premire notation la seconde en
changeant tes places des indices m et n et en notant donc les modes de cavit
section circulaire 77rlimp et TMnmp.
Les valeurs numriques donnes dans le tableau 1pour umn et unln devraient tre
conserves pour les unm et ujull correspondant aux modes TMmnp et TEnmp.

Le diagramme de la figure 5 trac partir des relations (13) et (14) permet :


soit de trouver les frquences de rsonance correspondant aux divers modes
possibles, pour une cavit dont on connat les dimensions (R, h) ;
soit de trouver la hauteur d une cavit correspondant aux divers modes pos
sibles, si lon connat son rayon et la frquence ;
soit de faire linventaire des modes de rsonance dune cavit dans une cer
taine bande de frquences.


C\J

C\J

Cvj

CNi

c \i

.OJ . 5 'r-

Figure 5.
Diagramme des modes
et des frquences de
rsonance dune cavit
cylindrique section
circulaire.
(Pour abrger on a uti
lis la notation E pour
TM et H pour TE).
(Figure extraite
de 168], p. 48)

15.3.3 Rsonateurs dilectriques


Ce sont des volumes dilectriques haute permittivit dont la forme est, en
gnral, cylindrique ou paralllpipdique. Ils peuvent tre considrs comme
des cavits rsonnantes pour lesquelles les conditions aux limites sont celles
imposes par linterface air-dilectrique haute permittivit. Cette interface
constitue un mur magntique alors que dans les cavits constitues partir de
guides d ondes mtalliques, linterface air-mtal constitue un mur lectrique.
Les rsonateurs dilectriques sont utiliss dans certains filtres et dans les oscilla
teurs micro-ondes pour stabiliser leur frquence.
Comme dans les cavits rsonnantes parois mtalliques, la rsonance stablit
selon diffrents modes dont les frquences sont dtermines par les dimensions
de la cavit et les conditions aux limites. Le mode fondamental est un mode
TE0)/J et les dimensions sont de lordre de grandeur de la longueur donde
guide Xs = XQJ\J~r - ,. tant lev, les champ E et H sont confins lint
rieur du rsonateur et son voisinage, si bien que les pertes par rayonnement
sont faibles.

Pour un matriau dilectrique faibles pertes, il est possible dobtenir un coeffi


cient de surtension vide lev. Les matriaux les plus utiliss sont des cra
miques dont la permittivit er peut atteindre 90. Les frquences d utilisation
vont de 3 GHz, au-dessous de laquelle les dimensions du rsonateur seraient
prohibitives, 30 GHz, au-dessus de laquelle le coefficient de surtension nest
plus suffisant.
Les rsonateurs dilectriques sont monts lintrieur dun botier mtallique et
la configuration la plus utilise en micro-ondes est celle reprsente la figure 6
car elle se prte bien au couplage avec une ligne microbande.
Figure 6.
Schma dun
rsonateur dilectrique

air
D
-------

D
T

Figure 7.
Dimensions dun
rsonateur dilectrique

e,

2f

~T~
1L
______ T
i L
/
y L2

Le calcul du diamtre D du rsonateur et de sa hauteur L (fig. 7) se conduit selon


la procdure approche suivante, d aprs Kajfez [72J :

Les hypothses de dpart sont :


la frquence de rsonance recherche,
lpaisseur Zo du substrat dilectrique,
la distance L, entre les faces en regard du dilectrique et du botier,
les constantes dilectriques relatives, er et e2r, du rsonateur et du substrat.
Le diamtre D doit tre choisi dans les limites suivantes :
_ 5 ,4
^ 5,4
<D<
1

(18)

k0 est la constante de propagation en espace libre. k0 = 2iI/X q.


K La hauteur L est donne par la formule :
a
a1
coth a-, L i
- coth a j L j + tg 1
L = tg

VP

VP

(19)

P est la constante de propagation dans le rsonateur dilectrique :


p 2 = A'y e r ~ h 2

(20)

CX| et cx9 sont les constantes dattnuation dans les guides d ondes sous cou
pure, de hauteurs
(dilectrique air) et L2 (substrat dilectrique).

et

^ =h2- k l

(21)

a \ = h 2 - k l t 2r

(22)

Le paramtre h de ces formules est calculer d'aprs :


>o

2,405 +

(23)

2,405 | 1 + - - - - + 0,291 v0

(24)

avec :

Rciproquement, les relations ci-dessus permettent de calculer la frquence de


rsonance du rsonateur dilectrique connaissant ses caractristiques gom
trique ainsi que (. et 2,.. La rsolution prcise de ce problme ne peut-tre que
numrique.

C o e f f ic ie n t s

d e s u r t e n s io n d ' u n e c a v it

15.4.1 Dfinition du coefficient de surtension


Par dfinition, le coefficient de surtension est le produit par 2 j du quotient de
lnergie emmagasine Wa par lnergie perdue W pendant une piiode, soit :
W
0

=2 * 5

(25)

Si P est la puissance perdue moyenne : W = PT et nous avons :


W
e =

(26)

a) nergie emmagasine dans la cavit


Si E et tL sont les amplitudes complexes des champs, lnergie emmagasine
dans un lment de volume dv est :
dWa = ~ z E - E * d v = \ v H

-tL*dv

(27)

Dans la cavit de volume V, nous aurons donc :


Wa = o1 c E E * dl' = M
V~
2

H - H dv

h>) Puissance moyenne perdue dans la cavit


Les pertes peuvent tre de diverses natures :
pertes ohmiques dans les parois, soit P-,

(28)

pertes dans !e dilectrique dont est remplie la cavit, soit Pd,


pertes travers les couplages vers lextrieur, soit Pex.
Les pertes totales sont donc :
(29)

p = Pj + Pd + Pcx
Le coefficient de surtension correspondant ces pertes totales est :
?ch = a>

K
Pj + Pd +PCX

(30)

Il est appel coefficient de surtension (ou de qualit) en charge.

15.4.2 Coefficients c e surtension (eu de qualit)


partiels
Linverse du coefficient de surtension en charge est :
! = pj
Qch

] pd

Pcx

CO Wu <Wa

coV^

(31)

Si Wu S> P-, Pj, 7Jex, nous pouvons considrer que ces pertes naffectent pas
lnergie emmagasine W et introduire des coefficients de surtension partiels
Qj, Qd' Qe\ en Panant pour rfrence le mme co Wa. Dans ces conditions :
I
(?Ch

Qj

Qd

(32)

Qcx

Les inverses des coefficients de surtension partiels sajoutent et linverse du


coefficient de surtension de la cavit est la somme des inverses des coefficients
de surtension partiels. Nous voyons donc que toute cause de pertes supplmen
taire a pour effet de diminuer la surtension de la cavit.
Le coefficient de surtension interne (ou propre) Qpr est dfini par :

JL - J
Qpr

Qj

(33)

Qd

Il est d uniquement aux pertes dans les parois et le dilectrique de la cavit.


Nous pouvons crire alors :
1
<2ch

avec :

1 _ 1

Qpr + Qcx ~ Qpr

(1+ P )

Q pr
P=

Qc

P est le coefficient de couplage de la cavit au circuit extrieur.

(34)

(35)

Si P < 1, la cavit est dite sous-couple : les pertes extrieures sont infrieures
aux pertes propres.
Si P > 1, la cavit est dite sur-couple : les pertes extrieures sont suprieures
aux pertes propres.
Si P = 1, la cavit est au couplage critique.

15.4.3 Caku! de

Qj

La puissance moyenne perdue par effet Joule dans les parois est :
2

(,

(36)

avec S, surface des parois de la cavit et Rs, rsistance superficielle des parois de
conductivit a ,. Si 8^. est la profondeur de pntration des courants dans les
parois, due leffet de Peau :

avec

*s = r
a ! 8.V

(37)

8S , 1 - ;
\ / k P, CT./

(38)

En crivant : Rs = (0 P[ 8^ 2, nous obtenons :


i to p , 8. j*
P - : ------H - H dS
J

2.

(39)

Le coefficient de surtension Qj d aux pertes dans les parois est donc :


>, = co

l jt
. 1 03f11 8i'
H H dv-

JS

H - H dS

' '

(40)

Compte tenu de ce que p = p ,, il vient :


1

1
>

'- S

Il H H* dv'l ! 1 H - H* dS

- 1

)S

(41)

En introduisant le volume V de la cavit et la surface S des parois :


2
8 S
Finalement :

V 1 f
ri f

H H dv H - H dS
S Js - ~
5 V Jv - V
F
S

(42)

(43)

F est un facteur de forme gal au rapport des valeurs moyennes du champ


magntique dans le volume V (1er crochet) et sur la surface S (2e crochet) de la

cavit. Ce facteur, qui dpend de la gomtrie de la cavit et du mode de rso


nance tudi, est compris entre 0,1 et 1. On en trouvera dans [68, p.74 et 75] les
expressions pour divers types de cavits.
La frquence qui intervient dans le calcul de 6^ est la frquence de rsonance de
la cavit. Donc, 8S est proportionnel la racine carre de la longueur donde de
rsonance. Par ailleurs, V/S qui est homogne une longueur, est donc propor
tionnel la longueur d'onde de rsonance ; finalement :
(44)
La surtension des cavits ne pourra donc pas atteindre des valeurs trs leves
en ondes millimtriques o lon devra utiliser des rsonateurs spcifiques (voir
15.8.2 Rsonateurs en ondes millimtriques ).
Notons enfin que, pour une frquence de rsonance donne, Qj est proportionnel
\ j G\ : il est donc oossible daugmenter Qj en levant la conductivit des
parois, ce qui peut tre obtenu basse temprature, avec des parois supraconductrices.

15.4.4 Calcu! de

Qd

Soit une cavit remplie dun dilectrique perles, prsentant donc une certaine
conductivit o. Ce dilectrique peut tre caractris par la pennittivit com
plexe :

est crit sous la forme 0 ( - je).


La puissance moyenne perdue dans le dilectrique est :
(46)
Le coefficient de qualit d aux pertes dans le dilectrique est donc :
r

- '

I-

Jv

Or e / o

= e / e

(47)

est linverse de la tangente de langle de pertes du dilectrique.


I

Do :

coe

(48)

Ce coefficient de qualit est donc indpendant des dimensions de la cavit et de


son mode de rsonance. Il ne dpend que des pertes du dilectrique remplissant
la cavit. En hyperfrquences, les meilleurs dilectriques homognes ont un

tg 6 = 10 4, ce qui limite Qd 104. Pour obtenir des coefficients de qualit plus


levs, il faudra donc utiliser des cavits vides ou remplies dair.

1 !5 .5

M odlisatio n d ' une


Im pd a n ce d ' en tre

cavit

Puur quune cavit rsonnante puisse tre utilise en pratique, elle doit tre cou
ple avec dautres lments extrieurs et tre excite par une source une fr
quence voisine de lune de ses frquences propres. La cavit fonctionne alors en
oscillations forces.

15.5.1 Modlisation
Vu de la ligne ou du guide daccs la cavit, lensemble situ au-del dun plan
de rfrence P j peut tre considr comme une charge. Celle-ci est caractrise
par son coefficient de rflexion T ou par limpdance ou ladmittance rduite
quelle ramne en ce plan. Ces paramtres sont relis par :
1 + T
z = -----=
i-r

(49)

et

1 - r
v = ------' i+E

(50)

F pourra tre dtermin par les mthodes de mesure classiques effectues sur la
voie daccs ; on en dduira z et y. Par ailleurs, sil est possible dexprimer z et y
en fonction des paramtres caractristiques de la cavit que sont co0, p et Cpr, on
pourra calculer ces derniers en dterminant T trois frquences diffrentes.
Le comportement dune cavit rsonnante tant identique celui dun circuit
rsonnant, nous allons nous servir des rsultats, bien connus, de ces circuits pour
identifier une cavit rsonnante un circuit rsonnant et calculer ainsi limp
dance ou ladmittance dentre dune cavit.
Soit une cavit qui, lorsquelle est isole, rsonne une pulsation propre co0 et a
un coefficient de qualit propre Qpr Celte cavit est un circuit accord
constantes (R, L, C) rparties. Reprsentons-la par un circuit constantes (R, L,
Q localises (fig. 8) : ce peut tre soit un circuit rsonnant srie (Rs, Ls, Cs) soit
un circuit rsonnant parallle (Rp, Lp, Cp).
Pour quil y ait quivalence entre la cavit et chacun des deux circuits, il faut
que ces derniers prsentent la mme frquence de rsonance et le mme coeffi
cient de qualit que la cavit, soit :
a)

t-sC s Co=l

(51)

Ls w0

b)

(52)

= e pr

Lp Cp Q

(53)

Figure 8.
Modlisation d une
cavit rsonnante par
un circuit rsonnant
srie (fig. 8a) ou
parallle (fig. 8b).

Remarquons que les couples de relations a) et b) contiennent les trois paramtres


caractristiques R, L, C et que, par consquent, ils ne pourront tre dtermins
qu une constante de proportionnalit prs.

15.5.2 Impdance d'entre


SI Supposons maintenant que la cavit est couple un circuit extrieur, mais
que le couplage est infinitsimal, c est--dire suffisamment petit pour que lon
puisse considrer que co0 et Qpr restent inchangs.
Pour tudier lquivalence de ce cas avec le circuit rsonnant couplage infini
tsimal, nous devons nous proccuper de calculer limpdance dentre des cir
cuits rsonnants srie et parallle entre leurs bornes/) et B :
limpdance dentre du circuit rsonnant srie est :
CO
Zs (eu) - R$ 1 + J C?pr

(55)

limpdance dentre du circuit rsonnant parallle est :


(
7 (co) = Rp 1 + j Q pr

(56)
0

Ces impdances doivent tre gales limpdance dentre Ze de la cavit. Or,


nettement en dehors de la rsonance (co l> cu0 ou ( < co0), Zs est trs leve et
Zy/ est trs faible tandis que Ze doit tre presque nulle puisque le trou de cou
plage qui est tout petit est quivalent un court-circuit.

Par consquent, le plan de rfrence o sera compte limpdance dentre de la


cavit doit tre pris, sur la ligne daccs la cavit :
en un maximum dimpdance, soit (2n + 1) X/A de linterface de couplage si
la modlisation est celle du circuit rsonnant srie ;
en un minimum dimpdance, soit nX 2 de linterface de couplage si la
modlisation est celle du circuit rsonnant parallle.
ct des paramtres co0 et (?pr qui sont accessibles au calcul ou lexprience,
il subsiste encore un paramtre Rp ou R^ qui est indtermin. Nous allons voir,
dans ce qui suit comment peut tre leve cette indtermination.
f Dans le cas o la cavit est couple au circuit extrieur par un couplage qui
nest plus ncessairement infinitsimal, supposons que la voie daccs, dimp
dance caractristique Z0, est adapte son extrmit oppose la cavit et rai
sonnons en impdances rduites.

Figure 9.
Modlisation
dune cavit
rsonnante couple
un circuit extrieur :
fl. en impdances
non rduites,
b. en impdances
rduites.

Dans le cas srie :


CO

Zs {() = rs

(57)

1 + J pr

Or, nous avons dans ce cas, puisque z0 = 1 :


1
Gex =

Z0

Z(j cs C0 0

e pr=-

1_
= es

0
I

rs

(58)

rs cs u

(59)

Cex = 1
<2pr " p

do :

(60)
(0

=> Z s(w )= p 1 + j Q pr

(61)

V o

Dans le cas parallle :


z{()=rp 1 + j Q Pr

CO

-1

w0

( 62 )

CO J

VC00

Nous avons, de mme, dans ce cas :


Q e x = 7t p^C70 = z c

w o = l p OJ0 =

2nr =

pr
do :

7 0

cp

wo

(64)

:rP CP< 0

(65)

=7=p
Z//

(c) = P ! + jO , pr

(63)

CO

vco0

CO

- 1
( 66)

Nous disposons dsormais dexpressions de limpdance d entre rduite qui


peuvent tre dtermines uniquement partir des paramtres p, Qpr et co0 acces
sibles au calcul ou lexprience. Lexpression zs (Z//) est utiliser si le plan de
rfrence des impdances rduites est pris en un maximum (minimum) d imp
dance sur la voie daccs.
Dans le cas o la cavit est couple par un petit iris, limpdance dentre dans
le plan de couplage est peu prs nulle en dehors de la rsonance. Il convient
donc dutiliser la modlisation de la cavit par un circuit rsonnant parallle, en
prenant le plan de couplage de la cavit comme plan de rfrence pour son imp
dance rduite. C'est donc l expression ip (co) et cette convention que nous utili
serons dsormais pour calculer l impdance d entre de la cavit rsonnante.

15.5.3 Reprsentation sur le diagramme


de Smith de y e
Ladmittance dentre de la cavit est donne par :
ye = ~ [ ^ i ^ Q p r ]

(67)

2x :

CO

CO,

Cn

CO

( 68)

Figure 10.
Reprsentation sur le
diagramme de Smith
des domaines d'une
cavit sur-couple ou
sous-couple.

Sur labaque de Smith, ye se dplace sur un cercle reprsentatif de la valeur


relle l/|3.
Ce cercle coupe laxe des abscisses en ye = 1 (3 pour co = co0 :
si ce point se trouve lextrieur du cercle g 1, 1/(3 < 1 et P > 1, la cavit est
dite sur-couple ;
si ce point se trouve lintrieur du cercle g = 1, l / p > 1 et p < 1, la cavit est
dite sous-couple ;
si ce point se trouve sur le cercle g = 1, [3 = I, la cavit est au couplage cri
tique.
Loin de la rsonance, ye = ( 1/ P) j 2x Qpc - jX avec X > 1.
Donc, le coefficient de rflexion ]Tf lentre de la cavit est :
r

i-j*
1 + jX

La cavit se comporte donc comme un court-circuit.

1 5 .0

C a vits

c o u ples par un a cc s
Une telle cavit (fig. Il) fonctionne
en rflexion ou en absorption et, pour
en expliquer le fonctionnem ent, il
faut faire ltude du coefficient de
rflexion qui sexprime en fonction
de limpdance ou de ladmittance
rduite de la cavit.

15.6.1

Puissance rflchie Puissance transmise la cavit

Nous venons de dmontrer que limpdance dentre de la cavit est donne, en


valeur rduite, par :
co
z c = P [l +j?nr 2-|-1 avec 2x = ----------1
H J
Cq CO

(69)

en prenant le plan du couplage comme plan de rfrence.


Rappelons que cette impdance a t calcule en modlisant la cavit par un cir
cuit rsonnant qui prend en compte toutes les causes de pertes, y compris celles
qui sont dues au couplage avec le circuit extrieur. Par consquent, le coefficient
de rflexion
qui lui correspond permettra de calculer la rflexion globale de
la cavit que lon pourrait mesurer en intercalant un coupleur directif entre elle
et le gnrateur.
a) Loin de la rsonance
CO

Si co > co0

G),,

> ---------- > o _>

co0 co

Donc :
Si co < co0
Donc :

Ze= m + j e pr(2 * > 0 ) ] - 1

(70a)

ze = re + j xe avec xe < 0
>

CO
C00

----- < 0 -
CO

ze = p | l + j e pr( 2 ^ < 0 ) |- 1
L

= re -t- j xe avec xe > 0

(70b)

Ds que co nest pas trs voisin de co0 - et comme Qpr est trs grand
vons ngliger 1 devant 2x <2pp alors :
JP _

avec

_P_
ZxQpr

nous pou-

(71)

(les valeurs de (3, en gnral voisines de 1, nexcdent jamais quelque dizaines).


Dans ces conditions :
(72)
T ;= 1. La puissance rflchie par la cavit est maximale et aucune puissance ne
pntre lintrieur de la cavit.
b) Au voisinage de la rsonance (to to0)

partir de ze = (3 [ 1 + (?pr 2v]

re =

nous pouvons calculer T, :


3 " 1 - j C?pr 2*
P+ 1 + j ?pr2x

1
-r+ 1

(73)

do nous dduisons :

>

4P

r; i

(74)

(p + l)- + (2 .v C V
qui correspond la puissance rflchie par la cavit.
-> i - r ~ =

4p

(75)

(p + l )- + (2.vC?pr)-

qui correspond la puissance transmise la cavit.

Comme P = <2pr ?ex et P + 1 = ?pi Qctr >' vient :


K
1 _ r - = -------------- * 1 + (2.v (?ch)2

(76)

A Qlh

(77)

avec:

La courbe des variations de 1 - V~ (fig. 12) qui reprsente la variation de la


puissance transmise ht cavit est donne par
Pi :iV
M

PiK

l+ (2 v (? th )2

CO- CO,
avec x =

(78)

co,

On peut galement tracer la courbe des variations (fig. 12) de r~ qui correspond
la puissance rflchie par la cavit ; elle est donne par :
r\ir = I

^cav

(79)

1 + (2v C?ch)2

Ces deux courbes sont complmentaires et la rsonance a = 0 ; donc PcmJ Pmc = K

et P ,tfl

Pw

= 1 - A'-

fafafai-HB.Ua3, 0
f 3

M icro - o n d es

La valeur de K peut tre talonne car nous, verrons (au c)) qu la rsonance :
2
(p-M

Ip+1j

K = 1 -

fp- \ \

lP+1i

(80)

La valeur maximale de K, qui est 1, est atteinte pour [3=1.

Figure 12.
Courbes des variations
de la puissance
transmise la cavit
et de la puissance
rflchie par la cavit.

Sur ces courbes, les points mi-niveau sont obtenus pour a- tel que 2a (2ch = 1.
Soit (Aco)| 2 la diffrence des deux pulsations pour lesquelles ce niveau est
obtenu :
co,
1
(81)
Qch 2a (Aco),.
Ainsi, (?ch se mesure partir de co0 et de (Aco)i /2-

c) la rsonance (to = co0)


P -> r

, z g- i = p - i

Zf+l

(3+1

(82)

D'aprs les courbes traces, nous voyons que la puissance transmise la cavit
est maximale tandis que la puissance rflchie est minimale :

S i P > 1,

P- 1
r =-
'

S i p < 1,

p+i
- p

r =
*

+p

> P =

P =

1 + r,
- re
i - re
l+ r e

R.O.S.

(83)

_1
R.O.S.

(84)

La mesure du R.O.S. permet donc de dterminer p ou 1/P sans quil soit pos
sible de savoir lequel. Cette indtermination ne pourra tre leve quen prenant
en considration le changement de phase la rflexion (voir 15.6.2).

15.6.2 Interprtation physique


du fonctionnement de la cavit
Loin de la rsonance
Nous avons vu que la cavit se comporte comme un court-circuit. Elle rflchit
donc la totalit de lnergie que lui envoie le gnrateur et elle naccumule
aucune nergie en son sein. Le R.O.S. dans la ligne dalimentation sera donc trs
lev.
Prs de la rsonance
ze est complexe et T,, < 1. La cavit ne sc comporte plus comme un court-circuit
et le R.O.S. diminue dans la ligne dalimentation. Par consquent, une partie de
lnergie envoye par le gnrateur est rflchie directement sur linterface den
tre ; une autre partie est transmise lintrieur de la cavit o londe quelle
produit subit de multiples rflexions sur les parois de la cavit.
Cette nergie transmise lintrieur de la cavit engendre :
de lnergie emmagasine sous forme stationnaire (ractive),
de lnergie active qui compense les pertes dans les parois et le dilectrique,
de lnergie active qui est rayonne vers lextrieur travers louverture de
couplage.
Dans la ligne qui relie la cavit au gnrateur, il y a une onde rflchie qui se
compose :
de londe rflchie directement sur linterface dentre de la cavit,
de londe rayonne par la cavit travers cet interface.
Il est trs important de noter que les puissances transportes par ces ondes ne
sadditionnent pas obligatoirement. En effet, elles correspondent des ondes se
propageant dans la mme direction mais qui ne sont pas forcment en phase. En
particulier, il peut se produire q"e ces ondes soient en opposition de phase,

auquel cas les puissances qu'elles transportent se retranchent et pourront san


nuler.
la rsonance

ze = P est relle. Le coefficient de rflexion


= (p - 1)/(P + 1) est rel et passe
par sa valeur minimale ainsi que le R.O.S. dans la ligne dalimentation de la
cavit. Londe transmise la cavit est maximale.
Notons que si p > 1,
est positif : il y a rflexion sans changement de phase ;
tandis que si P < 1, Tt, est ngatif : il y a rflexion avec un dphasage de 180.
Comme en dehors de la rsonance, la rflexion seffectue avec un dphasage de
180 (Lt, = - 1;, lexamen du comportement de la phase quand la cavit est
dsaccorde permettra de savoir si cette cavit est sur-couple (auquel cas la
phase change de 180 entre co0 et co * co0) ou si elle est sous-couple (auquel cas
la phase ne change pas entre co() et co ^ coy).
Dans le cas particulier o p = 1, Tt, = 0 : il ny a pas de rflexion sur la cavit
bien quil y ait une onde rayonne par la cavit vers lextrieur (puisque p = 1 :
Pg.JPpr = 1 d o / cx = /Jpi ). Ceci signifie que londe rayonne par la cavit vers
lextrieur et londe rflchie sur linterface de couplage sont damplitudes
gales et en opposition de phase, de sorte que les puissances quelles transpor
tent doivent tre retranches et quelles sannulent.

15.7

C a v it c o u p l e par d e u x a c c s
Considrons une cavit ayant deux
accs par lesquels elle se trouve
couple dune part une source et
dautre part une charge (fig. I3j.
Cette cavit fonctionne en absorp
tion laccs reli la source et en
mission laccs reli la charge.
Globalement, elle fonctionne en
transmission de la source vers la

Figure 13.
Cavit couple
par deux accs.

charge. Soient :
Pj, la puissance incidente,
Pj, la puissance dissipe dans lensemble cavit + charge,
Pr la puissance transmise la charge.
Nous nous proposons de dterminer :
X(l = P(l Pj, coefficient de transfert de la cavit en absorption ;
Tt = P,h PUi, coefficient de transfert de la cavit en mission ;

T, = P J Pj, coefficient de transfert de la cavit en transmission ou coefficient


de transmission.
Nous avons videmment :

xt = x(l Te

(85)

Pour rsoudre ce problme, nous allons utiliser le schma quivalent ce mon


tage (fig. 14) o :
les lments rp, i p, cp
sont propres la cavit
isole,

J
rp2 <
5

les lments r et r
p1
reprsentent les rsis
tances in tern es de la
source et de la charge
sur lesquelles est dissipe la puissance perdue travers les couplages dentre
et de sortie.
Toutes ces impdances sont normalises par rapport une impdance de rfrence
qui est souvent limpdance caractristique des lignes daccs la cavit.
Si la ligne est adapte la source du ct de lentre, nous avons r = l .
Compte tenu des rsultats tablis au paragraphe 15.5 :
les relations caractristiques de la cavit sont :
( 86)

(p cp i4 = 1
et

^pr = 7 ~

= ^ c/> o

(87)

les relations correspondant au couplage dentre sont :


n

r,h
V ^0

^exl

rP \ c p

rP

et

Pi =

(?ext

( 88)

(89)

Pt

les relations correspondant au couplage de sortie sont les mmes en rempla


ant les indices I par des indices 2,
le coefficient de surtension en charge est :
1 _ 1
(^ch

Qpr

1 f
(^exl

\_
(?ex2

1
pr

U + P l + P 2)
(90)

Limpdance dentre de lensemble cavit + charge se calcule par une relation


du type de celle donne au paragraphe 15.5 :
CO C0r
avec
2x

(91)
Zll = rp [l + } 0 Pr2x\
cof
co

Figure 14.
Modlisation par
un circuit parallle
dune cavit couple
par deux accs.

condition dy remplacer r par un r' tel que :


rP rP 2
1
1 1
, _
T = ----H
rP
rP
rP 2
P rp +rP2

Pi

(92)

1 + Pa

et Qpr par un <?pr


tel que :
Pr
1 _
Qpr

Q pr

Q ex2

Qpr

,+ )= i<1+w

(93)

Pi
1 + j ^ pr 2x
1 + P2
1+P2

Donc :

i .

I+P.

e t.

(94)

Qpr

1 +J ------- 2x
1+P,

(95)

ESDans ces conditions, le coefficient de rflexion lentre de la cavit est :

1-y e
r =
~ e 1 + y.

- ( l - P i + P 2) - j G p r 2^
1 + P, + P2 + j Qpr2x

(96)

Nous pouvons en dduire :


la puissance rflchie lentre de la cavit :
Pr

2 ( l - P , + P 2)2 + ^ r (2x)2
=r =
Pi
e (1 + p , + p 2) + 0 2r (Zv)2

(97)

la puissance dissipe dans la cavit :


Pd=

2 = _____ 4 P l ( 1 + P2>

P,

(98)

(1 + P, + p 0)2 + e 2 (Zv)2 ^

Par ailleurs, un calcul simple bas sur la considration du circuit quivalent o


les puissances dissipes sont inversement proportionnelles aux rsistances cor
respondantes, permet dtablir que :

P,

r.

P2

rP + rp2

l+p2

(99)

Finalement, nous trouvons le coefficient de transmission de la cavit utilise en


transmission :
4 ^ ,p 2
X, = Xa Xe =

(i + p ] + p2)2 + c>2r (2*y

( 100)

Comme >ch = Qpr ^(1 + Pj + P2)2. nous avons aussi :


4P,P2
( 101)

X' ~ ( l + P , + P 2)2 | l + 2h (2*)2

Remarque
Nous voyons que T, = 0,5 T, (C0) pour des pulsations a>, proches de C0(), pour les
quelles : Q2ch(2\)2 - l.
co0
d'o
Q.h=
(102)
.Mt 2
Acoi 2 est l cart entre les deux pulsations situes de part et d'autre de Cq, pour
lesquelles la puissance transmise par lu cavit a diminu de moiti par rapport
la rsonance.
I la rsonance, x 0 et par consquent :
1 ~ P | + P2

' 1 + P , + P2

(103)

4 P , (I + p 2)
x =

(10 )
(i + p , + p 2r
Po

T .= -

1+P2

(105)

4p,p2
(106)

'

(l+p.+p,)2

Nous pouvons faire, sur ces relations, quelques remarques intressantes :


1) Ladaptation
= 0) ncessite que P , = l -h P2. Il nest donc pas possible de
lobtenir avec des couplages identiques (p, = P-,). ladaptation :

1 ;

P2
x * 1+P,

P2
x, =
' 1+P2

P2
P!

2) Le coefficient de transmission est maximal pour Pi = P2 = P et alors :

4 P*
x,=
' (1 +2p)"

- ^
2
^p rj

(107)

3) Le coefficient de transmission est dautant plus proche de 1 que p] et p2 sont


grands (cavit fortement sur-couple) mais simultanment Qch diminue beau
coup et la rsonance de la cavit est moins nette.

Notons enfin que Boudouris [69, p. 145] a tabli la matrice S du quadriple que
constitue la cavit en transmission :
5=

(Pl -P o-Jc-)
2 v/m 7

____ 1____
Pl + P 3 +.vr

2v/p7p2
i [

(108)

' V)

o vc est ladmittance de la cavit isole avec r = 1 :


co0

CO
C

v( =>H Q Pr

1 5 .3

A pplications

(109)
CO

des cavits

15.8.1 Dtermination de l'indice


ou de la permittivit d'un gaz
Lindice de rfraction complexe est n = n j n".
La permittivit complexe est : e r = e - j e.
Calculons n = . / e

en tenant compte de ce que, pour un gaz e < e :

En identifiant avec t = n' - j n \ nous obtenons :


n n

110)

( 111)

Dtermination de n' et de e'


La frquence de rsonance est :
pour la cavit vide : / 0 = c/X,
pour la cavit remplie de dilectrique \ f tl = v/X - c j r i \ .
Do :

n.

fo

fcl

112)

Nous avons aussi :

i
A rt
n - 1/a l

avec A f = f0 - f d

A/
n = i +

Et si A/: / 0 < 1 :

(113a)

(113b)

JO

n' et e peuvent tre calculs partir de la variation de la frquence de rso


nance.
b) Dtermination de n" et e"
J _ _ J_

Rappelons que :

Q ch

Qo

( 1

pour la cavit vide :

QchJi

(114)

Qd

Q ex

Qo

(115)

Q<

pour la cavit remplie de dilectrique :


1 )

(116)

~r J -}---+
,Q c h ) 2 Q o
Q ex
Qu

Do :

-!-= ( U
<J.i I C c J ,

O r:

Q j~ tg ~ z

(117)

' 1
U J c h /,

1
et

A/

Gch"/

A/est la largeur de la courbe de rponse de la cavit - 3 dB.


Donc :

e _ A /2

A/,

fi

(118)

do e et n" = f /2 n \
n" et e peuvent tre calculs partir de llargissement de la courbe de
rponse de la cavit.

15.8.2 Rsonateurs en ondes millimtriques


[6 8 , p . 1 0 8 -1 1 4 ]
Les cavits conventionnelles ne conviennent pas en ondes millimtriques : dune
part leur fabrication devient impossible car leurs dimensions devraient tre de
lordre de grandeur de X ; dautre part, il nest pas possible dobtenir des coeffi
cients de qualit assez levs, vu quils sont proportionnels J X -

.Rappdns que te
rapport e / e ' = tg
es* apoel facteur de
pertes dv dilectrique
et que S est appel
l'angle de pertes de ,j
ce dilectripue.

En revanche, des rsonateurs analogues ceux utiliss en optique dans les interfromtres de Fabry-Perot conviennent bien en ondes millimtriques. Ces rso
nateurs sont des cavits ouvertes constitues de deux rflecteurs (plans, sph
riques ou parabolodaux, douverture 2a) placs face face et spars par une
distance cl (11g. 15)
Figure 15.
Cavit rsonnante
ouverte
a. rflecteurs
plans ou
b. rflecteurs
sphriques
Les rflecteurs, qui sont mtalliques, ont un coefficient de rflexion trs voisin
de 1. Le couplage avec la source millimtrique est effectu travers un petit trou
perc dans lun des rflecteurs. Londe excite subit des rflexions multiples
lintrieur de la cavit et lon dmontre [70, p. 37-45] que certaines distributions
de champ, de type quasi T.E.M., peuvent stablir et que ce champ se concentre
autour de laxe longitudinal de la cavit. Les modes de fonctionnement sont
nots TE , o les indices m et n sont lis la rpartition transversale du champ
tandis que lindice p est li la variation longitudinale.
La condition de rsonance impose que la phase de londe varie de 2n ou dun
multiple entier de 2k entre deux rflexions successives. Pour un rsonateur
ouvert rflecteurs sphriques, de rayon de courbure b, lcriture de cette condi
tion donne [68. p. 111 ] :
, 4
b -d
d = 2p + (2n + m + 1) 1 ---- arctg
b +d
n

X
4

Le coefficient de surtension en charge est donn par :


Wa
<2ch =

(119)

( 120)

Wa est lnergie emmagasine et Pp la puissance perdue dans le rsonateur. En


faisant intervenir les coefficients de qualit correspondant aux diverses causes
de pertes - par rflexion, par diffraction, par absorption dans le dilectrique et
travers les couplages - nous avons :
I
l
1
_1_
1
1 J_
?ch

Qr

Qd

Qa

Q ex

Q pr

Q ex

Les pertes par diffraction sont, en gnral, ngligeables. Les pertes par absorp
tion peuvent tre nulles dans le cas dune cavit vide : dans ces conditions
(2pr = Qr - Quant QR, on dmontre (voir exercice 15.8) que :
2vjd
( 122)
Qr =
X O .R

o.R est la fraction de la puissance Pp reprsentant les pertes dans les rflecteurs.

Les applications numriques de cette formule montrent que QR est de lordre de


10' 10 en ondes millimtriques, ce qui permet de disposer dexcellentes
cavits rsonnantes.

15.8.3 Frquencemtres cavit


La rsonance d une cavit cylindrique, lorsque sa longueur est gale un
nombre entier de demi-longueurs d onde, est utilise pour des mesures de la fr
quence de londe qui sont d autant plus prcises que la rsonance est plus
troite, donc que le coefficient de surtension est plus lev. Ces frquencemtres
cavit sont souvent appels ondemtres.
Lune des bases de la cavit est constitue par un court-circuit mobile dont la
position est repre par une vis micromtrique. Ltalonnage pralable peut tre
effectu par comparaison avec un talon de frquences. La cavit doit toujours
tre place en drivation afin de perturber le moins possible le circuit dans
lequel on veut mesurer la frquence.
Selon la position du dtecteur, la cavit peut tre utilise en absorption (fig. 16a)
ou en transmission (fig. 16b). Dans le premier cas, la rsonance est indique
par un minimum du signal dtect tandis que dans le second cas, le signal
dtect, qui tait nul en dehors de la rsonance, passe par un maximum la rso
nance. Le second montage est beaucoup plus sensible que le premier.
Figure 16.
Frquencemtre
cavit monte en
drivation sur une
ligne. Par rapport
au dtecteur, la cavit
est utilise :
a. en absorption, ou
b. en transmission.

Bien que les cavits cylindriques section rectangulaire fonctionnant en mode


TEI0^ conviennent trs bien, ce sont les cavits cylindriques section circulaire
fonctionnant en modes T E llp et TE01/., qui sont les plus utilises. Deux pro
blmes se posent dans la ralisation de ces cavits :
obtenir une surtension en charge aussi leve que possible, ce qui ncessite
dutiliser des couplages avec lextrieur caractriss par un coefficient de cou
plage p
1 et des longueurs de cavit gales plusieurs fois X /2 ;
rester en rsonance monomodt dans une certaine bande autour de la frquence
de rsonance. Pour cela, on a intrt utiliser le fonctionnement en mode
TEUp (qui correspond au mode fondamental T E j,). Le fonctionnement en
mode TE01p permettrait datteindre des prcisions de mesure plus leves (de

lordre de 10 " 6 au lieu de 10 " 5) mais il faudrait alors liminer les autres
modes en utilisant la proprit des modes TE0| dont les lignes de courant sont
des cercles transversaux.

15.3.4 Autres applications


Les cavits ont bien d autres applications quil nest pas possible de dvelopper
ici. 11 convient, toutefois, den citer quelques-unes :
la mesure de e et e dchantillons dont les dimensions sont trs petites par
rapport la longueur donde. Ces chantillons sont placs lintrieur dune
cavit rsonnante et la mesure est faite par une mthode de perturbations dont
on trouvera un expos dans [37, p. 115 121J ;
lutilisation de cavits, appeles rhumbatrons, dans les tubes vide tels que les
klystrons / 46, p. 2X5]. Ces cavits interviennent dans linteraction localise
entre le faisceau dlectrons qui les traverse et londe lectromagntique qui
rgne lintrieur delles ;
les circuits lectroniques pour micro-ondes o les cavits rsonnantes, et
notamment les rsonateurs dilectriques, sont utiliss en tant que circuits
accords, par exemple pour fixer trs prcisment la frquence des oscillateurs
ltat solide utilisant des transistors effet de champ lAsGa [71, p. 251 et
sq/.

*i$

EXERCICE 15.1

(Paragraphe 15.3.1 - Cavit cylindrique section


rectangulaire)
Une cavit rsonnante de section rectangulaire,
de dimensions a = 7,5 cm et b = 5 cm a une
longueur f = 15 cm. Rechercher les modes
dont les frquences de rsonance se trouvent
entre 3 et 4 GHz et donner leurs frquences de
rsonance (daprs Gardiol [37. p. J30]).

&

EXERCICE 15.2

(Paragraphe 15.3.1 - Cavit cylindrique section


rectangulaire)
Une cavit rsonnante paralllpipdique a ses
trois premires frquences de rsonance pour
5,196 GHz, 6 GHz et 6,708 GHz. Trouver les
dim ensions de ses trois cts a, h (section
droite) et 6 (longueur) (daprs Gardiol [37,
p.130]).

&

&

(Paragraphe 15.3 - Rsonateurs dilectriques)


On considre un rsonateur dilectrique sec
tion circulaire tel que celui schmatis sur les
figures 6 et 7. Les donnes sont les suivantes :
r = 40 ; e2r = 2,4 et/ = 10 GHz.
Par ailleurs, on prend le paramtre h = 5,4/D .
1) Quelle est la signification physique des condi
tions exprimes par la double ingalit (18) ?
Entre quelles limites doit tre compris D ?
2) On prend D = 10 mm ; L\ = 11,2 mm ;
L2 = 0,8 mm. Calculer la hauteur L du rso
nateur.

EXERCICE 15.5
(Paragraphe 15.4.3 - Calcul de Qj)
Le facteur de forme dune cavit cylindrique
section circulaire de rayon R et de longueur h,
fonctionnant en mode TEon^ est donn par :
3
u h, ( 1 + A *2 r 2)2
~ 2jt l + / t 2 r 3

EXERCICE 15.3

(Paragraphe 15.3.2 - Cavit cylindrique section


circulaire)

Quelle doit tre lulongueur hdune cavit cylin


drique, section circulaire de rayon R = 1,5 cm,
pour que sa frquence de rsonance en mode
fondamental TEn l soit de 9 GHz ?

EXERCICE 15.4

avec :

2R
r=
h

et

t
pn
A - -
2 u mn

1) Montrer que F(r) passe par un maximum


pour une valeur de r que lon calculera. En
dduire lexpression de FmiiX.

2) Calculer la valeur maximale (Q )M du coef


ficient de surtension d aux pertes dans les
parois, d une cavit de rayon R = 6 cm,
fonctionnant en mode TE01| la frquence
de 10 GHz. On prendra
= 5,7 107 S/m.

est 9 GHz e^ la largeur de bande - 3 dB


de sa courbe de rsonance est 4 MHz. Dter
miner lindice de rfraction n = n ~ j // de ce
gaz ainsi que sa permittivit = 8 - j .

EXERCICE 15.8

EXERCICE *15.6
(Paragraphe 15.6.1 - Puissance rflchie
Puissance transmise la cavit)

1) la frquence de rsonance dune cavit, le


module de son coefficient de rflexion en
champ est 0,33 ; dautre part, la phase de ce
coefficient est la mme que lorsque la cavit
est en dehors de la rsonance. Dterminer le
facteur de couplage |3.
2) Pour 8 988 MHz et pour 9 012 MHz, la
puissance absorbe par cette cavit est la
moiti de celle absorbe la rsonance.
Dterminer la frquence de rsonance, Qch,
Q pr e* f ex-

&

EXERCICE 15.7

(Paragraphe 15.S. I - Dtermination de l'indice ou


de la permittivit d un gaz)
La frquence de rsonance dune cavit remplie
dair est 10 GHz et son coefficient de surten
sion en charge est 8 333. Lorsque cette cavit
est remplie dun gaz, sa frquence de rsonance

(Paragraphe 15.8.2 - Rsonateurs en ondes


millimtriques)
Soit un rsonateur constitu par deux rflec
teurs mtalliques plans, distants de d, dilec
trique air. Les pertes par diffraction sont suppo
ses ngligeables. Les pertes par rflexion sont
caractrises par un coefficient a.R reprsentant
la fraction de puissance qui se dissipe par effet
'8 e0 co' 2
Joule dans les rflecteurs a.K

a , conductivit des rflecteurs.


1) Calculer le coefficient de qualit Qj li aux
pertes par effet Joule dans les rflecteurs en
fonction de (d, o.R et X0).
2) Dmontrer lexpression de Q: en f( d , .0)
dans le cas o la conductivit
des rflecteurs
-7
est a = 5,33 10 S/m. Application num
rique au cas o : d - 0,20 m et . = 1 mm.
3) Le facteur de pertes de lair qui se trouve
entre les deux rflecteurs est tg 8 = 2 10 _ 6.
Calculer le facteur de qualit Qd li aux
pertes dans le dilectrique. En dduire le
facteur de qualit propre la cavit.

E X E R C I C E S D J C H A P IT R E 2
P ro p a g a tio n

sur u n e lig n e e n h a u te fr q u e n c e

tp, H-iPj

EXERCICE 2.1

- J
p-, e

D aprs les relations (5) et (1 la), nous avons :


- y Vt- e ~ yx + y V r eyx = - (/?, + j L { co) /
Compte tenu de (9) et (11b), nous pouvons
crire :
- V'|. e - >r + V r eV
R\ + j L { 03

JS-

( / , e - ^ + / / ew)

y=\fpi
= 27.54 i o - 6 e J77I25
a = 6,137 10 6 Np/m
P = 26,848 10 6 rad/m.

EXERCICE 2.3

C, + j C, co
En identifiant les termes en e
et en e i l
vient :
V;
R | + j L I 03
c x +J C,

li

03

6*1 +J C] 03

avec p , =26,848 1 0 ~ 6

(rsultat de lexercice 2)
X, = 234 103 m = 234km

R l + j ^ i 03

yr

1) X l

V. =

03

2rc 103
= 234 000 km/s
26,848 10 ~6

- -

= 235 833 km/s


\fLl Q
03
P ,=
=26,64 O- 0 rad/m
1 l

2) v. =

EXERCICE 2.2

1) Pi = \ [ R \ + R 032 =20,'
L [ 03
<Pi = arctg = 70,25
P2 = ^ / g 2 + c 2 032 = 36.
C ( 03
i
IPI -<p2

2) z t.=

\ / P2
- e
V
Z,.. - 746,6 - j 90.

EXERCICE 2.<1
1) /n = Z0 + Zc

0
oc
II

cp2 arctg

10

. -752 e J 0.875

100
175- j 120

175- j 120 = 2 1 2 ,2 0 e "J34440


Do / 0 = 0,471 e J 3444

(A)

V0 = Z0 / 0 avec Z0 = 173,28 e ~i 4383


Do : ^Vi\
81,61 e ~ j9 -39
0=

(V)

2 ) D a p rs le s r e la tio n s ( 2 7 ) e t (2 8 )

V r = Y. o

cos

2k

----- j Z c. /

V 01

I / I_t\ c o s 2n

0 sin 2n
.

sin 2 tc

2 jt 2 4 0 n io d u lo 2 K (6 = 1 0 m ;

= 1 .0 4 W a tts

- 4 0 ,8 0 5 e j 9'39 + j 2 0 ,3 9 5 e

EXERCICE 2.6
X=

1,5 m )

j 34,44e

(Zo),, = Zt.th T^ e t(Z 0)0 = ^

= - 5 1 , 7 9 2 5 + j 2 3 ,4 7 7 5
- - 0 ,2 3 5 5 e j 3444 + j 1 ,4 1 3 5 e
0 ,0 3 7 + j 1 ,2 6 2

Lr

D o :
et : Lr

3)

7-r

VR =
=

Donc : Z~ = (Z0)L.C(,Z0)ci}

- j 9,39"

5 6 ,8 6 5 e j 24-38

(V )

1 ,2 6 3 e + j 8 8 3 2

(A )

= 4 5 ,0 2 e -* 112,7t>

(fi)

O n p e u t re tro u v e c e r su lta t p artir d e la fo r


m u le ( 2 9 ) q ui d o n n e

ZR p o u r

x =

1) Nous avons dmontr (paragraphe 6) que :

t.

2 (Z0\-c
et : (th yf)2 =
do y?
( / ^ (<)
Avec les valeurs mesures pour (Z0)cc et (Z0)H),
on trouve :
Zc = 753,06 e J 7 = 747,45 - j 91,77 (fi)
th yf = 0,704 e "-i 28 = 0,62 - j 0,33
O r- c -2v f= ] ~ t h ^ = 0.50e 40,97
1 + t h y f l,6 5 e - J H51C
= 0,30 e J 5248

EXERCICE 2.5

i ) P u is q u e

ZR =

Donc : e ~ 2ae = 0,30 et e

Z c n o u s a v o n s l e n tr e d e la

lig n e :
Z 0 = Z c = 8 0 0 - j 2 0 0 = 8 2 4 ,6 2 e j ,4
L e c o u r a n t l e n tr e d e la lig n e e s t d o n n p a r :

100
-

Z0 + Zc

8 7 5 - j 200

= 0,111 e J 12-875

R\ + j L\ c = yZt.et C l + j G, (o = ^~

2) Z#? = / 0 e - ^ e t ^ = V 0 e ^
e -7f = e - a f e -JPf

y = et + jP = 2,746 10 5 e j77-38
Zc = 753,06 e j7

ocf = 0 ,7 7 5 N p - > e a f = 0 ,4 6
P = 2 5 5 rad ( 2 tc) = 0 ,5 8 4 5 rad = 2 1 0 ,4 2 4

D o : Lr

= 0 ,4 6 e ->2 I0 '424

0 ,0 5 1

e " j 197-549

e t : 1 :^ = 4 2 , 2 6 5 e - J 211549

3) ^aCt = ^ [ V / ?/ ; j = ^ [ z / 2]

Do : eaf = 1,8125 > a = 0,6 10 5 Np/m.


et : 2fi = 52,48 + 360 puisque C doit tre
positif.
p = 153,76 = 2,6836 rad
> P = 2,68 10 5 rad/m
2) partir des relations de dfinition de y et Zt. :

o = Z () 70 = 9 1 ,8 8 e ~ J l,l2 5

D onc : e ~

= e *52,48

(A )
(V )

R , + j L x 03 = 2 068 10 5 e j7038
= 694,39 1 0 - 5 -hj 1 947,93 10 5
-* /?, =6,94 10 ~ 3 fi/m
et L x =3,10 10 6 H/m
G, + j G, 03 = 0,00365 10" 5 e j 84380
= 3,57 10~ 9 -t- j 0,00363 - 10 5
- > G , =3,57 10 9 S/m
et C] =5,78 - 10 12 F/m

fa

EXERCICE 2.7

Or : k =

1) Zc = v 75 x 300 = 75 x 2 = 150 Q

2n

03

et dk

do)
v

v est la vitesse de propagation en espace libre


dans le dilectrique dont est constitue la ligne
(v = c uniquement si le dilectrique est de lair).

2) Zri = 7 1 5 0 x 3 0 0 = 150 / 2 = 212 O


ZC2 = v/ 75 x 150 = 75 yj2 = 106 Ci

coder
03 dco
Donc : = p d p ou
v
P ' dp

2) v

A: d * = (3 d P

II

1) Diffrentions la relation le = (3 + A2 :

l6-

Do :

^ 0 EXERCICE 2.S

CO
= fK
P

E X E R C I C E S D U C H A P IT R E
tu d e d e la r fle x io n l'e x tr m it d 'u n e lig n e

fa

= /* [- 0,9595 - j (4 + j 2) 0,2818]

EXERCICE 3.H

= 0,0341 e - J45-17 1,1947 e J 70-65 e J 18

1) V ( x = ) = Y e = y R cos + j Zc L r Sin P
= Lr (Zr cos |3 + j Zt. sin p t)

Le = 0,0407 ej 205-48

P = 28,56 rad = 4,5455 271


= 0,5455 360 = 196,37

2 ) Z, = ^ = 1 8 5 1 e - J 40-82

Ve = l R 1 000 L- (2 + j) 0,9595

On retrouve ce rsultat par la formule

~e

- j 0,5 0,2818]
= 34,1 e - J 45-166 2,212 eJ 29-822 ei 180
Ve = 75,43 ej 16466
l e

cos

p + j

r z c + j z ^ tg p i

(V)

yR

z = z ZR + )Z c t g p i

s in p

3)

1=

^ ^ ^ ;]

C
= 75,43 0,041 cos 40,83
= / , cos pc + j sin p
Z(.

= 1,16 Watts

(A)
(O )

Or : arg (Z(. - j Zw) = - arg (Z(. + j XR)

EXERCICE 3.2

X r

= - arctg

z - z c- _ 5 0 + j 100 _ 0,5 + j
i) r R =
ZR + Zc ~ 200 + j 100 ~ 2 + j

X r

Donc : arg T R = n - 2 arctg

= 0,5 eJ 36'87 = 0,4 + j 0,3

2) a) arctg

2) Sur la charge (x = 0) :

M= ;

= V i ( \ +

1+E,

V R) et V* = V , | 1 + VR \
143,2
= 100 V
1,4+ j 0,3

vr = r w v /= 5 0 v

3)

Xp
50
b) arctg -y- = arctg - --- = - 33,69
-> arg

.f

V f = 100ej u
- > V r = r R V, = 50 e j 36-87"

(V)

~ = Zt ->/,- = 1,33 e j

(A)

j - = - Z c - * L r =-

(A)

0,667 e J 3687

100
= arctg - y = 53,13

> arg JTW= 180 2 53,13 = 73,74

L R = ~ eX- K = Y i + - r
Donc : V K

Z/?

= 180 + 2 33,69 = 247,38

EXERCICE 3. zs.

1) 1 N p = 8,68 dB ;
donc : a = 0,3 dB/m = 0,0345 Np/m
2K
P = = 20,1062 rad/m
X
ZR - Z..
2 ) a) T* = Z
^

250
350 = 714

r (( ) = rRe- 2y<:= r we 2a< e- 2jf5f

= 0,667 e J 216-87

e -2a< = e -0,0690 = 0>933

4 ) y * = y , + y , = y (. (l + r )
y * = 100 (1 .4 + j 0,3) a 143.2 ej l2' (V)
/ = /, + / , = / , ( ! - r wi
I R = 1,33 (0,6 - j 0,3) = 0,895 e j 26-56 (A)
'act = ^ ^ [ y /, / ] = 2 7* ^ fZ/?]
= ^ 0,895 2 x 125 = 50 W

2n
R( = ~ 2 n 3,2 0,4jc (modulo 27t)
X
e - 2j p = e - j O, 8K

Donc : (6) = 0,666 e


J l44
^ 1+T() ^ 0,461- j 0,3915
b) Z{C) = Zc
=50
J
1 - r tf )
1,539+ j 0,3915
0,6048 e "J 40-34
1,5880 e i 14,27
= 19,04 e J 54-61 = 11,03 - j 15,52

Z(() = 50

^ 0 EXERCICE 3.3

>

_ ZK~ Zc _ i XR~ Zc _

Zt--J X R

z R+ zc ~ iX R+ zc -

Zc+ j X R

rR=i
et arg R = aig (- 1) + arg (Zc - j XR)
- a rg (Zc + j XR)

d EXERCICE
i) r

3.5

65 + j 75 _ 99,25 e J 4908
165 -hj 75 181,25 e->24-,44

nous avons :

E * = 0,548 e j 2 4 ,6 4
i + rD
= 3,425
R.O.S. =i - r .K

P act 2

un

La position xM dun maximum de tension se


dtermine daprs :

/M =

<p= 24,64

_ ( ^ m)2
p zc

Nous obtenons donc les valeurs efficaces cher


ches :
VM =
= 1(
= 141,4 V

2 )Z M = p Z t.= 171,25 Q
Zt
Zm = = 14,6 2

<P- 2kn = 2 p x M > jcm =

p Z,

^ = I00 =
tw
P

'

V2

vM

_
= </2 = 1,414 A

kn
/m = = 4 ^ = 0,707 A
p V2

= 0,43 radians

<P

- f = 215
La position du premier maximum de tension
correspond k = 0
' M = l 5 C = 0-065m
Le premier minimum de tension est a \ ; 4 plus
loin.
Comme X = 2rc/ P = 1,9 m, nous avons :
xm = 0,065 + 0,475 = 0,54 m.

/Zd

e x e r c ic e
p

3.:

1) r = - 2 = ^ = 0,333
H n+1 3
cp -

2p Am = (2k + 1) tc cp = 4k + tc(2n)

. 0,75
, _
Donc : cp = 4 j i ----- + 7t = 1,3tc
10

J T = 0,333 e j 234

^0 EXERCICE 3.6
Puisque la li^ne na pas de perte, nous avons :
\
^aci = 2 a ^C0/CJ0*] V a
Plaons-nous, par exemple, en un maximum de
tension : la tension VM et 'e courant Zm sont en
phase, puisque ZM est relle, et nous avons
donc :

i+r
Z = Z - - A = 100 0,804 j 270
e 11 _ Lfl
r
1,196+ j 0,270
Z ff= 100

Comme / m = VM/Z m et ZM = p Z(

1,226 e j 2-72

Zfi = 69,17 e " i 31-28 = 59,115- j 35,915 Q


2 ) ^ = \ ^ \ . V r Lr] = \ R r I R
2
Donc : IR =

' Jact = 2 ^ [ V M'm] = 2

0,848 e "j 18>56

2P
= 1,84 A
R,

et: VK = \ZR\IR = 127,27 V

E X E R C I C E S D U C H A P IT R E 4

tiOS

D ia g ra m m e d e S m ith

&

E X E R C IC E 4 .1

( f i g . E .1 )

1) M est le point reprsentatif de


arg E* = 128.

0,54 et

Le R.O.S. se lit lintersection du cercle de


rayon TR 0,54 avec laxe des rels droite
> p = 3,35.
Impdance rduite correspondant au point M :
zR - 0,36 + j 0,44

Do : Zw = zR Zc - 18 + j 22

(2)

Admittance rduite correspondant au point M :


yR = 1.13 - j 1,35
Do : Yr = yR Yc = (2,26 - j 2,7) 10 - 2 S.
2) De la mme manire, on trouve : p = 2,35
zR = 1,07 - j 0,9

ZR = 53,5 - j 45 Q.

y R = 0,55 + j 0,45 - ^ = (1,1 + j 0,90) 10~2 S

Figure E.l.

EXERCICE 4.2 (fig. E.2)

ZR
1) z* = 7 = 0 ,8 + j 1,3
(Point M du diagramme)

Au point Z?, on lit le R.O.S. : p = 3,9.


La position du point M en coordonnes polaires
nous donne :

F/j = 0,59 eJ 6"'

2) De la mme manire, nous obtenons :

yR = 0,35 - j 0,55
(Point M \ symtrique de M)

- 0,6 - j 1,4
yR = 0,26 + j 0,60 ->YR = (0,52 + j 1,20) 10 ~ 2 S

YR=yRyc=(0;7-j 1.1) 10 - 2S

p = 5,3 et * = 0,68 e j 65

Zr

Figure E.2.

EXERCICE 4.3 (fig. E.3)*1


Z R

1) zR = ~ = 0,6 - j 1,1
z,c.
(Point M du diagramme)
Au point B, on lit le R.O.S. : p = 4,1.
La position de M en coordonnes polaires nous
donne : T R = 0,61 e j 76

2) 11 cm de la charge : x f k = 1,375 soit 0,375.


Nous devons donc tourner de 0,375 vers le
gnrateur en partant de la graduation 0,355
qui repere la position de la charge. Le point
darrive correspond donc la graduation :
0,355 + 0,375 = 0,730 soit 0,230
En ce point :
z ~ 3,3 + j 1,6 - > Z = 165 + j 80.

Figure E.3.

3) Un minimum de tension correspond au point


A dont la distance la charge est
= 0,500 - 0,355 = 0,145
Do : jcm = 0,1453, + - = 1,16 cm + 4k cm
(k, nombre entier positif ou nul pour le premier
minimum).
En ce point : zm - 0,24 Zm = 12 2.
Un maximum de tension correspond au point B
dont la distance la charge est :

T
Do :

+ 0,250 = 0.395

\
= 0,3953. + - = 3 , 16 cm + 4k cm
k

En ce point : cM = 4,1 >ZM = 205 Q.

EXERC3CE

4.4

(fig.

E.G)12

1) La position du point AL reprsentatif de


zR = 2,1 + j 1,35, est dtermine par la gradua
tion 0,212.
Pour trouver limpdance dentre ze, il faut
tourner de T./X = 1,184 soit 0,184 vers le gn
rateur ; ze correspond donc 5 la graduation :
0,212 +0,184 = 0,396.
Nous lisons :
ze = 0,48 - j 0,64 (Point N)
Do : Ze =

7C= 36 - j 48 Q.

2) Le R.O.S. se lit en B : p = 3,1.


r'R est donn par les coordonnes polaires du
point M :
V K = 0,51 e j 27

Figure E.4.

3) Le premier minimum dimpdance se trouve


en
Sa distance la charge est :
= 0,500 - 0,212 = 0,288
X

2) Le point reprsentatif du circuit ouvert est C


pour lequel
= . On tourne vers le gnra
teur jusquau point E reprsentatif de z = - 1,5 j.
- = 0,344 - 0,250 = 0,094 -> = 0,94 cm
X

D o : xni = 0,288 1,9 = 0,547 m.


La valeur de 7 m est : Zm = 0,325 75 = 24 Ci.

^
n

EXERCICE 4 .5 (fig. E.4)1

1) Le point reprsentatif du court-circuit est C


pour lequel
On tourne vers le gnra
teur jusquau point D reprsentatif de y = 2 j.
La longueur t de la ligne est elle que :
- = 0,250 + 0,176 = 0,426 ->

= 2,13 cm

EXERCICE 4.G (fig. E.5)

Pour le premier tronon de ligne :


Z, '
Z\ = - - = 1 - j 1,5 (P o in ts! <-> 0,324)
'L'i
On tourne de C^/Xt = 0,333 vers le gnrateur
jusquau point Z?j (<- 0,657 soit 0,157) repr
sentatif de limpdance dentre. Le point Cl
reprsente ladmittance dentre :

ye = 0,34 - j 0,60.
Pour le deuxime tronon de ligne :
Z2
z2 = 1,33 + j 1 (Point A2 <-* 0,182)
On tourne de C2 f/o = 0,300 vers le gnrateur
jusquau point B2 (- 0,482) reprsentatif de
limpdance dentre.
Le point C> reprsente ladmittance dentre :

3V, = 2,3 + j 0,52.


Pour trouver ladmittance rsultant de la mise
en parallle de ces deux tronons dimpdances
caractristiques diffrentes, nous devons obli
gatoirement passer en valeurs vraies ; nous
avons :
Ye*1 =yeM YcLl (6.8 - j 12) 1 0 - 3 S.
Ye = ye Yc = (30,7 + j 6,9) 1 0 - 3 S.
*2 ' L2 l2
Do : Ye = y + y ^ = (37,5 - j 5,1) 10 3 S.

Figure E.S.

E X E R C I C E 4 . 7 ( f i a . E .6 )
1) Nous partons du point A qui reprsente c:m et
nous tournons vers la charge de jc X 0,348
sur le cercle R.O.S. = 2,7. Nous obtenons
zH= 0,88 + j 0,95 (Point B
0,348).

2) Nous parions du point B et nous tournons


vers le gnrateur de L/X = 1,450 soit 0,450
sur le cercle R.O.S. - 2,7.
Nous obtenons le point C (<- 0,102 soit 0,450
+ 0,152 = 0,602) qui reprsente
ze 0,55 + j 0,60.

EXERCICE 4 .8 (fig. E.6)


1) Soit a | lu position d'un minimum de ten
sion, cest--dire dimpdance, lorsque la ligne
est termine par ZR.
Lorsque lon remplace ZR par un court-circuit,
le minimum de tension (qui dailleurs devient
nul) se trouve en x2 (voir fig. 8a et 8b du cha
pitre 3 tude de la rflexion lextrmit
dune ligne ) dont la distance au court-circuit
est nX, 2.
La distance entre le minimum dimpdance et
la charge est donc :

nX
nX
Xm = (AI - a2) + - = 5,25 cm + -0
Pour trouver zR, il faut donc partir de nl (point
A) et tourner vers la charge de 5,25, 50 = 0,105
sur le cercle R.O.S. = 5. Nous obtenons le
point B' pour lequel : zR = 0,32 j 0,72 do :
ZH= 16- j 36.
2) Avec un circuit-ouvert, les minima sont dca
ls de X/A par rapport au cas dun court-circuit.
Comme XjA 12,5 cm, le minimum de tension
se serait dplac de 5,25 + 12,5 = 17,75 cm vers
la charge ou de 12,5 - 5,25 cm = 7,25 cm vers
le gnrateur.

0,105

E X E R C I C E S D U C H A P IT R E 5

Le s d is p o s itifs d 'a d a p ta tio n

EXERCICE 5.1 (fig. E.7)

rateur, partir du point A, le premier point dont


limpdance est relle est le point B.

1) / = 500 MHz, X 60 cm donc , = X/ 4.

En B : zM = 2,95 - ZM = 295 Cl.

Impdance de charge .

La longueur Ey est donne par :


6,
= 0,250 - 0,102 = 0,148 do Ex = 8,88 cm
X

Zr = R+ jL( = 50 + j 62,8 Cl.


Impdance rduite :
ZR = 0,5 + j 0,628 (Point A

0,102)

La longueur
doit tre telle que limpdance
Z, (Ej - e) soit relle ; en tournant vers le gn

Limpdance caractristique de la ligne X 4


doit tre :
_______
Z ; = 295 x 100 = 171.75 2

Figure E.7.

ter**
UHt
2) / = 6U0 MHz, V = 50 cm.
Zfl = 50 + j 75,40
-> z,< = 0,50 + j 0,754 (Point C <-> 0,116).
la distance
rateur :
(Point D

- E ( ff /E = 0,178) vers le gn
z, = 1,9- j 1,5
0,116 + 0,178 = 0.294).

la distance

+e :
Zc
Z \= z] = 1,106- j 0,873
^ c
(Point E
0,333).

la distance
gnrateur :

+ -> e ((;2/ ^ 0,300) vers le

z2= 0,79 + j 0,75


(Point 7 0,633 soit 0,133).
la distance

Figure E.8.

+e :

z c
z 2 = z'2 - y - = 1,357 + j 1,288
'"C
(Point G
0,185).
Le cercle de centre O passant par G correspond
un R.O.S. de 2,9.

EXERCICE 5.2 (fig. E.8)


Zc = v/50 7,5 = 19,365 Q
Limpdance d entre de la ligne doit tre
gale Z^, soit en impdance rduite :
z*G = 0,387 - j 0,568 (Point <-> 0,091 )
Limpdance de charge, en valeur rduite, est :

j_.J

ZK = 2 ,582- C( Zc

partir du point A, il faut se dplacer vers la


charge sur le cercle R.O.S. = 3,6, jusqu la
rencontre du demi-cercle r 2,382 et x < 0
Lintersection est le point B (<-> 0,221) pour
lequel : = 2,852 j 1,5
C
1
1
do : =0,130 e t -------- = 1,5
X
Cto A
donc: C = 0,130 X et C = 2,24 1 0 - 12F.

n E X E R C I C E 5 . 4 ( f i g . E .9 )

ZR
= = 0,3 - j 0,85 (Point A)
AyR = 0,37 + j 1,05 (Point B
0,134).
ZR

Le stub doit tre plac un endroit de la ligne


o la partie relle de ladmittance est gale 1.
11 y a deux solutions.
1) En C : y (d e)= l + j 2.

E X E R C IC E 5 .3

La distance de ce point la charge est :


En appliquant la procdure qui a t dtaille au
paragraphe 5.6.1, on trouve que :
l re solution :
d = 10,6 cm et s 5,3 cm,
2e solution :
d ' - 18,6 cm et s' = 19,7 cm.

d = 0,188 - 0,134 = 0,054 -> d= 1.62 cm


X
Ladm ittance d entre du stub devra tre :
y (s) = j 2. Or, cette admittance rduite est
normalise par rapport Zc l/L . = 50 Cl.

Figure E.9.

0,312

Donc y (s) = Y(s)r Yc, o Y(s) est ladmittance


dentre du stub en valeur vraie.

La longueur dun stub en circuit ouvert (y = 0)


dont ladmittance dentre est j 4 (Point E <->
0,211) est :

Comme le stub a une impdance caractris


tique Z = l / y t> 100 2, la mme admittance
Y(s) normalise par rapport 7 c' a pour valeur
rduite :
y(s) =

= y(.v) - y - = y(s)

c- = - j 4

Nous devons donc rechercher la longueur dun


stub en circuit ouvert (y = 0) dont ladmittance
dentre est - j 4 (Point D
0,289).
- = 0,289 - a- = 8,67 cm
X
2) E n C : y (d' - e ) = 1 - j 2.
= 0 ,3 1 2 -0 ,1 3 4 = 0,178 ->d'= 5,34 cm
X
Ladmittance d'entre du stub devra tre :
y(.v) = j 2
soit y(.v) = j 4 lorsque lon normalise par rappoit 100 Q.

Figure E.10.

=0,211 >s= 6,33 cm


X

fa

E X E R C IC E 5 .5

En appliquant la procdure qui a t dtaille


au paragraphe 5.6.2, on trouve :
l re solution : s J = 5,6 cm et s 16,75 cm,
2e solution : sj = 3,3 cm et = 5,35 cm.

fa

E X E R C I C E 5 . S ( f i g . E. 1 0 )

Nous allons donner une solution entirement


analytique :
zR - 1,5- j 1,2 et yR = 0,40 + j 0,33
y(0) = yR + ySf = 0,40 + j 0,33 + j 6,
Daprs la formule de transformation des admit| tances :

v()+j tg

1 + j y() tg
et :

PX
8

EXERCICE 5.7 (fig. E.11)


v(0)+j
1 + j v(U)

+ 2 (s2)

1 + j 0 ,4 0 -0 ,3 3 -/? ,
0,40 + j 0,33 + j | + j + J *2

soit : z

!X
,
O r: z g + e |= 1 + j
donc :

N orm alisons les im pdances par rapport


Z( = 40 Q. :
zR = 0,1 + j 0,05 (Point A -* 0,008) et zG = 1,25
Nous allons crire la condition dadaptation
linterface (, + ). Comme les admittances
de part et dautre de cette interface sont com
plexes, cette condition est :
/ ( f + ) = >-(O
Calculons y {) en pariant de la charge 2/j '

0,67 + j 0,40-/?,
0 ,4 0 + j 1,33+j /?,

= 2 cm, X = 25 cm, = 0,080


( - ) = 0,14 + j 0,61 (Point B <h >0,088)
v ( - ) = 0,36 - j 1,55 (Point C)
v () = 0,36 - j 1,55 + j Cp to Zt.
( 1)

En galant parties relles et parties im agi


naires, nous obtenons deux quations deux
inconnues (/>,, ,r2) :
(1) 0,93=0,40 x2 - b l
(2) 0,27 = 1,33 x 2 + /?, + /?, a2
do lquation du second degr en a 2 :

(3) x \ + 2jc2 - 3 = 0
qui a pour solutions :
*2 = 1 - b \ = - 0,53
jtj = - 3 -/?, = - 2,13
do lon dduit :
z(s%) = j 1 et = 0,125
X
2 (5

) = j 3 et

s "2

= 0,302

(distances lues sur le diagramme de Smith entre


le point 2 = 0 et les points reprsentatifs de j 1
e t j 3)
>'(sj) = - j 0,53 et = 0,174
,
>'(5 ,) = - j 2.13 e t- =0.070
A,
(distances lues sur le diagramme entre le point
v = 00 et les points reprsentatifs de - j 0,53 et
j 2,13.)

Le lieu de y() est un arc du cercle C, g = 0,36


et /? > - 1,55.
Calculons v (f + ) en partant du gnrateur
1
I
(2 )
z ( + )= L 2 5 - j
C , eu
Le lieu de c ( + ) est le demi-cercle r = 1,25
et x < 0 (demi-cercle C2) ; le lieu de v ( + )
est le dem i-cercle sym trique par rapport
au centre du diagram m e (dem i-cercle C2)
? le lieu de y*(t + ) est le demi-cercle sym
trique du prcdent par rapport laxe hori
zontal (demi-cercle C4).
lintersection de C, et de C4, nous lisons :
y(C) = y*( + e ) = 0,36 - j 0,4 (Point D) (.3)
et lon en dduit :
y (/' + ) (Point E)
et 2 (( + ) = ! ,25 - j 1,45 (Point F)

(4)

Daprs les relations (1) et (3), on trouve Cp :


j Cp tu Z( = j 1,15 do : Cp = 7,625 pF
Daprs les relations (2) et (4), on trouve Cs :
-j

Zt. Cveu

= - J 1>45 do : Cs = 4,573 pF

Figure E .ll.

E X E R C I C E S D U C '-W P 'T R E
L ig n e s a v e c p e rte s

nous avons :

E X EI?,C 5C E 6.1

a 4 ~ 2 a 2 p 2 + p4 = 7?r C f

Nous avuns dmontr que :


a 2 - P2 =

G, -

Ll C ,

2 cx p = to (L j G , +

2 R \ Gy Ly Cy

CO

Cy

CO

to2

C ,)

II faut tablir lexpression de a 2 + p2 ;

A a2 2P

2 ,2L [ G j

to

+ 2 co2 G , C j 7?j C j + co2 2

Cf

ttbU& d

do :
a

4.0

+2 a

2 n2

rj 4 _ r>2

p +p

2 ,

- A j G j + o)
,

+ co

Oj

2 n 2 r 2 , r2/-2
aj

Donc : ZL.

2 r 2 /^2

C j + L j C i co

ou : a " + P" \J {R \ + L j co~) (Cf + c f co~)


En faisant la somme et la diffrence de a 2 - P2
et a 2 + p2, on obtient les expressions de a 2 et
P2, do finalement les expressions (I) et (2) de
a et p.

fa

E X E R C IC E 6 . 3

Par dfinition des Nepers :


K (Np) = ln ^122 = 2 ln e ~ 2a^ = ocf
F

P (f)

Cette relation nous montre que lunit naturelle


de ex est le Neper par mtre (Np/m).
fa

E X E R C IC E 6 . 2

Par dfinition des Dcibels :

1) Dans lexpression (3) de Z i l est possible


yC
de ngliger les termes en C f et R l G, ; dans
ces conditions :
" ~
^ "
L\ C , co" + j io(L\ O' - /^ , C\)

K (db) = 10 lg
b ^p () = io lg
b e ~ 2at
Comme ln .v = 2,326 lg .v, nous avons :
P{0)

no

c 2 t2
L\

L\ G { - R { C,

1,1513 1g

no)
rih

J _ ig ^ 2
8,o8o

Donc: K ( N p ) = g ^ K ( d B )

C? co
.f- jC j- ^ jC ,
7 ^ \ / 1+j
C I C j co
Comme 7?, et Gj sont faibles et compte tenu des
ordres de grandeur de Cj, L l et co, la seconde
racine est de la forme

La valeur numrique de K en Dcibels est donc


gale 8,686 fois la valeur numrique de K en
Nepers ; do lon tire que :
1 Np = 8,686 c'B et ldB = 0,115 Np

1 + e = 1 + e/2 donc :
r g,

Z,.=

1+j

2 C j co

2 L , co;

1 -j
Comme a = -^R\

*l

EXERCICE 6.-C

o = 0,1 (dB/m) 10 (m) - 1 dB.

2 G j co

Ci
Ci

La perte due la dsadaptation est donne par


labaque de la figure 7. Calculons le R.O.S. :
(relation (6))
(relation (4))

et P = cov/L , C,
^
P

fa

La perte due laffaiblissement linique de la


ligne est :

2) Dans ce cas (Gj = 0) :

nous avons :

et : AT(dB) = 8,686 K (Np)

1
2 Lj co

+ r

1
P = --------avec
1 - 1r I?
K

\ZR ~ Zc\
|Z * + Z , |

Comme ZR et Zc. sont rels et que Z R > Z, nous


avons :

f*a-

ZK ~ Zc

ZA>

Si ZR < Zc, nous avons :

r = z ^ c d o u p = z r =4
Z c

Nous lisons sur labaque une perte de 0.9 dB


correspondant O' = 1 dB et p = 4.
La perte totale est donc de 1.9 dB.

Z R

ZR + Zt.

4.

do

Le rsultat reste le mme.

EXERCICES DU CHAPITRE S
Lignes bifilaires et coaxiales

^3

E X E R C S C E S . *5

De mme, la rsistance linique du conducteur


extrieur est :

a) Calcul de 7?!
La rsistance dun conducteur par unit de lon
gueur est :
] J
/? ,=
o *
o G est la conductivit et .v la section o circu
lent les courants.

Do :
/?, =/? J + /? ,

Dans notre cas, cause de leffet de Peau, les


courants pntrent une profondeur :
5 = - , - - -----\ J n Voc f
cest--dire pour le conducteur intrieur (ext
rieur) dans une couronne de section 2n r} 6
(2n r-, 5) puisque 8 < r (, r2La rsistance linique due au conducteur int
rieur est donc :
1
*1 =

1
\ f n P o O ,/
2nrx

b) Calcul de L l
Daprs le thorme dAmpre, le courant / qui
circule la surface du conducteur intrieur du
coaxial est gal la circulation du champ
magntique H sur la circonfrence d un cercle
de rayon r (r < r < r2) : / = 2tirH.
j

Do linduction magntique :
1
2nr

Le flux de B travers une surface de longueur


parallle laxe du coaxial et de largeur dr
est :

/ = CJV + jCoCV. La conductance linique peut


donc tre obtenue en remplaant e par o dans
la formule de C (.
Do : G, - 2no
ln 1

Le flux total est donc


K f r2
0 = |in
0 2n Jr,

\r\

dr
1 , r2
= p . - ln
r
2n
r,

Comme le facteur de pertes dun milieu dilec


trique est tg = O, coe, nous avons :

Linductance par unit de longueur est :

G, = 4 n 2 E /tg S -

1 O

L' - r ,
D o :

ln
n

1 2k

EXERC'CE S.2

c) Calcul de C l
Soit Q | la charge par unit de longueur sur le
conducteur central du coaxial. Appliquons le
thorme de Gauss un cylindre de rayon r
(r, < r < r2) et de longueur : le flux de lin
duction lectrique D = eE (qui est radiale)
travers cette surface est gal la charge qui se
trouve sur le conducteur central.
O,
InrTD = Q . do E = ----
1
2nre
La tension entre les deux conducteurs du coaxial
est donc :
r7
Q i f i dr Q\ , r 2
lCr2
Gi
V=\ E d r = ~
ln
jr|
2jte ->r
2;te
r]
La capacit par unit de longueur est C t = Q t V.
1
D'o : Cj = 27ie
ln

Nous avons vu au chapitre 6 que, pour une


ligne faibles pertes :
a =

R,

/ cT

+G

=a-

2
l RW

reprsente les pertes dans les conducteurs.


/-i

= ot.

reprsente les pertes dans le dilectrique.


Compte tenu des expressions (1) (4) pour les
param tres prim aires d une ligne coaxiale,
nous obtenons :
1

a t. (Np/m ) = vAjt?
'1

d) Calcul de G1
Pour un milieu dilectrique avec des pertes
conductrices caractrises par une conductivit
a, le courant en rgime variable est
2 = o + jme E.
En termes de circuit, cette relation s'crit :

ln
7t/

a . (Np/m ) = - - tg
u

Pour exprimer a f. et
en dB/m, il suffit de
multiplier ces expressions par 8,686 (voir lexer
cice 6.3). Dautre part, explicitons numrique-

ment les coefficients de

olc

et de

a .d

:
Or :

n e f= n e0 e rf =
d f
60
Do : 2L = y = ln

d o : 8,686 . n e f =

= 4,578-10 ~5 y f e J
Cest bien le coefficient de a t. (dB/m) dans (10).

Dans le cas trait :


Z^ 7 &

,n 3 l6 = 512 4 fi

nf
v = n f s f i = n / \ / e0 e r^ 0 = K f\T * r f
dou : 8,686 = n
V

8,686

fa

r 3 108

= 9,096- 1 0 8/ . / e
Cest bien le coefficient de a (/ (dB/m) dans (11).

fa

EXERCICE

S.3

L 1w

1
\J n ^0 a f

1
d|

S.f.

1) Lexpression de P peut tre mise sous la


forme :

et p2 tant imposs, nous pouvons crire :

1) Daprs les relations ( 1) et (2) nous obtenons :


*1 _

EXERCICE

1
d2

P=K

ln x avec K =

\ p 2 = cte

1
f a
W iJ

dont la valeur numiique est 2,36 10~3.


D aprs les relations (3) et (4), nous obtenons :
G l = tg 5 = 10 _31

Ci w
En ngligeant R l/ 2 L l co et G ,/2 C | , lerreur
effectue est de lordre du millime.

dP
dv

(ln x) x 2 - (ln x) (.v2)


a4

I - 2 ln .v
a3

La puissance transporte est maximale si


dB /dr = 0, soit pour :
ln x = ^ x = e0,5 = 1,65
Pt

Pt

2) = 1,65 p. = -r-r= = 6,06 mm


p,
L65
P = -p20 (I 5)2 (606 10 3)2 5
= 1,53 kW

EXERCICES DU CHAPITRE 9
Lignes bandes et fentes

ivi

; SL

&

L,

E X E R C IC E 9 .1
Zc = 1) vv| h = 0,5 ; nous devons donc utiliser la for
mule (2) daprs laquelle ee = 5,8.
Ce rsultat est confirm par labaque de Wheeler
(fig. 11 ) sur laquelle nous lisons :
= 2,4.
2) Daprs cette abaque, nous lisons Z0 = 170 Q.,
do :
= 70,83 n

Dautre part, la vitesse de propagation sur une


telle ligne est :
1
vp
Nous avons donc
Z ,=

VP C l

S -()

La formule la plus simple est celle dHammerstad


(10) daprs laquelle : Zm = 69,51 Q.
D aprs celle de Wheeler (7), nous trouvons
Zm = 69,20 Cl.
Ces formules sont donc en accord mieux que
! %.
La prcision de lecture sur labaque est de
lordre de 2 3 %.

Pour une ligne dilectrique Ee :


1
= z
(Z r) =
e (vp)e (c 0 E
Pour la ligne dilectrique air :
1
(Zc)n =
0
(v/>)0
r : (vP e = 7 = et (C l)e = (C l>0 ce-

V ee

Il en rsulte que :
z*-=

n E X E R C I C E 9 .2

Dans cette formule (8), Zm est limpdance


caractristique de la ligne microbande mod
lise par une structure dilectrique homo
gne, de permittivit ee ; Z0 est limpdance
caractristique de la mme ligne, mais dilec
trique air (ee = 1).
Limpdance caractristique dune ligne T.E.M.
est donne, dans lapproximation des lignes
sans perte, par :

EXERCICE 9.3

Dans la formule (12) on veut que :

e(f

< 4

S o it:( |) < 5 ' - * f < l V g

G = 0,6 + 0,009 Zm = 1,23


fd = ~ =4,386 1010
d 8rc
h
Do : / <

11,1

109 = 3,95 GHz

Daprs (21) : A = 3,93.


D autre part : Rs =

n p 0 p / = 8,425 10 3

do lon calcule, daprs (19) :


a c 1,35 dB/m
La formule (22), largement surestime, donne
rait : a c. = 2,3 dB/m.

E X E R C IC E 9 . 4
Daprs le graphique de Hammerstad (fg. 12),
nous voyons que w/h < 2. Le calcul de w/h se
fait donc daprs les formules (15) et ( 16;.

3) Les pertes dans le dilectrique sont donnes


par (24).
Compte tenu de ce que
= 6,11 en appliquant
(1) ou (2), nous avons : a d = 0,212 dB/m.

Daprs (16) :A = 2,15


et d aprs (15) : w /h = 0,96
do : w = 0,61 mm.
Compte tenu de lpaisseur h du ruban, sa lar
geur doit tre calcule daprs la formule (11)
dans laquelle x = h puisque w/h > l/2 rt ; donc
w = w + Aw avec :
Aw = f 1 + ln ^ j = 0,0186 mm
Cette correction est de 3 % en valeur relative.

E X E R C IC E 9 . 5

1) Cette frquence est donne par la relation


(25) daprs laquelle :
9

/ m = 2,14

E X E R C IC E 9 . 6
1) Les couples de solutions possibles sont lus
directement partir de labaque de la figure 15.

r = 0,05
h

-f- = 0 ,1 0
b
t
-r =0,15
b
t
--= 0,20
b

W
-> - =0,7
b
vv
- -,- = 0 ,6
b
w
-> = 0,52
b
vv
-4 ~r 0,45
b

v = 0,25 -4 x - 0,37
b
b

1/4

0,635

= 5,837 GHz

2) a.c se calcule daprs les formules (19) et


(21) dans lesquelles nous prendrons we = w et
B = h.

2) La frquence jusqu laquelle cette ligne


fonctionne en mode T.E.M., dans le cas o
w /b 0,6 et b = 1 cm, se dtermine daprs la
formule (35) : / = 7,22 GHz.

EXERCICES DU CHAPITRE 10 f tIM


Ralisation des impdances
et des circuits rsonnants

E X E R C IC E 1 0 . 2

E X E R C IC E 1 0 .1
1) Ralisation dune inductance L = z c. x ' v
Il faut une Zc leve, donc w/h 0,1.
Daprs labaque de Wheeler (fig. 11, chap. 9) :
Z0 = 260 Q.
e t: * f ~ = 1 ,6 5 >e =2,72
do : Zc = Zm = 157,6 Q.

Limpdance aux bornes du circuit srie (L \ C)


est :
Zs = j L 'co ~~ avec L C cOq = 1
C co
Cette impdance peut aussi scrire :
f(~\
Z5 = j L eo 1 [10 J
CO

e t: v = * = = 1,82 108 m/s

ou

A 1 GHz :
30
X
X = t= = = 18,18 cm > = 1,515 cm
12
e
>65
Linductance maximale ralisable est donc :
L =0,866 10 6 1,515 I0 ~ 2
= 1,312 1 0 H

( 2 )

C ' co Vw0

Ladmittance aux bornes du circuit parallle


(L, C) est :
=
+ j Ceo avec LC co?, = 1
>
j
Cette admittance peut aussi scrire :
c o \2
1
1
1ZP j Lco
Vwoj
2

2) Ralisation dune capacit C = (l/Z^) (x/v)


11 faut une Zc faible, donc w/h = 5.
Daprs labaque de Wheeler :
Z0 = 50 G et

en

= L8

do : Z(. = Zm = 27,8 Q.
et v = 1,667 108 m/s.

ou

4 ~ =j C( 1 -

Aprs la ligne X/A, nous avons : ZA = Z 2/Z P


soit :
2
( co )
(3)
j Ceo
r<>y

1 GHz :
X
X = 7= = 16,67 cm = 1,39 cm
e
1La capacit maximale ralisable est donc :
C 0,216 10~9 - 1,39- 10 2 = 3 1 0 ~ 12F.

l <0 j

ou

ZA = Zc

i- W
l

2
j

En identifiant (1) et (3), nous trouvons :


L con = ------ L (0r

M icro - ondes
En identifiant (2) et (4), nous trouvons :
K2
C =
0 C tou

to0
= R + 2j L Aw
to

D EXERCICE

10.3

to0
Donc : X((o) 2L Aco, relation (27)
t

Nous savons que :


e
7t
K
=
tg
Zc tg 2
X

dX

K
arctg
~Z~

do : - = 2L
dto
to

Cette relation permet donc de dterminer :


(K
<= arctg
71
i Zc-

D autre part, nous pouvons crire :


K
e
Zc
2
, e = 2 tg l6
1- m 2 ,
2
[%)
Comme tg 0 =

Or : o r - (Oq = (to + to0) tco - to0) 2to0 Aco


2to0 At
ZAB = R + j U
t

dX
et pour t t(), = 2L, relation (28)
dto
b) Limpdance dentre dune ligne en courtcircuit est :
Ze ] Z(. tg p f
dans notre cas : = X0/ 2 .

2X
, nous avons :
Z,.

Compte tenu de ce que 0 = arctg (2X ZL) ( 18),


les couples de solutions sont :
> 0 e t X < 0 o u f < 0 e t A , > 0

EXERCICE 'O.^

a) Limpdance dentre du circuit rsonnant


srie scrit :
l
z ab

=r+ i

to

t,

cq + Ato

'- i i r

^-o

= 71

K
=+ -

2n

t,

At
= 71+ 71'
t0

At\
tg p = tg 7t + 7t '
0)
Ato
1 Ato^
= tg 7t---- 7t
t 0
V w0 j
Compte tenu de la prsence de R en srie, nous
avons :
Ato
ZAB = R +}Zc n:----,
to0
de la forme R + jX avec :
Ato
X(to) = Zc 7t ---- , relation (29)
to,

1-

LC(
<t2 -w , 2\

= /? + j U
v

03

Pour to to0 :
dX

= Zc ---- = -, relation (30)


dto
to0 + /o

EXERCICES DO CHAPITRE 11 m
Rflexion et rfraction
des ondes lectromagntiques

E X E R C IC E 1 1 .1
CT

E X E R C I C E 1 * * .2

1) --------- > 10 --- ;------ > 10


2tc/ e 0 e ,.
e0 e r
do : / <
2 0 n

en
D er

er

36n 109

1,8a

2 0 k

10 '

Cas TE :

te

Cas TM :

COS \|/

TM = 0 cos \j/

Dans notre cas :


; Ho
1
() = ^ j = 1 27t et cos \|/ - cos 45 = j=

Pour leau de mer :


Donc : TE = 266,62 El et : TM = 533,24 El
/< -

2) R =

109 = 77,143 MHz

avec =

(|J.
U +j)
2a

E X E R C IC E 1 1 . 3
a
i
2Tt/e0 er
1) --------- < -s, - ------------->10
( 'U
, , r.
CT
10a

10a

36t 109

2 tc e D er

er

2 tc

do : / >

180a
/ 2k 1 0 7 4k 1 0 - 7

=\/ -------- 6-------- (l+i)


=2jiV 0,333 <1 +j) = 2 ji - 0,577(1 +j)
donc : - = 0,01 (I + j)
d
0,01 (1 + j) - 1 _ - 0,99 + 0,01 j
et : R =
0,01 (1 + j)+ l" 1,01 + 0,0 j
99 e
R=
= 0,98 e ~J I,15
lO le J'057'

10s

Pour le sol moyennement sec :


, 180 - 10 ~ 3 109
t > ------- 4--------- -- 12 MHz
15
e , cos G! - yj e -, cos 02
2) R . =
e

et : R//

c o s0 1 + ^ /e 2 cos02

Ej

cos09 -

^ / e9

cos0

Ej

cos0 o +

^ /e2

cos

0j

Dans ces deux formules, nous avons : e, = e0


et -> complexe de la forme :

e0=e -j

to

E.
__
H
i.

=e 1-- jj - 1 ,
(
me j

\ / Er COse?

cos Gj + \ f Er cos 02
et : R a -

0] = 60 et 02 se calcule daprs :
e, sin 6[ = \7 e 2 sin 02
sin 02 =

j - sin 0 = 0,2236 07 = 12,92e

A
Dans ces conditions, nous obtenons :

0 ,5 -3 ,7 7 5
3,275
6,5 + 3,775 ~\ 4,275 ~ 0,766
_ 0,975 - 1,936 _ _ 0.961
K" ~ 0,975 +1,936 2,911 J ' U

2 r

2 r r

* 1
, A , = 2

1) crivons les vecteurs de Poynting de chaque


onde, en tenant compte de ce que :

i / i .2
' ui=2 '
u1

P
. =P + P t
i r t
E\
E}
do : ~z u i ----- u r + y - M,
>i
9i
<32
2) En projection sur laxe ries nous avons :
r
E f u r z = - E f u r -z+ E f u r z
^>2

} COS0J = - E ^ C O S ( T t - 0 [ ) + E ^ C O S 0 2
^2
O
En divisant par E j cos 0, pour faire apparatre
/?*et 7, il vient :
,
cos 02
1= R 2 + - - T
e

E X E R C IC E T 1 .4

E,
et
",

La loi de conservation de la puissance scrit,


sur linterface X :

cos 02 Er cos 6 [
cos 02 + \ f r cos 0 t

P
i. =

-*
]
, =

Nous allons donc pouvoir ngliger la partie


imaginaire de e 2.
cos 0'i

1 E1
ia Hi.= 2- E . H*
' u :' = -2
Cl
** 1
* _
] Ej.
P = ~ E /\ H = - E H u = - - w r

Dans notre cas :


O
O
10 3 -3671- 109 18
10
CE G) n
2r. 109 15 _ _ I5
0 r

Donc : R =

Er

COS 0 J

3) On vrifie lexactitude de cette relation aussi


bien pour les coefficients 7? et T L donns par
(84) et (85) que pour les coefficients R//f et l'n
donns par (96) et (97).

EXERCICES DU CHAPITRE 12
Les guides d'ondes rectangulaires

d EXERCICE 12.1

Do : A (dB) = - 54,6

1) Les champs de londe qui se propage dans le


guide sont tous affects dun facteur en :
2n
-j zco sV
e
k

2) Pour z = A,
A (dB)

avec : cos vj/ =

1-

c
A

- 1

54,6

et A,C= 2a.
Si A = \[2 Ac. ><4 = - 54,6 dB

Si >
nous avons : 1 (A/Ar)- < 0 et lon
peut crire :

Si A. = 1,2

A = - 36,2 dB

EXERCICE **2.2
Les champs sont donc affects dun facteur en .

Seule la solution avec le signe - permet de tra


duire un affaiblissement exponentiel caract
ris par le paramtre :

Pour quil soit il polarisation circulaire, il faut


que : TLX j //,. Le signe plus (moins) corres
pond la polarisation circulaire droite (gauche),
,,
.7 0 %x
d ou : cos vi sin = sm v/ cos
T
a
T
a
70
et : tg
a

Donc : E(z) = E0 e az
Laffaiblissement en dcibels est donn par :
>1 (dB) = 20 log ^

1) Le champ magntique a deux composantes


orthogonales Hx et TE.

= 20 log e 4X2

= 8,686 l n e _:
La fonction qui donne laffaiblissement est donc :
A (dB) = - 8,686 o.z

sin xi/
=

-----------

cos y

.
, .
l&i kg
a
La solution tg ---- = X\ = - arctg
a
A.
K
7tX j

71

ci

correspond :0 < ---- < soit 0 < x, <


a
2
2
La solution

7C

correspond a : <
2

. a
< ^ soit - < x j< a
a
2

7TV2

2) Remarquons que :
71*2

b) Sur les faces latrales du guide, qui sont


parallles yOz, il ny a quune composante
J ; donc les lignes de courant sont verticales
(fig. 8).

TU' |

---- - = 7C--------- > * 2 = U - X ]

s> / = / v 2 , x = - ; V2
A

La composante J x est nulle tandis que J, a une


amplitude maximale. Donc, au milieu de la
face suprieure (ou infrieure) du guide, il y a
une ligne de courant parallle laxe Oz (fig- 8).

* si vi/ =
et cos y = ;=
A(. V2
v2

3
D'o : *i = 4 et .v2 = 4 .

EXERCICE 12.3

1) Le courant surfacique est donn par le vec


teur J =n s\H; n est la normale unitaire la
face considre du guide.
t

Les expressions des composantes de /-/ en mode


fondamental sont donnes par les relations (26)
pour !x et (27) pour Jr

F 2 - 4 20 10 3 3 7 7 ------------------232,26 0,747
= 173,87 103 -> := 417 V/m

a) Sur les faces parallles xOz


J y

{H y + H z)= y

Hx +

H,

Jz+ Jx

J a donc deux composantes :


lune en .v, damplitude complexe J_x = t t z
(relation 27)
lautre en z, damptude complexe : J z t l x
(relation 26)

~ J EM
rM = \ (15 105)2 3
X 0,747 - 232,26 10
211
- PM = 258,867 kW.

b) Sur les faces parallles yOz


J = x a (Tv+ / T ,) = ; ^H*z = Ty

fa

E X E R C IC E 1 2 . 5

J na quune composante en y dont lampli


tude complexe est J v = tL~ (relation 27).
2) a) Au milieu de la face suprieure (//xOz) du
guide, nous avons :
a
TC*
TT*
* = _ cos = 0 et sin = 1
2
u
a

1) Condition de oropagation du mode T E )0 :

A < (X p

= 2a > > 2

(C 1>

Condition de non propagation des modes TEnI


{m > 2) :
A > (A^.)'iq ci > ci <z A
(C 2)

Condition de non propagation des modes TEU#I


(>D:
=2 b ^ b < 0

(C3)

Condition de non propagation du mode TE(,


X > ( X j 11
2b
0 r : ( ^ n = - - / r n1 2l2 (_l V
2b

1+

do: b < \ J 1 + ( | ) 2

Comme (Xc)ol > (Xc)u, il en rsulte que le


mode TEu et les autres modes 7ZTmn ne peu
vent se propager.

Puisque a > 2b, la synthse de ces trois


conditions nous donne :
X{ > a = 47,55 mm do / t < 6,31 GHz
X-, < 2a - 95,10 mm dou / 2 >3,155 GHz

b \2

La bande passante du guide est donc :


3,155 GHz 6,31 GHz

(C4)

2) Si le guide est rempli de dilectrique, les


conditions concernant les longueurs d onde
restent inchanges. Par contre, puisque :

La synthse des conditions (Cl) (C4) nous


donne :
X
X
~ < a < X et / ? < 2
la frquence de 10 GHz : X = 3 cm ; donc :
1,5 cm < a < 3 cm et b < 1,5 cm.
2) 8 GHz :
12 GHz:

IB

Xs = 3,75 cm
12 = 2,5 cm

les conditions concernant les frquences sont


divises par :
J V r =sf2,25' = 1,5.
Par consquent, la bande passante du guide est :
2,103 GHz 4,206 GHz.

Les conditions crites au 1) deviennent :


^8
~ Y < a < X i2

et

, ^12
6<-y-

Soit : 1,875 cm < a < 2,5 cm et b < 1,25 cm.

E X E R C IC E 1 2 .6
1) Notons
la longueur d onde infrieure
(frquence/j) et X2 la longueur donde sup
rieure (frquence/2)
Condition de propagation du mode T E |()
X < (Xc) !Q do Xj < 2a
Condition de non propagation des modes TEJIIO

(w > 2) :

X > (XC)2 do Xj > a


Condition de non propagation des modes TEon
(n>l):
X > ( X j oi do
> 2b

EXERCICE T2.7
1) Condition de propagation des modes fonda
mentaux TE l et TEoi :
X < (X.c.)()1 = (A.( ) 10 = 2a X < 4,8 cm.
2) Recherche du premier mode suprieur :
(^r)l)2 (^vLo = u
2
2a
= a s[2
s! 2
Le premier mode suprieur est donc TEn .
Condition de non propagation de ce mode :
X > (XJ f ( X > a J 2 >X > 3,4 cm
3) Bande passante en mode fondamental
X < 4,8 cm - / > 6,25 GHz
X > 3,4 cm /< 8,82 GHz
La bande passante va donc de 6,25 8,82 GHz.

^3

E X E R C IC E 1 2 . 8

Le coefficient dattnuation se calcule daprs


la formule (50) dans laquelle :

Le champ magntique a deux composantes


dont les modules sont :
X
Ttx
X
itx
Hr = H sin et H, = H cos
Xg
*

Xc

Donc :

Au total : j> H j d = H 2 a + 2b

f X)

Le dnominateur reprsente le double de la


puissance transmise par le guide. En mode
fondamental, il est donn, daprs la relation
(31) par :
a

/ / Y zdS

La relation (50) scrit donc :


2

-y TVC ( X \ 2
-y TVC
s i n - ---- 1- cos-

Lintgrale du numrateur doit tre value


sur le contour de la section transversale du
guide :
sur la face suprieure ou infrieure, qui est
parallle xOz :

Rem plaons/par :
V_ V

sur une face latrale, parallle yOz, o


x 0 ou \ a
Hx (x = 0)=Hx (x a) 0

'v _ V

J~X~\'^X~^'~X

X
H,(x = 0) H, {x = a) = H
x
Donc : H J H ?

\b H} d = H2 f l ]

Jo

'

En remplaant X/ Xc par f j f nous obtenons


enfin la formule (51).

E X E R C IC E S DU C H A P IT R E 13

tude gnrale de !a propagation

&

2a
ni

2a
m < - = 3.2
M l )mo
X
Il y a donc propagation de T E 10, TE ? 0 et
TE3.

E X E R C IC E 1 3 .1

Le dphasage subi par londe est donn par :


9:
(0 = 27

a - ia '
, V
avec : X y.=

X,

VM

l
e
J * X*~Tc'

Les modes TE(W| propagatifs doivent satisfaire


la condition :

,
22b
b
2b ,
=
>n< =1,6
v clon n
yX
Il y a donc propagation de TE01.
Les modes TEmfl propagatifs doivent satis
faire la condition :

c = 3 10 8 m/s.
Donc : =
** C
Do : e r ( f )

12

r|/ J

N ous avons

C= 2

12 k

:/=

109

v/"2+"2()

fc
Hz ; / f. - 9

109

Hz

c m ; <p = 2 8 8
m _

[ 2 8 8 ______1_

" ^ M 9/

+ ^3 6 0

2 10 _ 2

3 1 0 8 12
9 109

S o i t : e = 2 5 .

^2

n
a } \b
2a

2 l a \~

=l+ f

l/c j

2k

E X E R C IC E 1 3 . 2

1) A 15 GHz :
3 10 10
x = - U

,/e T /

1,6 15 109

1,25 cm

Les modes TE!/ propagatifs doivent satis


faire la condition :

do : yj~rn +

2a
\J~n? + 4n~

< =3,2
X

Il y a donc propagation de TE(I et TE-,].


2) Sur la figure ci-aprs :
* les points situs en abscisses (ordonnes)
reprsentent la valeur de 2/Xc pour les modes
TEmo (TE) ;
les intersections des droites reprsentent 2/Xc
des modes TEmn
Traons un cercle de rayon 2/X = 1,6 cm - 1 (le
point reprsentatif de 1 / b correspond 1 cm ').
Les points reprsentatifs de 2/-Xc situs lint
rieur de ce cercle (et qui sont tels que 2/Xc < 2/X
<=> X < Xc) correspondent donc aux modes pro
pagatifs. Nous voyons ainsi que les modes
propagatifs sont : T E ,0, TE20, TE30, TE01,
TEj, et TE2|.

Figure E,12.

f'Z

"EXERCICE T3.3

La relation qui synthtise la condition de pro


pagation du m ode fo n d am en tal T E 10 et
les conditions de non propagation des modes
T E , (' 2). T E (n > 1) et TE( (m > 1 et
n > 1 ) est :
J i , 2 b<X
(Cl)
Dans une bande de frquences, dont les lon
gueurs donde infrieure et suprieure sont Xt
et Xv nous avons :
X
-y- < a, 2b < X.
La largeur de bande est donne par :
& x = x s- x i = x i

(C2)

La largeur de bande maximale


correspond
la valeur maximale du rapport Xs/X t ; comme
X < 2 Xj, il vient :

(K )
If

IM M

= 2 ~>a X m = X:

Dans ce cas, il rsulte de lingalit (C2) que :


= 2b.
Remarquons que, dans ces conditions, les modes
TL ,q et TE 01 apparaissent simultanment.
ci

EXERCICE 13.4
1 ) a) Soient X, et X2 (X~> < ^-[) les longueurs
donde correspondant et f 2 :

Condition de propagation du mode T E , 0 :


X< (Xc)w - * X l <2 a
(C l)

Condition de propagation du mode TE0, :


X < (Xc)ol >Xl < 2b

Pour le mode TE0,, le paramtre de phase est :

(C2)

Condition de non propagation du mode TE, , :

do lon dduit : a yj2 < X7

(C3)

Notons que cette condition implique, a fortiori,


la non propagation des modes TE , 0 et TE0,
dont les longueurs donde de coupure sont a et
b.
Finalement, la synthse des conditions (C l)
(C3)nous donne :
<b

(C4)

b) On dduit de (C4) la relation quil doit y


avoir entre / , et / , :
h <

V'2 ./,

/ , = 9 GHz - y

= 1,665 cm

K
/> = 11 GHz - = 1 ,9 3 cm
v2
La condition (C4) est bien vrifie pour
a = 1,8 cm et b 1,7 cm.
2) a) Pour le mode TE, q, le paramtre de phase
est :

P.

2n- avec a
y. =

*
*oi
Soi

U b)

Sur une longueur L de guide, le dphasage entre


ces deux modes est :
A(p = P 10C - P 01 L
2
2 1/2
' x\
1/2
L
1
1 - 2n
,2a l
,2b)
X
X
b) Pour avoir une polarisation circulaire, il faut
que Atp = TT,'2
1/2 - 1
2 1/2
X 1 ' x
1
1
L ~ 4 1 ~2a)
\2bl

(Ml

Pour a = 1,8 cm ; b 1,7 cm et X - 3 cm, nous


trouvons :
L0 9,125 cm.

EXERCICE 13.5

1) Pour/, = 16 GHz,

= 1,875 cm.

P our/, = 18 GHz, X2 = 1,667 cm.


Condition de propagation du mode fonda
mental T E ,, :
X<

2-na
< y ^ 4 4 ~ ^ a> 0.55 cm

Condition de non propagation du premier


mode suprieur TM q, :
2nct
X > a c}TM0| ^ ^ 2 > 2/105 ^ < 6 3 8 Cm
Donc : 0,55 cm < a < 0,638 cm.
2) Conditions de propagation dun mode TE ou
TM :
2na
TM

u,

D o : k

2na
et unm < = 4,4

et :

cos \|/ = -

Il en rsulte que peuvent se propager les modes :


TE ,,, TE2I, TE01, TE3j et TM01, TM n .

/
,0 ' / e 0
Calculons X"/7.0 :
2

1 11

1 1

E X E R C IC E 1 3 .6

[ K

avec Xn = -7 et X =2a
0 /

( M

VK 0)

1) Condition de propagation du mode TM 01 :


Lmi
2.405

2.405X
= 1,15 cm
? 7t

d'o :

Condition de non propagation du premier mode


suprieur TM u :

(A
X 0V

2 na

3.832A.
= 1,83 cm
X > a ^ n = X832 ^ 0 < ^
Do : 1,15 cm < a < 1,83 cm.

246 c m -> /2

12,20 GHz

/n

1 \2

>4
\ KgJ
v
avec :X = j =
d o :
( y

^"0

^ e t XQ= 2 a

-1/2

(*< 0

(* 0 }

11

>*1h

A partir de la condition de non propagation du


mode TM |[, nous trouvons la longueur donde
infrieure de la bande :
2 ;:a

Calculons X^/X0 :

2) partir de la condition de propagation du


mode TM01, nous trouvons la longueur donde
suprieure de la bande :
2n- 1,5
X l,= 2,405 = 3,92 cm > / | = 7,655 GHz

1/2

er"
U

[ K

2) Considrons un guide court-circuit rempli


d air : V R = - 1 e ty = j pfl.
Daprs la relation (152) :
E X E R C IC E 1 3 .7
1) Limpdance d onde d une mode TE est
donne par :
r*=TE =

*
cos \J/

r (z) = r a 1 ~ e ~2 JM
^ 10i + e - 2i P ^
2n
avec : HC
R I=
*a
g

avec =.

M-

Il y a un minimum nul pour tg Pa zni = 0 soit


Pzm=kK do :

nous avons : (^m + ) = 0 , ce qui implique


que :
( )+ j^ o tg P ^ m= 0
3) Considrons un guide court-circuit, rempli
de dilectrique jusqu z = et rempli dair audel.
Dans le tronon rempli de dilectrique :

/? =- 1etY=j P,/Daprs la relation (152) :

soit : j f0 tg 0 f/ + j i0 tg Pc Am = 0
do : ^ t g

= - * ." tg \ia sm

xa
avec : sm = (zm ) + d = k ~ + (d )

1 _ e - 2JP,/
10

+ e 2->Paz

O r: tg p ^ s m = tg

= ^ * 6 P</2
= tg

2n
avec: Pd = 77i
Dans le tronon rempli dair, plaons-nous
une distance s de linterface (soit .v + de
lorigine) sur laquelle () =
j tg
.
En appliquant la formule (154) de transfor
mation des impdances donde, nous avons :

()+j0tg pfls

(s + ) = " "------------------
C + K ^ P ^
nb

s tant compt partir de Vinterface, nous


avons donc : s = z .

Donc : - tg
2 k

= 1 ,5 cm ; d = 0,97 cm et
= 0,4682 avec u =

do :

= 3,975 cm
2 k

La solution est :
u = 4,247 -> Xdg = 2,22 cm
r

4) En un minimum nul de champ lectrique,

(d -()

[Kl

v d
\ K g)

e r = 2,25.

E X E R C I C E S D U C H A P IT R E 1 4
L e s g u id e s d 'o n d e s d i le c triq u e s

E X E R C IC E 1 4 .1

2nu
:,45

01

1) Pour quil y ait propagation, il faut que :


4 nu

cos 0 + 2 <p = 2mn

( 1)

Al
et que :

Soit :

La rflexion totale seffectue sans dphasage


(tp = 0) dans le cas TE et avec un dphasage
de n (cp = 7t) dans le cas TM (voir 14.2.1 ).

4m

Avec ci X0 = 50, n, = 1,5 et 0 1Z^ = 69, nous


trouvons :
mL = 54,8 54 modes.
2)

partir de la formule

nous retrouvons ce rsultat.

&V E X E R C I C E 1 4 . 2

Pour que seul le mode fondamental puisse se


propager, il faut crire la condition de non pro
pagation des premiers modes suprieurs TM0I
et TE01, soit :

X
X { - -f- =

V e !r

do :
(2)

i1r E2tr

2c/M = 2,3

, 2a
do : m L - 1 + " 1 cos 0 1L
o

n V

X\ est la longueur donde dans le cur du guide ;


donc :

Pour 0 = 0 1L, la relation (1) va nous donner la


valeur maximale de ///, soit niL, qui correspond
au cas o <p = n :
-----cos 0 . + 2n = 2 niL n
'L

lr

Do lon dduit le diamtre maximal du cur :


2,405 X-l.
Ir
2 M-

. 2
> 0 ., = arcsm cos 0 < cos G
n|

t r - 2r

3 10l

1.5

2 3 0 - 1 0 14
*
2 VM

= 0,87 pm

gm.

E X E R C IC E 1 4 3

1) La frquence normalise dune fibre optique


e s t.
2na / o
T
V=
V "T "5
50 O^ 6
7x \! 1,45 2 1,40"
1,3 10 - 6
do : V = 45,613
Pour une fibre saut dindice :
V2
M = = 1 040 modes,
pour une fibre gradient dindice :
M - = 520 modes.
4

2) M est inversement proportionnel au carr de


la longueur donde. Donc, le nombre de modes
est multipli par le rapport fit,3 ,83)*2 = 2,453.
Nous avons donc respectivement :

2) Dans ce cas, llargissement dimpulsion est


donn par :
AT =

2 551 modes et 1 275 modes.

^0

E X E R C IC E 1 4 . 4

1) Louverture numrique est calcule daprs :

(An) 2 L
c
r

( 0 , 1)2

1,5

- ' c =2,777 10 ~ 9 s
3 105
dbit numrique maximal = 360 Mbit/s,
bande passante maximale = 360 MHz.

NA = x/ hj - n i = v 0,29 = 0,5385

EXERCICE 7 4 . 6

0M - arcsin NA = arcsin 0,5385 = 32,58


2) Le rendement nergtique du couplage est
donn par :

Le rapport des bandes passantes de ces deux


fibres est 8 n(. ^An. Il faut donc que :

1 - cosm + 1 0M = 1 - cos3*32,58 = 0,40

La perte due au couplage est donc :


10 1g 0,40 = -3 ,9 6 dB
3) Le pourcentage de pB&ance trantunfSfc est :
<>l l - n 0
^ 4 )~ = 0,96
1
2,5
1 +'(
La perte de Fresnel est donc :
10 1g 0,96 = -0 ,1 8 dB.

E X E R C IC E 1 4 . 5

O'-A

U)

1) Llargissement dimpulsion est donn par :


L
~-
"1
AT
(/ j , - n-,) ~
C
n2
I
1,5 0,1
1,4
1* ;

1 000

En prenant nL. = 1,5, nous trouvons que


An = nc s
doit tre gale 0 ,0 1 2 .
Do : 2 = 1 ,5 -0 ,0 1 2 = 1,488.

&

EXERCICE

74.7

*) AT = - (M ) K ^
c
dk~
=

1 5

3 105

10

1,3 10

0 ,5 10 10

On trouve : AT = 0,1083 10 ~ 9 s
-n
7

donc : * = 2 ,8 l 6 s
AT

An

AT

=9,230 109 s 1 ;

do lon dduit :

le dbit numrique maximal = 2,8 Mbit/s,


la bande passante maximale = 2,8 MHz.

do lon dduit :
le dbit numrique maximal = 9,230 Gbit/s
la bande passante maximale = 9,230 GHz.

EXERCICES DU CHAPITRE 15
Les cavits lectromagntiques

E X E R C IC E 1 5 . 2

E X E R C IC E 1 5 .1
La longueur donde de rsonance d un mode
mnp est donne par :

Pour 3 GHz,
- 7.5 cm.

= 10 cm et pour 4 GHz,

Nous recherchons donc les \ np telles que :


1
0,75 10 -1

1 \
10 -1

mnpj

La frquence de rsonance dun mode mnp est


donne par :

Les trois premires frquences de rsonance


correspondent aux modes :
( f l 0 l ) 2 _ ( 1 \2 ( n 2
TE 101
CD
(20) +(2j

-2

TE,'011

/o u
v

TM 110

( f l 10
V v

Soit :

100" f +
+ fi0 0 ^ 2< f 4 f 102
^ < 1 15 J + h o ~ J + h o J <[*
m
Ou : 1 <
1,5

r Y
1

(3)

\ 2 m 2 (foi 1
( , y 2 - 1 2a
1

(2)-( I )

Les solutions sont les suivantes :

100

-(iW

/lo i = 5,196 GHz <yu


= 6 GHz < / 110= 6,708 GHz

d o : 9 < 4 m2 + 9 n2 + p2 < 16.

Valeurs
de
Mode
p m n

( 2)

]2 + ( 1 ' 2
2

Supposons que > > b ; nous avons :

4 \2

+ n +' P
2

1
2b

^ mnp Jfmnp
(cm) (GHz)

14

8,02

En faisant la diffrence (3) - (4), nous obte


nons :
2 v 2 L\* - f l
2a J

10 - / o

l +f

101

i T^oii

10

9,49 3,161

i TE012

13

8,32 3,606

Do : 2a

13

8,32 3,606

En faisant la somme (3) + (4) et en oprant de


mme, nous obtenons 2b = 5,77 cm.

TL 103

8,32 3,606

M V <I 4 ,

i i no
i i TE,,,.
TM,,,

13 m ^ 2

3,741

Soit : y/2 v ~ = (45 - 36 + 27) l/ i 109


= ,/36 109

r r 3 - 1010
io cm = 7,07 cm.
V

36

109

Enfin :

c) Nous trouvons : 4,OS mm < D < 16,65 mm.

l )2 I f l oi
Ml
[2i
= 0,03 - 0,02 = 0,01

2) Pour calculer la longueur L daprs la for


mule (19), nous devons dterminer dabord a ,,
ot'} et [3 daprs (21 ), (22) et (20) compte tenu
de ce que : h = 5,4/ D - 540 m ~

Do : 21 = 10 cm.

fa

ocT = (540)2 E X E R C I C E *55 .3

La frquence de rsonance dun mode TE,


est donne par :

Js'Zmnp_~~l2

a 2 = (540)2 -

'2

=247 734,89

2k

'

2,4 = 186 323,73

13 10 2)
kR

[2hJ

do : a-} = 431,65 m ~ *

,3 10*

3 10

40 - (540)' = 1 463 004,6

do : p = 1 209,55 m - 1 .

Comme lon veut q u e /( ( ( = 9 GHz :


=

3 10 - 2

Do : oc, = 497,73 m

Pour le mode fondamental TE ni


1,841 d p 1.

1f
21,1

2k

La hauteur /. est donc : L 2,35 mm.

1,841
tt-3 10 2

= 302 - 19,53 2

fa

E X E R C IC E 1 5 .5

Do : 2h = 4.3,92 mm >// = 21,96 mm.


1) Calculons dF/ dr et crivons que dF/ dr = 0.

fa

F est de la forme u
v

E X E R C IC E 1 5 .4

,i
z , 3/2
u - (1
+ A ' r~)
a

l) a ) p 2 =7,2 e r - * 2

~>

dr
_ .s
t , 1/2
u , =3A~
r(I + A, 2 r~)

-,.2 2
v = iI + A r > v, = 3A
r

=>P = (Ao v'e,-) ~ ( ' d )

On trouve que dF dr = 0 si /= 1, soit 2R = h.


Pour que p soit rel, il faut que p~ soit positif,
do :
,
r 5,4
5,4
k \
d
k0\, e r

b) oc; = //2-A'6e2). = > a ; - ( 5^4 ) -(* 0 \ e 2,-)2


9

...

Pour que a , soit rel, il faut que oer soit positif,


do :
5,4 ,
,---5,4
D > k \ ^ ^ 7 y 7t 2).
=

P o u rr= 1 >

11mu
,->,1/2
,
(1 +/A )
2K

2) Daprs la relation (43) :


2 V
<(?j)
M~ 5 R

= V/ Kp0 O i /
7 . io lo J/2

= ( k 4 k 10 _ 7 - 5 , 7 I 0 7 - 1 0 10)
= 15 1 0 5

Le volume V/ de la cavit est :


V = tc R2 x 2R = 271 R3
La surface 5 des parois est :
S 4 tt: /?2 + 271 R2 = 6k R2
M:

'oi

1+

V2 i

D'o :

Pc
i

1 _1
Do : Cch _

js

2x\j-,

Jo

=1

Ju

x \/2 2 f \/2 /o ~~ ( A /) , s

Donc: <?ch = ^

V2

= 0,61

Aux frquences telles que

=375.

0,66

= I06 - 2 10~2 0,66


= 40 000.

Comme : ^ = p + 1 -> <2pr = 1,5 Qch - 562,5


t/ch

E X E R C I C E 15.6
1) la rsonance, la relation entre le coeffi
cient de rflexion complexe de la cavit F t, et
le facteur de couplage P est donne par (82) :
P- 1
T =

p+ 1
En dehors de la rsonance, la cavit se comporte
comme un court-circuit et
= 1. .Donc, la
rsonance, arg
= 71 et P < 1 : la cavit est
sous-couple.

i + re

i + 0 ,3 3

La largeur de bande mi-puissance est :


(A / ) ,,, = 9 012 8 988 = 24 MHz
P.-.-

= ^ = - - = l , I l l e = 1,234
Daprs (113b) :
= l

= i -A33 =o s

2) La frquence de rsonance est :


f 0 = 9 000 MH/

O r:

1) Daprs (112) :

= 1,1 - * e = 1,21

Lapproximation est donc de 1,1 % sur n et de


2,4 % sur e.

^ -- I1I=t |I- -, +p

R=

E X E R C IC E 1 5 . 7

2)

f l \
1 ^
ch jga

/*

9 109

Daprs (118) :
e

= iAf^L = 1 = 1,2 - 10 ~4
f " ' 8 333
(Af ) g _ 4 106
= 4,44 10 ~4

(A /)

(A /)

=
- = ^,24 10 - 4
e 4
Ja

71ic 1 + (2r B ch)2

e = 4 10 ~4
" o /-./()
avec : a = ------- -
O)0
Jo
la rsonance : a = 0 et

77 =

= K.

-=1.82-10

Donc : h = 1,1 - j 1,82 10


et : e = 1,21 - j 4 10~4

E X E R C C E 1 5 . 8

(relation 122)

Finalement : Q . - 2jt
a A>^0

1) Par dfinition :
8 e

W
Qj=
i
Wa est lnergie emmagasine dans la cavit.
Pj reprsente les pertes par effet Joule dans les
rflecteurs.
to est la pulsation de rsonance de la cavit.
Soit Pa, la puissance moyenne transporte par
londe qui se dplace dans la cavit entre les
deux rflecteurs distants de d ; elle est gale
lnergie emmagasine, divise par le temps de
parcours :
W
cP =
=W
cl.
d

J
8 2ji 3 108

A>_ 3671- 109 -5,33 107 3.0 _ 4

Donc

2A

Qj = 2?t
'o

= 4n - l 4 O

Avec d = 0,20 m et
Qj=

= 1 mm, nous trouvons

12,56- 104 0,2


- = 800 000
J io-3

1 _ !
d
= 2nJ
a Kr

10^8

Qd = - 1 = 500 000
g 5

PJ

Do : Qj = to

1,2

3) Daprs (48) :

nDonc : nQ , = co /J" d
J
PJ c
or : a .

2) o . -

1.25

Qvr~ Qj + Q~ 106 ^ 10^


Do : ?pr = 307 692.

3^25
106

S-ar putea să vă placă și